Feat: DM pour les Maths complémentaires
continuous-integration/drone/push Build is passing Details

This commit is contained in:
Bertrand Benjamin 2021-05-20 07:20:32 +02:00
parent 6503d17bbd
commit 3c4a16a982
38 changed files with 12713 additions and 38 deletions

View File

@ -0,0 +1,367 @@
\documentclass[a4paper,10pt]{article}
\usepackage{myXsim}
% Title Page
\title{DM1 \hfill BALLARD Antoine}
\tribe{Maths complémentaire}
\date{\hfillÀ render pour le jeudi 27 mai}
\xsimsetup{
solution/print = false
}
\begin{document}
\maketitle
Les valeurs des exercices sont générés automatiquement. Si une valeur a un nombre adhérant de chiffres après la virgule, vous pouvez l'arrondir à l'entier le plus proche.
\begin{exercise}[subtitle={Optimisation de matière}]
\begin{minipage}{0.6\textwidth}
On se propose de fabriquer avec le moins de tôle possible une citerne fermée en forme de parallélépipède rectangle dont le volume intérieur doit être de $8m^3$. La longueur est aussi fixée à $4m$ par le cahier des charges.
On peut donc faire varier uniquement la largeur (notée $x$) et la hauteur (notée $h$) de la cuve.
\end{minipage}
\hfill
\begin{minipage}{0.3\textwidth}
\begin{tikzpicture}
\pgfmathsetmacro{\cubex}{3}
\pgfmathsetmacro{\cubey}{1}
\pgfmathsetmacro{\cubez}{2}
\draw[black,fill=gray] (0,0,0) -- ++(-\cubex,0,0) -- ++(0,-\cubey,0) node [midway, left] {$h$} -- ++(\cubex,0,0) node [midway, below] {$x$} -- cycle;
\draw[black,fill=gray] (0,0,0) -- ++(0,0,-\cubez) -- ++(0,-\cubey,0) -- ++(0,0,\cubez) node [midway, right] {$4m$} -- cycle;
\draw[black,fill=gray] (0,0,0) -- ++(-\cubex,0,0) -- ++(0,0,-\cubez) -- ++(\cubex,0,0) -- cycle;
\end{tikzpicture}
\end{minipage}
\begin{enumerate}
\item Expliquer pourquoi quand la largeur $x$ change, la hauteur $h$ doit elle aussi changer pour respecter les contraintes.
\item Démontrer que l'on doit avoir $h = \dfrac{2}{x}$.
\item On note $S(x)$ l'aire totale de la citerne (c'est à dire la somme des aires des six faces). Montrer que l'on peut écrire
\[
S(x) = 8x + 4 + \frac{16}{x}
\]
\item Démontrer que
\[
S(x) = \frac{8x^2 + 4x + 16}{x}
\]
\item Démontrer que
\[
S'(x) = \frac{8x^2 - 16}{x^2}
\]
\item En déduire le tableau de variation de $S(x)$ sur $\intOF{0}{10}$.
\item Déterminer les valeurs de $x$ et $h$ correspondant à une utilisation minimal de tôle.
\end{enumerate}
\end{exercise}
\begin{solution}
\begin{enumerate}
\item Le volume étant fixe si l'on fait varier $x$, $h$ doit aussi varier.
\begin{itemize}
\item Si $x = 2$ alors conserver un volume de $V=8$, $h$ doit être égale à $2 / 2$
\item Si $x = 3$ alors conserver un volume de $V=8$, $h$ doit être égale à $2 / 3$
\end{itemize}
\item Pour calculer le volume, on a
\begin{eqnarray*}
V &=& h\times x \times 4 \\
8 &=& h\times x \times 4 \\
x &=& \frac{8}{h\times 4} = \frac{2}{h}
\end{eqnarray*}
\item Pour calculer la surface totale, on ajoute la surface de chaque face. On a donc le calcul suivant
\begin{eqnarray*}
S(x) &=& x\times h \times 2 + x\times4\times2 + h\times 4\times 2\\
S(x) &=& x\times \frac{2}{x} \times 2 + x\times4\times2 + \frac{2}{x}\times 4\times 2\\
S(x) &=& 8x + 4 + \frac{16}{x}
\end{eqnarray*}
\item Pour trouver cette nouvelle forme, on met chaque élément sur le même dénominateur
\begin{eqnarray*}
S(x) &=& 8x + 4 + \frac{16}{x}\\
S(x) &=& \frac{8x\times x}{x} + \frac{4\times x}{x} + \frac{16}{x}\\
S(x) &=& \frac{8x^2 + 4x + 16}{x}
\end{eqnarray*}
\item On retrouve la formule $\frac{u}{v}$ à dériver
\[
u(x) = 8x^2 + 4x + 16 \Rightarrow u'(x) = 16x + 4
\]
\[
v(x) = x \Rightarrow v'(x) = 1
\]
Donc au numérateur on obtient
\begin{eqnarray*}
u'(x)\times v(x) - u(x)\times v'(x) &=& (16x + 4)\times x - (8x^2 + 4x + 16)\times 1\\
&=& 8x^2 - 16
\end{eqnarray*}
Donc
\[
S'(x) = \frac{8x^2 - 16}{x^2}
\]
\item Tableau de variations de $S$
\begin{itemize}
\item Valeur interdite: $x^2 = 0 \equiv x = 0$
\item Signe de $8x^2 - 16$: c'est un polynôme du 2e degré
\[
\Delta = 512 > 0
\]
Il y a donc 2 racines
\[
x_1 = - 1.4142135623730951 \qquad
x_2 = 1.4142135623730951
\]
Et on sait que $8x^2 - 16$ est du signe de $a$ donc positif en dehors des racines
\item Le dénominateur $x^2$ est toujours positif.
\item Tableau de variations
\begin{tikzpicture}[baseline=(a.north)]
\tkzTabInit[lgt=3,espcl=3]{$x$/1,$8x^2 - 16$/1, $x^2$/1, $S'$/1, $S$/2}{$0$, $- 1.4142135623730951$, $10$}
\tkzTabLine{d,-, z, +, }
\tkzTabLine{d,+, , +, }
\tkzTabLine{d,-, z, +, }
\tkzTabVar{D+/ , -/ , +/ }
\end{tikzpicture}
\end{itemize}
\item On a donc une surface minimal pour $x=1.4142135623730951$ et $h = 2.8284271247461902$.
\end{enumerate}
\end{solution}
%%% Local Variables:
%%% mode: latex
%%% TeX-master: "master"
%%% End:
\begin{exercise}[subtitle={Bassin}]
Le tour d'un bassin au niveau du sol présente deux axes de symétrie : laxe des abscisses et la droite déquation $x=4$. Il est obtenu par symétrie de la courbe $\mathcal{C}_f$ sur $\intFF{0}{4}$$f$ est la fonction définie par
\[
f(x) = \left(- x^{2} + 0.3 x - 7.3\right) e^{- x} + 7.3
\]
On admet que sur $\intFF{0}{4}$ la fonction $f$ est positive.
\begin{enumerate}
\item Sur un repère, tracer l'allure de la courbe $\mathcal{C}_f$, les axes de symétries puis compléter pour dessiner la forme du bassin.
\item Montrer que la fonction $f$ admet comme primitive sur $\R$ la fonction $F$ définie par
\[
F(x) = 7.3 x + \left( x^{2} + 1.7 x + 9.0\right) e^{- x}
\]
\item Calculer la quantité $\ds \int_0^4 f(x) \; dx$, vous donnerez le résultat sous forme exacte. Interpréter le résultat et reportez cette quantité sur le graphique.
\item On considère que l'échelle de votre graphique est de 1unité pour 15m. Calculer l'aire du bassin. Vous donnerez un résultat arrondi au $m^2$ près.
\end{enumerate}
\end{exercise}
\begin{solution}
\begin{enumerate}
\item
\begin{tikzpicture}[baseline=(a.north), xscale=1, yscale=0.5]
\tkzInit[xmin=0,xmax=5,xstep=1,
ymin=0,ymax=10,ystep=1]
\tkzGrid
\tkzAxeXY
\tkzFct[domain=0:10,color=red,very thick]%
{ (-x**2 + 0.3*x - 7.3)*exp(-x) + 7.3 };
\end{tikzpicture}
\item Il faut dériver $F(x)$ et vérifier que $F'(x) = f(x)$.
\item $\ds \int_0^4 f(x) \; dx = F(4) - F(0) = \frac{31.8}{e^{4}} + 20.2$
\item La quantité calculée à la question précédente se retrouve 4fois pour former le bassin. Il faut ensuite prendre en compte l'échelle, comme 1unité de longueur correspond à 15m, une unité d'air correspond à $15\times15 = 225m^2$. Ainsi l'aire du bassin est égale à
\[
(\frac{31.8}{e^{4}} + 20.2)\times 4 \times 15^2 = 18704.00000
\]
\end{enumerate}
\end{solution}
%%% Local Variables:
%%% mode: latex
%%% TeX-master: "master"
%%% End:
\begin{exercise}[subtitle={Stylos}]
\emph{Les parties {\rm A} et {\rm B} de cet exercice sont indépendantes.}
\bigskip
\begin{minipage}{0.6\linewidth}
\textbf{Partie A}
\medskip
Deux ateliers A et B fabriquent des stylos pour une entreprise.
L'atelier A fabrique 86.0\,\% des stylos, et parmi ceux-là, 34.0\,\% possèdent un défaut de fabrication.
De plus, 8.0\,\% des stylos possèdent un défaut de fabrication et sortent de l'atelier B.
Un stylo est prélevé au hasard dans le stock de l'entreprise.
On considère les évènements suivants:
\begin{itemize}
\item A : \og Le stylo a été fabriqué par l'atelier A \fg
\item B : \og Le stylo a été fabriqué par l'atelier B \fg
\item D : \og Le stylo possède un défaut de fabrication \fg
\end{itemize}
\end{minipage}
\begin{minipage}{0.4\linewidth}
\begin{center}
\begin{tikzpicture}[sloped]
\node {.}
child {node {$A$}
child {node {$D$}
edge from parent
node[above] {...}
}
child {node {$\overline{D}$}
edge from parent
node[above] {...}
}
edge from parent
node[above] {...}
}
child[missing] {}
child { node {$B$}
child {node {$D$}
edge from parent
node[above] {...}
}
child {node {$\overline{D}$}
edge from parent
node[above] {...}
}
edge from parent
node[above] {...}
} ;
\end{tikzpicture}
\end{center}
\end{minipage}
\medskip
\begin{enumerate}
\item Compléter l'arbre de probabilité ci-contre
\item Interpréter puis donner les probabilités $P(A)$, $P(B)$, $P_A(D)$ et $P(B \cap D)$.
\item
\begin{enumerate}
\item Calculer la probabilité qu'un stylo provienne de l'atelier A et possède un défaut de fabrication.
\item En déduire que la probabilité qu'un stylo possède un défaut de fabrication est de $0.37$.
\end{enumerate}
\item On prélève un stylo au hasard avec un défaut. Quelle est la probabilité qu'il vienne de l'atelier A?
\end{enumerate}
\bigskip
\textbf{Partie B}
\medskip
Dans cette partie, on suppose que 37.0\,\% des stylos possèdent un défaut de fabrication.
L'entreprise confectionne des paquets contenant chacun $4$~stylos.
Le fait qu'un stylo possède ou non un défaut de fabrication est indépendant des autres stylos.
On appelle $X$ la variable aléatoire donnant pour un paquet le nombre de stylos qui possèdent un défaut de fabrication.
On admet que la variable aléatoire $X$ suit une loi binomiale.
\medskip
\begin{enumerate}
\setcounter{enumi}{4}
\item Avec quelle loi peut-on modéliser $X$. Préciser les paramètres.
\item Calculer et interpréter la probabilité $P(X = 10)$.
\item Le directeur de l'entreprise affirme qu'il y a plus d'une chance sur deux qu'un paquet ne comporte aucun stylo défectueux. A-t-il raison ?
\item Combien de stylos peut-on espérer avoir en moyenne?
\end{enumerate}
\pagebreak
\end{exercise}
\begin{solution}
\begin{enumerate}
\item
\begin{center}
\begin{tikzpicture}[sloped]
\node {.}
child {node {$A$}
child {node {$D$}
edge from parent
node[above] {0.34}
}
child {node {$\overline{D}$}
edge from parent
node[above] {0.66}
}
edge from parent
node[above] {0.86}
}
child[missing] {}
child { node {$B$}
child {node {$D$}
edge from parent
node[above] {0.58}
}
child {node {$\overline{D}$}
edge from parent
node[above] {0.42}
}
edge from parent
node[above] {0.14}
} ;
\end{tikzpicture}
\end{center}
\item
\begin{itemize}
\item Probabilité que le stylo vienne de l'atelier A
\[
P(A) = 0.86
\]
\item Probabilité que le stylo vienne de l'atelier B
\[
P(B) = 0.14
\]
\item Probabilité que le stylo ait un défaut sachant qu'il vient de l'atelier A.
\[
P_A(D) = 0.34
\]
\item Probabilité que le stylo vienne de l'atelier B et qu'il ait un défaut.
\[
P(D \cap D) = 0.08
\]
\end{itemize}
\item
\begin{enumerate}
\item Probabilité qu'un stylo vienne de l'atelier A et qu'il ait un defaut
\[
P(A\cap D) = P(A) \times P_A(D) = 0.86 \times 0.34 = 0.29
\]
\item Probabilité que le stylo ai un défaut de fabrication.
\[
P(D) = P(A\cap D) + P(B\cap D) = 0.29 + 0.08 = 0.37
\]
\end{enumerate}
\item Probabilité qu'il vienne de l'atelier A sachant qu'il a un defaut
\[
P_D(A) = \frac{P(A\cap D)}{P(D)} = \frac{0.29}{0.37} = 0.78
\]
\item $X$ peut être modélisée par une loi binomiale de paramètres $n=20$ et $p=0.37$.
\item (\textit{par de correction automatique disponible pour le résultat final}
\[
P(X = 10) = \coefBino{20}{10}\times 0.37^{10} \times 0.63^{10}
\]
\item (\textit{par de correction automatique disponible pour le résultat final}
Il faut calculer la probabilité qu'il y ait 0 stylo avec un defaut.
\[
P(X = 0) = \coefBino{20}{0}\times 0.37^{0} \times 0.63^{20}
\]
Puis comparer ce nombre à 0,5.
\item Il faut calculer l'espérance
\[
E[X] = n\times p = 20 \times 0.37 = 7.4
\]
\end{enumerate}
\end{solution}
\end{document}
%%% Local Variables:
%%% mode: latex
%%% TeX-master: "master"
%%% End:

View File

@ -0,0 +1,367 @@
\documentclass[a4paper,10pt]{article}
\usepackage{myXsim}
% Title Page
\title{DM1 \hfill BALUKHATYY Alexandre}
\tribe{Maths complémentaire}
\date{\hfillÀ render pour le jeudi 27 mai}
\xsimsetup{
solution/print = false
}
\begin{document}
\maketitle
Les valeurs des exercices sont générés automatiquement. Si une valeur a un nombre adhérant de chiffres après la virgule, vous pouvez l'arrondir à l'entier le plus proche.
\begin{exercise}[subtitle={Optimisation de matière}]
\begin{minipage}{0.6\textwidth}
On se propose de fabriquer avec le moins de tôle possible une citerne fermée en forme de parallélépipède rectangle dont le volume intérieur doit être de $15m^3$. La longueur est aussi fixée à $3m$ par le cahier des charges.
On peut donc faire varier uniquement la largeur (notée $x$) et la hauteur (notée $h$) de la cuve.
\end{minipage}
\hfill
\begin{minipage}{0.3\textwidth}
\begin{tikzpicture}
\pgfmathsetmacro{\cubex}{3}
\pgfmathsetmacro{\cubey}{1}
\pgfmathsetmacro{\cubez}{2}
\draw[black,fill=gray] (0,0,0) -- ++(-\cubex,0,0) -- ++(0,-\cubey,0) node [midway, left] {$h$} -- ++(\cubex,0,0) node [midway, below] {$x$} -- cycle;
\draw[black,fill=gray] (0,0,0) -- ++(0,0,-\cubez) -- ++(0,-\cubey,0) -- ++(0,0,\cubez) node [midway, right] {$3m$} -- cycle;
\draw[black,fill=gray] (0,0,0) -- ++(-\cubex,0,0) -- ++(0,0,-\cubez) -- ++(\cubex,0,0) -- cycle;
\end{tikzpicture}
\end{minipage}
\begin{enumerate}
\item Expliquer pourquoi quand la largeur $x$ change, la hauteur $h$ doit elle aussi changer pour respecter les contraintes.
\item Démontrer que l'on doit avoir $h = \dfrac{5}{x}$.
\item On note $S(x)$ l'aire totale de la citerne (c'est à dire la somme des aires des six faces). Montrer que l'on peut écrire
\[
S(x) = 6x + 10 + \frac{30}{x}
\]
\item Démontrer que
\[
S(x) = \frac{6x^2 + 10x + 30}{x}
\]
\item Démontrer que
\[
S'(x) = \frac{6x^2 - 30}{x^2}
\]
\item En déduire le tableau de variation de $S(x)$ sur $\intOF{0}{10}$.
\item Déterminer les valeurs de $x$ et $h$ correspondant à une utilisation minimal de tôle.
\end{enumerate}
\end{exercise}
\begin{solution}
\begin{enumerate}
\item Le volume étant fixe si l'on fait varier $x$, $h$ doit aussi varier.
\begin{itemize}
\item Si $x = 2$ alors conserver un volume de $V=15$, $h$ doit être égale à $5 / 2$
\item Si $x = 3$ alors conserver un volume de $V=15$, $h$ doit être égale à $5 / 3$
\end{itemize}
\item Pour calculer le volume, on a
\begin{eqnarray*}
V &=& h\times x \times 3 \\
15 &=& h\times x \times 3 \\
x &=& \frac{15}{h\times 3} = \frac{5}{h}
\end{eqnarray*}
\item Pour calculer la surface totale, on ajoute la surface de chaque face. On a donc le calcul suivant
\begin{eqnarray*}
S(x) &=& x\times h \times 2 + x\times3\times2 + h\times 3\times 2\\
S(x) &=& x\times \frac{5}{x} \times 2 + x\times3\times2 + \frac{5}{x}\times 3\times 2\\
S(x) &=& 6x + 10 + \frac{30}{x}
\end{eqnarray*}
\item Pour trouver cette nouvelle forme, on met chaque élément sur le même dénominateur
\begin{eqnarray*}
S(x) &=& 6x + 10 + \frac{30}{x}\\
S(x) &=& \frac{6x\times x}{x} + \frac{10\times x}{x} + \frac{30}{x}\\
S(x) &=& \frac{6x^2 + 10x + 30}{x}
\end{eqnarray*}
\item On retrouve la formule $\frac{u}{v}$ à dériver
\[
u(x) = 6x^2 + 10x + 30 \Rightarrow u'(x) = 12x + 10
\]
\[
v(x) = x \Rightarrow v'(x) = 1
\]
Donc au numérateur on obtient
\begin{eqnarray*}
u'(x)\times v(x) - u(x)\times v'(x) &=& (12x + 10)\times x - (6x^2 + 10x + 30)\times 1\\
&=& 6x^2 - 30
\end{eqnarray*}
Donc
\[
S'(x) = \frac{6x^2 - 30}{x^2}
\]
\item Tableau de variations de $S$
\begin{itemize}
\item Valeur interdite: $x^2 = 0 \equiv x = 0$
\item Signe de $6x^2 - 30$: c'est un polynôme du 2e degré
\[
\Delta = 720 > 0
\]
Il y a donc 2 racines
\[
x_1 = - 2.23606797749979 \qquad
x_2 = 2.23606797749979
\]
Et on sait que $6x^2 - 30$ est du signe de $a$ donc positif en dehors des racines
\item Le dénominateur $x^2$ est toujours positif.
\item Tableau de variations
\begin{tikzpicture}[baseline=(a.north)]
\tkzTabInit[lgt=3,espcl=3]{$x$/1,$6x^2 - 30$/1, $x^2$/1, $S'$/1, $S$/2}{$0$, $- 2.23606797749979$, $10$}
\tkzTabLine{d,-, z, +, }
\tkzTabLine{d,+, , +, }
\tkzTabLine{d,-, z, +, }
\tkzTabVar{D+/ , -/ , +/ }
\end{tikzpicture}
\end{itemize}
\item On a donc une surface minimal pour $x=2.23606797749979$ et $h = 11.18033988749895$.
\end{enumerate}
\end{solution}
%%% Local Variables:
%%% mode: latex
%%% TeX-master: "master"
%%% End:
\begin{exercise}[subtitle={Bassin}]
Le tour d'un bassin au niveau du sol présente deux axes de symétrie : laxe des abscisses et la droite déquation $x=4$. Il est obtenu par symétrie de la courbe $\mathcal{C}_f$ sur $\intFF{0}{4}$$f$ est la fonction définie par
\[
f(x) = \left(- x^{2} + 8.4 x - 0.6\right) e^{- x} + 0.6
\]
On admet que sur $\intFF{0}{4}$ la fonction $f$ est positive.
\begin{enumerate}
\item Sur un repère, tracer l'allure de la courbe $\mathcal{C}_f$, les axes de symétries puis compléter pour dessiner la forme du bassin.
\item Montrer que la fonction $f$ admet comme primitive sur $\R$ la fonction $F$ définie par
\[
F(x) = 0.6 x + \left( x^{2} - 6.4 x - 5.8\right) e^{- x}
\]
\item Calculer la quantité $\ds \int_0^4 f(x) \; dx$, vous donnerez le résultat sous forme exacte. Interpréter le résultat et reportez cette quantité sur le graphique.
\item On considère que l'échelle de votre graphique est de 1unité pour 15m. Calculer l'aire du bassin. Vous donnerez un résultat arrondi au $m^2$ près.
\end{enumerate}
\end{exercise}
\begin{solution}
\begin{enumerate}
\item
\begin{tikzpicture}[baseline=(a.north), xscale=1, yscale=0.5]
\tkzInit[xmin=0,xmax=5,xstep=1,
ymin=0,ymax=10,ystep=1]
\tkzGrid
\tkzAxeXY
\tkzFct[domain=0:10,color=red,very thick]%
{ (-x**2 + 8.4*x - 0.6)*exp(-x) + 0.6 };
\end{tikzpicture}
\item Il faut dériver $F(x)$ et vérifier que $F'(x) = f(x)$.
\item $\ds \int_0^4 f(x) \; dx = F(4) - F(0) = 8.2 - \frac{15.4}{e^{4}}$
\item La quantité calculée à la question précédente se retrouve 4fois pour former le bassin. Il faut ensuite prendre en compte l'échelle, comme 1unité de longueur correspond à 15m, une unité d'air correspond à $15\times15 = 225m^2$. Ainsi l'aire du bassin est égale à
\[
(8.2 - \frac{15.4}{e^{4}})\times 4 \times 15^2 = 7126.000000
\]
\end{enumerate}
\end{solution}
%%% Local Variables:
%%% mode: latex
%%% TeX-master: "master"
%%% End:
\begin{exercise}[subtitle={Stylos}]
\emph{Les parties {\rm A} et {\rm B} de cet exercice sont indépendantes.}
\bigskip
\begin{minipage}{0.6\linewidth}
\textbf{Partie A}
\medskip
Deux ateliers A et B fabriquent des stylos pour une entreprise.
L'atelier A fabrique 25.0\,\% des stylos, et parmi ceux-là, 84.0\,\% possèdent un défaut de fabrication.
De plus, 57.99999999999999\,\% des stylos possèdent un défaut de fabrication et sortent de l'atelier B.
Un stylo est prélevé au hasard dans le stock de l'entreprise.
On considère les évènements suivants:
\begin{itemize}
\item A : \og Le stylo a été fabriqué par l'atelier A \fg
\item B : \og Le stylo a été fabriqué par l'atelier B \fg
\item D : \og Le stylo possède un défaut de fabrication \fg
\end{itemize}
\end{minipage}
\begin{minipage}{0.4\linewidth}
\begin{center}
\begin{tikzpicture}[sloped]
\node {.}
child {node {$A$}
child {node {$D$}
edge from parent
node[above] {...}
}
child {node {$\overline{D}$}
edge from parent
node[above] {...}
}
edge from parent
node[above] {...}
}
child[missing] {}
child { node {$B$}
child {node {$D$}
edge from parent
node[above] {...}
}
child {node {$\overline{D}$}
edge from parent
node[above] {...}
}
edge from parent
node[above] {...}
} ;
\end{tikzpicture}
\end{center}
\end{minipage}
\medskip
\begin{enumerate}
\item Compléter l'arbre de probabilité ci-contre
\item Interpréter puis donner les probabilités $P(A)$, $P(B)$, $P_A(D)$ et $P(B \cap D)$.
\item
\begin{enumerate}
\item Calculer la probabilité qu'un stylo provienne de l'atelier A et possède un défaut de fabrication.
\item En déduire que la probabilité qu'un stylo possède un défaut de fabrication est de $0.79$.
\end{enumerate}
\item On prélève un stylo au hasard avec un défaut. Quelle est la probabilité qu'il vienne de l'atelier A?
\end{enumerate}
\bigskip
\textbf{Partie B}
\medskip
Dans cette partie, on suppose que 79.0\,\% des stylos possèdent un défaut de fabrication.
L'entreprise confectionne des paquets contenant chacun $4$~stylos.
Le fait qu'un stylo possède ou non un défaut de fabrication est indépendant des autres stylos.
On appelle $X$ la variable aléatoire donnant pour un paquet le nombre de stylos qui possèdent un défaut de fabrication.
On admet que la variable aléatoire $X$ suit une loi binomiale.
\medskip
\begin{enumerate}
\setcounter{enumi}{4}
\item Avec quelle loi peut-on modéliser $X$. Préciser les paramètres.
\item Calculer et interpréter la probabilité $P(X = 11)$.
\item Le directeur de l'entreprise affirme qu'il y a plus d'une chance sur deux qu'un paquet ne comporte aucun stylo défectueux. A-t-il raison ?
\item Combien de stylos peut-on espérer avoir en moyenne?
\end{enumerate}
\pagebreak
\end{exercise}
\begin{solution}
\begin{enumerate}
\item
\begin{center}
\begin{tikzpicture}[sloped]
\node {.}
child {node {$A$}
child {node {$D$}
edge from parent
node[above] {0.84}
}
child {node {$\overline{D}$}
edge from parent
node[above] {0.16}
}
edge from parent
node[above] {0.25}
}
child[missing] {}
child { node {$B$}
child {node {$D$}
edge from parent
node[above] {0.78}
}
child {node {$\overline{D}$}
edge from parent
node[above] {0.22}
}
edge from parent
node[above] {0.75}
} ;
\end{tikzpicture}
\end{center}
\item
\begin{itemize}
\item Probabilité que le stylo vienne de l'atelier A
\[
P(A) = 0.25
\]
\item Probabilité que le stylo vienne de l'atelier B
\[
P(B) = 0.75
\]
\item Probabilité que le stylo ait un défaut sachant qu'il vient de l'atelier A.
\[
P_A(D) = 0.84
\]
\item Probabilité que le stylo vienne de l'atelier B et qu'il ait un défaut.
\[
P(D \cap D) = 0.58
\]
\end{itemize}
\item
\begin{enumerate}
\item Probabilité qu'un stylo vienne de l'atelier A et qu'il ait un defaut
\[
P(A\cap D) = P(A) \times P_A(D) = 0.25 \times 0.84 = 0.21
\]
\item Probabilité que le stylo ai un défaut de fabrication.
\[
P(D) = P(A\cap D) + P(B\cap D) = 0.21 + 0.58 = 0.79
\]
\end{enumerate}
\item Probabilité qu'il vienne de l'atelier A sachant qu'il a un defaut
\[
P_D(A) = \frac{P(A\cap D)}{P(D)} = \frac{0.21}{0.79} = 0.27
\]
\item $X$ peut être modélisée par une loi binomiale de paramètres $n=12$ et $p=0.79$.
\item (\textit{par de correction automatique disponible pour le résultat final}
\[
P(X = 11) = \coefBino{12}{11}\times 0.79^{11} \times 0.21^{1}
\]
\item (\textit{par de correction automatique disponible pour le résultat final}
Il faut calculer la probabilité qu'il y ait 0 stylo avec un defaut.
\[
P(X = 0) = \coefBino{12}{0}\times 0.79^{0} \times 0.21^{12}
\]
Puis comparer ce nombre à 0,5.
\item Il faut calculer l'espérance
\[
E[X] = n\times p = 12 \times 0.79 = 9.48
\]
\end{enumerate}
\end{solution}
\end{document}
%%% Local Variables:
%%% mode: latex
%%% TeX-master: "master"
%%% End:

View File

@ -0,0 +1,367 @@
\documentclass[a4paper,10pt]{article}
\usepackage{myXsim}
% Title Page
\title{DM1 \hfill CALES Mathis}
\tribe{Maths complémentaire}
\date{\hfillÀ render pour le jeudi 27 mai}
\xsimsetup{
solution/print = false
}
\begin{document}
\maketitle
Les valeurs des exercices sont générés automatiquement. Si une valeur a un nombre adhérant de chiffres après la virgule, vous pouvez l'arrondir à l'entier le plus proche.
\begin{exercise}[subtitle={Optimisation de matière}]
\begin{minipage}{0.6\textwidth}
On se propose de fabriquer avec le moins de tôle possible une citerne fermée en forme de parallélépipède rectangle dont le volume intérieur doit être de $6m^3$. La longueur est aussi fixée à $2m$ par le cahier des charges.
On peut donc faire varier uniquement la largeur (notée $x$) et la hauteur (notée $h$) de la cuve.
\end{minipage}
\hfill
\begin{minipage}{0.3\textwidth}
\begin{tikzpicture}
\pgfmathsetmacro{\cubex}{3}
\pgfmathsetmacro{\cubey}{1}
\pgfmathsetmacro{\cubez}{2}
\draw[black,fill=gray] (0,0,0) -- ++(-\cubex,0,0) -- ++(0,-\cubey,0) node [midway, left] {$h$} -- ++(\cubex,0,0) node [midway, below] {$x$} -- cycle;
\draw[black,fill=gray] (0,0,0) -- ++(0,0,-\cubez) -- ++(0,-\cubey,0) -- ++(0,0,\cubez) node [midway, right] {$2m$} -- cycle;
\draw[black,fill=gray] (0,0,0) -- ++(-\cubex,0,0) -- ++(0,0,-\cubez) -- ++(\cubex,0,0) -- cycle;
\end{tikzpicture}
\end{minipage}
\begin{enumerate}
\item Expliquer pourquoi quand la largeur $x$ change, la hauteur $h$ doit elle aussi changer pour respecter les contraintes.
\item Démontrer que l'on doit avoir $h = \dfrac{3}{x}$.
\item On note $S(x)$ l'aire totale de la citerne (c'est à dire la somme des aires des six faces). Montrer que l'on peut écrire
\[
S(x) = 4x + 6 + \frac{12}{x}
\]
\item Démontrer que
\[
S(x) = \frac{4x^2 + 6x + 12}{x}
\]
\item Démontrer que
\[
S'(x) = \frac{4x^2 - 12}{x^2}
\]
\item En déduire le tableau de variation de $S(x)$ sur $\intOF{0}{10}$.
\item Déterminer les valeurs de $x$ et $h$ correspondant à une utilisation minimal de tôle.
\end{enumerate}
\end{exercise}
\begin{solution}
\begin{enumerate}
\item Le volume étant fixe si l'on fait varier $x$, $h$ doit aussi varier.
\begin{itemize}
\item Si $x = 2$ alors conserver un volume de $V=6$, $h$ doit être égale à $3 / 2$
\item Si $x = 3$ alors conserver un volume de $V=6$, $h$ doit être égale à $3 / 3$
\end{itemize}
\item Pour calculer le volume, on a
\begin{eqnarray*}
V &=& h\times x \times 2 \\
6 &=& h\times x \times 2 \\
x &=& \frac{6}{h\times 2} = \frac{3}{h}
\end{eqnarray*}
\item Pour calculer la surface totale, on ajoute la surface de chaque face. On a donc le calcul suivant
\begin{eqnarray*}
S(x) &=& x\times h \times 2 + x\times2\times2 + h\times 2\times 2\\
S(x) &=& x\times \frac{3}{x} \times 2 + x\times2\times2 + \frac{3}{x}\times 2\times 2\\
S(x) &=& 4x + 6 + \frac{12}{x}
\end{eqnarray*}
\item Pour trouver cette nouvelle forme, on met chaque élément sur le même dénominateur
\begin{eqnarray*}
S(x) &=& 4x + 6 + \frac{12}{x}\\
S(x) &=& \frac{4x\times x}{x} + \frac{6\times x}{x} + \frac{12}{x}\\
S(x) &=& \frac{4x^2 + 6x + 12}{x}
\end{eqnarray*}
\item On retrouve la formule $\frac{u}{v}$ à dériver
\[
u(x) = 4x^2 + 6x + 12 \Rightarrow u'(x) = 8x + 6
\]
\[
v(x) = x \Rightarrow v'(x) = 1
\]
Donc au numérateur on obtient
\begin{eqnarray*}
u'(x)\times v(x) - u(x)\times v'(x) &=& (8x + 6)\times x - (4x^2 + 6x + 12)\times 1\\
&=& 4x^2 - 12
\end{eqnarray*}
Donc
\[
S'(x) = \frac{4x^2 - 12}{x^2}
\]
\item Tableau de variations de $S$
\begin{itemize}
\item Valeur interdite: $x^2 = 0 \equiv x = 0$
\item Signe de $4x^2 - 12$: c'est un polynôme du 2e degré
\[
\Delta = 192 > 0
\]
Il y a donc 2 racines
\[
x_1 = - 1.7320508075688772 \qquad
x_2 = 1.7320508075688772
\]
Et on sait que $4x^2 - 12$ est du signe de $a$ donc positif en dehors des racines
\item Le dénominateur $x^2$ est toujours positif.
\item Tableau de variations
\begin{tikzpicture}[baseline=(a.north)]
\tkzTabInit[lgt=3,espcl=3]{$x$/1,$4x^2 - 12$/1, $x^2$/1, $S'$/1, $S$/2}{$0$, $- 1.7320508075688772$, $10$}
\tkzTabLine{d,-, z, +, }
\tkzTabLine{d,+, , +, }
\tkzTabLine{d,-, z, +, }
\tkzTabVar{D+/ , -/ , +/ }
\end{tikzpicture}
\end{itemize}
\item On a donc une surface minimal pour $x=1.7320508075688772$ et $h = 5.1961524227066316$.
\end{enumerate}
\end{solution}
%%% Local Variables:
%%% mode: latex
%%% TeX-master: "master"
%%% End:
\begin{exercise}[subtitle={Bassin}]
Le tour d'un bassin au niveau du sol présente deux axes de symétrie : laxe des abscisses et la droite déquation $x=4$. Il est obtenu par symétrie de la courbe $\mathcal{C}_f$ sur $\intFF{0}{4}$$f$ est la fonction définie par
\[
f(x) = \left(- x^{2} + 2.8 x - 8.7\right) e^{- x} + 8.7
\]
On admet que sur $\intFF{0}{4}$ la fonction $f$ est positive.
\begin{enumerate}
\item Sur un repère, tracer l'allure de la courbe $\mathcal{C}_f$, les axes de symétries puis compléter pour dessiner la forme du bassin.
\item Montrer que la fonction $f$ admet comme primitive sur $\R$ la fonction $F$ définie par
\[
F(x) = 8.7 x + \left( x^{2} - 0.8 x + 7.9\right) e^{- x}
\]
\item Calculer la quantité $\ds \int_0^4 f(x) \; dx$, vous donnerez le résultat sous forme exacte. Interpréter le résultat et reportez cette quantité sur le graphique.
\item On considère que l'échelle de votre graphique est de 1unité pour 15m. Calculer l'aire du bassin. Vous donnerez un résultat arrondi au $m^2$ près.
\end{enumerate}
\end{exercise}
\begin{solution}
\begin{enumerate}
\item
\begin{tikzpicture}[baseline=(a.north), xscale=1, yscale=0.5]
\tkzInit[xmin=0,xmax=5,xstep=1,
ymin=0,ymax=10,ystep=1]
\tkzGrid
\tkzAxeXY
\tkzFct[domain=0:10,color=red,very thick]%
{ (-x**2 + 2.8*x - 8.7)*exp(-x) + 8.7 };
\end{tikzpicture}
\item Il faut dériver $F(x)$ et vérifier que $F'(x) = f(x)$.
\item $\ds \int_0^4 f(x) \; dx = F(4) - F(0) = \frac{20.7}{e^{4}} + 26.9$
\item La quantité calculée à la question précédente se retrouve 4fois pour former le bassin. Il faut ensuite prendre en compte l'échelle, comme 1unité de longueur correspond à 15m, une unité d'air correspond à $15\times15 = 225m^2$. Ainsi l'aire du bassin est égale à
\[
(\frac{20.7}{e^{4}} + 26.9)\times 4 \times 15^2 = 24551.00000
\]
\end{enumerate}
\end{solution}
%%% Local Variables:
%%% mode: latex
%%% TeX-master: "master"
%%% End:
\begin{exercise}[subtitle={Stylos}]
\emph{Les parties {\rm A} et {\rm B} de cet exercice sont indépendantes.}
\bigskip
\begin{minipage}{0.6\linewidth}
\textbf{Partie A}
\medskip
Deux ateliers A et B fabriquent des stylos pour une entreprise.
L'atelier A fabrique 48.0\,\% des stylos, et parmi ceux-là, 99.0\,\% possèdent un défaut de fabrication.
De plus, 10.0\,\% des stylos possèdent un défaut de fabrication et sortent de l'atelier B.
Un stylo est prélevé au hasard dans le stock de l'entreprise.
On considère les évènements suivants:
\begin{itemize}
\item A : \og Le stylo a été fabriqué par l'atelier A \fg
\item B : \og Le stylo a été fabriqué par l'atelier B \fg
\item D : \og Le stylo possède un défaut de fabrication \fg
\end{itemize}
\end{minipage}
\begin{minipage}{0.4\linewidth}
\begin{center}
\begin{tikzpicture}[sloped]
\node {.}
child {node {$A$}
child {node {$D$}
edge from parent
node[above] {...}
}
child {node {$\overline{D}$}
edge from parent
node[above] {...}
}
edge from parent
node[above] {...}
}
child[missing] {}
child { node {$B$}
child {node {$D$}
edge from parent
node[above] {...}
}
child {node {$\overline{D}$}
edge from parent
node[above] {...}
}
edge from parent
node[above] {...}
} ;
\end{tikzpicture}
\end{center}
\end{minipage}
\medskip
\begin{enumerate}
\item Compléter l'arbre de probabilité ci-contre
\item Interpréter puis donner les probabilités $P(A)$, $P(B)$, $P_A(D)$ et $P(B \cap D)$.
\item
\begin{enumerate}
\item Calculer la probabilité qu'un stylo provienne de l'atelier A et possède un défaut de fabrication.
\item En déduire que la probabilité qu'un stylo possède un défaut de fabrication est de $0.58$.
\end{enumerate}
\item On prélève un stylo au hasard avec un défaut. Quelle est la probabilité qu'il vienne de l'atelier A?
\end{enumerate}
\bigskip
\textbf{Partie B}
\medskip
Dans cette partie, on suppose que 57.99999999999999\,\% des stylos possèdent un défaut de fabrication.
L'entreprise confectionne des paquets contenant chacun $4$~stylos.
Le fait qu'un stylo possède ou non un défaut de fabrication est indépendant des autres stylos.
On appelle $X$ la variable aléatoire donnant pour un paquet le nombre de stylos qui possèdent un défaut de fabrication.
On admet que la variable aléatoire $X$ suit une loi binomiale.
\medskip
\begin{enumerate}
\setcounter{enumi}{4}
\item Avec quelle loi peut-on modéliser $X$. Préciser les paramètres.
\item Calculer et interpréter la probabilité $P(X = 10)$.
\item Le directeur de l'entreprise affirme qu'il y a plus d'une chance sur deux qu'un paquet ne comporte aucun stylo défectueux. A-t-il raison ?
\item Combien de stylos peut-on espérer avoir en moyenne?
\end{enumerate}
\pagebreak
\end{exercise}
\begin{solution}
\begin{enumerate}
\item
\begin{center}
\begin{tikzpicture}[sloped]
\node {.}
child {node {$A$}
child {node {$D$}
edge from parent
node[above] {0.99}
}
child {node {$\overline{D}$}
edge from parent
node[above] {0.01}
}
edge from parent
node[above] {0.48}
}
child[missing] {}
child { node {$B$}
child {node {$D$}
edge from parent
node[above] {0.2}
}
child {node {$\overline{D}$}
edge from parent
node[above] {0.8}
}
edge from parent
node[above] {0.52}
} ;
\end{tikzpicture}
\end{center}
\item
\begin{itemize}
\item Probabilité que le stylo vienne de l'atelier A
\[
P(A) = 0.48
\]
\item Probabilité que le stylo vienne de l'atelier B
\[
P(B) = 0.52
\]
\item Probabilité que le stylo ait un défaut sachant qu'il vient de l'atelier A.
\[
P_A(D) = 0.99
\]
\item Probabilité que le stylo vienne de l'atelier B et qu'il ait un défaut.
\[
P(D \cap D) = 0.1
\]
\end{itemize}
\item
\begin{enumerate}
\item Probabilité qu'un stylo vienne de l'atelier A et qu'il ait un defaut
\[
P(A\cap D) = P(A) \times P_A(D) = 0.48 \times 0.99 = 0.48
\]
\item Probabilité que le stylo ai un défaut de fabrication.
\[
P(D) = P(A\cap D) + P(B\cap D) = 0.48 + 0.1 = 0.58
\]
\end{enumerate}
\item Probabilité qu'il vienne de l'atelier A sachant qu'il a un defaut
\[
P_D(A) = \frac{P(A\cap D)}{P(D)} = \frac{0.48}{0.58} = 0.83
\]
\item $X$ peut être modélisée par une loi binomiale de paramètres $n=17$ et $p=0.58$.
\item (\textit{par de correction automatique disponible pour le résultat final}
\[
P(X = 10) = \coefBino{17}{10}\times 0.58^{10} \times 0.42^{7}
\]
\item (\textit{par de correction automatique disponible pour le résultat final}
Il faut calculer la probabilité qu'il y ait 0 stylo avec un defaut.
\[
P(X = 0) = \coefBino{17}{0}\times 0.58^{0} \times 0.42^{17}
\]
Puis comparer ce nombre à 0,5.
\item Il faut calculer l'espérance
\[
E[X] = n\times p = 17 \times 0.58 = 9.86
\]
\end{enumerate}
\end{solution}
\end{document}
%%% Local Variables:
%%% mode: latex
%%% TeX-master: "master"
%%% End:

View File

@ -0,0 +1,367 @@
\documentclass[a4paper,10pt]{article}
\usepackage{myXsim}
% Title Page
\title{DM1 \hfill CHAKIR Iman}
\tribe{Maths complémentaire}
\date{\hfillÀ render pour le jeudi 27 mai}
\xsimsetup{
solution/print = false
}
\begin{document}
\maketitle
Les valeurs des exercices sont générés automatiquement. Si une valeur a un nombre adhérant de chiffres après la virgule, vous pouvez l'arrondir à l'entier le plus proche.
\begin{exercise}[subtitle={Optimisation de matière}]
\begin{minipage}{0.6\textwidth}
On se propose de fabriquer avec le moins de tôle possible une citerne fermée en forme de parallélépipède rectangle dont le volume intérieur doit être de $20m^3$. La longueur est aussi fixée à $4m$ par le cahier des charges.
On peut donc faire varier uniquement la largeur (notée $x$) et la hauteur (notée $h$) de la cuve.
\end{minipage}
\hfill
\begin{minipage}{0.3\textwidth}
\begin{tikzpicture}
\pgfmathsetmacro{\cubex}{3}
\pgfmathsetmacro{\cubey}{1}
\pgfmathsetmacro{\cubez}{2}
\draw[black,fill=gray] (0,0,0) -- ++(-\cubex,0,0) -- ++(0,-\cubey,0) node [midway, left] {$h$} -- ++(\cubex,0,0) node [midway, below] {$x$} -- cycle;
\draw[black,fill=gray] (0,0,0) -- ++(0,0,-\cubez) -- ++(0,-\cubey,0) -- ++(0,0,\cubez) node [midway, right] {$4m$} -- cycle;
\draw[black,fill=gray] (0,0,0) -- ++(-\cubex,0,0) -- ++(0,0,-\cubez) -- ++(\cubex,0,0) -- cycle;
\end{tikzpicture}
\end{minipage}
\begin{enumerate}
\item Expliquer pourquoi quand la largeur $x$ change, la hauteur $h$ doit elle aussi changer pour respecter les contraintes.
\item Démontrer que l'on doit avoir $h = \dfrac{5}{x}$.
\item On note $S(x)$ l'aire totale de la citerne (c'est à dire la somme des aires des six faces). Montrer que l'on peut écrire
\[
S(x) = 8x + 10 + \frac{40}{x}
\]
\item Démontrer que
\[
S(x) = \frac{8x^2 + 10x + 40}{x}
\]
\item Démontrer que
\[
S'(x) = \frac{8x^2 - 40}{x^2}
\]
\item En déduire le tableau de variation de $S(x)$ sur $\intOF{0}{10}$.
\item Déterminer les valeurs de $x$ et $h$ correspondant à une utilisation minimal de tôle.
\end{enumerate}
\end{exercise}
\begin{solution}
\begin{enumerate}
\item Le volume étant fixe si l'on fait varier $x$, $h$ doit aussi varier.
\begin{itemize}
\item Si $x = 2$ alors conserver un volume de $V=20$, $h$ doit être égale à $5 / 2$
\item Si $x = 3$ alors conserver un volume de $V=20$, $h$ doit être égale à $5 / 3$
\end{itemize}
\item Pour calculer le volume, on a
\begin{eqnarray*}
V &=& h\times x \times 4 \\
20 &=& h\times x \times 4 \\
x &=& \frac{20}{h\times 4} = \frac{5}{h}
\end{eqnarray*}
\item Pour calculer la surface totale, on ajoute la surface de chaque face. On a donc le calcul suivant
\begin{eqnarray*}
S(x) &=& x\times h \times 2 + x\times4\times2 + h\times 4\times 2\\
S(x) &=& x\times \frac{5}{x} \times 2 + x\times4\times2 + \frac{5}{x}\times 4\times 2\\
S(x) &=& 8x + 10 + \frac{40}{x}
\end{eqnarray*}
\item Pour trouver cette nouvelle forme, on met chaque élément sur le même dénominateur
\begin{eqnarray*}
S(x) &=& 8x + 10 + \frac{40}{x}\\
S(x) &=& \frac{8x\times x}{x} + \frac{10\times x}{x} + \frac{40}{x}\\
S(x) &=& \frac{8x^2 + 10x + 40}{x}
\end{eqnarray*}
\item On retrouve la formule $\frac{u}{v}$ à dériver
\[
u(x) = 8x^2 + 10x + 40 \Rightarrow u'(x) = 16x + 10
\]
\[
v(x) = x \Rightarrow v'(x) = 1
\]
Donc au numérateur on obtient
\begin{eqnarray*}
u'(x)\times v(x) - u(x)\times v'(x) &=& (16x + 10)\times x - (8x^2 + 10x + 40)\times 1\\
&=& 8x^2 - 40
\end{eqnarray*}
Donc
\[
S'(x) = \frac{8x^2 - 40}{x^2}
\]
\item Tableau de variations de $S$
\begin{itemize}
\item Valeur interdite: $x^2 = 0 \equiv x = 0$
\item Signe de $8x^2 - 40$: c'est un polynôme du 2e degré
\[
\Delta = 1280 > 0
\]
Il y a donc 2 racines
\[
x_1 = - 2.23606797749979 \qquad
x_2 = 2.23606797749979
\]
Et on sait que $8x^2 - 40$ est du signe de $a$ donc positif en dehors des racines
\item Le dénominateur $x^2$ est toujours positif.
\item Tableau de variations
\begin{tikzpicture}[baseline=(a.north)]
\tkzTabInit[lgt=3,espcl=3]{$x$/1,$8x^2 - 40$/1, $x^2$/1, $S'$/1, $S$/2}{$0$, $- 2.23606797749979$, $10$}
\tkzTabLine{d,-, z, +, }
\tkzTabLine{d,+, , +, }
\tkzTabLine{d,-, z, +, }
\tkzTabVar{D+/ , -/ , +/ }
\end{tikzpicture}
\end{itemize}
\item On a donc une surface minimal pour $x=2.23606797749979$ et $h = 11.18033988749895$.
\end{enumerate}
\end{solution}
%%% Local Variables:
%%% mode: latex
%%% TeX-master: "master"
%%% End:
\begin{exercise}[subtitle={Bassin}]
Le tour d'un bassin au niveau du sol présente deux axes de symétrie : laxe des abscisses et la droite déquation $x=4$. Il est obtenu par symétrie de la courbe $\mathcal{C}_f$ sur $\intFF{0}{4}$$f$ est la fonction définie par
\[
f(x) = \left(- x^{2} + 9.0 x - 9.0\right) e^{- x} + 9.0
\]
On admet que sur $\intFF{0}{4}$ la fonction $f$ est positive.
\begin{enumerate}
\item Sur un repère, tracer l'allure de la courbe $\mathcal{C}_f$, les axes de symétries puis compléter pour dessiner la forme du bassin.
\item Montrer que la fonction $f$ admet comme primitive sur $\R$ la fonction $F$ définie par
\[
F(x) = 9.0 x + \left( x^{2} - 7.0 x + 2.0\right) e^{- x}
\]
\item Calculer la quantité $\ds \int_0^4 f(x) \; dx$, vous donnerez le résultat sous forme exacte. Interpréter le résultat et reportez cette quantité sur le graphique.
\item On considère que l'échelle de votre graphique est de 1unité pour 15m. Calculer l'aire du bassin. Vous donnerez un résultat arrondi au $m^2$ près.
\end{enumerate}
\end{exercise}
\begin{solution}
\begin{enumerate}
\item
\begin{tikzpicture}[baseline=(a.north), xscale=1, yscale=0.5]
\tkzInit[xmin=0,xmax=5,xstep=1,
ymin=0,ymax=10,ystep=1]
\tkzGrid
\tkzAxeXY
\tkzFct[domain=0:10,color=red,very thick]%
{ (-x**2 + 9.0*x - 9.0)*exp(-x) + 9.0 };
\end{tikzpicture}
\item Il faut dériver $F(x)$ et vérifier que $F'(x) = f(x)$.
\item $\ds \int_0^4 f(x) \; dx = F(4) - F(0) = 34.0 - \frac{10.0}{e^{4}}$
\item La quantité calculée à la question précédente se retrouve 4fois pour former le bassin. Il faut ensuite prendre en compte l'échelle, comme 1unité de longueur correspond à 15m, une unité d'air correspond à $15\times15 = 225m^2$. Ainsi l'aire du bassin est égale à
\[
(34.0 - \frac{10.0}{e^{4}})\times 4 \times 15^2 = 30435.00000
\]
\end{enumerate}
\end{solution}
%%% Local Variables:
%%% mode: latex
%%% TeX-master: "master"
%%% End:
\begin{exercise}[subtitle={Stylos}]
\emph{Les parties {\rm A} et {\rm B} de cet exercice sont indépendantes.}
\bigskip
\begin{minipage}{0.6\linewidth}
\textbf{Partie A}
\medskip
Deux ateliers A et B fabriquent des stylos pour une entreprise.
L'atelier A fabrique 27.0\,\% des stylos, et parmi ceux-là, 39.0\,\% possèdent un défaut de fabrication.
De plus, 4.0\,\% des stylos possèdent un défaut de fabrication et sortent de l'atelier B.
Un stylo est prélevé au hasard dans le stock de l'entreprise.
On considère les évènements suivants:
\begin{itemize}
\item A : \og Le stylo a été fabriqué par l'atelier A \fg
\item B : \og Le stylo a été fabriqué par l'atelier B \fg
\item D : \og Le stylo possède un défaut de fabrication \fg
\end{itemize}
\end{minipage}
\begin{minipage}{0.4\linewidth}
\begin{center}
\begin{tikzpicture}[sloped]
\node {.}
child {node {$A$}
child {node {$D$}
edge from parent
node[above] {...}
}
child {node {$\overline{D}$}
edge from parent
node[above] {...}
}
edge from parent
node[above] {...}
}
child[missing] {}
child { node {$B$}
child {node {$D$}
edge from parent
node[above] {...}
}
child {node {$\overline{D}$}
edge from parent
node[above] {...}
}
edge from parent
node[above] {...}
} ;
\end{tikzpicture}
\end{center}
\end{minipage}
\medskip
\begin{enumerate}
\item Compléter l'arbre de probabilité ci-contre
\item Interpréter puis donner les probabilités $P(A)$, $P(B)$, $P_A(D)$ et $P(B \cap D)$.
\item
\begin{enumerate}
\item Calculer la probabilité qu'un stylo provienne de l'atelier A et possède un défaut de fabrication.
\item En déduire que la probabilité qu'un stylo possède un défaut de fabrication est de $0.15$.
\end{enumerate}
\item On prélève un stylo au hasard avec un défaut. Quelle est la probabilité qu'il vienne de l'atelier A?
\end{enumerate}
\bigskip
\textbf{Partie B}
\medskip
Dans cette partie, on suppose que 15.0\,\% des stylos possèdent un défaut de fabrication.
L'entreprise confectionne des paquets contenant chacun $4$~stylos.
Le fait qu'un stylo possède ou non un défaut de fabrication est indépendant des autres stylos.
On appelle $X$ la variable aléatoire donnant pour un paquet le nombre de stylos qui possèdent un défaut de fabrication.
On admet que la variable aléatoire $X$ suit une loi binomiale.
\medskip
\begin{enumerate}
\setcounter{enumi}{4}
\item Avec quelle loi peut-on modéliser $X$. Préciser les paramètres.
\item Calculer et interpréter la probabilité $P(X = 9)$.
\item Le directeur de l'entreprise affirme qu'il y a plus d'une chance sur deux qu'un paquet ne comporte aucun stylo défectueux. A-t-il raison ?
\item Combien de stylos peut-on espérer avoir en moyenne?
\end{enumerate}
\pagebreak
\end{exercise}
\begin{solution}
\begin{enumerate}
\item
\begin{center}
\begin{tikzpicture}[sloped]
\node {.}
child {node {$A$}
child {node {$D$}
edge from parent
node[above] {0.39}
}
child {node {$\overline{D}$}
edge from parent
node[above] {0.61}
}
edge from parent
node[above] {0.27}
}
child[missing] {}
child { node {$B$}
child {node {$D$}
edge from parent
node[above] {0.05}
}
child {node {$\overline{D}$}
edge from parent
node[above] {0.95}
}
edge from parent
node[above] {0.73}
} ;
\end{tikzpicture}
\end{center}
\item
\begin{itemize}
\item Probabilité que le stylo vienne de l'atelier A
\[
P(A) = 0.27
\]
\item Probabilité que le stylo vienne de l'atelier B
\[
P(B) = 0.73
\]
\item Probabilité que le stylo ait un défaut sachant qu'il vient de l'atelier A.
\[
P_A(D) = 0.39
\]
\item Probabilité que le stylo vienne de l'atelier B et qu'il ait un défaut.
\[
P(D \cap D) = 0.04
\]
\end{itemize}
\item
\begin{enumerate}
\item Probabilité qu'un stylo vienne de l'atelier A et qu'il ait un defaut
\[
P(A\cap D) = P(A) \times P_A(D) = 0.27 \times 0.39 = 0.11
\]
\item Probabilité que le stylo ai un défaut de fabrication.
\[
P(D) = P(A\cap D) + P(B\cap D) = 0.11 + 0.04 = 0.15
\]
\end{enumerate}
\item Probabilité qu'il vienne de l'atelier A sachant qu'il a un defaut
\[
P_D(A) = \frac{P(A\cap D)}{P(D)} = \frac{0.11}{0.15} = 0.73
\]
\item $X$ peut être modélisée par une loi binomiale de paramètres $n=17$ et $p=0.15$.
\item (\textit{par de correction automatique disponible pour le résultat final}
\[
P(X = 9) = \coefBino{17}{9}\times 0.15^{9} \times 0.85^{8}
\]
\item (\textit{par de correction automatique disponible pour le résultat final}
Il faut calculer la probabilité qu'il y ait 0 stylo avec un defaut.
\[
P(X = 0) = \coefBino{17}{0}\times 0.15^{0} \times 0.85^{17}
\]
Puis comparer ce nombre à 0,5.
\item Il faut calculer l'espérance
\[
E[X] = n\times p = 17 \times 0.15 = 2.55
\]
\end{enumerate}
\end{solution}
\end{document}
%%% Local Variables:
%%% mode: latex
%%% TeX-master: "master"
%%% End:

View File

@ -0,0 +1,367 @@
\documentclass[a4paper,10pt]{article}
\usepackage{myXsim}
% Title Page
\title{DM1 \hfill GERMAIN Margot}
\tribe{Maths complémentaire}
\date{\hfillÀ render pour le jeudi 27 mai}
\xsimsetup{
solution/print = false
}
\begin{document}
\maketitle
Les valeurs des exercices sont générés automatiquement. Si une valeur a un nombre adhérant de chiffres après la virgule, vous pouvez l'arrondir à l'entier le plus proche.
\begin{exercise}[subtitle={Optimisation de matière}]
\begin{minipage}{0.6\textwidth}
On se propose de fabriquer avec le moins de tôle possible une citerne fermée en forme de parallélépipède rectangle dont le volume intérieur doit être de $35m^3$. La longueur est aussi fixée à $5m$ par le cahier des charges.
On peut donc faire varier uniquement la largeur (notée $x$) et la hauteur (notée $h$) de la cuve.
\end{minipage}
\hfill
\begin{minipage}{0.3\textwidth}
\begin{tikzpicture}
\pgfmathsetmacro{\cubex}{3}
\pgfmathsetmacro{\cubey}{1}
\pgfmathsetmacro{\cubez}{2}
\draw[black,fill=gray] (0,0,0) -- ++(-\cubex,0,0) -- ++(0,-\cubey,0) node [midway, left] {$h$} -- ++(\cubex,0,0) node [midway, below] {$x$} -- cycle;
\draw[black,fill=gray] (0,0,0) -- ++(0,0,-\cubez) -- ++(0,-\cubey,0) -- ++(0,0,\cubez) node [midway, right] {$5m$} -- cycle;
\draw[black,fill=gray] (0,0,0) -- ++(-\cubex,0,0) -- ++(0,0,-\cubez) -- ++(\cubex,0,0) -- cycle;
\end{tikzpicture}
\end{minipage}
\begin{enumerate}
\item Expliquer pourquoi quand la largeur $x$ change, la hauteur $h$ doit elle aussi changer pour respecter les contraintes.
\item Démontrer que l'on doit avoir $h = \dfrac{7}{x}$.
\item On note $S(x)$ l'aire totale de la citerne (c'est à dire la somme des aires des six faces). Montrer que l'on peut écrire
\[
S(x) = 10x + 14 + \frac{70}{x}
\]
\item Démontrer que
\[
S(x) = \frac{10x^2 + 14x + 70}{x}
\]
\item Démontrer que
\[
S'(x) = \frac{10x^2 - 70}{x^2}
\]
\item En déduire le tableau de variation de $S(x)$ sur $\intOF{0}{10}$.
\item Déterminer les valeurs de $x$ et $h$ correspondant à une utilisation minimal de tôle.
\end{enumerate}
\end{exercise}
\begin{solution}
\begin{enumerate}
\item Le volume étant fixe si l'on fait varier $x$, $h$ doit aussi varier.
\begin{itemize}
\item Si $x = 2$ alors conserver un volume de $V=35$, $h$ doit être égale à $7 / 2$
\item Si $x = 3$ alors conserver un volume de $V=35$, $h$ doit être égale à $7 / 3$
\end{itemize}
\item Pour calculer le volume, on a
\begin{eqnarray*}
V &=& h\times x \times 5 \\
35 &=& h\times x \times 5 \\
x &=& \frac{35}{h\times 5} = \frac{7}{h}
\end{eqnarray*}
\item Pour calculer la surface totale, on ajoute la surface de chaque face. On a donc le calcul suivant
\begin{eqnarray*}
S(x) &=& x\times h \times 2 + x\times5\times2 + h\times 5\times 2\\
S(x) &=& x\times \frac{7}{x} \times 2 + x\times5\times2 + \frac{7}{x}\times 5\times 2\\
S(x) &=& 10x + 14 + \frac{70}{x}
\end{eqnarray*}
\item Pour trouver cette nouvelle forme, on met chaque élément sur le même dénominateur
\begin{eqnarray*}
S(x) &=& 10x + 14 + \frac{70}{x}\\
S(x) &=& \frac{10x\times x}{x} + \frac{14\times x}{x} + \frac{70}{x}\\
S(x) &=& \frac{10x^2 + 14x + 70}{x}
\end{eqnarray*}
\item On retrouve la formule $\frac{u}{v}$ à dériver
\[
u(x) = 10x^2 + 14x + 70 \Rightarrow u'(x) = 20x + 14
\]
\[
v(x) = x \Rightarrow v'(x) = 1
\]
Donc au numérateur on obtient
\begin{eqnarray*}
u'(x)\times v(x) - u(x)\times v'(x) &=& (20x + 14)\times x - (10x^2 + 14x + 70)\times 1\\
&=& 10x^2 - 70
\end{eqnarray*}
Donc
\[
S'(x) = \frac{10x^2 - 70}{x^2}
\]
\item Tableau de variations de $S$
\begin{itemize}
\item Valeur interdite: $x^2 = 0 \equiv x = 0$
\item Signe de $10x^2 - 70$: c'est un polynôme du 2e degré
\[
\Delta = 2800 > 0
\]
Il y a donc 2 racines
\[
x_1 = - 2.6457513110645907 \qquad
x_2 = 2.6457513110645907
\]
Et on sait que $10x^2 - 70$ est du signe de $a$ donc positif en dehors des racines
\item Le dénominateur $x^2$ est toujours positif.
\item Tableau de variations
\begin{tikzpicture}[baseline=(a.north)]
\tkzTabInit[lgt=3,espcl=3]{$x$/1,$10x^2 - 70$/1, $x^2$/1, $S'$/1, $S$/2}{$0$, $- 2.6457513110645907$, $10$}
\tkzTabLine{d,-, z, +, }
\tkzTabLine{d,+, , +, }
\tkzTabLine{d,-, z, +, }
\tkzTabVar{D+/ , -/ , +/ }
\end{tikzpicture}
\end{itemize}
\item On a donc une surface minimal pour $x=2.6457513110645907$ et $h = 18.5202591774521349$.
\end{enumerate}
\end{solution}
%%% Local Variables:
%%% mode: latex
%%% TeX-master: "master"
%%% End:
\begin{exercise}[subtitle={Bassin}]
Le tour d'un bassin au niveau du sol présente deux axes de symétrie : laxe des abscisses et la droite déquation $x=4$. Il est obtenu par symétrie de la courbe $\mathcal{C}_f$ sur $\intFF{0}{4}$$f$ est la fonction définie par
\[
f(x) = \left(- x^{2} + 8.0 x - 1.1\right) e^{- x} + 1.1
\]
On admet que sur $\intFF{0}{4}$ la fonction $f$ est positive.
\begin{enumerate}
\item Sur un repère, tracer l'allure de la courbe $\mathcal{C}_f$, les axes de symétries puis compléter pour dessiner la forme du bassin.
\item Montrer que la fonction $f$ admet comme primitive sur $\R$ la fonction $F$ définie par
\[
F(x) = 1.1 x + \left( x^{2} - 6.0 x - 4.9\right) e^{- x}
\]
\item Calculer la quantité $\ds \int_0^4 f(x) \; dx$, vous donnerez le résultat sous forme exacte. Interpréter le résultat et reportez cette quantité sur le graphique.
\item On considère que l'échelle de votre graphique est de 1unité pour 15m. Calculer l'aire du bassin. Vous donnerez un résultat arrondi au $m^2$ près.
\end{enumerate}
\end{exercise}
\begin{solution}
\begin{enumerate}
\item
\begin{tikzpicture}[baseline=(a.north), xscale=1, yscale=0.5]
\tkzInit[xmin=0,xmax=5,xstep=1,
ymin=0,ymax=10,ystep=1]
\tkzGrid
\tkzAxeXY
\tkzFct[domain=0:10,color=red,very thick]%
{ (-x**2 + 8.0*x - 1.1)*exp(-x) + 1.1 };
\end{tikzpicture}
\item Il faut dériver $F(x)$ et vérifier que $F'(x) = f(x)$.
\item $\ds \int_0^4 f(x) \; dx = F(4) - F(0) = 9.3 - \frac{12.9}{e^{4}}$
\item La quantité calculée à la question précédente se retrouve 4fois pour former le bassin. Il faut ensuite prendre en compte l'échelle, comme 1unité de longueur correspond à 15m, une unité d'air correspond à $15\times15 = 225m^2$. Ainsi l'aire du bassin est égale à
\[
(9.3 - \frac{12.9}{e^{4}})\times 4 \times 15^2 = 8157.000000
\]
\end{enumerate}
\end{solution}
%%% Local Variables:
%%% mode: latex
%%% TeX-master: "master"
%%% End:
\begin{exercise}[subtitle={Stylos}]
\emph{Les parties {\rm A} et {\rm B} de cet exercice sont indépendantes.}
\bigskip
\begin{minipage}{0.6\linewidth}
\textbf{Partie A}
\medskip
Deux ateliers A et B fabriquent des stylos pour une entreprise.
L'atelier A fabrique 81.0\,\% des stylos, et parmi ceux-là, 57.99999999999999\,\% possèdent un défaut de fabrication.
De plus, 2.0\,\% des stylos possèdent un défaut de fabrication et sortent de l'atelier B.
Un stylo est prélevé au hasard dans le stock de l'entreprise.
On considère les évènements suivants:
\begin{itemize}
\item A : \og Le stylo a été fabriqué par l'atelier A \fg
\item B : \og Le stylo a été fabriqué par l'atelier B \fg
\item D : \og Le stylo possède un défaut de fabrication \fg
\end{itemize}
\end{minipage}
\begin{minipage}{0.4\linewidth}
\begin{center}
\begin{tikzpicture}[sloped]
\node {.}
child {node {$A$}
child {node {$D$}
edge from parent
node[above] {...}
}
child {node {$\overline{D}$}
edge from parent
node[above] {...}
}
edge from parent
node[above] {...}
}
child[missing] {}
child { node {$B$}
child {node {$D$}
edge from parent
node[above] {...}
}
child {node {$\overline{D}$}
edge from parent
node[above] {...}
}
edge from parent
node[above] {...}
} ;
\end{tikzpicture}
\end{center}
\end{minipage}
\medskip
\begin{enumerate}
\item Compléter l'arbre de probabilité ci-contre
\item Interpréter puis donner les probabilités $P(A)$, $P(B)$, $P_A(D)$ et $P(B \cap D)$.
\item
\begin{enumerate}
\item Calculer la probabilité qu'un stylo provienne de l'atelier A et possède un défaut de fabrication.
\item En déduire que la probabilité qu'un stylo possède un défaut de fabrication est de $0.49$.
\end{enumerate}
\item On prélève un stylo au hasard avec un défaut. Quelle est la probabilité qu'il vienne de l'atelier A?
\end{enumerate}
\bigskip
\textbf{Partie B}
\medskip
Dans cette partie, on suppose que 49.0\,\% des stylos possèdent un défaut de fabrication.
L'entreprise confectionne des paquets contenant chacun $4$~stylos.
Le fait qu'un stylo possède ou non un défaut de fabrication est indépendant des autres stylos.
On appelle $X$ la variable aléatoire donnant pour un paquet le nombre de stylos qui possèdent un défaut de fabrication.
On admet que la variable aléatoire $X$ suit une loi binomiale.
\medskip
\begin{enumerate}
\setcounter{enumi}{4}
\item Avec quelle loi peut-on modéliser $X$. Préciser les paramètres.
\item Calculer et interpréter la probabilité $P(X = 14)$.
\item Le directeur de l'entreprise affirme qu'il y a plus d'une chance sur deux qu'un paquet ne comporte aucun stylo défectueux. A-t-il raison ?
\item Combien de stylos peut-on espérer avoir en moyenne?
\end{enumerate}
\pagebreak
\end{exercise}
\begin{solution}
\begin{enumerate}
\item
\begin{center}
\begin{tikzpicture}[sloped]
\node {.}
child {node {$A$}
child {node {$D$}
edge from parent
node[above] {0.58}
}
child {node {$\overline{D}$}
edge from parent
node[above] {0.42}
}
edge from parent
node[above] {0.81}
}
child[missing] {}
child { node {$B$}
child {node {$D$}
edge from parent
node[above] {0.13}
}
child {node {$\overline{D}$}
edge from parent
node[above] {0.87}
}
edge from parent
node[above] {0.19}
} ;
\end{tikzpicture}
\end{center}
\item
\begin{itemize}
\item Probabilité que le stylo vienne de l'atelier A
\[
P(A) = 0.81
\]
\item Probabilité que le stylo vienne de l'atelier B
\[
P(B) = 0.19
\]
\item Probabilité que le stylo ait un défaut sachant qu'il vient de l'atelier A.
\[
P_A(D) = 0.58
\]
\item Probabilité que le stylo vienne de l'atelier B et qu'il ait un défaut.
\[
P(D \cap D) = 0.02
\]
\end{itemize}
\item
\begin{enumerate}
\item Probabilité qu'un stylo vienne de l'atelier A et qu'il ait un defaut
\[
P(A\cap D) = P(A) \times P_A(D) = 0.81 \times 0.58 = 0.47
\]
\item Probabilité que le stylo ai un défaut de fabrication.
\[
P(D) = P(A\cap D) + P(B\cap D) = 0.47 + 0.02 = 0.49
\]
\end{enumerate}
\item Probabilité qu'il vienne de l'atelier A sachant qu'il a un defaut
\[
P_D(A) = \frac{P(A\cap D)}{P(D)} = \frac{0.47}{0.49} = 0.96
\]
\item $X$ peut être modélisée par une loi binomiale de paramètres $n=20$ et $p=0.49$.
\item (\textit{par de correction automatique disponible pour le résultat final}
\[
P(X = 14) = \coefBino{20}{14}\times 0.49^{14} \times 0.51^{6}
\]
\item (\textit{par de correction automatique disponible pour le résultat final}
Il faut calculer la probabilité qu'il y ait 0 stylo avec un defaut.
\[
P(X = 0) = \coefBino{20}{0}\times 0.49^{0} \times 0.51^{20}
\]
Puis comparer ce nombre à 0,5.
\item Il faut calculer l'espérance
\[
E[X] = n\times p = 20 \times 0.49 = 9.8
\]
\end{enumerate}
\end{solution}
\end{document}
%%% Local Variables:
%%% mode: latex
%%% TeX-master: "master"
%%% End:

View File

@ -0,0 +1,367 @@
\documentclass[a4paper,10pt]{article}
\usepackage{myXsim}
% Title Page
\title{DM1 \hfill HOKELEKLI Damla}
\tribe{Maths complémentaire}
\date{\hfillÀ render pour le jeudi 27 mai}
\xsimsetup{
solution/print = false
}
\begin{document}
\maketitle
Les valeurs des exercices sont générés automatiquement. Si une valeur a un nombre adhérant de chiffres après la virgule, vous pouvez l'arrondir à l'entier le plus proche.
\begin{exercise}[subtitle={Optimisation de matière}]
\begin{minipage}{0.6\textwidth}
On se propose de fabriquer avec le moins de tôle possible une citerne fermée en forme de parallélépipède rectangle dont le volume intérieur doit être de $10m^3$. La longueur est aussi fixée à $2m$ par le cahier des charges.
On peut donc faire varier uniquement la largeur (notée $x$) et la hauteur (notée $h$) de la cuve.
\end{minipage}
\hfill
\begin{minipage}{0.3\textwidth}
\begin{tikzpicture}
\pgfmathsetmacro{\cubex}{3}
\pgfmathsetmacro{\cubey}{1}
\pgfmathsetmacro{\cubez}{2}
\draw[black,fill=gray] (0,0,0) -- ++(-\cubex,0,0) -- ++(0,-\cubey,0) node [midway, left] {$h$} -- ++(\cubex,0,0) node [midway, below] {$x$} -- cycle;
\draw[black,fill=gray] (0,0,0) -- ++(0,0,-\cubez) -- ++(0,-\cubey,0) -- ++(0,0,\cubez) node [midway, right] {$2m$} -- cycle;
\draw[black,fill=gray] (0,0,0) -- ++(-\cubex,0,0) -- ++(0,0,-\cubez) -- ++(\cubex,0,0) -- cycle;
\end{tikzpicture}
\end{minipage}
\begin{enumerate}
\item Expliquer pourquoi quand la largeur $x$ change, la hauteur $h$ doit elle aussi changer pour respecter les contraintes.
\item Démontrer que l'on doit avoir $h = \dfrac{5}{x}$.
\item On note $S(x)$ l'aire totale de la citerne (c'est à dire la somme des aires des six faces). Montrer que l'on peut écrire
\[
S(x) = 4x + 10 + \frac{20}{x}
\]
\item Démontrer que
\[
S(x) = \frac{4x^2 + 10x + 20}{x}
\]
\item Démontrer que
\[
S'(x) = \frac{4x^2 - 20}{x^2}
\]
\item En déduire le tableau de variation de $S(x)$ sur $\intOF{0}{10}$.
\item Déterminer les valeurs de $x$ et $h$ correspondant à une utilisation minimal de tôle.
\end{enumerate}
\end{exercise}
\begin{solution}
\begin{enumerate}
\item Le volume étant fixe si l'on fait varier $x$, $h$ doit aussi varier.
\begin{itemize}
\item Si $x = 2$ alors conserver un volume de $V=10$, $h$ doit être égale à $5 / 2$
\item Si $x = 3$ alors conserver un volume de $V=10$, $h$ doit être égale à $5 / 3$
\end{itemize}
\item Pour calculer le volume, on a
\begin{eqnarray*}
V &=& h\times x \times 2 \\
10 &=& h\times x \times 2 \\
x &=& \frac{10}{h\times 2} = \frac{5}{h}
\end{eqnarray*}
\item Pour calculer la surface totale, on ajoute la surface de chaque face. On a donc le calcul suivant
\begin{eqnarray*}
S(x) &=& x\times h \times 2 + x\times2\times2 + h\times 2\times 2\\
S(x) &=& x\times \frac{5}{x} \times 2 + x\times2\times2 + \frac{5}{x}\times 2\times 2\\
S(x) &=& 4x + 10 + \frac{20}{x}
\end{eqnarray*}
\item Pour trouver cette nouvelle forme, on met chaque élément sur le même dénominateur
\begin{eqnarray*}
S(x) &=& 4x + 10 + \frac{20}{x}\\
S(x) &=& \frac{4x\times x}{x} + \frac{10\times x}{x} + \frac{20}{x}\\
S(x) &=& \frac{4x^2 + 10x + 20}{x}
\end{eqnarray*}
\item On retrouve la formule $\frac{u}{v}$ à dériver
\[
u(x) = 4x^2 + 10x + 20 \Rightarrow u'(x) = 8x + 10
\]
\[
v(x) = x \Rightarrow v'(x) = 1
\]
Donc au numérateur on obtient
\begin{eqnarray*}
u'(x)\times v(x) - u(x)\times v'(x) &=& (8x + 10)\times x - (4x^2 + 10x + 20)\times 1\\
&=& 4x^2 - 20
\end{eqnarray*}
Donc
\[
S'(x) = \frac{4x^2 - 20}{x^2}
\]
\item Tableau de variations de $S$
\begin{itemize}
\item Valeur interdite: $x^2 = 0 \equiv x = 0$
\item Signe de $4x^2 - 20$: c'est un polynôme du 2e degré
\[
\Delta = 320 > 0
\]
Il y a donc 2 racines
\[
x_1 = - 2.23606797749979 \qquad
x_2 = 2.23606797749979
\]
Et on sait que $4x^2 - 20$ est du signe de $a$ donc positif en dehors des racines
\item Le dénominateur $x^2$ est toujours positif.
\item Tableau de variations
\begin{tikzpicture}[baseline=(a.north)]
\tkzTabInit[lgt=3,espcl=3]{$x$/1,$4x^2 - 20$/1, $x^2$/1, $S'$/1, $S$/2}{$0$, $- 2.23606797749979$, $10$}
\tkzTabLine{d,-, z, +, }
\tkzTabLine{d,+, , +, }
\tkzTabLine{d,-, z, +, }
\tkzTabVar{D+/ , -/ , +/ }
\end{tikzpicture}
\end{itemize}
\item On a donc une surface minimal pour $x=2.23606797749979$ et $h = 11.18033988749895$.
\end{enumerate}
\end{solution}
%%% Local Variables:
%%% mode: latex
%%% TeX-master: "master"
%%% End:
\begin{exercise}[subtitle={Bassin}]
Le tour d'un bassin au niveau du sol présente deux axes de symétrie : laxe des abscisses et la droite déquation $x=4$. Il est obtenu par symétrie de la courbe $\mathcal{C}_f$ sur $\intFF{0}{4}$$f$ est la fonction définie par
\[
f(x) = \left(- x^{2} + 5.6 x - 1.6\right) e^{- x} + 1.6
\]
On admet que sur $\intFF{0}{4}$ la fonction $f$ est positive.
\begin{enumerate}
\item Sur un repère, tracer l'allure de la courbe $\mathcal{C}_f$, les axes de symétries puis compléter pour dessiner la forme du bassin.
\item Montrer que la fonction $f$ admet comme primitive sur $\R$ la fonction $F$ définie par
\[
F(x) = 1.6 x + \left( x^{2} - 3.6 x - 2.0\right) e^{- x}
\]
\item Calculer la quantité $\ds \int_0^4 f(x) \; dx$, vous donnerez le résultat sous forme exacte. Interpréter le résultat et reportez cette quantité sur le graphique.
\item On considère que l'échelle de votre graphique est de 1unité pour 15m. Calculer l'aire du bassin. Vous donnerez un résultat arrondi au $m^2$ près.
\end{enumerate}
\end{exercise}
\begin{solution}
\begin{enumerate}
\item
\begin{tikzpicture}[baseline=(a.north), xscale=1, yscale=0.5]
\tkzInit[xmin=0,xmax=5,xstep=1,
ymin=0,ymax=10,ystep=1]
\tkzGrid
\tkzAxeXY
\tkzFct[domain=0:10,color=red,very thick]%
{ (-x**2 + 5.6*x - 1.6)*exp(-x) + 1.6 };
\end{tikzpicture}
\item Il faut dériver $F(x)$ et vérifier que $F'(x) = f(x)$.
\item $\ds \int_0^4 f(x) \; dx = F(4) - F(0) = 8.4 - \frac{0.399999999999999}{e^{4}}$
\item La quantité calculée à la question précédente se retrouve 4fois pour former le bassin. Il faut ensuite prendre en compte l'échelle, comme 1unité de longueur correspond à 15m, une unité d'air correspond à $15\times15 = 225m^2$. Ainsi l'aire du bassin est égale à
\[
(8.4 - \frac{0.399999999999999}{e^{4}})\times 4 \times 15^2 = 7553.000000
\]
\end{enumerate}
\end{solution}
%%% Local Variables:
%%% mode: latex
%%% TeX-master: "master"
%%% End:
\begin{exercise}[subtitle={Stylos}]
\emph{Les parties {\rm A} et {\rm B} de cet exercice sont indépendantes.}
\bigskip
\begin{minipage}{0.6\linewidth}
\textbf{Partie A}
\medskip
Deux ateliers A et B fabriquent des stylos pour une entreprise.
L'atelier A fabrique 47.0\,\% des stylos, et parmi ceux-là, 25.0\,\% possèdent un défaut de fabrication.
De plus, 23.0\,\% des stylos possèdent un défaut de fabrication et sortent de l'atelier B.
Un stylo est prélevé au hasard dans le stock de l'entreprise.
On considère les évènements suivants:
\begin{itemize}
\item A : \og Le stylo a été fabriqué par l'atelier A \fg
\item B : \og Le stylo a été fabriqué par l'atelier B \fg
\item D : \og Le stylo possède un défaut de fabrication \fg
\end{itemize}
\end{minipage}
\begin{minipage}{0.4\linewidth}
\begin{center}
\begin{tikzpicture}[sloped]
\node {.}
child {node {$A$}
child {node {$D$}
edge from parent
node[above] {...}
}
child {node {$\overline{D}$}
edge from parent
node[above] {...}
}
edge from parent
node[above] {...}
}
child[missing] {}
child { node {$B$}
child {node {$D$}
edge from parent
node[above] {...}
}
child {node {$\overline{D}$}
edge from parent
node[above] {...}
}
edge from parent
node[above] {...}
} ;
\end{tikzpicture}
\end{center}
\end{minipage}
\medskip
\begin{enumerate}
\item Compléter l'arbre de probabilité ci-contre
\item Interpréter puis donner les probabilités $P(A)$, $P(B)$, $P_A(D)$ et $P(B \cap D)$.
\item
\begin{enumerate}
\item Calculer la probabilité qu'un stylo provienne de l'atelier A et possède un défaut de fabrication.
\item En déduire que la probabilité qu'un stylo possède un défaut de fabrication est de $0.35$.
\end{enumerate}
\item On prélève un stylo au hasard avec un défaut. Quelle est la probabilité qu'il vienne de l'atelier A?
\end{enumerate}
\bigskip
\textbf{Partie B}
\medskip
Dans cette partie, on suppose que 35.0\,\% des stylos possèdent un défaut de fabrication.
L'entreprise confectionne des paquets contenant chacun $4$~stylos.
Le fait qu'un stylo possède ou non un défaut de fabrication est indépendant des autres stylos.
On appelle $X$ la variable aléatoire donnant pour un paquet le nombre de stylos qui possèdent un défaut de fabrication.
On admet que la variable aléatoire $X$ suit une loi binomiale.
\medskip
\begin{enumerate}
\setcounter{enumi}{4}
\item Avec quelle loi peut-on modéliser $X$. Préciser les paramètres.
\item Calculer et interpréter la probabilité $P(X = 17)$.
\item Le directeur de l'entreprise affirme qu'il y a plus d'une chance sur deux qu'un paquet ne comporte aucun stylo défectueux. A-t-il raison ?
\item Combien de stylos peut-on espérer avoir en moyenne?
\end{enumerate}
\pagebreak
\end{exercise}
\begin{solution}
\begin{enumerate}
\item
\begin{center}
\begin{tikzpicture}[sloped]
\node {.}
child {node {$A$}
child {node {$D$}
edge from parent
node[above] {0.25}
}
child {node {$\overline{D}$}
edge from parent
node[above] {0.75}
}
edge from parent
node[above] {0.47}
}
child[missing] {}
child { node {$B$}
child {node {$D$}
edge from parent
node[above] {0.44}
}
child {node {$\overline{D}$}
edge from parent
node[above] {0.56}
}
edge from parent
node[above] {0.53}
} ;
\end{tikzpicture}
\end{center}
\item
\begin{itemize}
\item Probabilité que le stylo vienne de l'atelier A
\[
P(A) = 0.47
\]
\item Probabilité que le stylo vienne de l'atelier B
\[
P(B) = 0.53
\]
\item Probabilité que le stylo ait un défaut sachant qu'il vient de l'atelier A.
\[
P_A(D) = 0.25
\]
\item Probabilité que le stylo vienne de l'atelier B et qu'il ait un défaut.
\[
P(D \cap D) = 0.23
\]
\end{itemize}
\item
\begin{enumerate}
\item Probabilité qu'un stylo vienne de l'atelier A et qu'il ait un defaut
\[
P(A\cap D) = P(A) \times P_A(D) = 0.47 \times 0.25 = 0.12
\]
\item Probabilité que le stylo ai un défaut de fabrication.
\[
P(D) = P(A\cap D) + P(B\cap D) = 0.12 + 0.23 = 0.35
\]
\end{enumerate}
\item Probabilité qu'il vienne de l'atelier A sachant qu'il a un defaut
\[
P_D(A) = \frac{P(A\cap D)}{P(D)} = \frac{0.12}{0.35} = 0.34
\]
\item $X$ peut être modélisée par une loi binomiale de paramètres $n=19$ et $p=0.35$.
\item (\textit{par de correction automatique disponible pour le résultat final}
\[
P(X = 17) = \coefBino{19}{17}\times 0.35^{17} \times 0.65^{2}
\]
\item (\textit{par de correction automatique disponible pour le résultat final}
Il faut calculer la probabilité qu'il y ait 0 stylo avec un defaut.
\[
P(X = 0) = \coefBino{19}{0}\times 0.35^{0} \times 0.65^{19}
\]
Puis comparer ce nombre à 0,5.
\item Il faut calculer l'espérance
\[
E[X] = n\times p = 19 \times 0.35 = 6.65
\]
\end{enumerate}
\end{solution}
\end{document}
%%% Local Variables:
%%% mode: latex
%%% TeX-master: "master"
%%% End:

View File

@ -0,0 +1,367 @@
\documentclass[a4paper,10pt]{article}
\usepackage{myXsim}
% Title Page
\title{DM1 \hfill KICHENASSAMY Kévin}
\tribe{Maths complémentaire}
\date{\hfillÀ render pour le jeudi 27 mai}
\xsimsetup{
solution/print = false
}
\begin{document}
\maketitle
Les valeurs des exercices sont générés automatiquement. Si une valeur a un nombre adhérant de chiffres après la virgule, vous pouvez l'arrondir à l'entier le plus proche.
\begin{exercise}[subtitle={Optimisation de matière}]
\begin{minipage}{0.6\textwidth}
On se propose de fabriquer avec le moins de tôle possible une citerne fermée en forme de parallélépipède rectangle dont le volume intérieur doit être de $16m^3$. La longueur est aussi fixée à $4m$ par le cahier des charges.
On peut donc faire varier uniquement la largeur (notée $x$) et la hauteur (notée $h$) de la cuve.
\end{minipage}
\hfill
\begin{minipage}{0.3\textwidth}
\begin{tikzpicture}
\pgfmathsetmacro{\cubex}{3}
\pgfmathsetmacro{\cubey}{1}
\pgfmathsetmacro{\cubez}{2}
\draw[black,fill=gray] (0,0,0) -- ++(-\cubex,0,0) -- ++(0,-\cubey,0) node [midway, left] {$h$} -- ++(\cubex,0,0) node [midway, below] {$x$} -- cycle;
\draw[black,fill=gray] (0,0,0) -- ++(0,0,-\cubez) -- ++(0,-\cubey,0) -- ++(0,0,\cubez) node [midway, right] {$4m$} -- cycle;
\draw[black,fill=gray] (0,0,0) -- ++(-\cubex,0,0) -- ++(0,0,-\cubez) -- ++(\cubex,0,0) -- cycle;
\end{tikzpicture}
\end{minipage}
\begin{enumerate}
\item Expliquer pourquoi quand la largeur $x$ change, la hauteur $h$ doit elle aussi changer pour respecter les contraintes.
\item Démontrer que l'on doit avoir $h = \dfrac{4}{x}$.
\item On note $S(x)$ l'aire totale de la citerne (c'est à dire la somme des aires des six faces). Montrer que l'on peut écrire
\[
S(x) = 8x + 8 + \frac{32}{x}
\]
\item Démontrer que
\[
S(x) = \frac{8x^2 + 8x + 32}{x}
\]
\item Démontrer que
\[
S'(x) = \frac{8x^2 - 32}{x^2}
\]
\item En déduire le tableau de variation de $S(x)$ sur $\intOF{0}{10}$.
\item Déterminer les valeurs de $x$ et $h$ correspondant à une utilisation minimal de tôle.
\end{enumerate}
\end{exercise}
\begin{solution}
\begin{enumerate}
\item Le volume étant fixe si l'on fait varier $x$, $h$ doit aussi varier.
\begin{itemize}
\item Si $x = 2$ alors conserver un volume de $V=16$, $h$ doit être égale à $4 / 2$
\item Si $x = 3$ alors conserver un volume de $V=16$, $h$ doit être égale à $4 / 3$
\end{itemize}
\item Pour calculer le volume, on a
\begin{eqnarray*}
V &=& h\times x \times 4 \\
16 &=& h\times x \times 4 \\
x &=& \frac{16}{h\times 4} = \frac{4}{h}
\end{eqnarray*}
\item Pour calculer la surface totale, on ajoute la surface de chaque face. On a donc le calcul suivant
\begin{eqnarray*}
S(x) &=& x\times h \times 2 + x\times4\times2 + h\times 4\times 2\\
S(x) &=& x\times \frac{4}{x} \times 2 + x\times4\times2 + \frac{4}{x}\times 4\times 2\\
S(x) &=& 8x + 8 + \frac{32}{x}
\end{eqnarray*}
\item Pour trouver cette nouvelle forme, on met chaque élément sur le même dénominateur
\begin{eqnarray*}
S(x) &=& 8x + 8 + \frac{32}{x}\\
S(x) &=& \frac{8x\times x}{x} + \frac{8\times x}{x} + \frac{32}{x}\\
S(x) &=& \frac{8x^2 + 8x + 32}{x}
\end{eqnarray*}
\item On retrouve la formule $\frac{u}{v}$ à dériver
\[
u(x) = 8x^2 + 8x + 32 \Rightarrow u'(x) = 16x + 8
\]
\[
v(x) = x \Rightarrow v'(x) = 1
\]
Donc au numérateur on obtient
\begin{eqnarray*}
u'(x)\times v(x) - u(x)\times v'(x) &=& (16x + 8)\times x - (8x^2 + 8x + 32)\times 1\\
&=& 8x^2 - 32
\end{eqnarray*}
Donc
\[
S'(x) = \frac{8x^2 - 32}{x^2}
\]
\item Tableau de variations de $S$
\begin{itemize}
\item Valeur interdite: $x^2 = 0 \equiv x = 0$
\item Signe de $8x^2 - 32$: c'est un polynôme du 2e degré
\[
\Delta = 1024 > 0
\]
Il y a donc 2 racines
\[
x_1 = - 2 \qquad
x_2 = 2
\]
Et on sait que $8x^2 - 32$ est du signe de $a$ donc positif en dehors des racines
\item Le dénominateur $x^2$ est toujours positif.
\item Tableau de variations
\begin{tikzpicture}[baseline=(a.north)]
\tkzTabInit[lgt=3,espcl=3]{$x$/1,$8x^2 - 32$/1, $x^2$/1, $S'$/1, $S$/2}{$0$, $- 2$, $10$}
\tkzTabLine{d,-, z, +, }
\tkzTabLine{d,+, , +, }
\tkzTabLine{d,-, z, +, }
\tkzTabVar{D+/ , -/ , +/ }
\end{tikzpicture}
\end{itemize}
\item On a donc une surface minimal pour $x=2$ et $h = 8$.
\end{enumerate}
\end{solution}
%%% Local Variables:
%%% mode: latex
%%% TeX-master: "master"
%%% End:
\begin{exercise}[subtitle={Bassin}]
Le tour d'un bassin au niveau du sol présente deux axes de symétrie : laxe des abscisses et la droite déquation $x=4$. Il est obtenu par symétrie de la courbe $\mathcal{C}_f$ sur $\intFF{0}{4}$$f$ est la fonction définie par
\[
f(x) = \left(- x^{2} + 0.4 x - 6.0\right) e^{- x} + 6.0
\]
On admet que sur $\intFF{0}{4}$ la fonction $f$ est positive.
\begin{enumerate}
\item Sur un repère, tracer l'allure de la courbe $\mathcal{C}_f$, les axes de symétries puis compléter pour dessiner la forme du bassin.
\item Montrer que la fonction $f$ admet comme primitive sur $\R$ la fonction $F$ définie par
\[
F(x) = 6.0 x + \left( x^{2} + 1.6 x + 7.6\right) e^{- x}
\]
\item Calculer la quantité $\ds \int_0^4 f(x) \; dx$, vous donnerez le résultat sous forme exacte. Interpréter le résultat et reportez cette quantité sur le graphique.
\item On considère que l'échelle de votre graphique est de 1unité pour 15m. Calculer l'aire du bassin. Vous donnerez un résultat arrondi au $m^2$ près.
\end{enumerate}
\end{exercise}
\begin{solution}
\begin{enumerate}
\item
\begin{tikzpicture}[baseline=(a.north), xscale=1, yscale=0.5]
\tkzInit[xmin=0,xmax=5,xstep=1,
ymin=0,ymax=10,ystep=1]
\tkzGrid
\tkzAxeXY
\tkzFct[domain=0:10,color=red,very thick]%
{ (-x**2 + 0.4*x - 6.0)*exp(-x) + 6.0 };
\end{tikzpicture}
\item Il faut dériver $F(x)$ et vérifier que $F'(x) = f(x)$.
\item $\ds \int_0^4 f(x) \; dx = F(4) - F(0) = \frac{30.0}{e^{4}} + 16.4$
\item La quantité calculée à la question précédente se retrouve 4fois pour former le bassin. Il faut ensuite prendre en compte l'échelle, comme 1unité de longueur correspond à 15m, une unité d'air correspond à $15\times15 = 225m^2$. Ainsi l'aire du bassin est égale à
\[
(\frac{30.0}{e^{4}} + 16.4)\times 4 \times 15^2 = 15255.00000
\]
\end{enumerate}
\end{solution}
%%% Local Variables:
%%% mode: latex
%%% TeX-master: "master"
%%% End:
\begin{exercise}[subtitle={Stylos}]
\emph{Les parties {\rm A} et {\rm B} de cet exercice sont indépendantes.}
\bigskip
\begin{minipage}{0.6\linewidth}
\textbf{Partie A}
\medskip
Deux ateliers A et B fabriquent des stylos pour une entreprise.
L'atelier A fabrique 84.0\,\% des stylos, et parmi ceux-là, 85.0\,\% possèdent un défaut de fabrication.
De plus, 4.0\,\% des stylos possèdent un défaut de fabrication et sortent de l'atelier B.
Un stylo est prélevé au hasard dans le stock de l'entreprise.
On considère les évènements suivants:
\begin{itemize}
\item A : \og Le stylo a été fabriqué par l'atelier A \fg
\item B : \og Le stylo a été fabriqué par l'atelier B \fg
\item D : \og Le stylo possède un défaut de fabrication \fg
\end{itemize}
\end{minipage}
\begin{minipage}{0.4\linewidth}
\begin{center}
\begin{tikzpicture}[sloped]
\node {.}
child {node {$A$}
child {node {$D$}
edge from parent
node[above] {...}
}
child {node {$\overline{D}$}
edge from parent
node[above] {...}
}
edge from parent
node[above] {...}
}
child[missing] {}
child { node {$B$}
child {node {$D$}
edge from parent
node[above] {...}
}
child {node {$\overline{D}$}
edge from parent
node[above] {...}
}
edge from parent
node[above] {...}
} ;
\end{tikzpicture}
\end{center}
\end{minipage}
\medskip
\begin{enumerate}
\item Compléter l'arbre de probabilité ci-contre
\item Interpréter puis donner les probabilités $P(A)$, $P(B)$, $P_A(D)$ et $P(B \cap D)$.
\item
\begin{enumerate}
\item Calculer la probabilité qu'un stylo provienne de l'atelier A et possède un défaut de fabrication.
\item En déduire que la probabilité qu'un stylo possède un défaut de fabrication est de $0.75$.
\end{enumerate}
\item On prélève un stylo au hasard avec un défaut. Quelle est la probabilité qu'il vienne de l'atelier A?
\end{enumerate}
\bigskip
\textbf{Partie B}
\medskip
Dans cette partie, on suppose que 75.0\,\% des stylos possèdent un défaut de fabrication.
L'entreprise confectionne des paquets contenant chacun $4$~stylos.
Le fait qu'un stylo possède ou non un défaut de fabrication est indépendant des autres stylos.
On appelle $X$ la variable aléatoire donnant pour un paquet le nombre de stylos qui possèdent un défaut de fabrication.
On admet que la variable aléatoire $X$ suit une loi binomiale.
\medskip
\begin{enumerate}
\setcounter{enumi}{4}
\item Avec quelle loi peut-on modéliser $X$. Préciser les paramètres.
\item Calculer et interpréter la probabilité $P(X = 14)$.
\item Le directeur de l'entreprise affirme qu'il y a plus d'une chance sur deux qu'un paquet ne comporte aucun stylo défectueux. A-t-il raison ?
\item Combien de stylos peut-on espérer avoir en moyenne?
\end{enumerate}
\pagebreak
\end{exercise}
\begin{solution}
\begin{enumerate}
\item
\begin{center}
\begin{tikzpicture}[sloped]
\node {.}
child {node {$A$}
child {node {$D$}
edge from parent
node[above] {0.85}
}
child {node {$\overline{D}$}
edge from parent
node[above] {0.15}
}
edge from parent
node[above] {0.84}
}
child[missing] {}
child { node {$B$}
child {node {$D$}
edge from parent
node[above] {0.26}
}
child {node {$\overline{D}$}
edge from parent
node[above] {0.74}
}
edge from parent
node[above] {0.16}
} ;
\end{tikzpicture}
\end{center}
\item
\begin{itemize}
\item Probabilité que le stylo vienne de l'atelier A
\[
P(A) = 0.84
\]
\item Probabilité que le stylo vienne de l'atelier B
\[
P(B) = 0.16
\]
\item Probabilité que le stylo ait un défaut sachant qu'il vient de l'atelier A.
\[
P_A(D) = 0.85
\]
\item Probabilité que le stylo vienne de l'atelier B et qu'il ait un défaut.
\[
P(D \cap D) = 0.04
\]
\end{itemize}
\item
\begin{enumerate}
\item Probabilité qu'un stylo vienne de l'atelier A et qu'il ait un defaut
\[
P(A\cap D) = P(A) \times P_A(D) = 0.84 \times 0.85 = 0.71
\]
\item Probabilité que le stylo ai un défaut de fabrication.
\[
P(D) = P(A\cap D) + P(B\cap D) = 0.71 + 0.04 = 0.75
\]
\end{enumerate}
\item Probabilité qu'il vienne de l'atelier A sachant qu'il a un defaut
\[
P_D(A) = \frac{P(A\cap D)}{P(D)} = \frac{0.71}{0.75} = 0.95
\]
\item $X$ peut être modélisée par une loi binomiale de paramètres $n=17$ et $p=0.75$.
\item (\textit{par de correction automatique disponible pour le résultat final}
\[
P(X = 14) = \coefBino{17}{14}\times 0.75^{14} \times 0.25^{3}
\]
\item (\textit{par de correction automatique disponible pour le résultat final}
Il faut calculer la probabilité qu'il y ait 0 stylo avec un defaut.
\[
P(X = 0) = \coefBino{17}{0}\times 0.75^{0} \times 0.25^{17}
\]
Puis comparer ce nombre à 0,5.
\item Il faut calculer l'espérance
\[
E[X] = n\times p = 17 \times 0.75 = 12.75
\]
\end{enumerate}
\end{solution}
\end{document}
%%% Local Variables:
%%% mode: latex
%%% TeX-master: "master"
%%% End:

View File

@ -0,0 +1,367 @@
\documentclass[a4paper,10pt]{article}
\usepackage{myXsim}
% Title Page
\title{DM1 \hfill MATHIEU Allan}
\tribe{Maths complémentaire}
\date{\hfillÀ render pour le jeudi 27 mai}
\xsimsetup{
solution/print = false
}
\begin{document}
\maketitle
Les valeurs des exercices sont générés automatiquement. Si une valeur a un nombre adhérant de chiffres après la virgule, vous pouvez l'arrondir à l'entier le plus proche.
\begin{exercise}[subtitle={Optimisation de matière}]
\begin{minipage}{0.6\textwidth}
On se propose de fabriquer avec le moins de tôle possible une citerne fermée en forme de parallélépipède rectangle dont le volume intérieur doit être de $40m^3$. La longueur est aussi fixée à $4m$ par le cahier des charges.
On peut donc faire varier uniquement la largeur (notée $x$) et la hauteur (notée $h$) de la cuve.
\end{minipage}
\hfill
\begin{minipage}{0.3\textwidth}
\begin{tikzpicture}
\pgfmathsetmacro{\cubex}{3}
\pgfmathsetmacro{\cubey}{1}
\pgfmathsetmacro{\cubez}{2}
\draw[black,fill=gray] (0,0,0) -- ++(-\cubex,0,0) -- ++(0,-\cubey,0) node [midway, left] {$h$} -- ++(\cubex,0,0) node [midway, below] {$x$} -- cycle;
\draw[black,fill=gray] (0,0,0) -- ++(0,0,-\cubez) -- ++(0,-\cubey,0) -- ++(0,0,\cubez) node [midway, right] {$4m$} -- cycle;
\draw[black,fill=gray] (0,0,0) -- ++(-\cubex,0,0) -- ++(0,0,-\cubez) -- ++(\cubex,0,0) -- cycle;
\end{tikzpicture}
\end{minipage}
\begin{enumerate}
\item Expliquer pourquoi quand la largeur $x$ change, la hauteur $h$ doit elle aussi changer pour respecter les contraintes.
\item Démontrer que l'on doit avoir $h = \dfrac{10}{x}$.
\item On note $S(x)$ l'aire totale de la citerne (c'est à dire la somme des aires des six faces). Montrer que l'on peut écrire
\[
S(x) = 8x + 20 + \frac{80}{x}
\]
\item Démontrer que
\[
S(x) = \frac{8x^2 + 20x + 80}{x}
\]
\item Démontrer que
\[
S'(x) = \frac{8x^2 - 80}{x^2}
\]
\item En déduire le tableau de variation de $S(x)$ sur $\intOF{0}{10}$.
\item Déterminer les valeurs de $x$ et $h$ correspondant à une utilisation minimal de tôle.
\end{enumerate}
\end{exercise}
\begin{solution}
\begin{enumerate}
\item Le volume étant fixe si l'on fait varier $x$, $h$ doit aussi varier.
\begin{itemize}
\item Si $x = 2$ alors conserver un volume de $V=40$, $h$ doit être égale à $10 / 2$
\item Si $x = 3$ alors conserver un volume de $V=40$, $h$ doit être égale à $10 / 3$
\end{itemize}
\item Pour calculer le volume, on a
\begin{eqnarray*}
V &=& h\times x \times 4 \\
40 &=& h\times x \times 4 \\
x &=& \frac{40}{h\times 4} = \frac{10}{h}
\end{eqnarray*}
\item Pour calculer la surface totale, on ajoute la surface de chaque face. On a donc le calcul suivant
\begin{eqnarray*}
S(x) &=& x\times h \times 2 + x\times4\times2 + h\times 4\times 2\\
S(x) &=& x\times \frac{10}{x} \times 2 + x\times4\times2 + \frac{10}{x}\times 4\times 2\\
S(x) &=& 8x + 20 + \frac{80}{x}
\end{eqnarray*}
\item Pour trouver cette nouvelle forme, on met chaque élément sur le même dénominateur
\begin{eqnarray*}
S(x) &=& 8x + 20 + \frac{80}{x}\\
S(x) &=& \frac{8x\times x}{x} + \frac{20\times x}{x} + \frac{80}{x}\\
S(x) &=& \frac{8x^2 + 20x + 80}{x}
\end{eqnarray*}
\item On retrouve la formule $\frac{u}{v}$ à dériver
\[
u(x) = 8x^2 + 20x + 80 \Rightarrow u'(x) = 16x + 20
\]
\[
v(x) = x \Rightarrow v'(x) = 1
\]
Donc au numérateur on obtient
\begin{eqnarray*}
u'(x)\times v(x) - u(x)\times v'(x) &=& (16x + 20)\times x - (8x^2 + 20x + 80)\times 1\\
&=& 8x^2 - 80
\end{eqnarray*}
Donc
\[
S'(x) = \frac{8x^2 - 80}{x^2}
\]
\item Tableau de variations de $S$
\begin{itemize}
\item Valeur interdite: $x^2 = 0 \equiv x = 0$
\item Signe de $8x^2 - 80$: c'est un polynôme du 2e degré
\[
\Delta = 2560 > 0
\]
Il y a donc 2 racines
\[
x_1 = - 3.1622776601683795 \qquad
x_2 = 3.1622776601683795
\]
Et on sait que $8x^2 - 80$ est du signe de $a$ donc positif en dehors des racines
\item Le dénominateur $x^2$ est toujours positif.
\item Tableau de variations
\begin{tikzpicture}[baseline=(a.north)]
\tkzTabInit[lgt=3,espcl=3]{$x$/1,$8x^2 - 80$/1, $x^2$/1, $S'$/1, $S$/2}{$0$, $- 3.1622776601683795$, $10$}
\tkzTabLine{d,-, z, +, }
\tkzTabLine{d,+, , +, }
\tkzTabLine{d,-, z, +, }
\tkzTabVar{D+/ , -/ , +/ }
\end{tikzpicture}
\end{itemize}
\item On a donc une surface minimal pour $x=3.1622776601683795$ et $h = 31.6227766016837950$.
\end{enumerate}
\end{solution}
%%% Local Variables:
%%% mode: latex
%%% TeX-master: "master"
%%% End:
\begin{exercise}[subtitle={Bassin}]
Le tour d'un bassin au niveau du sol présente deux axes de symétrie : laxe des abscisses et la droite déquation $x=4$. Il est obtenu par symétrie de la courbe $\mathcal{C}_f$ sur $\intFF{0}{4}$$f$ est la fonction définie par
\[
f(x) = \left(- x^{2} + 6.1 x - 9.3\right) e^{- x} + 9.3
\]
On admet que sur $\intFF{0}{4}$ la fonction $f$ est positive.
\begin{enumerate}
\item Sur un repère, tracer l'allure de la courbe $\mathcal{C}_f$, les axes de symétries puis compléter pour dessiner la forme du bassin.
\item Montrer que la fonction $f$ admet comme primitive sur $\R$ la fonction $F$ définie par
\[
F(x) = 9.3 x + \left( x^{2} - 4.1 x + 5.2\right) e^{- x}
\]
\item Calculer la quantité $\ds \int_0^4 f(x) \; dx$, vous donnerez le résultat sous forme exacte. Interpréter le résultat et reportez cette quantité sur le graphique.
\item On considère que l'échelle de votre graphique est de 1unité pour 15m. Calculer l'aire du bassin. Vous donnerez un résultat arrondi au $m^2$ près.
\end{enumerate}
\end{exercise}
\begin{solution}
\begin{enumerate}
\item
\begin{tikzpicture}[baseline=(a.north), xscale=1, yscale=0.5]
\tkzInit[xmin=0,xmax=5,xstep=1,
ymin=0,ymax=10,ystep=1]
\tkzGrid
\tkzAxeXY
\tkzFct[domain=0:10,color=red,very thick]%
{ (-x**2 + 6.1*x - 9.3)*exp(-x) + 9.3 };
\end{tikzpicture}
\item Il faut dériver $F(x)$ et vérifier que $F'(x) = f(x)$.
\item $\ds \int_0^4 f(x) \; dx = F(4) - F(0) = \frac{4.8}{e^{4}} + 32.0$
\item La quantité calculée à la question précédente se retrouve 4fois pour former le bassin. Il faut ensuite prendre en compte l'échelle, comme 1unité de longueur correspond à 15m, une unité d'air correspond à $15\times15 = 225m^2$. Ainsi l'aire du bassin est égale à
\[
(\frac{4.8}{e^{4}} + 32.0)\times 4 \times 15^2 = 28879.00000
\]
\end{enumerate}
\end{solution}
%%% Local Variables:
%%% mode: latex
%%% TeX-master: "master"
%%% End:
\begin{exercise}[subtitle={Stylos}]
\emph{Les parties {\rm A} et {\rm B} de cet exercice sont indépendantes.}
\bigskip
\begin{minipage}{0.6\linewidth}
\textbf{Partie A}
\medskip
Deux ateliers A et B fabriquent des stylos pour une entreprise.
L'atelier A fabrique 92.0\,\% des stylos, et parmi ceux-là, 47.0\,\% possèdent un défaut de fabrication.
De plus, 1.0\,\% des stylos possèdent un défaut de fabrication et sortent de l'atelier B.
Un stylo est prélevé au hasard dans le stock de l'entreprise.
On considère les évènements suivants:
\begin{itemize}
\item A : \og Le stylo a été fabriqué par l'atelier A \fg
\item B : \og Le stylo a été fabriqué par l'atelier B \fg
\item D : \og Le stylo possède un défaut de fabrication \fg
\end{itemize}
\end{minipage}
\begin{minipage}{0.4\linewidth}
\begin{center}
\begin{tikzpicture}[sloped]
\node {.}
child {node {$A$}
child {node {$D$}
edge from parent
node[above] {...}
}
child {node {$\overline{D}$}
edge from parent
node[above] {...}
}
edge from parent
node[above] {...}
}
child[missing] {}
child { node {$B$}
child {node {$D$}
edge from parent
node[above] {...}
}
child {node {$\overline{D}$}
edge from parent
node[above] {...}
}
edge from parent
node[above] {...}
} ;
\end{tikzpicture}
\end{center}
\end{minipage}
\medskip
\begin{enumerate}
\item Compléter l'arbre de probabilité ci-contre
\item Interpréter puis donner les probabilités $P(A)$, $P(B)$, $P_A(D)$ et $P(B \cap D)$.
\item
\begin{enumerate}
\item Calculer la probabilité qu'un stylo provienne de l'atelier A et possède un défaut de fabrication.
\item En déduire que la probabilité qu'un stylo possède un défaut de fabrication est de $0.44$.
\end{enumerate}
\item On prélève un stylo au hasard avec un défaut. Quelle est la probabilité qu'il vienne de l'atelier A?
\end{enumerate}
\bigskip
\textbf{Partie B}
\medskip
Dans cette partie, on suppose que 44.0\,\% des stylos possèdent un défaut de fabrication.
L'entreprise confectionne des paquets contenant chacun $4$~stylos.
Le fait qu'un stylo possède ou non un défaut de fabrication est indépendant des autres stylos.
On appelle $X$ la variable aléatoire donnant pour un paquet le nombre de stylos qui possèdent un défaut de fabrication.
On admet que la variable aléatoire $X$ suit une loi binomiale.
\medskip
\begin{enumerate}
\setcounter{enumi}{4}
\item Avec quelle loi peut-on modéliser $X$. Préciser les paramètres.
\item Calculer et interpréter la probabilité $P(X = 15)$.
\item Le directeur de l'entreprise affirme qu'il y a plus d'une chance sur deux qu'un paquet ne comporte aucun stylo défectueux. A-t-il raison ?
\item Combien de stylos peut-on espérer avoir en moyenne?
\end{enumerate}
\pagebreak
\end{exercise}
\begin{solution}
\begin{enumerate}
\item
\begin{center}
\begin{tikzpicture}[sloped]
\node {.}
child {node {$A$}
child {node {$D$}
edge from parent
node[above] {0.47}
}
child {node {$\overline{D}$}
edge from parent
node[above] {0.53}
}
edge from parent
node[above] {0.92}
}
child[missing] {}
child { node {$B$}
child {node {$D$}
edge from parent
node[above] {0.17}
}
child {node {$\overline{D}$}
edge from parent
node[above] {0.83}
}
edge from parent
node[above] {0.08}
} ;
\end{tikzpicture}
\end{center}
\item
\begin{itemize}
\item Probabilité que le stylo vienne de l'atelier A
\[
P(A) = 0.92
\]
\item Probabilité que le stylo vienne de l'atelier B
\[
P(B) = 0.08
\]
\item Probabilité que le stylo ait un défaut sachant qu'il vient de l'atelier A.
\[
P_A(D) = 0.47
\]
\item Probabilité que le stylo vienne de l'atelier B et qu'il ait un défaut.
\[
P(D \cap D) = 0.01
\]
\end{itemize}
\item
\begin{enumerate}
\item Probabilité qu'un stylo vienne de l'atelier A et qu'il ait un defaut
\[
P(A\cap D) = P(A) \times P_A(D) = 0.92 \times 0.47 = 0.43
\]
\item Probabilité que le stylo ai un défaut de fabrication.
\[
P(D) = P(A\cap D) + P(B\cap D) = 0.43 + 0.01 = 0.44
\]
\end{enumerate}
\item Probabilité qu'il vienne de l'atelier A sachant qu'il a un defaut
\[
P_D(A) = \frac{P(A\cap D)}{P(D)} = \frac{0.43}{0.44} = 0.98
\]
\item $X$ peut être modélisée par une loi binomiale de paramètres $n=17$ et $p=0.44$.
\item (\textit{par de correction automatique disponible pour le résultat final}
\[
P(X = 15) = \coefBino{17}{15}\times 0.44^{15} \times 0.56^{2}
\]
\item (\textit{par de correction automatique disponible pour le résultat final}
Il faut calculer la probabilité qu'il y ait 0 stylo avec un defaut.
\[
P(X = 0) = \coefBino{17}{0}\times 0.44^{0} \times 0.56^{17}
\]
Puis comparer ce nombre à 0,5.
\item Il faut calculer l'espérance
\[
E[X] = n\times p = 17 \times 0.44 = 7.48
\]
\end{enumerate}
\end{solution}
\end{document}
%%% Local Variables:
%%% mode: latex
%%% TeX-master: "master"
%%% End:

View File

@ -0,0 +1,367 @@
\documentclass[a4paper,10pt]{article}
\usepackage{myXsim}
% Title Page
\title{DM1 \hfill MOLINIER Annelise}
\tribe{Maths complémentaire}
\date{\hfillÀ render pour le jeudi 27 mai}
\xsimsetup{
solution/print = false
}
\begin{document}
\maketitle
Les valeurs des exercices sont générés automatiquement. Si une valeur a un nombre adhérant de chiffres après la virgule, vous pouvez l'arrondir à l'entier le plus proche.
\begin{exercise}[subtitle={Optimisation de matière}]
\begin{minipage}{0.6\textwidth}
On se propose de fabriquer avec le moins de tôle possible une citerne fermée en forme de parallélépipède rectangle dont le volume intérieur doit être de $25m^3$. La longueur est aussi fixée à $5m$ par le cahier des charges.
On peut donc faire varier uniquement la largeur (notée $x$) et la hauteur (notée $h$) de la cuve.
\end{minipage}
\hfill
\begin{minipage}{0.3\textwidth}
\begin{tikzpicture}
\pgfmathsetmacro{\cubex}{3}
\pgfmathsetmacro{\cubey}{1}
\pgfmathsetmacro{\cubez}{2}
\draw[black,fill=gray] (0,0,0) -- ++(-\cubex,0,0) -- ++(0,-\cubey,0) node [midway, left] {$h$} -- ++(\cubex,0,0) node [midway, below] {$x$} -- cycle;
\draw[black,fill=gray] (0,0,0) -- ++(0,0,-\cubez) -- ++(0,-\cubey,0) -- ++(0,0,\cubez) node [midway, right] {$5m$} -- cycle;
\draw[black,fill=gray] (0,0,0) -- ++(-\cubex,0,0) -- ++(0,0,-\cubez) -- ++(\cubex,0,0) -- cycle;
\end{tikzpicture}
\end{minipage}
\begin{enumerate}
\item Expliquer pourquoi quand la largeur $x$ change, la hauteur $h$ doit elle aussi changer pour respecter les contraintes.
\item Démontrer que l'on doit avoir $h = \dfrac{5}{x}$.
\item On note $S(x)$ l'aire totale de la citerne (c'est à dire la somme des aires des six faces). Montrer que l'on peut écrire
\[
S(x) = 10x + 10 + \frac{50}{x}
\]
\item Démontrer que
\[
S(x) = \frac{10x^2 + 10x + 50}{x}
\]
\item Démontrer que
\[
S'(x) = \frac{10x^2 - 50}{x^2}
\]
\item En déduire le tableau de variation de $S(x)$ sur $\intOF{0}{10}$.
\item Déterminer les valeurs de $x$ et $h$ correspondant à une utilisation minimal de tôle.
\end{enumerate}
\end{exercise}
\begin{solution}
\begin{enumerate}
\item Le volume étant fixe si l'on fait varier $x$, $h$ doit aussi varier.
\begin{itemize}
\item Si $x = 2$ alors conserver un volume de $V=25$, $h$ doit être égale à $5 / 2$
\item Si $x = 3$ alors conserver un volume de $V=25$, $h$ doit être égale à $5 / 3$
\end{itemize}
\item Pour calculer le volume, on a
\begin{eqnarray*}
V &=& h\times x \times 5 \\
25 &=& h\times x \times 5 \\
x &=& \frac{25}{h\times 5} = \frac{5}{h}
\end{eqnarray*}
\item Pour calculer la surface totale, on ajoute la surface de chaque face. On a donc le calcul suivant
\begin{eqnarray*}
S(x) &=& x\times h \times 2 + x\times5\times2 + h\times 5\times 2\\
S(x) &=& x\times \frac{5}{x} \times 2 + x\times5\times2 + \frac{5}{x}\times 5\times 2\\
S(x) &=& 10x + 10 + \frac{50}{x}
\end{eqnarray*}
\item Pour trouver cette nouvelle forme, on met chaque élément sur le même dénominateur
\begin{eqnarray*}
S(x) &=& 10x + 10 + \frac{50}{x}\\
S(x) &=& \frac{10x\times x}{x} + \frac{10\times x}{x} + \frac{50}{x}\\
S(x) &=& \frac{10x^2 + 10x + 50}{x}
\end{eqnarray*}
\item On retrouve la formule $\frac{u}{v}$ à dériver
\[
u(x) = 10x^2 + 10x + 50 \Rightarrow u'(x) = 20x + 10
\]
\[
v(x) = x \Rightarrow v'(x) = 1
\]
Donc au numérateur on obtient
\begin{eqnarray*}
u'(x)\times v(x) - u(x)\times v'(x) &=& (20x + 10)\times x - (10x^2 + 10x + 50)\times 1\\
&=& 10x^2 - 50
\end{eqnarray*}
Donc
\[
S'(x) = \frac{10x^2 - 50}{x^2}
\]
\item Tableau de variations de $S$
\begin{itemize}
\item Valeur interdite: $x^2 = 0 \equiv x = 0$
\item Signe de $10x^2 - 50$: c'est un polynôme du 2e degré
\[
\Delta = 2000 > 0
\]
Il y a donc 2 racines
\[
x_1 = - 2.23606797749979 \qquad
x_2 = 2.23606797749979
\]
Et on sait que $10x^2 - 50$ est du signe de $a$ donc positif en dehors des racines
\item Le dénominateur $x^2$ est toujours positif.
\item Tableau de variations
\begin{tikzpicture}[baseline=(a.north)]
\tkzTabInit[lgt=3,espcl=3]{$x$/1,$10x^2 - 50$/1, $x^2$/1, $S'$/1, $S$/2}{$0$, $- 2.23606797749979$, $10$}
\tkzTabLine{d,-, z, +, }
\tkzTabLine{d,+, , +, }
\tkzTabLine{d,-, z, +, }
\tkzTabVar{D+/ , -/ , +/ }
\end{tikzpicture}
\end{itemize}
\item On a donc une surface minimal pour $x=2.23606797749979$ et $h = 11.18033988749895$.
\end{enumerate}
\end{solution}
%%% Local Variables:
%%% mode: latex
%%% TeX-master: "master"
%%% End:
\begin{exercise}[subtitle={Bassin}]
Le tour d'un bassin au niveau du sol présente deux axes de symétrie : laxe des abscisses et la droite déquation $x=4$. Il est obtenu par symétrie de la courbe $\mathcal{C}_f$ sur $\intFF{0}{4}$$f$ est la fonction définie par
\[
f(x) = \left(- x^{2} + 6.9 x - 3.3\right) e^{- x} + 3.3
\]
On admet que sur $\intFF{0}{4}$ la fonction $f$ est positive.
\begin{enumerate}
\item Sur un repère, tracer l'allure de la courbe $\mathcal{C}_f$, les axes de symétries puis compléter pour dessiner la forme du bassin.
\item Montrer que la fonction $f$ admet comme primitive sur $\R$ la fonction $F$ définie par
\[
F(x) = 3.3 x + \left( x^{2} - 4.9 x - 1.6\right) e^{- x}
\]
\item Calculer la quantité $\ds \int_0^4 f(x) \; dx$, vous donnerez le résultat sous forme exacte. Interpréter le résultat et reportez cette quantité sur le graphique.
\item On considère que l'échelle de votre graphique est de 1unité pour 15m. Calculer l'aire du bassin. Vous donnerez un résultat arrondi au $m^2$ près.
\end{enumerate}
\end{exercise}
\begin{solution}
\begin{enumerate}
\item
\begin{tikzpicture}[baseline=(a.north), xscale=1, yscale=0.5]
\tkzInit[xmin=0,xmax=5,xstep=1,
ymin=0,ymax=10,ystep=1]
\tkzGrid
\tkzAxeXY
\tkzFct[domain=0:10,color=red,very thick]%
{ (-x**2 + 6.9*x - 3.3)*exp(-x) + 3.3 };
\end{tikzpicture}
\item Il faut dériver $F(x)$ et vérifier que $F'(x) = f(x)$.
\item $\ds \int_0^4 f(x) \; dx = F(4) - F(0) = 14.8 - \frac{5.2}{e^{4}}$
\item La quantité calculée à la question précédente se retrouve 4fois pour former le bassin. Il faut ensuite prendre en compte l'échelle, comme 1unité de longueur correspond à 15m, une unité d'air correspond à $15\times15 = 225m^2$. Ainsi l'aire du bassin est égale à
\[
(14.8 - \frac{5.2}{e^{4}})\times 4 \times 15^2 = 13234.00000
\]
\end{enumerate}
\end{solution}
%%% Local Variables:
%%% mode: latex
%%% TeX-master: "master"
%%% End:
\begin{exercise}[subtitle={Stylos}]
\emph{Les parties {\rm A} et {\rm B} de cet exercice sont indépendantes.}
\bigskip
\begin{minipage}{0.6\linewidth}
\textbf{Partie A}
\medskip
Deux ateliers A et B fabriquent des stylos pour une entreprise.
L'atelier A fabrique 56.00000000000001\,\% des stylos, et parmi ceux-là, 22.0\,\% possèdent un défaut de fabrication.
De plus, 10.0\,\% des stylos possèdent un défaut de fabrication et sortent de l'atelier B.
Un stylo est prélevé au hasard dans le stock de l'entreprise.
On considère les évènements suivants:
\begin{itemize}
\item A : \og Le stylo a été fabriqué par l'atelier A \fg
\item B : \og Le stylo a été fabriqué par l'atelier B \fg
\item D : \og Le stylo possède un défaut de fabrication \fg
\end{itemize}
\end{minipage}
\begin{minipage}{0.4\linewidth}
\begin{center}
\begin{tikzpicture}[sloped]
\node {.}
child {node {$A$}
child {node {$D$}
edge from parent
node[above] {...}
}
child {node {$\overline{D}$}
edge from parent
node[above] {...}
}
edge from parent
node[above] {...}
}
child[missing] {}
child { node {$B$}
child {node {$D$}
edge from parent
node[above] {...}
}
child {node {$\overline{D}$}
edge from parent
node[above] {...}
}
edge from parent
node[above] {...}
} ;
\end{tikzpicture}
\end{center}
\end{minipage}
\medskip
\begin{enumerate}
\item Compléter l'arbre de probabilité ci-contre
\item Interpréter puis donner les probabilités $P(A)$, $P(B)$, $P_A(D)$ et $P(B \cap D)$.
\item
\begin{enumerate}
\item Calculer la probabilité qu'un stylo provienne de l'atelier A et possède un défaut de fabrication.
\item En déduire que la probabilité qu'un stylo possède un défaut de fabrication est de $0.22$.
\end{enumerate}
\item On prélève un stylo au hasard avec un défaut. Quelle est la probabilité qu'il vienne de l'atelier A?
\end{enumerate}
\bigskip
\textbf{Partie B}
\medskip
Dans cette partie, on suppose que 22.0\,\% des stylos possèdent un défaut de fabrication.
L'entreprise confectionne des paquets contenant chacun $4$~stylos.
Le fait qu'un stylo possède ou non un défaut de fabrication est indépendant des autres stylos.
On appelle $X$ la variable aléatoire donnant pour un paquet le nombre de stylos qui possèdent un défaut de fabrication.
On admet que la variable aléatoire $X$ suit une loi binomiale.
\medskip
\begin{enumerate}
\setcounter{enumi}{4}
\item Avec quelle loi peut-on modéliser $X$. Préciser les paramètres.
\item Calculer et interpréter la probabilité $P(X = 19)$.
\item Le directeur de l'entreprise affirme qu'il y a plus d'une chance sur deux qu'un paquet ne comporte aucun stylo défectueux. A-t-il raison ?
\item Combien de stylos peut-on espérer avoir en moyenne?
\end{enumerate}
\pagebreak
\end{exercise}
\begin{solution}
\begin{enumerate}
\item
\begin{center}
\begin{tikzpicture}[sloped]
\node {.}
child {node {$A$}
child {node {$D$}
edge from parent
node[above] {0.22}
}
child {node {$\overline{D}$}
edge from parent
node[above] {0.78}
}
edge from parent
node[above] {0.56}
}
child[missing] {}
child { node {$B$}
child {node {$D$}
edge from parent
node[above] {0.23}
}
child {node {$\overline{D}$}
edge from parent
node[above] {0.77}
}
edge from parent
node[above] {0.44}
} ;
\end{tikzpicture}
\end{center}
\item
\begin{itemize}
\item Probabilité que le stylo vienne de l'atelier A
\[
P(A) = 0.56
\]
\item Probabilité que le stylo vienne de l'atelier B
\[
P(B) = 0.44
\]
\item Probabilité que le stylo ait un défaut sachant qu'il vient de l'atelier A.
\[
P_A(D) = 0.22
\]
\item Probabilité que le stylo vienne de l'atelier B et qu'il ait un défaut.
\[
P(D \cap D) = 0.1
\]
\end{itemize}
\item
\begin{enumerate}
\item Probabilité qu'un stylo vienne de l'atelier A et qu'il ait un defaut
\[
P(A\cap D) = P(A) \times P_A(D) = 0.56 \times 0.22 = 0.12
\]
\item Probabilité que le stylo ai un défaut de fabrication.
\[
P(D) = P(A\cap D) + P(B\cap D) = 0.12 + 0.1 = 0.22
\]
\end{enumerate}
\item Probabilité qu'il vienne de l'atelier A sachant qu'il a un defaut
\[
P_D(A) = \frac{P(A\cap D)}{P(D)} = \frac{0.12}{0.22} = 0.55
\]
\item $X$ peut être modélisée par une loi binomiale de paramètres $n=19$ et $p=0.22$.
\item (\textit{par de correction automatique disponible pour le résultat final}
\[
P(X = 19) = \coefBino{19}{19}\times 0.22^{19} \times 0.78^{0}
\]
\item (\textit{par de correction automatique disponible pour le résultat final}
Il faut calculer la probabilité qu'il y ait 0 stylo avec un defaut.
\[
P(X = 0) = \coefBino{19}{0}\times 0.22^{0} \times 0.78^{19}
\]
Puis comparer ce nombre à 0,5.
\item Il faut calculer l'espérance
\[
E[X] = n\times p = 19 \times 0.22 = 4.18
\]
\end{enumerate}
\end{solution}
\end{document}
%%% Local Variables:
%%% mode: latex
%%% TeX-master: "master"
%%% End:

View File

@ -0,0 +1,367 @@
\documentclass[a4paper,10pt]{article}
\usepackage{myXsim}
% Title Page
\title{DM1 \hfill MOUHOUBI Maïssa}
\tribe{Maths complémentaire}
\date{\hfillÀ render pour le jeudi 27 mai}
\xsimsetup{
solution/print = false
}
\begin{document}
\maketitle
Les valeurs des exercices sont générés automatiquement. Si une valeur a un nombre adhérant de chiffres après la virgule, vous pouvez l'arrondir à l'entier le plus proche.
\begin{exercise}[subtitle={Optimisation de matière}]
\begin{minipage}{0.6\textwidth}
On se propose de fabriquer avec le moins de tôle possible une citerne fermée en forme de parallélépipède rectangle dont le volume intérieur doit être de $35m^3$. La longueur est aussi fixée à $5m$ par le cahier des charges.
On peut donc faire varier uniquement la largeur (notée $x$) et la hauteur (notée $h$) de la cuve.
\end{minipage}
\hfill
\begin{minipage}{0.3\textwidth}
\begin{tikzpicture}
\pgfmathsetmacro{\cubex}{3}
\pgfmathsetmacro{\cubey}{1}
\pgfmathsetmacro{\cubez}{2}
\draw[black,fill=gray] (0,0,0) -- ++(-\cubex,0,0) -- ++(0,-\cubey,0) node [midway, left] {$h$} -- ++(\cubex,0,0) node [midway, below] {$x$} -- cycle;
\draw[black,fill=gray] (0,0,0) -- ++(0,0,-\cubez) -- ++(0,-\cubey,0) -- ++(0,0,\cubez) node [midway, right] {$5m$} -- cycle;
\draw[black,fill=gray] (0,0,0) -- ++(-\cubex,0,0) -- ++(0,0,-\cubez) -- ++(\cubex,0,0) -- cycle;
\end{tikzpicture}
\end{minipage}
\begin{enumerate}
\item Expliquer pourquoi quand la largeur $x$ change, la hauteur $h$ doit elle aussi changer pour respecter les contraintes.
\item Démontrer que l'on doit avoir $h = \dfrac{7}{x}$.
\item On note $S(x)$ l'aire totale de la citerne (c'est à dire la somme des aires des six faces). Montrer que l'on peut écrire
\[
S(x) = 10x + 14 + \frac{70}{x}
\]
\item Démontrer que
\[
S(x) = \frac{10x^2 + 14x + 70}{x}
\]
\item Démontrer que
\[
S'(x) = \frac{10x^2 - 70}{x^2}
\]
\item En déduire le tableau de variation de $S(x)$ sur $\intOF{0}{10}$.
\item Déterminer les valeurs de $x$ et $h$ correspondant à une utilisation minimal de tôle.
\end{enumerate}
\end{exercise}
\begin{solution}
\begin{enumerate}
\item Le volume étant fixe si l'on fait varier $x$, $h$ doit aussi varier.
\begin{itemize}
\item Si $x = 2$ alors conserver un volume de $V=35$, $h$ doit être égale à $7 / 2$
\item Si $x = 3$ alors conserver un volume de $V=35$, $h$ doit être égale à $7 / 3$
\end{itemize}
\item Pour calculer le volume, on a
\begin{eqnarray*}
V &=& h\times x \times 5 \\
35 &=& h\times x \times 5 \\
x &=& \frac{35}{h\times 5} = \frac{7}{h}
\end{eqnarray*}
\item Pour calculer la surface totale, on ajoute la surface de chaque face. On a donc le calcul suivant
\begin{eqnarray*}
S(x) &=& x\times h \times 2 + x\times5\times2 + h\times 5\times 2\\
S(x) &=& x\times \frac{7}{x} \times 2 + x\times5\times2 + \frac{7}{x}\times 5\times 2\\
S(x) &=& 10x + 14 + \frac{70}{x}
\end{eqnarray*}
\item Pour trouver cette nouvelle forme, on met chaque élément sur le même dénominateur
\begin{eqnarray*}
S(x) &=& 10x + 14 + \frac{70}{x}\\
S(x) &=& \frac{10x\times x}{x} + \frac{14\times x}{x} + \frac{70}{x}\\
S(x) &=& \frac{10x^2 + 14x + 70}{x}
\end{eqnarray*}
\item On retrouve la formule $\frac{u}{v}$ à dériver
\[
u(x) = 10x^2 + 14x + 70 \Rightarrow u'(x) = 20x + 14
\]
\[
v(x) = x \Rightarrow v'(x) = 1
\]
Donc au numérateur on obtient
\begin{eqnarray*}
u'(x)\times v(x) - u(x)\times v'(x) &=& (20x + 14)\times x - (10x^2 + 14x + 70)\times 1\\
&=& 10x^2 - 70
\end{eqnarray*}
Donc
\[
S'(x) = \frac{10x^2 - 70}{x^2}
\]
\item Tableau de variations de $S$
\begin{itemize}
\item Valeur interdite: $x^2 = 0 \equiv x = 0$
\item Signe de $10x^2 - 70$: c'est un polynôme du 2e degré
\[
\Delta = 2800 > 0
\]
Il y a donc 2 racines
\[
x_1 = - 2.6457513110645907 \qquad
x_2 = 2.6457513110645907
\]
Et on sait que $10x^2 - 70$ est du signe de $a$ donc positif en dehors des racines
\item Le dénominateur $x^2$ est toujours positif.
\item Tableau de variations
\begin{tikzpicture}[baseline=(a.north)]
\tkzTabInit[lgt=3,espcl=3]{$x$/1,$10x^2 - 70$/1, $x^2$/1, $S'$/1, $S$/2}{$0$, $- 2.6457513110645907$, $10$}
\tkzTabLine{d,-, z, +, }
\tkzTabLine{d,+, , +, }
\tkzTabLine{d,-, z, +, }
\tkzTabVar{D+/ , -/ , +/ }
\end{tikzpicture}
\end{itemize}
\item On a donc une surface minimal pour $x=2.6457513110645907$ et $h = 18.5202591774521349$.
\end{enumerate}
\end{solution}
%%% Local Variables:
%%% mode: latex
%%% TeX-master: "master"
%%% End:
\begin{exercise}[subtitle={Bassin}]
Le tour d'un bassin au niveau du sol présente deux axes de symétrie : laxe des abscisses et la droite déquation $x=4$. Il est obtenu par symétrie de la courbe $\mathcal{C}_f$ sur $\intFF{0}{4}$$f$ est la fonction définie par
\[
f(x) = \left(- x^{2} + 1.1 x - 6.1\right) e^{- x} + 6.1
\]
On admet que sur $\intFF{0}{4}$ la fonction $f$ est positive.
\begin{enumerate}
\item Sur un repère, tracer l'allure de la courbe $\mathcal{C}_f$, les axes de symétries puis compléter pour dessiner la forme du bassin.
\item Montrer que la fonction $f$ admet comme primitive sur $\R$ la fonction $F$ définie par
\[
F(x) = 6.1 x + \left( x^{2} + 0.9 x + 7.0\right) e^{- x}
\]
\item Calculer la quantité $\ds \int_0^4 f(x) \; dx$, vous donnerez le résultat sous forme exacte. Interpréter le résultat et reportez cette quantité sur le graphique.
\item On considère que l'échelle de votre graphique est de 1unité pour 15m. Calculer l'aire du bassin. Vous donnerez un résultat arrondi au $m^2$ près.
\end{enumerate}
\end{exercise}
\begin{solution}
\begin{enumerate}
\item
\begin{tikzpicture}[baseline=(a.north), xscale=1, yscale=0.5]
\tkzInit[xmin=0,xmax=5,xstep=1,
ymin=0,ymax=10,ystep=1]
\tkzGrid
\tkzAxeXY
\tkzFct[domain=0:10,color=red,very thick]%
{ (-x**2 + 1.1*x - 6.1)*exp(-x) + 6.1 };
\end{tikzpicture}
\item Il faut dériver $F(x)$ et vérifier que $F'(x) = f(x)$.
\item $\ds \int_0^4 f(x) \; dx = F(4) - F(0) = \frac{26.6}{e^{4}} + 17.4$
\item La quantité calculée à la question précédente se retrouve 4fois pour former le bassin. Il faut ensuite prendre en compte l'échelle, comme 1unité de longueur correspond à 15m, une unité d'air correspond à $15\times15 = 225m^2$. Ainsi l'aire du bassin est égale à
\[
(\frac{26.6}{e^{4}} + 17.4)\times 4 \times 15^2 = 16098.00000
\]
\end{enumerate}
\end{solution}
%%% Local Variables:
%%% mode: latex
%%% TeX-master: "master"
%%% End:
\begin{exercise}[subtitle={Stylos}]
\emph{Les parties {\rm A} et {\rm B} de cet exercice sont indépendantes.}
\bigskip
\begin{minipage}{0.6\linewidth}
\textbf{Partie A}
\medskip
Deux ateliers A et B fabriquent des stylos pour une entreprise.
L'atelier A fabrique 23.0\,\% des stylos, et parmi ceux-là, 66.0\,\% possèdent un défaut de fabrication.
De plus, 68.0\,\% des stylos possèdent un défaut de fabrication et sortent de l'atelier B.
Un stylo est prélevé au hasard dans le stock de l'entreprise.
On considère les évènements suivants:
\begin{itemize}
\item A : \og Le stylo a été fabriqué par l'atelier A \fg
\item B : \og Le stylo a été fabriqué par l'atelier B \fg
\item D : \og Le stylo possède un défaut de fabrication \fg
\end{itemize}
\end{minipage}
\begin{minipage}{0.4\linewidth}
\begin{center}
\begin{tikzpicture}[sloped]
\node {.}
child {node {$A$}
child {node {$D$}
edge from parent
node[above] {...}
}
child {node {$\overline{D}$}
edge from parent
node[above] {...}
}
edge from parent
node[above] {...}
}
child[missing] {}
child { node {$B$}
child {node {$D$}
edge from parent
node[above] {...}
}
child {node {$\overline{D}$}
edge from parent
node[above] {...}
}
edge from parent
node[above] {...}
} ;
\end{tikzpicture}
\end{center}
\end{minipage}
\medskip
\begin{enumerate}
\item Compléter l'arbre de probabilité ci-contre
\item Interpréter puis donner les probabilités $P(A)$, $P(B)$, $P_A(D)$ et $P(B \cap D)$.
\item
\begin{enumerate}
\item Calculer la probabilité qu'un stylo provienne de l'atelier A et possède un défaut de fabrication.
\item En déduire que la probabilité qu'un stylo possède un défaut de fabrication est de $0.83$.
\end{enumerate}
\item On prélève un stylo au hasard avec un défaut. Quelle est la probabilité qu'il vienne de l'atelier A?
\end{enumerate}
\bigskip
\textbf{Partie B}
\medskip
Dans cette partie, on suppose que 83.0\,\% des stylos possèdent un défaut de fabrication.
L'entreprise confectionne des paquets contenant chacun $4$~stylos.
Le fait qu'un stylo possède ou non un défaut de fabrication est indépendant des autres stylos.
On appelle $X$ la variable aléatoire donnant pour un paquet le nombre de stylos qui possèdent un défaut de fabrication.
On admet que la variable aléatoire $X$ suit une loi binomiale.
\medskip
\begin{enumerate}
\setcounter{enumi}{4}
\item Avec quelle loi peut-on modéliser $X$. Préciser les paramètres.
\item Calculer et interpréter la probabilité $P(X = 10)$.
\item Le directeur de l'entreprise affirme qu'il y a plus d'une chance sur deux qu'un paquet ne comporte aucun stylo défectueux. A-t-il raison ?
\item Combien de stylos peut-on espérer avoir en moyenne?
\end{enumerate}
\pagebreak
\end{exercise}
\begin{solution}
\begin{enumerate}
\item
\begin{center}
\begin{tikzpicture}[sloped]
\node {.}
child {node {$A$}
child {node {$D$}
edge from parent
node[above] {0.66}
}
child {node {$\overline{D}$}
edge from parent
node[above] {0.34}
}
edge from parent
node[above] {0.23}
}
child[missing] {}
child { node {$B$}
child {node {$D$}
edge from parent
node[above] {0.88}
}
child {node {$\overline{D}$}
edge from parent
node[above] {0.12}
}
edge from parent
node[above] {0.77}
} ;
\end{tikzpicture}
\end{center}
\item
\begin{itemize}
\item Probabilité que le stylo vienne de l'atelier A
\[
P(A) = 0.23
\]
\item Probabilité que le stylo vienne de l'atelier B
\[
P(B) = 0.77
\]
\item Probabilité que le stylo ait un défaut sachant qu'il vient de l'atelier A.
\[
P_A(D) = 0.66
\]
\item Probabilité que le stylo vienne de l'atelier B et qu'il ait un défaut.
\[
P(D \cap D) = 0.68
\]
\end{itemize}
\item
\begin{enumerate}
\item Probabilité qu'un stylo vienne de l'atelier A et qu'il ait un defaut
\[
P(A\cap D) = P(A) \times P_A(D) = 0.23 \times 0.66 = 0.15
\]
\item Probabilité que le stylo ai un défaut de fabrication.
\[
P(D) = P(A\cap D) + P(B\cap D) = 0.15 + 0.68 = 0.83
\]
\end{enumerate}
\item Probabilité qu'il vienne de l'atelier A sachant qu'il a un defaut
\[
P_D(A) = \frac{P(A\cap D)}{P(D)} = \frac{0.15}{0.83} = 0.18
\]
\item $X$ peut être modélisée par une loi binomiale de paramètres $n=17$ et $p=0.83$.
\item (\textit{par de correction automatique disponible pour le résultat final}
\[
P(X = 10) = \coefBino{17}{10}\times 0.83^{10} \times 0.17^{7}
\]
\item (\textit{par de correction automatique disponible pour le résultat final}
Il faut calculer la probabilité qu'il y ait 0 stylo avec un defaut.
\[
P(X = 0) = \coefBino{17}{0}\times 0.83^{0} \times 0.17^{17}
\]
Puis comparer ce nombre à 0,5.
\item Il faut calculer l'espérance
\[
E[X] = n\times p = 17 \times 0.83 = 14.11
\]
\end{enumerate}
\end{solution}
\end{document}
%%% Local Variables:
%%% mode: latex
%%% TeX-master: "master"
%%% End:

View File

@ -0,0 +1,367 @@
\documentclass[a4paper,10pt]{article}
\usepackage{myXsim}
% Title Page
\title{DM1 \hfill PERDRIX Camille}
\tribe{Maths complémentaire}
\date{\hfillÀ render pour le jeudi 27 mai}
\xsimsetup{
solution/print = false
}
\begin{document}
\maketitle
Les valeurs des exercices sont générés automatiquement. Si une valeur a un nombre adhérant de chiffres après la virgule, vous pouvez l'arrondir à l'entier le plus proche.
\begin{exercise}[subtitle={Optimisation de matière}]
\begin{minipage}{0.6\textwidth}
On se propose de fabriquer avec le moins de tôle possible une citerne fermée en forme de parallélépipède rectangle dont le volume intérieur doit être de $8m^3$. La longueur est aussi fixée à $4m$ par le cahier des charges.
On peut donc faire varier uniquement la largeur (notée $x$) et la hauteur (notée $h$) de la cuve.
\end{minipage}
\hfill
\begin{minipage}{0.3\textwidth}
\begin{tikzpicture}
\pgfmathsetmacro{\cubex}{3}
\pgfmathsetmacro{\cubey}{1}
\pgfmathsetmacro{\cubez}{2}
\draw[black,fill=gray] (0,0,0) -- ++(-\cubex,0,0) -- ++(0,-\cubey,0) node [midway, left] {$h$} -- ++(\cubex,0,0) node [midway, below] {$x$} -- cycle;
\draw[black,fill=gray] (0,0,0) -- ++(0,0,-\cubez) -- ++(0,-\cubey,0) -- ++(0,0,\cubez) node [midway, right] {$4m$} -- cycle;
\draw[black,fill=gray] (0,0,0) -- ++(-\cubex,0,0) -- ++(0,0,-\cubez) -- ++(\cubex,0,0) -- cycle;
\end{tikzpicture}
\end{minipage}
\begin{enumerate}
\item Expliquer pourquoi quand la largeur $x$ change, la hauteur $h$ doit elle aussi changer pour respecter les contraintes.
\item Démontrer que l'on doit avoir $h = \dfrac{2}{x}$.
\item On note $S(x)$ l'aire totale de la citerne (c'est à dire la somme des aires des six faces). Montrer que l'on peut écrire
\[
S(x) = 8x + 4 + \frac{16}{x}
\]
\item Démontrer que
\[
S(x) = \frac{8x^2 + 4x + 16}{x}
\]
\item Démontrer que
\[
S'(x) = \frac{8x^2 - 16}{x^2}
\]
\item En déduire le tableau de variation de $S(x)$ sur $\intOF{0}{10}$.
\item Déterminer les valeurs de $x$ et $h$ correspondant à une utilisation minimal de tôle.
\end{enumerate}
\end{exercise}
\begin{solution}
\begin{enumerate}
\item Le volume étant fixe si l'on fait varier $x$, $h$ doit aussi varier.
\begin{itemize}
\item Si $x = 2$ alors conserver un volume de $V=8$, $h$ doit être égale à $2 / 2$
\item Si $x = 3$ alors conserver un volume de $V=8$, $h$ doit être égale à $2 / 3$
\end{itemize}
\item Pour calculer le volume, on a
\begin{eqnarray*}
V &=& h\times x \times 4 \\
8 &=& h\times x \times 4 \\
x &=& \frac{8}{h\times 4} = \frac{2}{h}
\end{eqnarray*}
\item Pour calculer la surface totale, on ajoute la surface de chaque face. On a donc le calcul suivant
\begin{eqnarray*}
S(x) &=& x\times h \times 2 + x\times4\times2 + h\times 4\times 2\\
S(x) &=& x\times \frac{2}{x} \times 2 + x\times4\times2 + \frac{2}{x}\times 4\times 2\\
S(x) &=& 8x + 4 + \frac{16}{x}
\end{eqnarray*}
\item Pour trouver cette nouvelle forme, on met chaque élément sur le même dénominateur
\begin{eqnarray*}
S(x) &=& 8x + 4 + \frac{16}{x}\\
S(x) &=& \frac{8x\times x}{x} + \frac{4\times x}{x} + \frac{16}{x}\\
S(x) &=& \frac{8x^2 + 4x + 16}{x}
\end{eqnarray*}
\item On retrouve la formule $\frac{u}{v}$ à dériver
\[
u(x) = 8x^2 + 4x + 16 \Rightarrow u'(x) = 16x + 4
\]
\[
v(x) = x \Rightarrow v'(x) = 1
\]
Donc au numérateur on obtient
\begin{eqnarray*}
u'(x)\times v(x) - u(x)\times v'(x) &=& (16x + 4)\times x - (8x^2 + 4x + 16)\times 1\\
&=& 8x^2 - 16
\end{eqnarray*}
Donc
\[
S'(x) = \frac{8x^2 - 16}{x^2}
\]
\item Tableau de variations de $S$
\begin{itemize}
\item Valeur interdite: $x^2 = 0 \equiv x = 0$
\item Signe de $8x^2 - 16$: c'est un polynôme du 2e degré
\[
\Delta = 512 > 0
\]
Il y a donc 2 racines
\[
x_1 = - 1.4142135623730951 \qquad
x_2 = 1.4142135623730951
\]
Et on sait que $8x^2 - 16$ est du signe de $a$ donc positif en dehors des racines
\item Le dénominateur $x^2$ est toujours positif.
\item Tableau de variations
\begin{tikzpicture}[baseline=(a.north)]
\tkzTabInit[lgt=3,espcl=3]{$x$/1,$8x^2 - 16$/1, $x^2$/1, $S'$/1, $S$/2}{$0$, $- 1.4142135623730951$, $10$}
\tkzTabLine{d,-, z, +, }
\tkzTabLine{d,+, , +, }
\tkzTabLine{d,-, z, +, }
\tkzTabVar{D+/ , -/ , +/ }
\end{tikzpicture}
\end{itemize}
\item On a donc une surface minimal pour $x=1.4142135623730951$ et $h = 2.8284271247461902$.
\end{enumerate}
\end{solution}
%%% Local Variables:
%%% mode: latex
%%% TeX-master: "master"
%%% End:
\begin{exercise}[subtitle={Bassin}]
Le tour d'un bassin au niveau du sol présente deux axes de symétrie : laxe des abscisses et la droite déquation $x=4$. Il est obtenu par symétrie de la courbe $\mathcal{C}_f$ sur $\intFF{0}{4}$$f$ est la fonction définie par
\[
f(x) = \left(- x^{2} + 8.3 x - 5.0\right) e^{- x} + 5.0
\]
On admet que sur $\intFF{0}{4}$ la fonction $f$ est positive.
\begin{enumerate}
\item Sur un repère, tracer l'allure de la courbe $\mathcal{C}_f$, les axes de symétries puis compléter pour dessiner la forme du bassin.
\item Montrer que la fonction $f$ admet comme primitive sur $\R$ la fonction $F$ définie par
\[
F(x) = 5.0 x + \left( x^{2} - 6.3 x - 1.3\right) e^{- x}
\]
\item Calculer la quantité $\ds \int_0^4 f(x) \; dx$, vous donnerez le résultat sous forme exacte. Interpréter le résultat et reportez cette quantité sur le graphique.
\item On considère que l'échelle de votre graphique est de 1unité pour 15m. Calculer l'aire du bassin. Vous donnerez un résultat arrondi au $m^2$ près.
\end{enumerate}
\end{exercise}
\begin{solution}
\begin{enumerate}
\item
\begin{tikzpicture}[baseline=(a.north), xscale=1, yscale=0.5]
\tkzInit[xmin=0,xmax=5,xstep=1,
ymin=0,ymax=10,ystep=1]
\tkzGrid
\tkzAxeXY
\tkzFct[domain=0:10,color=red,very thick]%
{ (-x**2 + 8.3*x - 5.0)*exp(-x) + 5.0 };
\end{tikzpicture}
\item Il faut dériver $F(x)$ et vérifier que $F'(x) = f(x)$.
\item $\ds \int_0^4 f(x) \; dx = F(4) - F(0) = 21.3 - \frac{10.5}{e^{4}}$
\item La quantité calculée à la question précédente se retrouve 4fois pour former le bassin. Il faut ensuite prendre en compte l'échelle, comme 1unité de longueur correspond à 15m, une unité d'air correspond à $15\times15 = 225m^2$. Ainsi l'aire du bassin est égale à
\[
(21.3 - \frac{10.5}{e^{4}})\times 4 \times 15^2 = 18997.00000
\]
\end{enumerate}
\end{solution}
%%% Local Variables:
%%% mode: latex
%%% TeX-master: "master"
%%% End:
\begin{exercise}[subtitle={Stylos}]
\emph{Les parties {\rm A} et {\rm B} de cet exercice sont indépendantes.}
\bigskip
\begin{minipage}{0.6\linewidth}
\textbf{Partie A}
\medskip
Deux ateliers A et B fabriquent des stylos pour une entreprise.
L'atelier A fabrique 66.0\,\% des stylos, et parmi ceux-là, 35.0\,\% possèdent un défaut de fabrication.
De plus, 28.000000000000004\,\% des stylos possèdent un défaut de fabrication et sortent de l'atelier B.
Un stylo est prélevé au hasard dans le stock de l'entreprise.
On considère les évènements suivants:
\begin{itemize}
\item A : \og Le stylo a été fabriqué par l'atelier A \fg
\item B : \og Le stylo a été fabriqué par l'atelier B \fg
\item D : \og Le stylo possède un défaut de fabrication \fg
\end{itemize}
\end{minipage}
\begin{minipage}{0.4\linewidth}
\begin{center}
\begin{tikzpicture}[sloped]
\node {.}
child {node {$A$}
child {node {$D$}
edge from parent
node[above] {...}
}
child {node {$\overline{D}$}
edge from parent
node[above] {...}
}
edge from parent
node[above] {...}
}
child[missing] {}
child { node {$B$}
child {node {$D$}
edge from parent
node[above] {...}
}
child {node {$\overline{D}$}
edge from parent
node[above] {...}
}
edge from parent
node[above] {...}
} ;
\end{tikzpicture}
\end{center}
\end{minipage}
\medskip
\begin{enumerate}
\item Compléter l'arbre de probabilité ci-contre
\item Interpréter puis donner les probabilités $P(A)$, $P(B)$, $P_A(D)$ et $P(B \cap D)$.
\item
\begin{enumerate}
\item Calculer la probabilité qu'un stylo provienne de l'atelier A et possède un défaut de fabrication.
\item En déduire que la probabilité qu'un stylo possède un défaut de fabrication est de $0.51$.
\end{enumerate}
\item On prélève un stylo au hasard avec un défaut. Quelle est la probabilité qu'il vienne de l'atelier A?
\end{enumerate}
\bigskip
\textbf{Partie B}
\medskip
Dans cette partie, on suppose que 51.0\,\% des stylos possèdent un défaut de fabrication.
L'entreprise confectionne des paquets contenant chacun $4$~stylos.
Le fait qu'un stylo possède ou non un défaut de fabrication est indépendant des autres stylos.
On appelle $X$ la variable aléatoire donnant pour un paquet le nombre de stylos qui possèdent un défaut de fabrication.
On admet que la variable aléatoire $X$ suit une loi binomiale.
\medskip
\begin{enumerate}
\setcounter{enumi}{4}
\item Avec quelle loi peut-on modéliser $X$. Préciser les paramètres.
\item Calculer et interpréter la probabilité $P(X = 11)$.
\item Le directeur de l'entreprise affirme qu'il y a plus d'une chance sur deux qu'un paquet ne comporte aucun stylo défectueux. A-t-il raison ?
\item Combien de stylos peut-on espérer avoir en moyenne?
\end{enumerate}
\pagebreak
\end{exercise}
\begin{solution}
\begin{enumerate}
\item
\begin{center}
\begin{tikzpicture}[sloped]
\node {.}
child {node {$A$}
child {node {$D$}
edge from parent
node[above] {0.35}
}
child {node {$\overline{D}$}
edge from parent
node[above] {0.65}
}
edge from parent
node[above] {0.66}
}
child[missing] {}
child { node {$B$}
child {node {$D$}
edge from parent
node[above] {0.81}
}
child {node {$\overline{D}$}
edge from parent
node[above] {0.19}
}
edge from parent
node[above] {0.34}
} ;
\end{tikzpicture}
\end{center}
\item
\begin{itemize}
\item Probabilité que le stylo vienne de l'atelier A
\[
P(A) = 0.66
\]
\item Probabilité que le stylo vienne de l'atelier B
\[
P(B) = 0.34
\]
\item Probabilité que le stylo ait un défaut sachant qu'il vient de l'atelier A.
\[
P_A(D) = 0.35
\]
\item Probabilité que le stylo vienne de l'atelier B et qu'il ait un défaut.
\[
P(D \cap D) = 0.28
\]
\end{itemize}
\item
\begin{enumerate}
\item Probabilité qu'un stylo vienne de l'atelier A et qu'il ait un defaut
\[
P(A\cap D) = P(A) \times P_A(D) = 0.66 \times 0.35 = 0.23
\]
\item Probabilité que le stylo ai un défaut de fabrication.
\[
P(D) = P(A\cap D) + P(B\cap D) = 0.23 + 0.28 = 0.51
\]
\end{enumerate}
\item Probabilité qu'il vienne de l'atelier A sachant qu'il a un defaut
\[
P_D(A) = \frac{P(A\cap D)}{P(D)} = \frac{0.23}{0.51} = 0.45
\]
\item $X$ peut être modélisée par une loi binomiale de paramètres $n=19$ et $p=0.51$.
\item (\textit{par de correction automatique disponible pour le résultat final}
\[
P(X = 11) = \coefBino{19}{11}\times 0.51^{11} \times 0.49^{8}
\]
\item (\textit{par de correction automatique disponible pour le résultat final}
Il faut calculer la probabilité qu'il y ait 0 stylo avec un defaut.
\[
P(X = 0) = \coefBino{19}{0}\times 0.51^{0} \times 0.49^{19}
\]
Puis comparer ce nombre à 0,5.
\item Il faut calculer l'espérance
\[
E[X] = n\times p = 19 \times 0.51 = 9.69
\]
\end{enumerate}
\end{solution}
\end{document}
%%% Local Variables:
%%% mode: latex
%%% TeX-master: "master"
%%% End:

View File

@ -0,0 +1,367 @@
\documentclass[a4paper,10pt]{article}
\usepackage{myXsim}
% Title Page
\title{DM1 \hfill POISON Lorette}
\tribe{Maths complémentaire}
\date{\hfillÀ render pour le jeudi 27 mai}
\xsimsetup{
solution/print = false
}
\begin{document}
\maketitle
Les valeurs des exercices sont générés automatiquement. Si une valeur a un nombre adhérant de chiffres après la virgule, vous pouvez l'arrondir à l'entier le plus proche.
\begin{exercise}[subtitle={Optimisation de matière}]
\begin{minipage}{0.6\textwidth}
On se propose de fabriquer avec le moins de tôle possible une citerne fermée en forme de parallélépipède rectangle dont le volume intérieur doit être de $35m^3$. La longueur est aussi fixée à $5m$ par le cahier des charges.
On peut donc faire varier uniquement la largeur (notée $x$) et la hauteur (notée $h$) de la cuve.
\end{minipage}
\hfill
\begin{minipage}{0.3\textwidth}
\begin{tikzpicture}
\pgfmathsetmacro{\cubex}{3}
\pgfmathsetmacro{\cubey}{1}
\pgfmathsetmacro{\cubez}{2}
\draw[black,fill=gray] (0,0,0) -- ++(-\cubex,0,0) -- ++(0,-\cubey,0) node [midway, left] {$h$} -- ++(\cubex,0,0) node [midway, below] {$x$} -- cycle;
\draw[black,fill=gray] (0,0,0) -- ++(0,0,-\cubez) -- ++(0,-\cubey,0) -- ++(0,0,\cubez) node [midway, right] {$5m$} -- cycle;
\draw[black,fill=gray] (0,0,0) -- ++(-\cubex,0,0) -- ++(0,0,-\cubez) -- ++(\cubex,0,0) -- cycle;
\end{tikzpicture}
\end{minipage}
\begin{enumerate}
\item Expliquer pourquoi quand la largeur $x$ change, la hauteur $h$ doit elle aussi changer pour respecter les contraintes.
\item Démontrer que l'on doit avoir $h = \dfrac{7}{x}$.
\item On note $S(x)$ l'aire totale de la citerne (c'est à dire la somme des aires des six faces). Montrer que l'on peut écrire
\[
S(x) = 10x + 14 + \frac{70}{x}
\]
\item Démontrer que
\[
S(x) = \frac{10x^2 + 14x + 70}{x}
\]
\item Démontrer que
\[
S'(x) = \frac{10x^2 - 70}{x^2}
\]
\item En déduire le tableau de variation de $S(x)$ sur $\intOF{0}{10}$.
\item Déterminer les valeurs de $x$ et $h$ correspondant à une utilisation minimal de tôle.
\end{enumerate}
\end{exercise}
\begin{solution}
\begin{enumerate}
\item Le volume étant fixe si l'on fait varier $x$, $h$ doit aussi varier.
\begin{itemize}
\item Si $x = 2$ alors conserver un volume de $V=35$, $h$ doit être égale à $7 / 2$
\item Si $x = 3$ alors conserver un volume de $V=35$, $h$ doit être égale à $7 / 3$
\end{itemize}
\item Pour calculer le volume, on a
\begin{eqnarray*}
V &=& h\times x \times 5 \\
35 &=& h\times x \times 5 \\
x &=& \frac{35}{h\times 5} = \frac{7}{h}
\end{eqnarray*}
\item Pour calculer la surface totale, on ajoute la surface de chaque face. On a donc le calcul suivant
\begin{eqnarray*}
S(x) &=& x\times h \times 2 + x\times5\times2 + h\times 5\times 2\\
S(x) &=& x\times \frac{7}{x} \times 2 + x\times5\times2 + \frac{7}{x}\times 5\times 2\\
S(x) &=& 10x + 14 + \frac{70}{x}
\end{eqnarray*}
\item Pour trouver cette nouvelle forme, on met chaque élément sur le même dénominateur
\begin{eqnarray*}
S(x) &=& 10x + 14 + \frac{70}{x}\\
S(x) &=& \frac{10x\times x}{x} + \frac{14\times x}{x} + \frac{70}{x}\\
S(x) &=& \frac{10x^2 + 14x + 70}{x}
\end{eqnarray*}
\item On retrouve la formule $\frac{u}{v}$ à dériver
\[
u(x) = 10x^2 + 14x + 70 \Rightarrow u'(x) = 20x + 14
\]
\[
v(x) = x \Rightarrow v'(x) = 1
\]
Donc au numérateur on obtient
\begin{eqnarray*}
u'(x)\times v(x) - u(x)\times v'(x) &=& (20x + 14)\times x - (10x^2 + 14x + 70)\times 1\\
&=& 10x^2 - 70
\end{eqnarray*}
Donc
\[
S'(x) = \frac{10x^2 - 70}{x^2}
\]
\item Tableau de variations de $S$
\begin{itemize}
\item Valeur interdite: $x^2 = 0 \equiv x = 0$
\item Signe de $10x^2 - 70$: c'est un polynôme du 2e degré
\[
\Delta = 2800 > 0
\]
Il y a donc 2 racines
\[
x_1 = - 2.6457513110645907 \qquad
x_2 = 2.6457513110645907
\]
Et on sait que $10x^2 - 70$ est du signe de $a$ donc positif en dehors des racines
\item Le dénominateur $x^2$ est toujours positif.
\item Tableau de variations
\begin{tikzpicture}[baseline=(a.north)]
\tkzTabInit[lgt=3,espcl=3]{$x$/1,$10x^2 - 70$/1, $x^2$/1, $S'$/1, $S$/2}{$0$, $- 2.6457513110645907$, $10$}
\tkzTabLine{d,-, z, +, }
\tkzTabLine{d,+, , +, }
\tkzTabLine{d,-, z, +, }
\tkzTabVar{D+/ , -/ , +/ }
\end{tikzpicture}
\end{itemize}
\item On a donc une surface minimal pour $x=2.6457513110645907$ et $h = 18.5202591774521349$.
\end{enumerate}
\end{solution}
%%% Local Variables:
%%% mode: latex
%%% TeX-master: "master"
%%% End:
\begin{exercise}[subtitle={Bassin}]
Le tour d'un bassin au niveau du sol présente deux axes de symétrie : laxe des abscisses et la droite déquation $x=4$. Il est obtenu par symétrie de la courbe $\mathcal{C}_f$ sur $\intFF{0}{4}$$f$ est la fonction définie par
\[
f(x) = \left(- x^{2} + 6.1 x - 3.4\right) e^{- x} + 3.4
\]
On admet que sur $\intFF{0}{4}$ la fonction $f$ est positive.
\begin{enumerate}
\item Sur un repère, tracer l'allure de la courbe $\mathcal{C}_f$, les axes de symétries puis compléter pour dessiner la forme du bassin.
\item Montrer que la fonction $f$ admet comme primitive sur $\R$ la fonction $F$ définie par
\[
F(x) = 3.4 x + \left( x^{2} - 4.1 x - 0.7\right) e^{- x}
\]
\item Calculer la quantité $\ds \int_0^4 f(x) \; dx$, vous donnerez le résultat sous forme exacte. Interpréter le résultat et reportez cette quantité sur le graphique.
\item On considère que l'échelle de votre graphique est de 1unité pour 15m. Calculer l'aire du bassin. Vous donnerez un résultat arrondi au $m^2$ près.
\end{enumerate}
\end{exercise}
\begin{solution}
\begin{enumerate}
\item
\begin{tikzpicture}[baseline=(a.north), xscale=1, yscale=0.5]
\tkzInit[xmin=0,xmax=5,xstep=1,
ymin=0,ymax=10,ystep=1]
\tkzGrid
\tkzAxeXY
\tkzFct[domain=0:10,color=red,very thick]%
{ (-x**2 + 6.1*x - 3.4)*exp(-x) + 3.4 };
\end{tikzpicture}
\item Il faut dériver $F(x)$ et vérifier que $F'(x) = f(x)$.
\item $\ds \int_0^4 f(x) \; dx = F(4) - F(0) = 14.3 - \frac{1.1}{e^{4}}$
\item La quantité calculée à la question précédente se retrouve 4fois pour former le bassin. Il faut ensuite prendre en compte l'échelle, comme 1unité de longueur correspond à 15m, une unité d'air correspond à $15\times15 = 225m^2$. Ainsi l'aire du bassin est égale à
\[
(14.3 - \frac{1.1}{e^{4}})\times 4 \times 15^2 = 12852.00000
\]
\end{enumerate}
\end{solution}
%%% Local Variables:
%%% mode: latex
%%% TeX-master: "master"
%%% End:
\begin{exercise}[subtitle={Stylos}]
\emph{Les parties {\rm A} et {\rm B} de cet exercice sont indépendantes.}
\bigskip
\begin{minipage}{0.6\linewidth}
\textbf{Partie A}
\medskip
Deux ateliers A et B fabriquent des stylos pour une entreprise.
L'atelier A fabrique 47.0\,\% des stylos, et parmi ceux-là, 95.0\,\% possèdent un défaut de fabrication.
De plus, 50.0\,\% des stylos possèdent un défaut de fabrication et sortent de l'atelier B.
Un stylo est prélevé au hasard dans le stock de l'entreprise.
On considère les évènements suivants:
\begin{itemize}
\item A : \og Le stylo a été fabriqué par l'atelier A \fg
\item B : \og Le stylo a été fabriqué par l'atelier B \fg
\item D : \og Le stylo possède un défaut de fabrication \fg
\end{itemize}
\end{minipage}
\begin{minipage}{0.4\linewidth}
\begin{center}
\begin{tikzpicture}[sloped]
\node {.}
child {node {$A$}
child {node {$D$}
edge from parent
node[above] {...}
}
child {node {$\overline{D}$}
edge from parent
node[above] {...}
}
edge from parent
node[above] {...}
}
child[missing] {}
child { node {$B$}
child {node {$D$}
edge from parent
node[above] {...}
}
child {node {$\overline{D}$}
edge from parent
node[above] {...}
}
edge from parent
node[above] {...}
} ;
\end{tikzpicture}
\end{center}
\end{minipage}
\medskip
\begin{enumerate}
\item Compléter l'arbre de probabilité ci-contre
\item Interpréter puis donner les probabilités $P(A)$, $P(B)$, $P_A(D)$ et $P(B \cap D)$.
\item
\begin{enumerate}
\item Calculer la probabilité qu'un stylo provienne de l'atelier A et possède un défaut de fabrication.
\item En déduire que la probabilité qu'un stylo possède un défaut de fabrication est de $0.95$.
\end{enumerate}
\item On prélève un stylo au hasard avec un défaut. Quelle est la probabilité qu'il vienne de l'atelier A?
\end{enumerate}
\bigskip
\textbf{Partie B}
\medskip
Dans cette partie, on suppose que 95.0\,\% des stylos possèdent un défaut de fabrication.
L'entreprise confectionne des paquets contenant chacun $4$~stylos.
Le fait qu'un stylo possède ou non un défaut de fabrication est indépendant des autres stylos.
On appelle $X$ la variable aléatoire donnant pour un paquet le nombre de stylos qui possèdent un défaut de fabrication.
On admet que la variable aléatoire $X$ suit une loi binomiale.
\medskip
\begin{enumerate}
\setcounter{enumi}{4}
\item Avec quelle loi peut-on modéliser $X$. Préciser les paramètres.
\item Calculer et interpréter la probabilité $P(X = 12)$.
\item Le directeur de l'entreprise affirme qu'il y a plus d'une chance sur deux qu'un paquet ne comporte aucun stylo défectueux. A-t-il raison ?
\item Combien de stylos peut-on espérer avoir en moyenne?
\end{enumerate}
\pagebreak
\end{exercise}
\begin{solution}
\begin{enumerate}
\item
\begin{center}
\begin{tikzpicture}[sloped]
\node {.}
child {node {$A$}
child {node {$D$}
edge from parent
node[above] {0.95}
}
child {node {$\overline{D}$}
edge from parent
node[above] {0.05}
}
edge from parent
node[above] {0.47}
}
child[missing] {}
child { node {$B$}
child {node {$D$}
edge from parent
node[above] {0.95}
}
child {node {$\overline{D}$}
edge from parent
node[above] {0.05}
}
edge from parent
node[above] {0.53}
} ;
\end{tikzpicture}
\end{center}
\item
\begin{itemize}
\item Probabilité que le stylo vienne de l'atelier A
\[
P(A) = 0.47
\]
\item Probabilité que le stylo vienne de l'atelier B
\[
P(B) = 0.53
\]
\item Probabilité que le stylo ait un défaut sachant qu'il vient de l'atelier A.
\[
P_A(D) = 0.95
\]
\item Probabilité que le stylo vienne de l'atelier B et qu'il ait un défaut.
\[
P(D \cap D) = 0.5
\]
\end{itemize}
\item
\begin{enumerate}
\item Probabilité qu'un stylo vienne de l'atelier A et qu'il ait un defaut
\[
P(A\cap D) = P(A) \times P_A(D) = 0.47 \times 0.95 = 0.45
\]
\item Probabilité que le stylo ai un défaut de fabrication.
\[
P(D) = P(A\cap D) + P(B\cap D) = 0.45 + 0.5 = 0.95
\]
\end{enumerate}
\item Probabilité qu'il vienne de l'atelier A sachant qu'il a un defaut
\[
P_D(A) = \frac{P(A\cap D)}{P(D)} = \frac{0.45}{0.95} = 0.47
\]
\item $X$ peut être modélisée par une loi binomiale de paramètres $n=15$ et $p=0.95$.
\item (\textit{par de correction automatique disponible pour le résultat final}
\[
P(X = 12) = \coefBino{15}{12}\times 0.95^{12} \times 0.05^{3}
\]
\item (\textit{par de correction automatique disponible pour le résultat final}
Il faut calculer la probabilité qu'il y ait 0 stylo avec un defaut.
\[
P(X = 0) = \coefBino{15}{0}\times 0.95^{0} \times 0.05^{15}
\]
Puis comparer ce nombre à 0,5.
\item Il faut calculer l'espérance
\[
E[X] = n\times p = 15 \times 0.95 = 14.25
\]
\end{enumerate}
\end{solution}
\end{document}
%%% Local Variables:
%%% mode: latex
%%% TeX-master: "master"
%%% End:

View File

@ -0,0 +1,367 @@
\documentclass[a4paper,10pt]{article}
\usepackage{myXsim}
% Title Page
\title{DM1 \hfill RODRIGUEZ Teddy}
\tribe{Maths complémentaire}
\date{\hfillÀ render pour le jeudi 27 mai}
\xsimsetup{
solution/print = false
}
\begin{document}
\maketitle
Les valeurs des exercices sont générés automatiquement. Si une valeur a un nombre adhérant de chiffres après la virgule, vous pouvez l'arrondir à l'entier le plus proche.
\begin{exercise}[subtitle={Optimisation de matière}]
\begin{minipage}{0.6\textwidth}
On se propose de fabriquer avec le moins de tôle possible une citerne fermée en forme de parallélépipède rectangle dont le volume intérieur doit être de $50m^3$. La longueur est aussi fixée à $5m$ par le cahier des charges.
On peut donc faire varier uniquement la largeur (notée $x$) et la hauteur (notée $h$) de la cuve.
\end{minipage}
\hfill
\begin{minipage}{0.3\textwidth}
\begin{tikzpicture}
\pgfmathsetmacro{\cubex}{3}
\pgfmathsetmacro{\cubey}{1}
\pgfmathsetmacro{\cubez}{2}
\draw[black,fill=gray] (0,0,0) -- ++(-\cubex,0,0) -- ++(0,-\cubey,0) node [midway, left] {$h$} -- ++(\cubex,0,0) node [midway, below] {$x$} -- cycle;
\draw[black,fill=gray] (0,0,0) -- ++(0,0,-\cubez) -- ++(0,-\cubey,0) -- ++(0,0,\cubez) node [midway, right] {$5m$} -- cycle;
\draw[black,fill=gray] (0,0,0) -- ++(-\cubex,0,0) -- ++(0,0,-\cubez) -- ++(\cubex,0,0) -- cycle;
\end{tikzpicture}
\end{minipage}
\begin{enumerate}
\item Expliquer pourquoi quand la largeur $x$ change, la hauteur $h$ doit elle aussi changer pour respecter les contraintes.
\item Démontrer que l'on doit avoir $h = \dfrac{10}{x}$.
\item On note $S(x)$ l'aire totale de la citerne (c'est à dire la somme des aires des six faces). Montrer que l'on peut écrire
\[
S(x) = 10x + 20 + \frac{100}{x}
\]
\item Démontrer que
\[
S(x) = \frac{10x^2 + 20x + 100}{x}
\]
\item Démontrer que
\[
S'(x) = \frac{10x^2 - 100}{x^2}
\]
\item En déduire le tableau de variation de $S(x)$ sur $\intOF{0}{10}$.
\item Déterminer les valeurs de $x$ et $h$ correspondant à une utilisation minimal de tôle.
\end{enumerate}
\end{exercise}
\begin{solution}
\begin{enumerate}
\item Le volume étant fixe si l'on fait varier $x$, $h$ doit aussi varier.
\begin{itemize}
\item Si $x = 2$ alors conserver un volume de $V=50$, $h$ doit être égale à $10 / 2$
\item Si $x = 3$ alors conserver un volume de $V=50$, $h$ doit être égale à $10 / 3$
\end{itemize}
\item Pour calculer le volume, on a
\begin{eqnarray*}
V &=& h\times x \times 5 \\
50 &=& h\times x \times 5 \\
x &=& \frac{50}{h\times 5} = \frac{10}{h}
\end{eqnarray*}
\item Pour calculer la surface totale, on ajoute la surface de chaque face. On a donc le calcul suivant
\begin{eqnarray*}
S(x) &=& x\times h \times 2 + x\times5\times2 + h\times 5\times 2\\
S(x) &=& x\times \frac{10}{x} \times 2 + x\times5\times2 + \frac{10}{x}\times 5\times 2\\
S(x) &=& 10x + 20 + \frac{100}{x}
\end{eqnarray*}
\item Pour trouver cette nouvelle forme, on met chaque élément sur le même dénominateur
\begin{eqnarray*}
S(x) &=& 10x + 20 + \frac{100}{x}\\
S(x) &=& \frac{10x\times x}{x} + \frac{20\times x}{x} + \frac{100}{x}\\
S(x) &=& \frac{10x^2 + 20x + 100}{x}
\end{eqnarray*}
\item On retrouve la formule $\frac{u}{v}$ à dériver
\[
u(x) = 10x^2 + 20x + 100 \Rightarrow u'(x) = 20x + 20
\]
\[
v(x) = x \Rightarrow v'(x) = 1
\]
Donc au numérateur on obtient
\begin{eqnarray*}
u'(x)\times v(x) - u(x)\times v'(x) &=& (20x + 20)\times x - (10x^2 + 20x + 100)\times 1\\
&=& 10x^2 - 100
\end{eqnarray*}
Donc
\[
S'(x) = \frac{10x^2 - 100}{x^2}
\]
\item Tableau de variations de $S$
\begin{itemize}
\item Valeur interdite: $x^2 = 0 \equiv x = 0$
\item Signe de $10x^2 - 100$: c'est un polynôme du 2e degré
\[
\Delta = 4000 > 0
\]
Il y a donc 2 racines
\[
x_1 = - 3.162277660168379 \qquad
x_2 = 3.162277660168379
\]
Et on sait que $10x^2 - 100$ est du signe de $a$ donc positif en dehors des racines
\item Le dénominateur $x^2$ est toujours positif.
\item Tableau de variations
\begin{tikzpicture}[baseline=(a.north)]
\tkzTabInit[lgt=3,espcl=3]{$x$/1,$10x^2 - 100$/1, $x^2$/1, $S'$/1, $S$/2}{$0$, $- 3.162277660168379$, $10$}
\tkzTabLine{d,-, z, +, }
\tkzTabLine{d,+, , +, }
\tkzTabLine{d,-, z, +, }
\tkzTabVar{D+/ , -/ , +/ }
\end{tikzpicture}
\end{itemize}
\item On a donc une surface minimal pour $x=3.162277660168379$ et $h = 31.622776601683790$.
\end{enumerate}
\end{solution}
%%% Local Variables:
%%% mode: latex
%%% TeX-master: "master"
%%% End:
\begin{exercise}[subtitle={Bassin}]
Le tour d'un bassin au niveau du sol présente deux axes de symétrie : laxe des abscisses et la droite déquation $x=4$. Il est obtenu par symétrie de la courbe $\mathcal{C}_f$ sur $\intFF{0}{4}$$f$ est la fonction définie par
\[
f(x) = \left(- x^{2} + 1.0 x - 3.6\right) e^{- x} + 3.6
\]
On admet que sur $\intFF{0}{4}$ la fonction $f$ est positive.
\begin{enumerate}
\item Sur un repère, tracer l'allure de la courbe $\mathcal{C}_f$, les axes de symétries puis compléter pour dessiner la forme du bassin.
\item Montrer que la fonction $f$ admet comme primitive sur $\R$ la fonction $F$ définie par
\[
F(x) = 3.6 x + \left( x^{2} + x + 4.6\right) e^{- x}
\]
\item Calculer la quantité $\ds \int_0^4 f(x) \; dx$, vous donnerez le résultat sous forme exacte. Interpréter le résultat et reportez cette quantité sur le graphique.
\item On considère que l'échelle de votre graphique est de 1unité pour 15m. Calculer l'aire du bassin. Vous donnerez un résultat arrondi au $m^2$ près.
\end{enumerate}
\end{exercise}
\begin{solution}
\begin{enumerate}
\item
\begin{tikzpicture}[baseline=(a.north), xscale=1, yscale=0.5]
\tkzInit[xmin=0,xmax=5,xstep=1,
ymin=0,ymax=10,ystep=1]
\tkzGrid
\tkzAxeXY
\tkzFct[domain=0:10,color=red,very thick]%
{ (-x**2 + 1.0*x - 3.6)*exp(-x) + 3.6 };
\end{tikzpicture}
\item Il faut dériver $F(x)$ et vérifier que $F'(x) = f(x)$.
\item $\ds \int_0^4 f(x) \; dx = F(4) - F(0) = \frac{24.6}{e^{4}} + 9.8$
\item La quantité calculée à la question précédente se retrouve 4fois pour former le bassin. Il faut ensuite prendre en compte l'échelle, comme 1unité de longueur correspond à 15m, une unité d'air correspond à $15\times15 = 225m^2$. Ainsi l'aire du bassin est égale à
\[
(\frac{24.6}{e^{4}} + 9.8)\times 4 \times 15^2 = 9226.000000
\]
\end{enumerate}
\end{solution}
%%% Local Variables:
%%% mode: latex
%%% TeX-master: "master"
%%% End:
\begin{exercise}[subtitle={Stylos}]
\emph{Les parties {\rm A} et {\rm B} de cet exercice sont indépendantes.}
\bigskip
\begin{minipage}{0.6\linewidth}
\textbf{Partie A}
\medskip
Deux ateliers A et B fabriquent des stylos pour une entreprise.
L'atelier A fabrique 64.0\,\% des stylos, et parmi ceux-là, 4.0\,\% possèdent un défaut de fabrication.
De plus, 6.0\,\% des stylos possèdent un défaut de fabrication et sortent de l'atelier B.
Un stylo est prélevé au hasard dans le stock de l'entreprise.
On considère les évènements suivants:
\begin{itemize}
\item A : \og Le stylo a été fabriqué par l'atelier A \fg
\item B : \og Le stylo a été fabriqué par l'atelier B \fg
\item D : \og Le stylo possède un défaut de fabrication \fg
\end{itemize}
\end{minipage}
\begin{minipage}{0.4\linewidth}
\begin{center}
\begin{tikzpicture}[sloped]
\node {.}
child {node {$A$}
child {node {$D$}
edge from parent
node[above] {...}
}
child {node {$\overline{D}$}
edge from parent
node[above] {...}
}
edge from parent
node[above] {...}
}
child[missing] {}
child { node {$B$}
child {node {$D$}
edge from parent
node[above] {...}
}
child {node {$\overline{D}$}
edge from parent
node[above] {...}
}
edge from parent
node[above] {...}
} ;
\end{tikzpicture}
\end{center}
\end{minipage}
\medskip
\begin{enumerate}
\item Compléter l'arbre de probabilité ci-contre
\item Interpréter puis donner les probabilités $P(A)$, $P(B)$, $P_A(D)$ et $P(B \cap D)$.
\item
\begin{enumerate}
\item Calculer la probabilité qu'un stylo provienne de l'atelier A et possède un défaut de fabrication.
\item En déduire que la probabilité qu'un stylo possède un défaut de fabrication est de $0.09$.
\end{enumerate}
\item On prélève un stylo au hasard avec un défaut. Quelle est la probabilité qu'il vienne de l'atelier A?
\end{enumerate}
\bigskip
\textbf{Partie B}
\medskip
Dans cette partie, on suppose que 9.0\,\% des stylos possèdent un défaut de fabrication.
L'entreprise confectionne des paquets contenant chacun $4$~stylos.
Le fait qu'un stylo possède ou non un défaut de fabrication est indépendant des autres stylos.
On appelle $X$ la variable aléatoire donnant pour un paquet le nombre de stylos qui possèdent un défaut de fabrication.
On admet que la variable aléatoire $X$ suit une loi binomiale.
\medskip
\begin{enumerate}
\setcounter{enumi}{4}
\item Avec quelle loi peut-on modéliser $X$. Préciser les paramètres.
\item Calculer et interpréter la probabilité $P(X = 9)$.
\item Le directeur de l'entreprise affirme qu'il y a plus d'une chance sur deux qu'un paquet ne comporte aucun stylo défectueux. A-t-il raison ?
\item Combien de stylos peut-on espérer avoir en moyenne?
\end{enumerate}
\pagebreak
\end{exercise}
\begin{solution}
\begin{enumerate}
\item
\begin{center}
\begin{tikzpicture}[sloped]
\node {.}
child {node {$A$}
child {node {$D$}
edge from parent
node[above] {0.04}
}
child {node {$\overline{D}$}
edge from parent
node[above] {0.96}
}
edge from parent
node[above] {0.64}
}
child[missing] {}
child { node {$B$}
child {node {$D$}
edge from parent
node[above] {0.18}
}
child {node {$\overline{D}$}
edge from parent
node[above] {0.82}
}
edge from parent
node[above] {0.36}
} ;
\end{tikzpicture}
\end{center}
\item
\begin{itemize}
\item Probabilité que le stylo vienne de l'atelier A
\[
P(A) = 0.64
\]
\item Probabilité que le stylo vienne de l'atelier B
\[
P(B) = 0.36
\]
\item Probabilité que le stylo ait un défaut sachant qu'il vient de l'atelier A.
\[
P_A(D) = 0.04
\]
\item Probabilité que le stylo vienne de l'atelier B et qu'il ait un défaut.
\[
P(D \cap D) = 0.06
\]
\end{itemize}
\item
\begin{enumerate}
\item Probabilité qu'un stylo vienne de l'atelier A et qu'il ait un defaut
\[
P(A\cap D) = P(A) \times P_A(D) = 0.64 \times 0.04 = 0.03
\]
\item Probabilité que le stylo ai un défaut de fabrication.
\[
P(D) = P(A\cap D) + P(B\cap D) = 0.03 + 0.06 = 0.09
\]
\end{enumerate}
\item Probabilité qu'il vienne de l'atelier A sachant qu'il a un defaut
\[
P_D(A) = \frac{P(A\cap D)}{P(D)} = \frac{0.03}{0.09} = 0.33
\]
\item $X$ peut être modélisée par une loi binomiale de paramètres $n=16$ et $p=0.09$.
\item (\textit{par de correction automatique disponible pour le résultat final}
\[
P(X = 9) = \coefBino{16}{9}\times 0.09^{9} \times 0.91^{7}
\]
\item (\textit{par de correction automatique disponible pour le résultat final}
Il faut calculer la probabilité qu'il y ait 0 stylo avec un defaut.
\[
P(X = 0) = \coefBino{16}{0}\times 0.09^{0} \times 0.91^{16}
\]
Puis comparer ce nombre à 0,5.
\item Il faut calculer l'espérance
\[
E[X] = n\times p = 16 \times 0.09 = 1.44
\]
\end{enumerate}
\end{solution}
\end{document}
%%% Local Variables:
%%% mode: latex
%%% TeX-master: "master"
%%% End:

View File

@ -0,0 +1,367 @@
\documentclass[a4paper,10pt]{article}
\usepackage{myXsim}
% Title Page
\title{DM1 \hfill SAINT CYR Louis}
\tribe{Maths complémentaire}
\date{\hfillÀ render pour le jeudi 27 mai}
\xsimsetup{
solution/print = false
}
\begin{document}
\maketitle
Les valeurs des exercices sont générés automatiquement. Si une valeur a un nombre adhérant de chiffres après la virgule, vous pouvez l'arrondir à l'entier le plus proche.
\begin{exercise}[subtitle={Optimisation de matière}]
\begin{minipage}{0.6\textwidth}
On se propose de fabriquer avec le moins de tôle possible une citerne fermée en forme de parallélépipède rectangle dont le volume intérieur doit être de $45m^3$. La longueur est aussi fixée à $5m$ par le cahier des charges.
On peut donc faire varier uniquement la largeur (notée $x$) et la hauteur (notée $h$) de la cuve.
\end{minipage}
\hfill
\begin{minipage}{0.3\textwidth}
\begin{tikzpicture}
\pgfmathsetmacro{\cubex}{3}
\pgfmathsetmacro{\cubey}{1}
\pgfmathsetmacro{\cubez}{2}
\draw[black,fill=gray] (0,0,0) -- ++(-\cubex,0,0) -- ++(0,-\cubey,0) node [midway, left] {$h$} -- ++(\cubex,0,0) node [midway, below] {$x$} -- cycle;
\draw[black,fill=gray] (0,0,0) -- ++(0,0,-\cubez) -- ++(0,-\cubey,0) -- ++(0,0,\cubez) node [midway, right] {$5m$} -- cycle;
\draw[black,fill=gray] (0,0,0) -- ++(-\cubex,0,0) -- ++(0,0,-\cubez) -- ++(\cubex,0,0) -- cycle;
\end{tikzpicture}
\end{minipage}
\begin{enumerate}
\item Expliquer pourquoi quand la largeur $x$ change, la hauteur $h$ doit elle aussi changer pour respecter les contraintes.
\item Démontrer que l'on doit avoir $h = \dfrac{9}{x}$.
\item On note $S(x)$ l'aire totale de la citerne (c'est à dire la somme des aires des six faces). Montrer que l'on peut écrire
\[
S(x) = 10x + 18 + \frac{90}{x}
\]
\item Démontrer que
\[
S(x) = \frac{10x^2 + 18x + 90}{x}
\]
\item Démontrer que
\[
S'(x) = \frac{10x^2 - 90}{x^2}
\]
\item En déduire le tableau de variation de $S(x)$ sur $\intOF{0}{10}$.
\item Déterminer les valeurs de $x$ et $h$ correspondant à une utilisation minimal de tôle.
\end{enumerate}
\end{exercise}
\begin{solution}
\begin{enumerate}
\item Le volume étant fixe si l'on fait varier $x$, $h$ doit aussi varier.
\begin{itemize}
\item Si $x = 2$ alors conserver un volume de $V=45$, $h$ doit être égale à $9 / 2$
\item Si $x = 3$ alors conserver un volume de $V=45$, $h$ doit être égale à $9 / 3$
\end{itemize}
\item Pour calculer le volume, on a
\begin{eqnarray*}
V &=& h\times x \times 5 \\
45 &=& h\times x \times 5 \\
x &=& \frac{45}{h\times 5} = \frac{9}{h}
\end{eqnarray*}
\item Pour calculer la surface totale, on ajoute la surface de chaque face. On a donc le calcul suivant
\begin{eqnarray*}
S(x) &=& x\times h \times 2 + x\times5\times2 + h\times 5\times 2\\
S(x) &=& x\times \frac{9}{x} \times 2 + x\times5\times2 + \frac{9}{x}\times 5\times 2\\
S(x) &=& 10x + 18 + \frac{90}{x}
\end{eqnarray*}
\item Pour trouver cette nouvelle forme, on met chaque élément sur le même dénominateur
\begin{eqnarray*}
S(x) &=& 10x + 18 + \frac{90}{x}\\
S(x) &=& \frac{10x\times x}{x} + \frac{18\times x}{x} + \frac{90}{x}\\
S(x) &=& \frac{10x^2 + 18x + 90}{x}
\end{eqnarray*}
\item On retrouve la formule $\frac{u}{v}$ à dériver
\[
u(x) = 10x^2 + 18x + 90 \Rightarrow u'(x) = 20x + 18
\]
\[
v(x) = x \Rightarrow v'(x) = 1
\]
Donc au numérateur on obtient
\begin{eqnarray*}
u'(x)\times v(x) - u(x)\times v'(x) &=& (20x + 18)\times x - (10x^2 + 18x + 90)\times 1\\
&=& 10x^2 - 90
\end{eqnarray*}
Donc
\[
S'(x) = \frac{10x^2 - 90}{x^2}
\]
\item Tableau de variations de $S$
\begin{itemize}
\item Valeur interdite: $x^2 = 0 \equiv x = 0$
\item Signe de $10x^2 - 90$: c'est un polynôme du 2e degré
\[
\Delta = 3600 > 0
\]
Il y a donc 2 racines
\[
x_1 = - 3 \qquad
x_2 = 3
\]
Et on sait que $10x^2 - 90$ est du signe de $a$ donc positif en dehors des racines
\item Le dénominateur $x^2$ est toujours positif.
\item Tableau de variations
\begin{tikzpicture}[baseline=(a.north)]
\tkzTabInit[lgt=3,espcl=3]{$x$/1,$10x^2 - 90$/1, $x^2$/1, $S'$/1, $S$/2}{$0$, $- 3$, $10$}
\tkzTabLine{d,-, z, +, }
\tkzTabLine{d,+, , +, }
\tkzTabLine{d,-, z, +, }
\tkzTabVar{D+/ , -/ , +/ }
\end{tikzpicture}
\end{itemize}
\item On a donc une surface minimal pour $x=3$ et $h = 27$.
\end{enumerate}
\end{solution}
%%% Local Variables:
%%% mode: latex
%%% TeX-master: "master"
%%% End:
\begin{exercise}[subtitle={Bassin}]
Le tour d'un bassin au niveau du sol présente deux axes de symétrie : laxe des abscisses et la droite déquation $x=4$. Il est obtenu par symétrie de la courbe $\mathcal{C}_f$ sur $\intFF{0}{4}$$f$ est la fonction définie par
\[
f(x) = \left(- x^{2} + 8.0 x - 0.9\right) e^{- x} + 0.9
\]
On admet que sur $\intFF{0}{4}$ la fonction $f$ est positive.
\begin{enumerate}
\item Sur un repère, tracer l'allure de la courbe $\mathcal{C}_f$, les axes de symétries puis compléter pour dessiner la forme du bassin.
\item Montrer que la fonction $f$ admet comme primitive sur $\R$ la fonction $F$ définie par
\[
F(x) = 0.9 x + \left( x^{2} - 6.0 x - 5.1\right) e^{- x}
\]
\item Calculer la quantité $\ds \int_0^4 f(x) \; dx$, vous donnerez le résultat sous forme exacte. Interpréter le résultat et reportez cette quantité sur le graphique.
\item On considère que l'échelle de votre graphique est de 1unité pour 15m. Calculer l'aire du bassin. Vous donnerez un résultat arrondi au $m^2$ près.
\end{enumerate}
\end{exercise}
\begin{solution}
\begin{enumerate}
\item
\begin{tikzpicture}[baseline=(a.north), xscale=1, yscale=0.5]
\tkzInit[xmin=0,xmax=5,xstep=1,
ymin=0,ymax=10,ystep=1]
\tkzGrid
\tkzAxeXY
\tkzFct[domain=0:10,color=red,very thick]%
{ (-x**2 + 8.0*x - 0.9)*exp(-x) + 0.9 };
\end{tikzpicture}
\item Il faut dériver $F(x)$ et vérifier que $F'(x) = f(x)$.
\item $\ds \int_0^4 f(x) \; dx = F(4) - F(0) = 8.7 - \frac{13.1}{e^{4}}$
\item La quantité calculée à la question précédente se retrouve 4fois pour former le bassin. Il faut ensuite prendre en compte l'échelle, comme 1unité de longueur correspond à 15m, une unité d'air correspond à $15\times15 = 225m^2$. Ainsi l'aire du bassin est égale à
\[
(8.7 - \frac{13.1}{e^{4}})\times 4 \times 15^2 = 7614.000000
\]
\end{enumerate}
\end{solution}
%%% Local Variables:
%%% mode: latex
%%% TeX-master: "master"
%%% End:
\begin{exercise}[subtitle={Stylos}]
\emph{Les parties {\rm A} et {\rm B} de cet exercice sont indépendantes.}
\bigskip
\begin{minipage}{0.6\linewidth}
\textbf{Partie A}
\medskip
Deux ateliers A et B fabriquent des stylos pour une entreprise.
L'atelier A fabrique 88.0\,\% des stylos, et parmi ceux-là, 75.0\,\% possèdent un défaut de fabrication.
De plus, 4.0\,\% des stylos possèdent un défaut de fabrication et sortent de l'atelier B.
Un stylo est prélevé au hasard dans le stock de l'entreprise.
On considère les évènements suivants:
\begin{itemize}
\item A : \og Le stylo a été fabriqué par l'atelier A \fg
\item B : \og Le stylo a été fabriqué par l'atelier B \fg
\item D : \og Le stylo possède un défaut de fabrication \fg
\end{itemize}
\end{minipage}
\begin{minipage}{0.4\linewidth}
\begin{center}
\begin{tikzpicture}[sloped]
\node {.}
child {node {$A$}
child {node {$D$}
edge from parent
node[above] {...}
}
child {node {$\overline{D}$}
edge from parent
node[above] {...}
}
edge from parent
node[above] {...}
}
child[missing] {}
child { node {$B$}
child {node {$D$}
edge from parent
node[above] {...}
}
child {node {$\overline{D}$}
edge from parent
node[above] {...}
}
edge from parent
node[above] {...}
} ;
\end{tikzpicture}
\end{center}
\end{minipage}
\medskip
\begin{enumerate}
\item Compléter l'arbre de probabilité ci-contre
\item Interpréter puis donner les probabilités $P(A)$, $P(B)$, $P_A(D)$ et $P(B \cap D)$.
\item
\begin{enumerate}
\item Calculer la probabilité qu'un stylo provienne de l'atelier A et possède un défaut de fabrication.
\item En déduire que la probabilité qu'un stylo possède un défaut de fabrication est de $0.7$.
\end{enumerate}
\item On prélève un stylo au hasard avec un défaut. Quelle est la probabilité qu'il vienne de l'atelier A?
\end{enumerate}
\bigskip
\textbf{Partie B}
\medskip
Dans cette partie, on suppose que 70.0\,\% des stylos possèdent un défaut de fabrication.
L'entreprise confectionne des paquets contenant chacun $4$~stylos.
Le fait qu'un stylo possède ou non un défaut de fabrication est indépendant des autres stylos.
On appelle $X$ la variable aléatoire donnant pour un paquet le nombre de stylos qui possèdent un défaut de fabrication.
On admet que la variable aléatoire $X$ suit une loi binomiale.
\medskip
\begin{enumerate}
\setcounter{enumi}{4}
\item Avec quelle loi peut-on modéliser $X$. Préciser les paramètres.
\item Calculer et interpréter la probabilité $P(X = 14)$.
\item Le directeur de l'entreprise affirme qu'il y a plus d'une chance sur deux qu'un paquet ne comporte aucun stylo défectueux. A-t-il raison ?
\item Combien de stylos peut-on espérer avoir en moyenne?
\end{enumerate}
\pagebreak
\end{exercise}
\begin{solution}
\begin{enumerate}
\item
\begin{center}
\begin{tikzpicture}[sloped]
\node {.}
child {node {$A$}
child {node {$D$}
edge from parent
node[above] {0.75}
}
child {node {$\overline{D}$}
edge from parent
node[above] {0.25}
}
edge from parent
node[above] {0.88}
}
child[missing] {}
child { node {$B$}
child {node {$D$}
edge from parent
node[above] {0.34}
}
child {node {$\overline{D}$}
edge from parent
node[above] {0.66}
}
edge from parent
node[above] {0.12}
} ;
\end{tikzpicture}
\end{center}
\item
\begin{itemize}
\item Probabilité que le stylo vienne de l'atelier A
\[
P(A) = 0.88
\]
\item Probabilité que le stylo vienne de l'atelier B
\[
P(B) = 0.12
\]
\item Probabilité que le stylo ait un défaut sachant qu'il vient de l'atelier A.
\[
P_A(D) = 0.75
\]
\item Probabilité que le stylo vienne de l'atelier B et qu'il ait un défaut.
\[
P(D \cap D) = 0.04
\]
\end{itemize}
\item
\begin{enumerate}
\item Probabilité qu'un stylo vienne de l'atelier A et qu'il ait un defaut
\[
P(A\cap D) = P(A) \times P_A(D) = 0.88 \times 0.75 = 0.66
\]
\item Probabilité que le stylo ai un défaut de fabrication.
\[
P(D) = P(A\cap D) + P(B\cap D) = 0.66 + 0.04 = 0.7
\]
\end{enumerate}
\item Probabilité qu'il vienne de l'atelier A sachant qu'il a un defaut
\[
P_D(A) = \frac{P(A\cap D)}{P(D)} = \frac{0.66}{0.7} = 0.94
\]
\item $X$ peut être modélisée par une loi binomiale de paramètres $n=15$ et $p=0.7$.
\item (\textit{par de correction automatique disponible pour le résultat final}
\[
P(X = 14) = \coefBino{15}{14}\times 0.7^{14} \times 0.3^{1}
\]
\item (\textit{par de correction automatique disponible pour le résultat final}
Il faut calculer la probabilité qu'il y ait 0 stylo avec un defaut.
\[
P(X = 0) = \coefBino{15}{0}\times 0.7^{0} \times 0.3^{15}
\]
Puis comparer ce nombre à 0,5.
\item Il faut calculer l'espérance
\[
E[X] = n\times p = 15 \times 0.7 = 10.5
\]
\end{enumerate}
\end{solution}
\end{document}
%%% Local Variables:
%%% mode: latex
%%% TeX-master: "master"
%%% End:

View File

@ -0,0 +1,367 @@
\documentclass[a4paper,10pt]{article}
\usepackage{myXsim}
% Title Page
\title{DM1 \hfill SAVIN Lou-Ann}
\tribe{Maths complémentaire}
\date{\hfillÀ render pour le jeudi 27 mai}
\xsimsetup{
solution/print = false
}
\begin{document}
\maketitle
Les valeurs des exercices sont générés automatiquement. Si une valeur a un nombre adhérant de chiffres après la virgule, vous pouvez l'arrondir à l'entier le plus proche.
\begin{exercise}[subtitle={Optimisation de matière}]
\begin{minipage}{0.6\textwidth}
On se propose de fabriquer avec le moins de tôle possible une citerne fermée en forme de parallélépipède rectangle dont le volume intérieur doit être de $18m^3$. La longueur est aussi fixée à $3m$ par le cahier des charges.
On peut donc faire varier uniquement la largeur (notée $x$) et la hauteur (notée $h$) de la cuve.
\end{minipage}
\hfill
\begin{minipage}{0.3\textwidth}
\begin{tikzpicture}
\pgfmathsetmacro{\cubex}{3}
\pgfmathsetmacro{\cubey}{1}
\pgfmathsetmacro{\cubez}{2}
\draw[black,fill=gray] (0,0,0) -- ++(-\cubex,0,0) -- ++(0,-\cubey,0) node [midway, left] {$h$} -- ++(\cubex,0,0) node [midway, below] {$x$} -- cycle;
\draw[black,fill=gray] (0,0,0) -- ++(0,0,-\cubez) -- ++(0,-\cubey,0) -- ++(0,0,\cubez) node [midway, right] {$3m$} -- cycle;
\draw[black,fill=gray] (0,0,0) -- ++(-\cubex,0,0) -- ++(0,0,-\cubez) -- ++(\cubex,0,0) -- cycle;
\end{tikzpicture}
\end{minipage}
\begin{enumerate}
\item Expliquer pourquoi quand la largeur $x$ change, la hauteur $h$ doit elle aussi changer pour respecter les contraintes.
\item Démontrer que l'on doit avoir $h = \dfrac{6}{x}$.
\item On note $S(x)$ l'aire totale de la citerne (c'est à dire la somme des aires des six faces). Montrer que l'on peut écrire
\[
S(x) = 6x + 12 + \frac{36}{x}
\]
\item Démontrer que
\[
S(x) = \frac{6x^2 + 12x + 36}{x}
\]
\item Démontrer que
\[
S'(x) = \frac{6x^2 - 36}{x^2}
\]
\item En déduire le tableau de variation de $S(x)$ sur $\intOF{0}{10}$.
\item Déterminer les valeurs de $x$ et $h$ correspondant à une utilisation minimal de tôle.
\end{enumerate}
\end{exercise}
\begin{solution}
\begin{enumerate}
\item Le volume étant fixe si l'on fait varier $x$, $h$ doit aussi varier.
\begin{itemize}
\item Si $x = 2$ alors conserver un volume de $V=18$, $h$ doit être égale à $6 / 2$
\item Si $x = 3$ alors conserver un volume de $V=18$, $h$ doit être égale à $6 / 3$
\end{itemize}
\item Pour calculer le volume, on a
\begin{eqnarray*}
V &=& h\times x \times 3 \\
18 &=& h\times x \times 3 \\
x &=& \frac{18}{h\times 3} = \frac{6}{h}
\end{eqnarray*}
\item Pour calculer la surface totale, on ajoute la surface de chaque face. On a donc le calcul suivant
\begin{eqnarray*}
S(x) &=& x\times h \times 2 + x\times3\times2 + h\times 3\times 2\\
S(x) &=& x\times \frac{6}{x} \times 2 + x\times3\times2 + \frac{6}{x}\times 3\times 2\\
S(x) &=& 6x + 12 + \frac{36}{x}
\end{eqnarray*}
\item Pour trouver cette nouvelle forme, on met chaque élément sur le même dénominateur
\begin{eqnarray*}
S(x) &=& 6x + 12 + \frac{36}{x}\\
S(x) &=& \frac{6x\times x}{x} + \frac{12\times x}{x} + \frac{36}{x}\\
S(x) &=& \frac{6x^2 + 12x + 36}{x}
\end{eqnarray*}
\item On retrouve la formule $\frac{u}{v}$ à dériver
\[
u(x) = 6x^2 + 12x + 36 \Rightarrow u'(x) = 12x + 12
\]
\[
v(x) = x \Rightarrow v'(x) = 1
\]
Donc au numérateur on obtient
\begin{eqnarray*}
u'(x)\times v(x) - u(x)\times v'(x) &=& (12x + 12)\times x - (6x^2 + 12x + 36)\times 1\\
&=& 6x^2 - 36
\end{eqnarray*}
Donc
\[
S'(x) = \frac{6x^2 - 36}{x^2}
\]
\item Tableau de variations de $S$
\begin{itemize}
\item Valeur interdite: $x^2 = 0 \equiv x = 0$
\item Signe de $6x^2 - 36$: c'est un polynôme du 2e degré
\[
\Delta = 864 > 0
\]
Il y a donc 2 racines
\[
x_1 = - 2.4494897427831783 \qquad
x_2 = 2.4494897427831783
\]
Et on sait que $6x^2 - 36$ est du signe de $a$ donc positif en dehors des racines
\item Le dénominateur $x^2$ est toujours positif.
\item Tableau de variations
\begin{tikzpicture}[baseline=(a.north)]
\tkzTabInit[lgt=3,espcl=3]{$x$/1,$6x^2 - 36$/1, $x^2$/1, $S'$/1, $S$/2}{$0$, $- 2.4494897427831783$, $10$}
\tkzTabLine{d,-, z, +, }
\tkzTabLine{d,+, , +, }
\tkzTabLine{d,-, z, +, }
\tkzTabVar{D+/ , -/ , +/ }
\end{tikzpicture}
\end{itemize}
\item On a donc une surface minimal pour $x=2.4494897427831783$ et $h = 14.6969384566990698$.
\end{enumerate}
\end{solution}
%%% Local Variables:
%%% mode: latex
%%% TeX-master: "master"
%%% End:
\begin{exercise}[subtitle={Bassin}]
Le tour d'un bassin au niveau du sol présente deux axes de symétrie : laxe des abscisses et la droite déquation $x=4$. Il est obtenu par symétrie de la courbe $\mathcal{C}_f$ sur $\intFF{0}{4}$$f$ est la fonction définie par
\[
f(x) = \left(- x^{2} + 2.7 x - 6.1\right) e^{- x} + 6.1
\]
On admet que sur $\intFF{0}{4}$ la fonction $f$ est positive.
\begin{enumerate}
\item Sur un repère, tracer l'allure de la courbe $\mathcal{C}_f$, les axes de symétries puis compléter pour dessiner la forme du bassin.
\item Montrer que la fonction $f$ admet comme primitive sur $\R$ la fonction $F$ définie par
\[
F(x) = 6.1 x + \left( x^{2} - 0.7 x + 5.4\right) e^{- x}
\]
\item Calculer la quantité $\ds \int_0^4 f(x) \; dx$, vous donnerez le résultat sous forme exacte. Interpréter le résultat et reportez cette quantité sur le graphique.
\item On considère que l'échelle de votre graphique est de 1unité pour 15m. Calculer l'aire du bassin. Vous donnerez un résultat arrondi au $m^2$ près.
\end{enumerate}
\end{exercise}
\begin{solution}
\begin{enumerate}
\item
\begin{tikzpicture}[baseline=(a.north), xscale=1, yscale=0.5]
\tkzInit[xmin=0,xmax=5,xstep=1,
ymin=0,ymax=10,ystep=1]
\tkzGrid
\tkzAxeXY
\tkzFct[domain=0:10,color=red,very thick]%
{ (-x**2 + 2.7*x - 6.1)*exp(-x) + 6.1 };
\end{tikzpicture}
\item Il faut dériver $F(x)$ et vérifier que $F'(x) = f(x)$.
\item $\ds \int_0^4 f(x) \; dx = F(4) - F(0) = \frac{18.6}{e^{4}} + 19.0$
\item La quantité calculée à la question précédente se retrouve 4fois pour former le bassin. Il faut ensuite prendre en compte l'échelle, comme 1unité de longueur correspond à 15m, une unité d'air correspond à $15\times15 = 225m^2$. Ainsi l'aire du bassin est égale à
\[
(\frac{18.6}{e^{4}} + 19.0)\times 4 \times 15^2 = 17407.00000
\]
\end{enumerate}
\end{solution}
%%% Local Variables:
%%% mode: latex
%%% TeX-master: "master"
%%% End:
\begin{exercise}[subtitle={Stylos}]
\emph{Les parties {\rm A} et {\rm B} de cet exercice sont indépendantes.}
\bigskip
\begin{minipage}{0.6\linewidth}
\textbf{Partie A}
\medskip
Deux ateliers A et B fabriquent des stylos pour une entreprise.
L'atelier A fabrique 47.0\,\% des stylos, et parmi ceux-là, 45.0\,\% possèdent un défaut de fabrication.
De plus, 3.0\,\% des stylos possèdent un défaut de fabrication et sortent de l'atelier B.
Un stylo est prélevé au hasard dans le stock de l'entreprise.
On considère les évènements suivants:
\begin{itemize}
\item A : \og Le stylo a été fabriqué par l'atelier A \fg
\item B : \og Le stylo a été fabriqué par l'atelier B \fg
\item D : \og Le stylo possède un défaut de fabrication \fg
\end{itemize}
\end{minipage}
\begin{minipage}{0.4\linewidth}
\begin{center}
\begin{tikzpicture}[sloped]
\node {.}
child {node {$A$}
child {node {$D$}
edge from parent
node[above] {...}
}
child {node {$\overline{D}$}
edge from parent
node[above] {...}
}
edge from parent
node[above] {...}
}
child[missing] {}
child { node {$B$}
child {node {$D$}
edge from parent
node[above] {...}
}
child {node {$\overline{D}$}
edge from parent
node[above] {...}
}
edge from parent
node[above] {...}
} ;
\end{tikzpicture}
\end{center}
\end{minipage}
\medskip
\begin{enumerate}
\item Compléter l'arbre de probabilité ci-contre
\item Interpréter puis donner les probabilités $P(A)$, $P(B)$, $P_A(D)$ et $P(B \cap D)$.
\item
\begin{enumerate}
\item Calculer la probabilité qu'un stylo provienne de l'atelier A et possède un défaut de fabrication.
\item En déduire que la probabilité qu'un stylo possède un défaut de fabrication est de $0.24$.
\end{enumerate}
\item On prélève un stylo au hasard avec un défaut. Quelle est la probabilité qu'il vienne de l'atelier A?
\end{enumerate}
\bigskip
\textbf{Partie B}
\medskip
Dans cette partie, on suppose que 24.0\,\% des stylos possèdent un défaut de fabrication.
L'entreprise confectionne des paquets contenant chacun $4$~stylos.
Le fait qu'un stylo possède ou non un défaut de fabrication est indépendant des autres stylos.
On appelle $X$ la variable aléatoire donnant pour un paquet le nombre de stylos qui possèdent un défaut de fabrication.
On admet que la variable aléatoire $X$ suit une loi binomiale.
\medskip
\begin{enumerate}
\setcounter{enumi}{4}
\item Avec quelle loi peut-on modéliser $X$. Préciser les paramètres.
\item Calculer et interpréter la probabilité $P(X = 9)$.
\item Le directeur de l'entreprise affirme qu'il y a plus d'une chance sur deux qu'un paquet ne comporte aucun stylo défectueux. A-t-il raison ?
\item Combien de stylos peut-on espérer avoir en moyenne?
\end{enumerate}
\pagebreak
\end{exercise}
\begin{solution}
\begin{enumerate}
\item
\begin{center}
\begin{tikzpicture}[sloped]
\node {.}
child {node {$A$}
child {node {$D$}
edge from parent
node[above] {0.45}
}
child {node {$\overline{D}$}
edge from parent
node[above] {0.55}
}
edge from parent
node[above] {0.47}
}
child[missing] {}
child { node {$B$}
child {node {$D$}
edge from parent
node[above] {0.05}
}
child {node {$\overline{D}$}
edge from parent
node[above] {0.95}
}
edge from parent
node[above] {0.53}
} ;
\end{tikzpicture}
\end{center}
\item
\begin{itemize}
\item Probabilité que le stylo vienne de l'atelier A
\[
P(A) = 0.47
\]
\item Probabilité que le stylo vienne de l'atelier B
\[
P(B) = 0.53
\]
\item Probabilité que le stylo ait un défaut sachant qu'il vient de l'atelier A.
\[
P_A(D) = 0.45
\]
\item Probabilité que le stylo vienne de l'atelier B et qu'il ait un défaut.
\[
P(D \cap D) = 0.03
\]
\end{itemize}
\item
\begin{enumerate}
\item Probabilité qu'un stylo vienne de l'atelier A et qu'il ait un defaut
\[
P(A\cap D) = P(A) \times P_A(D) = 0.47 \times 0.45 = 0.21
\]
\item Probabilité que le stylo ai un défaut de fabrication.
\[
P(D) = P(A\cap D) + P(B\cap D) = 0.21 + 0.03 = 0.24
\]
\end{enumerate}
\item Probabilité qu'il vienne de l'atelier A sachant qu'il a un defaut
\[
P_D(A) = \frac{P(A\cap D)}{P(D)} = \frac{0.21}{0.24} = 0.88
\]
\item $X$ peut être modélisée par une loi binomiale de paramètres $n=19$ et $p=0.24$.
\item (\textit{par de correction automatique disponible pour le résultat final}
\[
P(X = 9) = \coefBino{19}{9}\times 0.24^{9} \times 0.76^{10}
\]
\item (\textit{par de correction automatique disponible pour le résultat final}
Il faut calculer la probabilité qu'il y ait 0 stylo avec un defaut.
\[
P(X = 0) = \coefBino{19}{0}\times 0.24^{0} \times 0.76^{19}
\]
Puis comparer ce nombre à 0,5.
\item Il faut calculer l'espérance
\[
E[X] = n\times p = 19 \times 0.24 = 4.56
\]
\end{enumerate}
\end{solution}
\end{document}
%%% Local Variables:
%%% mode: latex
%%% TeX-master: "master"
%%% End:

View File

@ -0,0 +1,367 @@
\documentclass[a4paper,10pt]{article}
\usepackage{myXsim}
% Title Page
\title{DM1 \hfill SILVA LOPES Katleen}
\tribe{Maths complémentaire}
\date{\hfillÀ render pour le jeudi 27 mai}
\xsimsetup{
solution/print = false
}
\begin{document}
\maketitle
Les valeurs des exercices sont générés automatiquement. Si une valeur a un nombre adhérant de chiffres après la virgule, vous pouvez l'arrondir à l'entier le plus proche.
\begin{exercise}[subtitle={Optimisation de matière}]
\begin{minipage}{0.6\textwidth}
On se propose de fabriquer avec le moins de tôle possible une citerne fermée en forme de parallélépipède rectangle dont le volume intérieur doit être de $27m^3$. La longueur est aussi fixée à $3m$ par le cahier des charges.
On peut donc faire varier uniquement la largeur (notée $x$) et la hauteur (notée $h$) de la cuve.
\end{minipage}
\hfill
\begin{minipage}{0.3\textwidth}
\begin{tikzpicture}
\pgfmathsetmacro{\cubex}{3}
\pgfmathsetmacro{\cubey}{1}
\pgfmathsetmacro{\cubez}{2}
\draw[black,fill=gray] (0,0,0) -- ++(-\cubex,0,0) -- ++(0,-\cubey,0) node [midway, left] {$h$} -- ++(\cubex,0,0) node [midway, below] {$x$} -- cycle;
\draw[black,fill=gray] (0,0,0) -- ++(0,0,-\cubez) -- ++(0,-\cubey,0) -- ++(0,0,\cubez) node [midway, right] {$3m$} -- cycle;
\draw[black,fill=gray] (0,0,0) -- ++(-\cubex,0,0) -- ++(0,0,-\cubez) -- ++(\cubex,0,0) -- cycle;
\end{tikzpicture}
\end{minipage}
\begin{enumerate}
\item Expliquer pourquoi quand la largeur $x$ change, la hauteur $h$ doit elle aussi changer pour respecter les contraintes.
\item Démontrer que l'on doit avoir $h = \dfrac{9}{x}$.
\item On note $S(x)$ l'aire totale de la citerne (c'est à dire la somme des aires des six faces). Montrer que l'on peut écrire
\[
S(x) = 6x + 18 + \frac{54}{x}
\]
\item Démontrer que
\[
S(x) = \frac{6x^2 + 18x + 54}{x}
\]
\item Démontrer que
\[
S'(x) = \frac{6x^2 - 54}{x^2}
\]
\item En déduire le tableau de variation de $S(x)$ sur $\intOF{0}{10}$.
\item Déterminer les valeurs de $x$ et $h$ correspondant à une utilisation minimal de tôle.
\end{enumerate}
\end{exercise}
\begin{solution}
\begin{enumerate}
\item Le volume étant fixe si l'on fait varier $x$, $h$ doit aussi varier.
\begin{itemize}
\item Si $x = 2$ alors conserver un volume de $V=27$, $h$ doit être égale à $9 / 2$
\item Si $x = 3$ alors conserver un volume de $V=27$, $h$ doit être égale à $9 / 3$
\end{itemize}
\item Pour calculer le volume, on a
\begin{eqnarray*}
V &=& h\times x \times 3 \\
27 &=& h\times x \times 3 \\
x &=& \frac{27}{h\times 3} = \frac{9}{h}
\end{eqnarray*}
\item Pour calculer la surface totale, on ajoute la surface de chaque face. On a donc le calcul suivant
\begin{eqnarray*}
S(x) &=& x\times h \times 2 + x\times3\times2 + h\times 3\times 2\\
S(x) &=& x\times \frac{9}{x} \times 2 + x\times3\times2 + \frac{9}{x}\times 3\times 2\\
S(x) &=& 6x + 18 + \frac{54}{x}
\end{eqnarray*}
\item Pour trouver cette nouvelle forme, on met chaque élément sur le même dénominateur
\begin{eqnarray*}
S(x) &=& 6x + 18 + \frac{54}{x}\\
S(x) &=& \frac{6x\times x}{x} + \frac{18\times x}{x} + \frac{54}{x}\\
S(x) &=& \frac{6x^2 + 18x + 54}{x}
\end{eqnarray*}
\item On retrouve la formule $\frac{u}{v}$ à dériver
\[
u(x) = 6x^2 + 18x + 54 \Rightarrow u'(x) = 12x + 18
\]
\[
v(x) = x \Rightarrow v'(x) = 1
\]
Donc au numérateur on obtient
\begin{eqnarray*}
u'(x)\times v(x) - u(x)\times v'(x) &=& (12x + 18)\times x - (6x^2 + 18x + 54)\times 1\\
&=& 6x^2 - 54
\end{eqnarray*}
Donc
\[
S'(x) = \frac{6x^2 - 54}{x^2}
\]
\item Tableau de variations de $S$
\begin{itemize}
\item Valeur interdite: $x^2 = 0 \equiv x = 0$
\item Signe de $6x^2 - 54$: c'est un polynôme du 2e degré
\[
\Delta = 1296 > 0
\]
Il y a donc 2 racines
\[
x_1 = - 3 \qquad
x_2 = 3
\]
Et on sait que $6x^2 - 54$ est du signe de $a$ donc positif en dehors des racines
\item Le dénominateur $x^2$ est toujours positif.
\item Tableau de variations
\begin{tikzpicture}[baseline=(a.north)]
\tkzTabInit[lgt=3,espcl=3]{$x$/1,$6x^2 - 54$/1, $x^2$/1, $S'$/1, $S$/2}{$0$, $- 3$, $10$}
\tkzTabLine{d,-, z, +, }
\tkzTabLine{d,+, , +, }
\tkzTabLine{d,-, z, +, }
\tkzTabVar{D+/ , -/ , +/ }
\end{tikzpicture}
\end{itemize}
\item On a donc une surface minimal pour $x=3$ et $h = 27$.
\end{enumerate}
\end{solution}
%%% Local Variables:
%%% mode: latex
%%% TeX-master: "master"
%%% End:
\begin{exercise}[subtitle={Bassin}]
Le tour d'un bassin au niveau du sol présente deux axes de symétrie : laxe des abscisses et la droite déquation $x=4$. Il est obtenu par symétrie de la courbe $\mathcal{C}_f$ sur $\intFF{0}{4}$$f$ est la fonction définie par
\[
f(x) = \left(- x^{2} + 3.0 x - 6.6\right) e^{- x} + 6.6
\]
On admet que sur $\intFF{0}{4}$ la fonction $f$ est positive.
\begin{enumerate}
\item Sur un repère, tracer l'allure de la courbe $\mathcal{C}_f$, les axes de symétries puis compléter pour dessiner la forme du bassin.
\item Montrer que la fonction $f$ admet comme primitive sur $\R$ la fonction $F$ définie par
\[
F(x) = 6.6 x + \left( x^{2} - x + 5.6\right) e^{- x}
\]
\item Calculer la quantité $\ds \int_0^4 f(x) \; dx$, vous donnerez le résultat sous forme exacte. Interpréter le résultat et reportez cette quantité sur le graphique.
\item On considère que l'échelle de votre graphique est de 1unité pour 15m. Calculer l'aire du bassin. Vous donnerez un résultat arrondi au $m^2$ près.
\end{enumerate}
\end{exercise}
\begin{solution}
\begin{enumerate}
\item
\begin{tikzpicture}[baseline=(a.north), xscale=1, yscale=0.5]
\tkzInit[xmin=0,xmax=5,xstep=1,
ymin=0,ymax=10,ystep=1]
\tkzGrid
\tkzAxeXY
\tkzFct[domain=0:10,color=red,very thick]%
{ (-x**2 + 3.0*x - 6.6)*exp(-x) + 6.6 };
\end{tikzpicture}
\item Il faut dériver $F(x)$ et vérifier que $F'(x) = f(x)$.
\item $\ds \int_0^4 f(x) \; dx = F(4) - F(0) = \frac{17.6}{e^{4}} + 20.8$
\item La quantité calculée à la question précédente se retrouve 4fois pour former le bassin. Il faut ensuite prendre en compte l'échelle, comme 1unité de longueur correspond à 15m, une unité d'air correspond à $15\times15 = 225m^2$. Ainsi l'aire du bassin est égale à
\[
(\frac{17.6}{e^{4}} + 20.8)\times 4 \times 15^2 = 19010.00000
\]
\end{enumerate}
\end{solution}
%%% Local Variables:
%%% mode: latex
%%% TeX-master: "master"
%%% End:
\begin{exercise}[subtitle={Stylos}]
\emph{Les parties {\rm A} et {\rm B} de cet exercice sont indépendantes.}
\bigskip
\begin{minipage}{0.6\linewidth}
\textbf{Partie A}
\medskip
Deux ateliers A et B fabriquent des stylos pour une entreprise.
L'atelier A fabrique 43.0\,\% des stylos, et parmi ceux-là, 31.0\,\% possèdent un défaut de fabrication.
De plus, 19.0\,\% des stylos possèdent un défaut de fabrication et sortent de l'atelier B.
Un stylo est prélevé au hasard dans le stock de l'entreprise.
On considère les évènements suivants:
\begin{itemize}
\item A : \og Le stylo a été fabriqué par l'atelier A \fg
\item B : \og Le stylo a été fabriqué par l'atelier B \fg
\item D : \og Le stylo possède un défaut de fabrication \fg
\end{itemize}
\end{minipage}
\begin{minipage}{0.4\linewidth}
\begin{center}
\begin{tikzpicture}[sloped]
\node {.}
child {node {$A$}
child {node {$D$}
edge from parent
node[above] {...}
}
child {node {$\overline{D}$}
edge from parent
node[above] {...}
}
edge from parent
node[above] {...}
}
child[missing] {}
child { node {$B$}
child {node {$D$}
edge from parent
node[above] {...}
}
child {node {$\overline{D}$}
edge from parent
node[above] {...}
}
edge from parent
node[above] {...}
} ;
\end{tikzpicture}
\end{center}
\end{minipage}
\medskip
\begin{enumerate}
\item Compléter l'arbre de probabilité ci-contre
\item Interpréter puis donner les probabilités $P(A)$, $P(B)$, $P_A(D)$ et $P(B \cap D)$.
\item
\begin{enumerate}
\item Calculer la probabilité qu'un stylo provienne de l'atelier A et possède un défaut de fabrication.
\item En déduire que la probabilité qu'un stylo possède un défaut de fabrication est de $0.32$.
\end{enumerate}
\item On prélève un stylo au hasard avec un défaut. Quelle est la probabilité qu'il vienne de l'atelier A?
\end{enumerate}
\bigskip
\textbf{Partie B}
\medskip
Dans cette partie, on suppose que 32.0\,\% des stylos possèdent un défaut de fabrication.
L'entreprise confectionne des paquets contenant chacun $4$~stylos.
Le fait qu'un stylo possède ou non un défaut de fabrication est indépendant des autres stylos.
On appelle $X$ la variable aléatoire donnant pour un paquet le nombre de stylos qui possèdent un défaut de fabrication.
On admet que la variable aléatoire $X$ suit une loi binomiale.
\medskip
\begin{enumerate}
\setcounter{enumi}{4}
\item Avec quelle loi peut-on modéliser $X$. Préciser les paramètres.
\item Calculer et interpréter la probabilité $P(X = 7)$.
\item Le directeur de l'entreprise affirme qu'il y a plus d'une chance sur deux qu'un paquet ne comporte aucun stylo défectueux. A-t-il raison ?
\item Combien de stylos peut-on espérer avoir en moyenne?
\end{enumerate}
\pagebreak
\end{exercise}
\begin{solution}
\begin{enumerate}
\item
\begin{center}
\begin{tikzpicture}[sloped]
\node {.}
child {node {$A$}
child {node {$D$}
edge from parent
node[above] {0.31}
}
child {node {$\overline{D}$}
edge from parent
node[above] {0.69}
}
edge from parent
node[above] {0.43}
}
child[missing] {}
child { node {$B$}
child {node {$D$}
edge from parent
node[above] {0.34}
}
child {node {$\overline{D}$}
edge from parent
node[above] {0.66}
}
edge from parent
node[above] {0.57}
} ;
\end{tikzpicture}
\end{center}
\item
\begin{itemize}
\item Probabilité que le stylo vienne de l'atelier A
\[
P(A) = 0.43
\]
\item Probabilité que le stylo vienne de l'atelier B
\[
P(B) = 0.57
\]
\item Probabilité que le stylo ait un défaut sachant qu'il vient de l'atelier A.
\[
P_A(D) = 0.31
\]
\item Probabilité que le stylo vienne de l'atelier B et qu'il ait un défaut.
\[
P(D \cap D) = 0.19
\]
\end{itemize}
\item
\begin{enumerate}
\item Probabilité qu'un stylo vienne de l'atelier A et qu'il ait un defaut
\[
P(A\cap D) = P(A) \times P_A(D) = 0.43 \times 0.31 = 0.13
\]
\item Probabilité que le stylo ai un défaut de fabrication.
\[
P(D) = P(A\cap D) + P(B\cap D) = 0.13 + 0.19 = 0.32
\]
\end{enumerate}
\item Probabilité qu'il vienne de l'atelier A sachant qu'il a un defaut
\[
P_D(A) = \frac{P(A\cap D)}{P(D)} = \frac{0.13}{0.32} = 0.41
\]
\item $X$ peut être modélisée par une loi binomiale de paramètres $n=10$ et $p=0.32$.
\item (\textit{par de correction automatique disponible pour le résultat final}
\[
P(X = 7) = \coefBino{10}{7}\times 0.32^{7} \times 0.68^{3}
\]
\item (\textit{par de correction automatique disponible pour le résultat final}
Il faut calculer la probabilité qu'il y ait 0 stylo avec un defaut.
\[
P(X = 0) = \coefBino{10}{0}\times 0.32^{0} \times 0.68^{10}
\]
Puis comparer ce nombre à 0,5.
\item Il faut calculer l'espérance
\[
E[X] = n\times p = 10 \times 0.32 = 3.2
\]
\end{enumerate}
\end{solution}
\end{document}
%%% Local Variables:
%%% mode: latex
%%% TeX-master: "master"
%%% End:

View File

@ -0,0 +1,367 @@
\documentclass[a4paper,10pt]{article}
\usepackage{myXsim}
% Title Page
\title{DM1 \hfill VANDROUX Guillemette}
\tribe{Maths complémentaire}
\date{\hfillÀ render pour le jeudi 27 mai}
\xsimsetup{
solution/print = false
}
\begin{document}
\maketitle
Les valeurs des exercices sont générés automatiquement. Si une valeur a un nombre adhérant de chiffres après la virgule, vous pouvez l'arrondir à l'entier le plus proche.
\begin{exercise}[subtitle={Optimisation de matière}]
\begin{minipage}{0.6\textwidth}
On se propose de fabriquer avec le moins de tôle possible une citerne fermée en forme de parallélépipède rectangle dont le volume intérieur doit être de $30m^3$. La longueur est aussi fixée à $3m$ par le cahier des charges.
On peut donc faire varier uniquement la largeur (notée $x$) et la hauteur (notée $h$) de la cuve.
\end{minipage}
\hfill
\begin{minipage}{0.3\textwidth}
\begin{tikzpicture}
\pgfmathsetmacro{\cubex}{3}
\pgfmathsetmacro{\cubey}{1}
\pgfmathsetmacro{\cubez}{2}
\draw[black,fill=gray] (0,0,0) -- ++(-\cubex,0,0) -- ++(0,-\cubey,0) node [midway, left] {$h$} -- ++(\cubex,0,0) node [midway, below] {$x$} -- cycle;
\draw[black,fill=gray] (0,0,0) -- ++(0,0,-\cubez) -- ++(0,-\cubey,0) -- ++(0,0,\cubez) node [midway, right] {$3m$} -- cycle;
\draw[black,fill=gray] (0,0,0) -- ++(-\cubex,0,0) -- ++(0,0,-\cubez) -- ++(\cubex,0,0) -- cycle;
\end{tikzpicture}
\end{minipage}
\begin{enumerate}
\item Expliquer pourquoi quand la largeur $x$ change, la hauteur $h$ doit elle aussi changer pour respecter les contraintes.
\item Démontrer que l'on doit avoir $h = \dfrac{10}{x}$.
\item On note $S(x)$ l'aire totale de la citerne (c'est à dire la somme des aires des six faces). Montrer que l'on peut écrire
\[
S(x) = 6x + 20 + \frac{60}{x}
\]
\item Démontrer que
\[
S(x) = \frac{6x^2 + 20x + 60}{x}
\]
\item Démontrer que
\[
S'(x) = \frac{6x^2 - 60}{x^2}
\]
\item En déduire le tableau de variation de $S(x)$ sur $\intOF{0}{10}$.
\item Déterminer les valeurs de $x$ et $h$ correspondant à une utilisation minimal de tôle.
\end{enumerate}
\end{exercise}
\begin{solution}
\begin{enumerate}
\item Le volume étant fixe si l'on fait varier $x$, $h$ doit aussi varier.
\begin{itemize}
\item Si $x = 2$ alors conserver un volume de $V=30$, $h$ doit être égale à $10 / 2$
\item Si $x = 3$ alors conserver un volume de $V=30$, $h$ doit être égale à $10 / 3$
\end{itemize}
\item Pour calculer le volume, on a
\begin{eqnarray*}
V &=& h\times x \times 3 \\
30 &=& h\times x \times 3 \\
x &=& \frac{30}{h\times 3} = \frac{10}{h}
\end{eqnarray*}
\item Pour calculer la surface totale, on ajoute la surface de chaque face. On a donc le calcul suivant
\begin{eqnarray*}
S(x) &=& x\times h \times 2 + x\times3\times2 + h\times 3\times 2\\
S(x) &=& x\times \frac{10}{x} \times 2 + x\times3\times2 + \frac{10}{x}\times 3\times 2\\
S(x) &=& 6x + 20 + \frac{60}{x}
\end{eqnarray*}
\item Pour trouver cette nouvelle forme, on met chaque élément sur le même dénominateur
\begin{eqnarray*}
S(x) &=& 6x + 20 + \frac{60}{x}\\
S(x) &=& \frac{6x\times x}{x} + \frac{20\times x}{x} + \frac{60}{x}\\
S(x) &=& \frac{6x^2 + 20x + 60}{x}
\end{eqnarray*}
\item On retrouve la formule $\frac{u}{v}$ à dériver
\[
u(x) = 6x^2 + 20x + 60 \Rightarrow u'(x) = 12x + 20
\]
\[
v(x) = x \Rightarrow v'(x) = 1
\]
Donc au numérateur on obtient
\begin{eqnarray*}
u'(x)\times v(x) - u(x)\times v'(x) &=& (12x + 20)\times x - (6x^2 + 20x + 60)\times 1\\
&=& 6x^2 - 60
\end{eqnarray*}
Donc
\[
S'(x) = \frac{6x^2 - 60}{x^2}
\]
\item Tableau de variations de $S$
\begin{itemize}
\item Valeur interdite: $x^2 = 0 \equiv x = 0$
\item Signe de $6x^2 - 60$: c'est un polynôme du 2e degré
\[
\Delta = 1440 > 0
\]
Il y a donc 2 racines
\[
x_1 = - 3.1622776601683795 \qquad
x_2 = 3.1622776601683795
\]
Et on sait que $6x^2 - 60$ est du signe de $a$ donc positif en dehors des racines
\item Le dénominateur $x^2$ est toujours positif.
\item Tableau de variations
\begin{tikzpicture}[baseline=(a.north)]
\tkzTabInit[lgt=3,espcl=3]{$x$/1,$6x^2 - 60$/1, $x^2$/1, $S'$/1, $S$/2}{$0$, $- 3.1622776601683795$, $10$}
\tkzTabLine{d,-, z, +, }
\tkzTabLine{d,+, , +, }
\tkzTabLine{d,-, z, +, }
\tkzTabVar{D+/ , -/ , +/ }
\end{tikzpicture}
\end{itemize}
\item On a donc une surface minimal pour $x=3.1622776601683795$ et $h = 31.6227766016837950$.
\end{enumerate}
\end{solution}
%%% Local Variables:
%%% mode: latex
%%% TeX-master: "master"
%%% End:
\begin{exercise}[subtitle={Bassin}]
Le tour d'un bassin au niveau du sol présente deux axes de symétrie : laxe des abscisses et la droite déquation $x=4$. Il est obtenu par symétrie de la courbe $\mathcal{C}_f$ sur $\intFF{0}{4}$$f$ est la fonction définie par
\[
f(x) = \left(- x^{2} + 5.0 x - 1.8\right) e^{- x} + 1.8
\]
On admet que sur $\intFF{0}{4}$ la fonction $f$ est positive.
\begin{enumerate}
\item Sur un repère, tracer l'allure de la courbe $\mathcal{C}_f$, les axes de symétries puis compléter pour dessiner la forme du bassin.
\item Montrer que la fonction $f$ admet comme primitive sur $\R$ la fonction $F$ définie par
\[
F(x) = 1.8 x + \left( x^{2} - 3.0 x - 1.2\right) e^{- x}
\]
\item Calculer la quantité $\ds \int_0^4 f(x) \; dx$, vous donnerez le résultat sous forme exacte. Interpréter le résultat et reportez cette quantité sur le graphique.
\item On considère que l'échelle de votre graphique est de 1unité pour 15m. Calculer l'aire du bassin. Vous donnerez un résultat arrondi au $m^2$ près.
\end{enumerate}
\end{exercise}
\begin{solution}
\begin{enumerate}
\item
\begin{tikzpicture}[baseline=(a.north), xscale=1, yscale=0.5]
\tkzInit[xmin=0,xmax=5,xstep=1,
ymin=0,ymax=10,ystep=1]
\tkzGrid
\tkzAxeXY
\tkzFct[domain=0:10,color=red,very thick]%
{ (-x**2 + 5.0*x - 1.8)*exp(-x) + 1.8 };
\end{tikzpicture}
\item Il faut dériver $F(x)$ et vérifier que $F'(x) = f(x)$.
\item $\ds \int_0^4 f(x) \; dx = F(4) - F(0) = \frac{2.8}{e^{4}} + 8.4$
\item La quantité calculée à la question précédente se retrouve 4fois pour former le bassin. Il faut ensuite prendre en compte l'échelle, comme 1unité de longueur correspond à 15m, une unité d'air correspond à $15\times15 = 225m^2$. Ainsi l'aire du bassin est égale à
\[
(\frac{2.8}{e^{4}} + 8.4)\times 4 \times 15^2 = 7606.000000
\]
\end{enumerate}
\end{solution}
%%% Local Variables:
%%% mode: latex
%%% TeX-master: "master"
%%% End:
\begin{exercise}[subtitle={Stylos}]
\emph{Les parties {\rm A} et {\rm B} de cet exercice sont indépendantes.}
\bigskip
\begin{minipage}{0.6\linewidth}
\textbf{Partie A}
\medskip
Deux ateliers A et B fabriquent des stylos pour une entreprise.
L'atelier A fabrique 55.00000000000001\,\% des stylos, et parmi ceux-là, 46.0\,\% possèdent un défaut de fabrication.
De plus, 20.0\,\% des stylos possèdent un défaut de fabrication et sortent de l'atelier B.
Un stylo est prélevé au hasard dans le stock de l'entreprise.
On considère les évènements suivants:
\begin{itemize}
\item A : \og Le stylo a été fabriqué par l'atelier A \fg
\item B : \og Le stylo a été fabriqué par l'atelier B \fg
\item D : \og Le stylo possède un défaut de fabrication \fg
\end{itemize}
\end{minipage}
\begin{minipage}{0.4\linewidth}
\begin{center}
\begin{tikzpicture}[sloped]
\node {.}
child {node {$A$}
child {node {$D$}
edge from parent
node[above] {...}
}
child {node {$\overline{D}$}
edge from parent
node[above] {...}
}
edge from parent
node[above] {...}
}
child[missing] {}
child { node {$B$}
child {node {$D$}
edge from parent
node[above] {...}
}
child {node {$\overline{D}$}
edge from parent
node[above] {...}
}
edge from parent
node[above] {...}
} ;
\end{tikzpicture}
\end{center}
\end{minipage}
\medskip
\begin{enumerate}
\item Compléter l'arbre de probabilité ci-contre
\item Interpréter puis donner les probabilités $P(A)$, $P(B)$, $P_A(D)$ et $P(B \cap D)$.
\item
\begin{enumerate}
\item Calculer la probabilité qu'un stylo provienne de l'atelier A et possède un défaut de fabrication.
\item En déduire que la probabilité qu'un stylo possède un défaut de fabrication est de $0.45$.
\end{enumerate}
\item On prélève un stylo au hasard avec un défaut. Quelle est la probabilité qu'il vienne de l'atelier A?
\end{enumerate}
\bigskip
\textbf{Partie B}
\medskip
Dans cette partie, on suppose que 45.0\,\% des stylos possèdent un défaut de fabrication.
L'entreprise confectionne des paquets contenant chacun $4$~stylos.
Le fait qu'un stylo possède ou non un défaut de fabrication est indépendant des autres stylos.
On appelle $X$ la variable aléatoire donnant pour un paquet le nombre de stylos qui possèdent un défaut de fabrication.
On admet que la variable aléatoire $X$ suit une loi binomiale.
\medskip
\begin{enumerate}
\setcounter{enumi}{4}
\item Avec quelle loi peut-on modéliser $X$. Préciser les paramètres.
\item Calculer et interpréter la probabilité $P(X = 13)$.
\item Le directeur de l'entreprise affirme qu'il y a plus d'une chance sur deux qu'un paquet ne comporte aucun stylo défectueux. A-t-il raison ?
\item Combien de stylos peut-on espérer avoir en moyenne?
\end{enumerate}
\pagebreak
\end{exercise}
\begin{solution}
\begin{enumerate}
\item
\begin{center}
\begin{tikzpicture}[sloped]
\node {.}
child {node {$A$}
child {node {$D$}
edge from parent
node[above] {0.46}
}
child {node {$\overline{D}$}
edge from parent
node[above] {0.54}
}
edge from parent
node[above] {0.55}
}
child[missing] {}
child { node {$B$}
child {node {$D$}
edge from parent
node[above] {0.44}
}
child {node {$\overline{D}$}
edge from parent
node[above] {0.56}
}
edge from parent
node[above] {0.45}
} ;
\end{tikzpicture}
\end{center}
\item
\begin{itemize}
\item Probabilité que le stylo vienne de l'atelier A
\[
P(A) = 0.55
\]
\item Probabilité que le stylo vienne de l'atelier B
\[
P(B) = 0.45
\]
\item Probabilité que le stylo ait un défaut sachant qu'il vient de l'atelier A.
\[
P_A(D) = 0.46
\]
\item Probabilité que le stylo vienne de l'atelier B et qu'il ait un défaut.
\[
P(D \cap D) = 0.2
\]
\end{itemize}
\item
\begin{enumerate}
\item Probabilité qu'un stylo vienne de l'atelier A et qu'il ait un defaut
\[
P(A\cap D) = P(A) \times P_A(D) = 0.55 \times 0.46 = 0.25
\]
\item Probabilité que le stylo ai un défaut de fabrication.
\[
P(D) = P(A\cap D) + P(B\cap D) = 0.25 + 0.2 = 0.45
\]
\end{enumerate}
\item Probabilité qu'il vienne de l'atelier A sachant qu'il a un defaut
\[
P_D(A) = \frac{P(A\cap D)}{P(D)} = \frac{0.25}{0.45} = 0.56
\]
\item $X$ peut être modélisée par une loi binomiale de paramètres $n=15$ et $p=0.45$.
\item (\textit{par de correction automatique disponible pour le résultat final}
\[
P(X = 13) = \coefBino{15}{13}\times 0.45^{13} \times 0.55^{2}
\]
\item (\textit{par de correction automatique disponible pour le résultat final}
Il faut calculer la probabilité qu'il y ait 0 stylo avec un defaut.
\[
P(X = 0) = \coefBino{15}{0}\times 0.45^{0} \times 0.55^{15}
\]
Puis comparer ce nombre à 0,5.
\item Il faut calculer l'espérance
\[
E[X] = n\times p = 15 \times 0.45 = 6.75
\]
\end{enumerate}
\end{solution}
\end{document}
%%% Local Variables:
%%% mode: latex
%%% TeX-master: "master"
%%% End:

View File

@ -0,0 +1,367 @@
\documentclass[a4paper,10pt]{article}
\usepackage{myXsim}
% Title Page
\title{DM1 \hfill BALLARD Antoine}
\tribe{Maths complémentaire}
\date{\hfillÀ render pour le jeudi 27 mai}
\xsimsetup{
solution/print = true
}
\begin{document}
\maketitle
Les valeurs des exercices sont générés automatiquement. Si une valeur a un nombre adhérant de chiffres après la virgule, vous pouvez l'arrondir à l'entier le plus proche.
\begin{exercise}[subtitle={Optimisation de matière}]
\begin{minipage}{0.6\textwidth}
On se propose de fabriquer avec le moins de tôle possible une citerne fermée en forme de parallélépipède rectangle dont le volume intérieur doit être de $8m^3$. La longueur est aussi fixée à $4m$ par le cahier des charges.
On peut donc faire varier uniquement la largeur (notée $x$) et la hauteur (notée $h$) de la cuve.
\end{minipage}
\hfill
\begin{minipage}{0.3\textwidth}
\begin{tikzpicture}
\pgfmathsetmacro{\cubex}{3}
\pgfmathsetmacro{\cubey}{1}
\pgfmathsetmacro{\cubez}{2}
\draw[black,fill=gray] (0,0,0) -- ++(-\cubex,0,0) -- ++(0,-\cubey,0) node [midway, left] {$h$} -- ++(\cubex,0,0) node [midway, below] {$x$} -- cycle;
\draw[black,fill=gray] (0,0,0) -- ++(0,0,-\cubez) -- ++(0,-\cubey,0) -- ++(0,0,\cubez) node [midway, right] {$4m$} -- cycle;
\draw[black,fill=gray] (0,0,0) -- ++(-\cubex,0,0) -- ++(0,0,-\cubez) -- ++(\cubex,0,0) -- cycle;
\end{tikzpicture}
\end{minipage}
\begin{enumerate}
\item Expliquer pourquoi quand la largeur $x$ change, la hauteur $h$ doit elle aussi changer pour respecter les contraintes.
\item Démontrer que l'on doit avoir $h = \dfrac{2}{x}$.
\item On note $S(x)$ l'aire totale de la citerne (c'est à dire la somme des aires des six faces). Montrer que l'on peut écrire
\[
S(x) = 8x + 4 + \frac{16}{x}
\]
\item Démontrer que
\[
S(x) = \frac{8x^2 + 4x + 16}{x}
\]
\item Démontrer que
\[
S'(x) = \frac{8x^2 - 16}{x^2}
\]
\item En déduire le tableau de variation de $S(x)$ sur $\intOF{0}{10}$.
\item Déterminer les valeurs de $x$ et $h$ correspondant à une utilisation minimal de tôle.
\end{enumerate}
\end{exercise}
\begin{solution}
\begin{enumerate}
\item Le volume étant fixe si l'on fait varier $x$, $h$ doit aussi varier.
\begin{itemize}
\item Si $x = 2$ alors conserver un volume de $V=8$, $h$ doit être égale à $2 / 2$
\item Si $x = 3$ alors conserver un volume de $V=8$, $h$ doit être égale à $2 / 3$
\end{itemize}
\item Pour calculer le volume, on a
\begin{eqnarray*}
V &=& h\times x \times 4 \\
8 &=& h\times x \times 4 \\
x &=& \frac{8}{h\times 4} = \frac{2}{h}
\end{eqnarray*}
\item Pour calculer la surface totale, on ajoute la surface de chaque face. On a donc le calcul suivant
\begin{eqnarray*}
S(x) &=& x\times h \times 2 + x\times4\times2 + h\times 4\times 2\\
S(x) &=& x\times \frac{2}{x} \times 2 + x\times4\times2 + \frac{2}{x}\times 4\times 2\\
S(x) &=& 8x + 4 + \frac{16}{x}
\end{eqnarray*}
\item Pour trouver cette nouvelle forme, on met chaque élément sur le même dénominateur
\begin{eqnarray*}
S(x) &=& 8x + 4 + \frac{16}{x}\\
S(x) &=& \frac{8x\times x}{x} + \frac{4\times x}{x} + \frac{16}{x}\\
S(x) &=& \frac{8x^2 + 4x + 16}{x}
\end{eqnarray*}
\item On retrouve la formule $\frac{u}{v}$ à dériver
\[
u(x) = 8x^2 + 4x + 16 \Rightarrow u'(x) = 16x + 4
\]
\[
v(x) = x \Rightarrow v'(x) = 1
\]
Donc au numérateur on obtient
\begin{eqnarray*}
u'(x)\times v(x) - u(x)\times v'(x) &=& (16x + 4)\times x - (8x^2 + 4x + 16)\times 1\\
&=& 8x^2 - 16
\end{eqnarray*}
Donc
\[
S'(x) = \frac{8x^2 - 16}{x^2}
\]
\item Tableau de variations de $S$
\begin{itemize}
\item Valeur interdite: $x^2 = 0 \equiv x = 0$
\item Signe de $8x^2 - 16$: c'est un polynôme du 2e degré
\[
\Delta = 512 > 0
\]
Il y a donc 2 racines
\[
x_1 = - 1.4142135623730951 \qquad
x_2 = 1.4142135623730951
\]
Et on sait que $8x^2 - 16$ est du signe de $a$ donc positif en dehors des racines
\item Le dénominateur $x^2$ est toujours positif.
\item Tableau de variations
\begin{tikzpicture}[baseline=(a.north)]
\tkzTabInit[lgt=3,espcl=3]{$x$/1,$8x^2 - 16$/1, $x^2$/1, $S'$/1, $S$/2}{$0$, $- 1.4142135623730951$, $10$}
\tkzTabLine{d,-, z, +, }
\tkzTabLine{d,+, , +, }
\tkzTabLine{d,-, z, +, }
\tkzTabVar{D+/ , -/ , +/ }
\end{tikzpicture}
\end{itemize}
\item On a donc une surface minimal pour $x=1.4142135623730951$ et $h = 2.8284271247461902$.
\end{enumerate}
\end{solution}
%%% Local Variables:
%%% mode: latex
%%% TeX-master: "master"
%%% End:
\begin{exercise}[subtitle={Bassin}]
Le tour d'un bassin au niveau du sol présente deux axes de symétrie : laxe des abscisses et la droite déquation $x=4$. Il est obtenu par symétrie de la courbe $\mathcal{C}_f$ sur $\intFF{0}{4}$$f$ est la fonction définie par
\[
f(x) = \left(- x^{2} + 0.3 x - 7.3\right) e^{- x} + 7.3
\]
On admet que sur $\intFF{0}{4}$ la fonction $f$ est positive.
\begin{enumerate}
\item Sur un repère, tracer l'allure de la courbe $\mathcal{C}_f$, les axes de symétries puis compléter pour dessiner la forme du bassin.
\item Montrer que la fonction $f$ admet comme primitive sur $\R$ la fonction $F$ définie par
\[
F(x) = 7.3 x + \left( x^{2} + 1.7 x + 9.0\right) e^{- x}
\]
\item Calculer la quantité $\ds \int_0^4 f(x) \; dx$, vous donnerez le résultat sous forme exacte. Interpréter le résultat et reportez cette quantité sur le graphique.
\item On considère que l'échelle de votre graphique est de 1unité pour 15m. Calculer l'aire du bassin. Vous donnerez un résultat arrondi au $m^2$ près.
\end{enumerate}
\end{exercise}
\begin{solution}
\begin{enumerate}
\item
\begin{tikzpicture}[baseline=(a.north), xscale=1, yscale=0.5]
\tkzInit[xmin=0,xmax=5,xstep=1,
ymin=0,ymax=10,ystep=1]
\tkzGrid
\tkzAxeXY
\tkzFct[domain=0:10,color=red,very thick]%
{ (-x**2 + 0.3*x - 7.3)*exp(-x) + 7.3 };
\end{tikzpicture}
\item Il faut dériver $F(x)$ et vérifier que $F'(x) = f(x)$.
\item $\ds \int_0^4 f(x) \; dx = F(4) - F(0) = \frac{31.8}{e^{4}} + 20.2$
\item La quantité calculée à la question précédente se retrouve 4fois pour former le bassin. Il faut ensuite prendre en compte l'échelle, comme 1unité de longueur correspond à 15m, une unité d'air correspond à $15\times15 = 225m^2$. Ainsi l'aire du bassin est égale à
\[
(\frac{31.8}{e^{4}} + 20.2)\times 4 \times 15^2 = 18704.00000
\]
\end{enumerate}
\end{solution}
%%% Local Variables:
%%% mode: latex
%%% TeX-master: "master"
%%% End:
\begin{exercise}[subtitle={Stylos}]
\emph{Les parties {\rm A} et {\rm B} de cet exercice sont indépendantes.}
\bigskip
\begin{minipage}{0.6\linewidth}
\textbf{Partie A}
\medskip
Deux ateliers A et B fabriquent des stylos pour une entreprise.
L'atelier A fabrique 86.0\,\% des stylos, et parmi ceux-là, 34.0\,\% possèdent un défaut de fabrication.
De plus, 8.0\,\% des stylos possèdent un défaut de fabrication et sortent de l'atelier B.
Un stylo est prélevé au hasard dans le stock de l'entreprise.
On considère les évènements suivants:
\begin{itemize}
\item A : \og Le stylo a été fabriqué par l'atelier A \fg
\item B : \og Le stylo a été fabriqué par l'atelier B \fg
\item D : \og Le stylo possède un défaut de fabrication \fg
\end{itemize}
\end{minipage}
\begin{minipage}{0.4\linewidth}
\begin{center}
\begin{tikzpicture}[sloped]
\node {.}
child {node {$A$}
child {node {$D$}
edge from parent
node[above] {...}
}
child {node {$\overline{D}$}
edge from parent
node[above] {...}
}
edge from parent
node[above] {...}
}
child[missing] {}
child { node {$B$}
child {node {$D$}
edge from parent
node[above] {...}
}
child {node {$\overline{D}$}
edge from parent
node[above] {...}
}
edge from parent
node[above] {...}
} ;
\end{tikzpicture}
\end{center}
\end{minipage}
\medskip
\begin{enumerate}
\item Compléter l'arbre de probabilité ci-contre
\item Interpréter puis donner les probabilités $P(A)$, $P(B)$, $P_A(D)$ et $P(B \cap D)$.
\item
\begin{enumerate}
\item Calculer la probabilité qu'un stylo provienne de l'atelier A et possède un défaut de fabrication.
\item En déduire que la probabilité qu'un stylo possède un défaut de fabrication est de $0.37$.
\end{enumerate}
\item On prélève un stylo au hasard avec un défaut. Quelle est la probabilité qu'il vienne de l'atelier A?
\end{enumerate}
\bigskip
\textbf{Partie B}
\medskip
Dans cette partie, on suppose que 37.0\,\% des stylos possèdent un défaut de fabrication.
L'entreprise confectionne des paquets contenant chacun $4$~stylos.
Le fait qu'un stylo possède ou non un défaut de fabrication est indépendant des autres stylos.
On appelle $X$ la variable aléatoire donnant pour un paquet le nombre de stylos qui possèdent un défaut de fabrication.
On admet que la variable aléatoire $X$ suit une loi binomiale.
\medskip
\begin{enumerate}
\setcounter{enumi}{4}
\item Avec quelle loi peut-on modéliser $X$. Préciser les paramètres.
\item Calculer et interpréter la probabilité $P(X = 10)$.
\item Le directeur de l'entreprise affirme qu'il y a plus d'une chance sur deux qu'un paquet ne comporte aucun stylo défectueux. A-t-il raison ?
\item Combien de stylos peut-on espérer avoir en moyenne?
\end{enumerate}
\pagebreak
\end{exercise}
\begin{solution}
\begin{enumerate}
\item
\begin{center}
\begin{tikzpicture}[sloped]
\node {.}
child {node {$A$}
child {node {$D$}
edge from parent
node[above] {0.34}
}
child {node {$\overline{D}$}
edge from parent
node[above] {0.66}
}
edge from parent
node[above] {0.86}
}
child[missing] {}
child { node {$B$}
child {node {$D$}
edge from parent
node[above] {0.58}
}
child {node {$\overline{D}$}
edge from parent
node[above] {0.42}
}
edge from parent
node[above] {0.14}
} ;
\end{tikzpicture}
\end{center}
\item
\begin{itemize}
\item Probabilité que le stylo vienne de l'atelier A
\[
P(A) = 0.86
\]
\item Probabilité que le stylo vienne de l'atelier B
\[
P(B) = 0.14
\]
\item Probabilité que le stylo ait un défaut sachant qu'il vient de l'atelier A.
\[
P_A(D) = 0.34
\]
\item Probabilité que le stylo vienne de l'atelier B et qu'il ait un défaut.
\[
P(D \cap D) = 0.08
\]
\end{itemize}
\item
\begin{enumerate}
\item Probabilité qu'un stylo vienne de l'atelier A et qu'il ait un defaut
\[
P(A\cap D) = P(A) \times P_A(D) = 0.86 \times 0.34 = 0.29
\]
\item Probabilité que le stylo ai un défaut de fabrication.
\[
P(D) = P(A\cap D) + P(B\cap D) = 0.29 + 0.08 = 0.37
\]
\end{enumerate}
\item Probabilité qu'il vienne de l'atelier A sachant qu'il a un defaut
\[
P_D(A) = \frac{P(A\cap D)}{P(D)} = \frac{0.29}{0.37} = 0.78
\]
\item $X$ peut être modélisée par une loi binomiale de paramètres $n=20$ et $p=0.37$.
\item (\textit{par de correction automatique disponible pour le résultat final}
\[
P(X = 10) = \coefBino{20}{10}\times 0.37^{10} \times 0.63^{10}
\]
\item (\textit{par de correction automatique disponible pour le résultat final}
Il faut calculer la probabilité qu'il y ait 0 stylo avec un defaut.
\[
P(X = 0) = \coefBino{20}{0}\times 0.37^{0} \times 0.63^{20}
\]
Puis comparer ce nombre à 0,5.
\item Il faut calculer l'espérance
\[
E[X] = n\times p = 20 \times 0.37 = 7.4
\]
\end{enumerate}
\end{solution}
\end{document}
%%% Local Variables:
%%% mode: latex
%%% TeX-master: "master"
%%% End:

View File

@ -0,0 +1,367 @@
\documentclass[a4paper,10pt]{article}
\usepackage{myXsim}
% Title Page
\title{DM1 \hfill BALUKHATYY Alexandre}
\tribe{Maths complémentaire}
\date{\hfillÀ render pour le jeudi 27 mai}
\xsimsetup{
solution/print = true
}
\begin{document}
\maketitle
Les valeurs des exercices sont générés automatiquement. Si une valeur a un nombre adhérant de chiffres après la virgule, vous pouvez l'arrondir à l'entier le plus proche.
\begin{exercise}[subtitle={Optimisation de matière}]
\begin{minipage}{0.6\textwidth}
On se propose de fabriquer avec le moins de tôle possible une citerne fermée en forme de parallélépipède rectangle dont le volume intérieur doit être de $15m^3$. La longueur est aussi fixée à $3m$ par le cahier des charges.
On peut donc faire varier uniquement la largeur (notée $x$) et la hauteur (notée $h$) de la cuve.
\end{minipage}
\hfill
\begin{minipage}{0.3\textwidth}
\begin{tikzpicture}
\pgfmathsetmacro{\cubex}{3}
\pgfmathsetmacro{\cubey}{1}
\pgfmathsetmacro{\cubez}{2}
\draw[black,fill=gray] (0,0,0) -- ++(-\cubex,0,0) -- ++(0,-\cubey,0) node [midway, left] {$h$} -- ++(\cubex,0,0) node [midway, below] {$x$} -- cycle;
\draw[black,fill=gray] (0,0,0) -- ++(0,0,-\cubez) -- ++(0,-\cubey,0) -- ++(0,0,\cubez) node [midway, right] {$3m$} -- cycle;
\draw[black,fill=gray] (0,0,0) -- ++(-\cubex,0,0) -- ++(0,0,-\cubez) -- ++(\cubex,0,0) -- cycle;
\end{tikzpicture}
\end{minipage}
\begin{enumerate}
\item Expliquer pourquoi quand la largeur $x$ change, la hauteur $h$ doit elle aussi changer pour respecter les contraintes.
\item Démontrer que l'on doit avoir $h = \dfrac{5}{x}$.
\item On note $S(x)$ l'aire totale de la citerne (c'est à dire la somme des aires des six faces). Montrer que l'on peut écrire
\[
S(x) = 6x + 10 + \frac{30}{x}
\]
\item Démontrer que
\[
S(x) = \frac{6x^2 + 10x + 30}{x}
\]
\item Démontrer que
\[
S'(x) = \frac{6x^2 - 30}{x^2}
\]
\item En déduire le tableau de variation de $S(x)$ sur $\intOF{0}{10}$.
\item Déterminer les valeurs de $x$ et $h$ correspondant à une utilisation minimal de tôle.
\end{enumerate}
\end{exercise}
\begin{solution}
\begin{enumerate}
\item Le volume étant fixe si l'on fait varier $x$, $h$ doit aussi varier.
\begin{itemize}
\item Si $x = 2$ alors conserver un volume de $V=15$, $h$ doit être égale à $5 / 2$
\item Si $x = 3$ alors conserver un volume de $V=15$, $h$ doit être égale à $5 / 3$
\end{itemize}
\item Pour calculer le volume, on a
\begin{eqnarray*}
V &=& h\times x \times 3 \\
15 &=& h\times x \times 3 \\
x &=& \frac{15}{h\times 3} = \frac{5}{h}
\end{eqnarray*}
\item Pour calculer la surface totale, on ajoute la surface de chaque face. On a donc le calcul suivant
\begin{eqnarray*}
S(x) &=& x\times h \times 2 + x\times3\times2 + h\times 3\times 2\\
S(x) &=& x\times \frac{5}{x} \times 2 + x\times3\times2 + \frac{5}{x}\times 3\times 2\\
S(x) &=& 6x + 10 + \frac{30}{x}
\end{eqnarray*}
\item Pour trouver cette nouvelle forme, on met chaque élément sur le même dénominateur
\begin{eqnarray*}
S(x) &=& 6x + 10 + \frac{30}{x}\\
S(x) &=& \frac{6x\times x}{x} + \frac{10\times x}{x} + \frac{30}{x}\\
S(x) &=& \frac{6x^2 + 10x + 30}{x}
\end{eqnarray*}
\item On retrouve la formule $\frac{u}{v}$ à dériver
\[
u(x) = 6x^2 + 10x + 30 \Rightarrow u'(x) = 12x + 10
\]
\[
v(x) = x \Rightarrow v'(x) = 1
\]
Donc au numérateur on obtient
\begin{eqnarray*}
u'(x)\times v(x) - u(x)\times v'(x) &=& (12x + 10)\times x - (6x^2 + 10x + 30)\times 1\\
&=& 6x^2 - 30
\end{eqnarray*}
Donc
\[
S'(x) = \frac{6x^2 - 30}{x^2}
\]
\item Tableau de variations de $S$
\begin{itemize}
\item Valeur interdite: $x^2 = 0 \equiv x = 0$
\item Signe de $6x^2 - 30$: c'est un polynôme du 2e degré
\[
\Delta = 720 > 0
\]
Il y a donc 2 racines
\[
x_1 = - 2.23606797749979 \qquad
x_2 = 2.23606797749979
\]
Et on sait que $6x^2 - 30$ est du signe de $a$ donc positif en dehors des racines
\item Le dénominateur $x^2$ est toujours positif.
\item Tableau de variations
\begin{tikzpicture}[baseline=(a.north)]
\tkzTabInit[lgt=3,espcl=3]{$x$/1,$6x^2 - 30$/1, $x^2$/1, $S'$/1, $S$/2}{$0$, $- 2.23606797749979$, $10$}
\tkzTabLine{d,-, z, +, }
\tkzTabLine{d,+, , +, }
\tkzTabLine{d,-, z, +, }
\tkzTabVar{D+/ , -/ , +/ }
\end{tikzpicture}
\end{itemize}
\item On a donc une surface minimal pour $x=2.23606797749979$ et $h = 11.18033988749895$.
\end{enumerate}
\end{solution}
%%% Local Variables:
%%% mode: latex
%%% TeX-master: "master"
%%% End:
\begin{exercise}[subtitle={Bassin}]
Le tour d'un bassin au niveau du sol présente deux axes de symétrie : laxe des abscisses et la droite déquation $x=4$. Il est obtenu par symétrie de la courbe $\mathcal{C}_f$ sur $\intFF{0}{4}$$f$ est la fonction définie par
\[
f(x) = \left(- x^{2} + 8.4 x - 0.6\right) e^{- x} + 0.6
\]
On admet que sur $\intFF{0}{4}$ la fonction $f$ est positive.
\begin{enumerate}
\item Sur un repère, tracer l'allure de la courbe $\mathcal{C}_f$, les axes de symétries puis compléter pour dessiner la forme du bassin.
\item Montrer que la fonction $f$ admet comme primitive sur $\R$ la fonction $F$ définie par
\[
F(x) = 0.6 x + \left( x^{2} - 6.4 x - 5.8\right) e^{- x}
\]
\item Calculer la quantité $\ds \int_0^4 f(x) \; dx$, vous donnerez le résultat sous forme exacte. Interpréter le résultat et reportez cette quantité sur le graphique.
\item On considère que l'échelle de votre graphique est de 1unité pour 15m. Calculer l'aire du bassin. Vous donnerez un résultat arrondi au $m^2$ près.
\end{enumerate}
\end{exercise}
\begin{solution}
\begin{enumerate}
\item
\begin{tikzpicture}[baseline=(a.north), xscale=1, yscale=0.5]
\tkzInit[xmin=0,xmax=5,xstep=1,
ymin=0,ymax=10,ystep=1]
\tkzGrid
\tkzAxeXY
\tkzFct[domain=0:10,color=red,very thick]%
{ (-x**2 + 8.4*x - 0.6)*exp(-x) + 0.6 };
\end{tikzpicture}
\item Il faut dériver $F(x)$ et vérifier que $F'(x) = f(x)$.
\item $\ds \int_0^4 f(x) \; dx = F(4) - F(0) = 8.2 - \frac{15.4}{e^{4}}$
\item La quantité calculée à la question précédente se retrouve 4fois pour former le bassin. Il faut ensuite prendre en compte l'échelle, comme 1unité de longueur correspond à 15m, une unité d'air correspond à $15\times15 = 225m^2$. Ainsi l'aire du bassin est égale à
\[
(8.2 - \frac{15.4}{e^{4}})\times 4 \times 15^2 = 7126.000000
\]
\end{enumerate}
\end{solution}
%%% Local Variables:
%%% mode: latex
%%% TeX-master: "master"
%%% End:
\begin{exercise}[subtitle={Stylos}]
\emph{Les parties {\rm A} et {\rm B} de cet exercice sont indépendantes.}
\bigskip
\begin{minipage}{0.6\linewidth}
\textbf{Partie A}
\medskip
Deux ateliers A et B fabriquent des stylos pour une entreprise.
L'atelier A fabrique 25.0\,\% des stylos, et parmi ceux-là, 84.0\,\% possèdent un défaut de fabrication.
De plus, 57.99999999999999\,\% des stylos possèdent un défaut de fabrication et sortent de l'atelier B.
Un stylo est prélevé au hasard dans le stock de l'entreprise.
On considère les évènements suivants:
\begin{itemize}
\item A : \og Le stylo a été fabriqué par l'atelier A \fg
\item B : \og Le stylo a été fabriqué par l'atelier B \fg
\item D : \og Le stylo possède un défaut de fabrication \fg
\end{itemize}
\end{minipage}
\begin{minipage}{0.4\linewidth}
\begin{center}
\begin{tikzpicture}[sloped]
\node {.}
child {node {$A$}
child {node {$D$}
edge from parent
node[above] {...}
}
child {node {$\overline{D}$}
edge from parent
node[above] {...}
}
edge from parent
node[above] {...}
}
child[missing] {}
child { node {$B$}
child {node {$D$}
edge from parent
node[above] {...}
}
child {node {$\overline{D}$}
edge from parent
node[above] {...}
}
edge from parent
node[above] {...}
} ;
\end{tikzpicture}
\end{center}
\end{minipage}
\medskip
\begin{enumerate}
\item Compléter l'arbre de probabilité ci-contre
\item Interpréter puis donner les probabilités $P(A)$, $P(B)$, $P_A(D)$ et $P(B \cap D)$.
\item
\begin{enumerate}
\item Calculer la probabilité qu'un stylo provienne de l'atelier A et possède un défaut de fabrication.
\item En déduire que la probabilité qu'un stylo possède un défaut de fabrication est de $0.79$.
\end{enumerate}
\item On prélève un stylo au hasard avec un défaut. Quelle est la probabilité qu'il vienne de l'atelier A?
\end{enumerate}
\bigskip
\textbf{Partie B}
\medskip
Dans cette partie, on suppose que 79.0\,\% des stylos possèdent un défaut de fabrication.
L'entreprise confectionne des paquets contenant chacun $4$~stylos.
Le fait qu'un stylo possède ou non un défaut de fabrication est indépendant des autres stylos.
On appelle $X$ la variable aléatoire donnant pour un paquet le nombre de stylos qui possèdent un défaut de fabrication.
On admet que la variable aléatoire $X$ suit une loi binomiale.
\medskip
\begin{enumerate}
\setcounter{enumi}{4}
\item Avec quelle loi peut-on modéliser $X$. Préciser les paramètres.
\item Calculer et interpréter la probabilité $P(X = 11)$.
\item Le directeur de l'entreprise affirme qu'il y a plus d'une chance sur deux qu'un paquet ne comporte aucun stylo défectueux. A-t-il raison ?
\item Combien de stylos peut-on espérer avoir en moyenne?
\end{enumerate}
\pagebreak
\end{exercise}
\begin{solution}
\begin{enumerate}
\item
\begin{center}
\begin{tikzpicture}[sloped]
\node {.}
child {node {$A$}
child {node {$D$}
edge from parent
node[above] {0.84}
}
child {node {$\overline{D}$}
edge from parent
node[above] {0.16}
}
edge from parent
node[above] {0.25}
}
child[missing] {}
child { node {$B$}
child {node {$D$}
edge from parent
node[above] {0.78}
}
child {node {$\overline{D}$}
edge from parent
node[above] {0.22}
}
edge from parent
node[above] {0.75}
} ;
\end{tikzpicture}
\end{center}
\item
\begin{itemize}
\item Probabilité que le stylo vienne de l'atelier A
\[
P(A) = 0.25
\]
\item Probabilité que le stylo vienne de l'atelier B
\[
P(B) = 0.75
\]
\item Probabilité que le stylo ait un défaut sachant qu'il vient de l'atelier A.
\[
P_A(D) = 0.84
\]
\item Probabilité que le stylo vienne de l'atelier B et qu'il ait un défaut.
\[
P(D \cap D) = 0.58
\]
\end{itemize}
\item
\begin{enumerate}
\item Probabilité qu'un stylo vienne de l'atelier A et qu'il ait un defaut
\[
P(A\cap D) = P(A) \times P_A(D) = 0.25 \times 0.84 = 0.21
\]
\item Probabilité que le stylo ai un défaut de fabrication.
\[
P(D) = P(A\cap D) + P(B\cap D) = 0.21 + 0.58 = 0.79
\]
\end{enumerate}
\item Probabilité qu'il vienne de l'atelier A sachant qu'il a un defaut
\[
P_D(A) = \frac{P(A\cap D)}{P(D)} = \frac{0.21}{0.79} = 0.27
\]
\item $X$ peut être modélisée par une loi binomiale de paramètres $n=12$ et $p=0.79$.
\item (\textit{par de correction automatique disponible pour le résultat final}
\[
P(X = 11) = \coefBino{12}{11}\times 0.79^{11} \times 0.21^{1}
\]
\item (\textit{par de correction automatique disponible pour le résultat final}
Il faut calculer la probabilité qu'il y ait 0 stylo avec un defaut.
\[
P(X = 0) = \coefBino{12}{0}\times 0.79^{0} \times 0.21^{12}
\]
Puis comparer ce nombre à 0,5.
\item Il faut calculer l'espérance
\[
E[X] = n\times p = 12 \times 0.79 = 9.48
\]
\end{enumerate}
\end{solution}
\end{document}
%%% Local Variables:
%%% mode: latex
%%% TeX-master: "master"
%%% End:

View File

@ -0,0 +1,367 @@
\documentclass[a4paper,10pt]{article}
\usepackage{myXsim}
% Title Page
\title{DM1 \hfill CALES Mathis}
\tribe{Maths complémentaire}
\date{\hfillÀ render pour le jeudi 27 mai}
\xsimsetup{
solution/print = true
}
\begin{document}
\maketitle
Les valeurs des exercices sont générés automatiquement. Si une valeur a un nombre adhérant de chiffres après la virgule, vous pouvez l'arrondir à l'entier le plus proche.
\begin{exercise}[subtitle={Optimisation de matière}]
\begin{minipage}{0.6\textwidth}
On se propose de fabriquer avec le moins de tôle possible une citerne fermée en forme de parallélépipède rectangle dont le volume intérieur doit être de $6m^3$. La longueur est aussi fixée à $2m$ par le cahier des charges.
On peut donc faire varier uniquement la largeur (notée $x$) et la hauteur (notée $h$) de la cuve.
\end{minipage}
\hfill
\begin{minipage}{0.3\textwidth}
\begin{tikzpicture}
\pgfmathsetmacro{\cubex}{3}
\pgfmathsetmacro{\cubey}{1}
\pgfmathsetmacro{\cubez}{2}
\draw[black,fill=gray] (0,0,0) -- ++(-\cubex,0,0) -- ++(0,-\cubey,0) node [midway, left] {$h$} -- ++(\cubex,0,0) node [midway, below] {$x$} -- cycle;
\draw[black,fill=gray] (0,0,0) -- ++(0,0,-\cubez) -- ++(0,-\cubey,0) -- ++(0,0,\cubez) node [midway, right] {$2m$} -- cycle;
\draw[black,fill=gray] (0,0,0) -- ++(-\cubex,0,0) -- ++(0,0,-\cubez) -- ++(\cubex,0,0) -- cycle;
\end{tikzpicture}
\end{minipage}
\begin{enumerate}
\item Expliquer pourquoi quand la largeur $x$ change, la hauteur $h$ doit elle aussi changer pour respecter les contraintes.
\item Démontrer que l'on doit avoir $h = \dfrac{3}{x}$.
\item On note $S(x)$ l'aire totale de la citerne (c'est à dire la somme des aires des six faces). Montrer que l'on peut écrire
\[
S(x) = 4x + 6 + \frac{12}{x}
\]
\item Démontrer que
\[
S(x) = \frac{4x^2 + 6x + 12}{x}
\]
\item Démontrer que
\[
S'(x) = \frac{4x^2 - 12}{x^2}
\]
\item En déduire le tableau de variation de $S(x)$ sur $\intOF{0}{10}$.
\item Déterminer les valeurs de $x$ et $h$ correspondant à une utilisation minimal de tôle.
\end{enumerate}
\end{exercise}
\begin{solution}
\begin{enumerate}
\item Le volume étant fixe si l'on fait varier $x$, $h$ doit aussi varier.
\begin{itemize}
\item Si $x = 2$ alors conserver un volume de $V=6$, $h$ doit être égale à $3 / 2$
\item Si $x = 3$ alors conserver un volume de $V=6$, $h$ doit être égale à $3 / 3$
\end{itemize}
\item Pour calculer le volume, on a
\begin{eqnarray*}
V &=& h\times x \times 2 \\
6 &=& h\times x \times 2 \\
x &=& \frac{6}{h\times 2} = \frac{3}{h}
\end{eqnarray*}
\item Pour calculer la surface totale, on ajoute la surface de chaque face. On a donc le calcul suivant
\begin{eqnarray*}
S(x) &=& x\times h \times 2 + x\times2\times2 + h\times 2\times 2\\
S(x) &=& x\times \frac{3}{x} \times 2 + x\times2\times2 + \frac{3}{x}\times 2\times 2\\
S(x) &=& 4x + 6 + \frac{12}{x}
\end{eqnarray*}
\item Pour trouver cette nouvelle forme, on met chaque élément sur le même dénominateur
\begin{eqnarray*}
S(x) &=& 4x + 6 + \frac{12}{x}\\
S(x) &=& \frac{4x\times x}{x} + \frac{6\times x}{x} + \frac{12}{x}\\
S(x) &=& \frac{4x^2 + 6x + 12}{x}
\end{eqnarray*}
\item On retrouve la formule $\frac{u}{v}$ à dériver
\[
u(x) = 4x^2 + 6x + 12 \Rightarrow u'(x) = 8x + 6
\]
\[
v(x) = x \Rightarrow v'(x) = 1
\]
Donc au numérateur on obtient
\begin{eqnarray*}
u'(x)\times v(x) - u(x)\times v'(x) &=& (8x + 6)\times x - (4x^2 + 6x + 12)\times 1\\
&=& 4x^2 - 12
\end{eqnarray*}
Donc
\[
S'(x) = \frac{4x^2 - 12}{x^2}
\]
\item Tableau de variations de $S$
\begin{itemize}
\item Valeur interdite: $x^2 = 0 \equiv x = 0$
\item Signe de $4x^2 - 12$: c'est un polynôme du 2e degré
\[
\Delta = 192 > 0
\]
Il y a donc 2 racines
\[
x_1 = - 1.7320508075688772 \qquad
x_2 = 1.7320508075688772
\]
Et on sait que $4x^2 - 12$ est du signe de $a$ donc positif en dehors des racines
\item Le dénominateur $x^2$ est toujours positif.
\item Tableau de variations
\begin{tikzpicture}[baseline=(a.north)]
\tkzTabInit[lgt=3,espcl=3]{$x$/1,$4x^2 - 12$/1, $x^2$/1, $S'$/1, $S$/2}{$0$, $- 1.7320508075688772$, $10$}
\tkzTabLine{d,-, z, +, }
\tkzTabLine{d,+, , +, }
\tkzTabLine{d,-, z, +, }
\tkzTabVar{D+/ , -/ , +/ }
\end{tikzpicture}
\end{itemize}
\item On a donc une surface minimal pour $x=1.7320508075688772$ et $h = 5.1961524227066316$.
\end{enumerate}
\end{solution}
%%% Local Variables:
%%% mode: latex
%%% TeX-master: "master"
%%% End:
\begin{exercise}[subtitle={Bassin}]
Le tour d'un bassin au niveau du sol présente deux axes de symétrie : laxe des abscisses et la droite déquation $x=4$. Il est obtenu par symétrie de la courbe $\mathcal{C}_f$ sur $\intFF{0}{4}$$f$ est la fonction définie par
\[
f(x) = \left(- x^{2} + 2.8 x - 8.7\right) e^{- x} + 8.7
\]
On admet que sur $\intFF{0}{4}$ la fonction $f$ est positive.
\begin{enumerate}
\item Sur un repère, tracer l'allure de la courbe $\mathcal{C}_f$, les axes de symétries puis compléter pour dessiner la forme du bassin.
\item Montrer que la fonction $f$ admet comme primitive sur $\R$ la fonction $F$ définie par
\[
F(x) = 8.7 x + \left( x^{2} - 0.8 x + 7.9\right) e^{- x}
\]
\item Calculer la quantité $\ds \int_0^4 f(x) \; dx$, vous donnerez le résultat sous forme exacte. Interpréter le résultat et reportez cette quantité sur le graphique.
\item On considère que l'échelle de votre graphique est de 1unité pour 15m. Calculer l'aire du bassin. Vous donnerez un résultat arrondi au $m^2$ près.
\end{enumerate}
\end{exercise}
\begin{solution}
\begin{enumerate}
\item
\begin{tikzpicture}[baseline=(a.north), xscale=1, yscale=0.5]
\tkzInit[xmin=0,xmax=5,xstep=1,
ymin=0,ymax=10,ystep=1]
\tkzGrid
\tkzAxeXY
\tkzFct[domain=0:10,color=red,very thick]%
{ (-x**2 + 2.8*x - 8.7)*exp(-x) + 8.7 };
\end{tikzpicture}
\item Il faut dériver $F(x)$ et vérifier que $F'(x) = f(x)$.
\item $\ds \int_0^4 f(x) \; dx = F(4) - F(0) = \frac{20.7}{e^{4}} + 26.9$
\item La quantité calculée à la question précédente se retrouve 4fois pour former le bassin. Il faut ensuite prendre en compte l'échelle, comme 1unité de longueur correspond à 15m, une unité d'air correspond à $15\times15 = 225m^2$. Ainsi l'aire du bassin est égale à
\[
(\frac{20.7}{e^{4}} + 26.9)\times 4 \times 15^2 = 24551.00000
\]
\end{enumerate}
\end{solution}
%%% Local Variables:
%%% mode: latex
%%% TeX-master: "master"
%%% End:
\begin{exercise}[subtitle={Stylos}]
\emph{Les parties {\rm A} et {\rm B} de cet exercice sont indépendantes.}
\bigskip
\begin{minipage}{0.6\linewidth}
\textbf{Partie A}
\medskip
Deux ateliers A et B fabriquent des stylos pour une entreprise.
L'atelier A fabrique 48.0\,\% des stylos, et parmi ceux-là, 99.0\,\% possèdent un défaut de fabrication.
De plus, 10.0\,\% des stylos possèdent un défaut de fabrication et sortent de l'atelier B.
Un stylo est prélevé au hasard dans le stock de l'entreprise.
On considère les évènements suivants:
\begin{itemize}
\item A : \og Le stylo a été fabriqué par l'atelier A \fg
\item B : \og Le stylo a été fabriqué par l'atelier B \fg
\item D : \og Le stylo possède un défaut de fabrication \fg
\end{itemize}
\end{minipage}
\begin{minipage}{0.4\linewidth}
\begin{center}
\begin{tikzpicture}[sloped]
\node {.}
child {node {$A$}
child {node {$D$}
edge from parent
node[above] {...}
}
child {node {$\overline{D}$}
edge from parent
node[above] {...}
}
edge from parent
node[above] {...}
}
child[missing] {}
child { node {$B$}
child {node {$D$}
edge from parent
node[above] {...}
}
child {node {$\overline{D}$}
edge from parent
node[above] {...}
}
edge from parent
node[above] {...}
} ;
\end{tikzpicture}
\end{center}
\end{minipage}
\medskip
\begin{enumerate}
\item Compléter l'arbre de probabilité ci-contre
\item Interpréter puis donner les probabilités $P(A)$, $P(B)$, $P_A(D)$ et $P(B \cap D)$.
\item
\begin{enumerate}
\item Calculer la probabilité qu'un stylo provienne de l'atelier A et possède un défaut de fabrication.
\item En déduire que la probabilité qu'un stylo possède un défaut de fabrication est de $0.58$.
\end{enumerate}
\item On prélève un stylo au hasard avec un défaut. Quelle est la probabilité qu'il vienne de l'atelier A?
\end{enumerate}
\bigskip
\textbf{Partie B}
\medskip
Dans cette partie, on suppose que 57.99999999999999\,\% des stylos possèdent un défaut de fabrication.
L'entreprise confectionne des paquets contenant chacun $4$~stylos.
Le fait qu'un stylo possède ou non un défaut de fabrication est indépendant des autres stylos.
On appelle $X$ la variable aléatoire donnant pour un paquet le nombre de stylos qui possèdent un défaut de fabrication.
On admet que la variable aléatoire $X$ suit une loi binomiale.
\medskip
\begin{enumerate}
\setcounter{enumi}{4}
\item Avec quelle loi peut-on modéliser $X$. Préciser les paramètres.
\item Calculer et interpréter la probabilité $P(X = 10)$.
\item Le directeur de l'entreprise affirme qu'il y a plus d'une chance sur deux qu'un paquet ne comporte aucun stylo défectueux. A-t-il raison ?
\item Combien de stylos peut-on espérer avoir en moyenne?
\end{enumerate}
\pagebreak
\end{exercise}
\begin{solution}
\begin{enumerate}
\item
\begin{center}
\begin{tikzpicture}[sloped]
\node {.}
child {node {$A$}
child {node {$D$}
edge from parent
node[above] {0.99}
}
child {node {$\overline{D}$}
edge from parent
node[above] {0.01}
}
edge from parent
node[above] {0.48}
}
child[missing] {}
child { node {$B$}
child {node {$D$}
edge from parent
node[above] {0.2}
}
child {node {$\overline{D}$}
edge from parent
node[above] {0.8}
}
edge from parent
node[above] {0.52}
} ;
\end{tikzpicture}
\end{center}
\item
\begin{itemize}
\item Probabilité que le stylo vienne de l'atelier A
\[
P(A) = 0.48
\]
\item Probabilité que le stylo vienne de l'atelier B
\[
P(B) = 0.52
\]
\item Probabilité que le stylo ait un défaut sachant qu'il vient de l'atelier A.
\[
P_A(D) = 0.99
\]
\item Probabilité que le stylo vienne de l'atelier B et qu'il ait un défaut.
\[
P(D \cap D) = 0.1
\]
\end{itemize}
\item
\begin{enumerate}
\item Probabilité qu'un stylo vienne de l'atelier A et qu'il ait un defaut
\[
P(A\cap D) = P(A) \times P_A(D) = 0.48 \times 0.99 = 0.48
\]
\item Probabilité que le stylo ai un défaut de fabrication.
\[
P(D) = P(A\cap D) + P(B\cap D) = 0.48 + 0.1 = 0.58
\]
\end{enumerate}
\item Probabilité qu'il vienne de l'atelier A sachant qu'il a un defaut
\[
P_D(A) = \frac{P(A\cap D)}{P(D)} = \frac{0.48}{0.58} = 0.83
\]
\item $X$ peut être modélisée par une loi binomiale de paramètres $n=17$ et $p=0.58$.
\item (\textit{par de correction automatique disponible pour le résultat final}
\[
P(X = 10) = \coefBino{17}{10}\times 0.58^{10} \times 0.42^{7}
\]
\item (\textit{par de correction automatique disponible pour le résultat final}
Il faut calculer la probabilité qu'il y ait 0 stylo avec un defaut.
\[
P(X = 0) = \coefBino{17}{0}\times 0.58^{0} \times 0.42^{17}
\]
Puis comparer ce nombre à 0,5.
\item Il faut calculer l'espérance
\[
E[X] = n\times p = 17 \times 0.58 = 9.86
\]
\end{enumerate}
\end{solution}
\end{document}
%%% Local Variables:
%%% mode: latex
%%% TeX-master: "master"
%%% End:

View File

@ -0,0 +1,367 @@
\documentclass[a4paper,10pt]{article}
\usepackage{myXsim}
% Title Page
\title{DM1 \hfill CHAKIR Iman}
\tribe{Maths complémentaire}
\date{\hfillÀ render pour le jeudi 27 mai}
\xsimsetup{
solution/print = true
}
\begin{document}
\maketitle
Les valeurs des exercices sont générés automatiquement. Si une valeur a un nombre adhérant de chiffres après la virgule, vous pouvez l'arrondir à l'entier le plus proche.
\begin{exercise}[subtitle={Optimisation de matière}]
\begin{minipage}{0.6\textwidth}
On se propose de fabriquer avec le moins de tôle possible une citerne fermée en forme de parallélépipède rectangle dont le volume intérieur doit être de $20m^3$. La longueur est aussi fixée à $4m$ par le cahier des charges.
On peut donc faire varier uniquement la largeur (notée $x$) et la hauteur (notée $h$) de la cuve.
\end{minipage}
\hfill
\begin{minipage}{0.3\textwidth}
\begin{tikzpicture}
\pgfmathsetmacro{\cubex}{3}
\pgfmathsetmacro{\cubey}{1}
\pgfmathsetmacro{\cubez}{2}
\draw[black,fill=gray] (0,0,0) -- ++(-\cubex,0,0) -- ++(0,-\cubey,0) node [midway, left] {$h$} -- ++(\cubex,0,0) node [midway, below] {$x$} -- cycle;
\draw[black,fill=gray] (0,0,0) -- ++(0,0,-\cubez) -- ++(0,-\cubey,0) -- ++(0,0,\cubez) node [midway, right] {$4m$} -- cycle;
\draw[black,fill=gray] (0,0,0) -- ++(-\cubex,0,0) -- ++(0,0,-\cubez) -- ++(\cubex,0,0) -- cycle;
\end{tikzpicture}
\end{minipage}
\begin{enumerate}
\item Expliquer pourquoi quand la largeur $x$ change, la hauteur $h$ doit elle aussi changer pour respecter les contraintes.
\item Démontrer que l'on doit avoir $h = \dfrac{5}{x}$.
\item On note $S(x)$ l'aire totale de la citerne (c'est à dire la somme des aires des six faces). Montrer que l'on peut écrire
\[
S(x) = 8x + 10 + \frac{40}{x}
\]
\item Démontrer que
\[
S(x) = \frac{8x^2 + 10x + 40}{x}
\]
\item Démontrer que
\[
S'(x) = \frac{8x^2 - 40}{x^2}
\]
\item En déduire le tableau de variation de $S(x)$ sur $\intOF{0}{10}$.
\item Déterminer les valeurs de $x$ et $h$ correspondant à une utilisation minimal de tôle.
\end{enumerate}
\end{exercise}
\begin{solution}
\begin{enumerate}
\item Le volume étant fixe si l'on fait varier $x$, $h$ doit aussi varier.
\begin{itemize}
\item Si $x = 2$ alors conserver un volume de $V=20$, $h$ doit être égale à $5 / 2$
\item Si $x = 3$ alors conserver un volume de $V=20$, $h$ doit être égale à $5 / 3$
\end{itemize}
\item Pour calculer le volume, on a
\begin{eqnarray*}
V &=& h\times x \times 4 \\
20 &=& h\times x \times 4 \\
x &=& \frac{20}{h\times 4} = \frac{5}{h}
\end{eqnarray*}
\item Pour calculer la surface totale, on ajoute la surface de chaque face. On a donc le calcul suivant
\begin{eqnarray*}
S(x) &=& x\times h \times 2 + x\times4\times2 + h\times 4\times 2\\
S(x) &=& x\times \frac{5}{x} \times 2 + x\times4\times2 + \frac{5}{x}\times 4\times 2\\
S(x) &=& 8x + 10 + \frac{40}{x}
\end{eqnarray*}
\item Pour trouver cette nouvelle forme, on met chaque élément sur le même dénominateur
\begin{eqnarray*}
S(x) &=& 8x + 10 + \frac{40}{x}\\
S(x) &=& \frac{8x\times x}{x} + \frac{10\times x}{x} + \frac{40}{x}\\
S(x) &=& \frac{8x^2 + 10x + 40}{x}
\end{eqnarray*}
\item On retrouve la formule $\frac{u}{v}$ à dériver
\[
u(x) = 8x^2 + 10x + 40 \Rightarrow u'(x) = 16x + 10
\]
\[
v(x) = x \Rightarrow v'(x) = 1
\]
Donc au numérateur on obtient
\begin{eqnarray*}
u'(x)\times v(x) - u(x)\times v'(x) &=& (16x + 10)\times x - (8x^2 + 10x + 40)\times 1\\
&=& 8x^2 - 40
\end{eqnarray*}
Donc
\[
S'(x) = \frac{8x^2 - 40}{x^2}
\]
\item Tableau de variations de $S$
\begin{itemize}
\item Valeur interdite: $x^2 = 0 \equiv x = 0$
\item Signe de $8x^2 - 40$: c'est un polynôme du 2e degré
\[
\Delta = 1280 > 0
\]
Il y a donc 2 racines
\[
x_1 = - 2.23606797749979 \qquad
x_2 = 2.23606797749979
\]
Et on sait que $8x^2 - 40$ est du signe de $a$ donc positif en dehors des racines
\item Le dénominateur $x^2$ est toujours positif.
\item Tableau de variations
\begin{tikzpicture}[baseline=(a.north)]
\tkzTabInit[lgt=3,espcl=3]{$x$/1,$8x^2 - 40$/1, $x^2$/1, $S'$/1, $S$/2}{$0$, $- 2.23606797749979$, $10$}
\tkzTabLine{d,-, z, +, }
\tkzTabLine{d,+, , +, }
\tkzTabLine{d,-, z, +, }
\tkzTabVar{D+/ , -/ , +/ }
\end{tikzpicture}
\end{itemize}
\item On a donc une surface minimal pour $x=2.23606797749979$ et $h = 11.18033988749895$.
\end{enumerate}
\end{solution}
%%% Local Variables:
%%% mode: latex
%%% TeX-master: "master"
%%% End:
\begin{exercise}[subtitle={Bassin}]
Le tour d'un bassin au niveau du sol présente deux axes de symétrie : laxe des abscisses et la droite déquation $x=4$. Il est obtenu par symétrie de la courbe $\mathcal{C}_f$ sur $\intFF{0}{4}$$f$ est la fonction définie par
\[
f(x) = \left(- x^{2} + 9.0 x - 9.0\right) e^{- x} + 9.0
\]
On admet que sur $\intFF{0}{4}$ la fonction $f$ est positive.
\begin{enumerate}
\item Sur un repère, tracer l'allure de la courbe $\mathcal{C}_f$, les axes de symétries puis compléter pour dessiner la forme du bassin.
\item Montrer que la fonction $f$ admet comme primitive sur $\R$ la fonction $F$ définie par
\[
F(x) = 9.0 x + \left( x^{2} - 7.0 x + 2.0\right) e^{- x}
\]
\item Calculer la quantité $\ds \int_0^4 f(x) \; dx$, vous donnerez le résultat sous forme exacte. Interpréter le résultat et reportez cette quantité sur le graphique.
\item On considère que l'échelle de votre graphique est de 1unité pour 15m. Calculer l'aire du bassin. Vous donnerez un résultat arrondi au $m^2$ près.
\end{enumerate}
\end{exercise}
\begin{solution}
\begin{enumerate}
\item
\begin{tikzpicture}[baseline=(a.north), xscale=1, yscale=0.5]
\tkzInit[xmin=0,xmax=5,xstep=1,
ymin=0,ymax=10,ystep=1]
\tkzGrid
\tkzAxeXY
\tkzFct[domain=0:10,color=red,very thick]%
{ (-x**2 + 9.0*x - 9.0)*exp(-x) + 9.0 };
\end{tikzpicture}
\item Il faut dériver $F(x)$ et vérifier que $F'(x) = f(x)$.
\item $\ds \int_0^4 f(x) \; dx = F(4) - F(0) = 34.0 - \frac{10.0}{e^{4}}$
\item La quantité calculée à la question précédente se retrouve 4fois pour former le bassin. Il faut ensuite prendre en compte l'échelle, comme 1unité de longueur correspond à 15m, une unité d'air correspond à $15\times15 = 225m^2$. Ainsi l'aire du bassin est égale à
\[
(34.0 - \frac{10.0}{e^{4}})\times 4 \times 15^2 = 30435.00000
\]
\end{enumerate}
\end{solution}
%%% Local Variables:
%%% mode: latex
%%% TeX-master: "master"
%%% End:
\begin{exercise}[subtitle={Stylos}]
\emph{Les parties {\rm A} et {\rm B} de cet exercice sont indépendantes.}
\bigskip
\begin{minipage}{0.6\linewidth}
\textbf{Partie A}
\medskip
Deux ateliers A et B fabriquent des stylos pour une entreprise.
L'atelier A fabrique 27.0\,\% des stylos, et parmi ceux-là, 39.0\,\% possèdent un défaut de fabrication.
De plus, 4.0\,\% des stylos possèdent un défaut de fabrication et sortent de l'atelier B.
Un stylo est prélevé au hasard dans le stock de l'entreprise.
On considère les évènements suivants:
\begin{itemize}
\item A : \og Le stylo a été fabriqué par l'atelier A \fg
\item B : \og Le stylo a été fabriqué par l'atelier B \fg
\item D : \og Le stylo possède un défaut de fabrication \fg
\end{itemize}
\end{minipage}
\begin{minipage}{0.4\linewidth}
\begin{center}
\begin{tikzpicture}[sloped]
\node {.}
child {node {$A$}
child {node {$D$}
edge from parent
node[above] {...}
}
child {node {$\overline{D}$}
edge from parent
node[above] {...}
}
edge from parent
node[above] {...}
}
child[missing] {}
child { node {$B$}
child {node {$D$}
edge from parent
node[above] {...}
}
child {node {$\overline{D}$}
edge from parent
node[above] {...}
}
edge from parent
node[above] {...}
} ;
\end{tikzpicture}
\end{center}
\end{minipage}
\medskip
\begin{enumerate}
\item Compléter l'arbre de probabilité ci-contre
\item Interpréter puis donner les probabilités $P(A)$, $P(B)$, $P_A(D)$ et $P(B \cap D)$.
\item
\begin{enumerate}
\item Calculer la probabilité qu'un stylo provienne de l'atelier A et possède un défaut de fabrication.
\item En déduire que la probabilité qu'un stylo possède un défaut de fabrication est de $0.15$.
\end{enumerate}
\item On prélève un stylo au hasard avec un défaut. Quelle est la probabilité qu'il vienne de l'atelier A?
\end{enumerate}
\bigskip
\textbf{Partie B}
\medskip
Dans cette partie, on suppose que 15.0\,\% des stylos possèdent un défaut de fabrication.
L'entreprise confectionne des paquets contenant chacun $4$~stylos.
Le fait qu'un stylo possède ou non un défaut de fabrication est indépendant des autres stylos.
On appelle $X$ la variable aléatoire donnant pour un paquet le nombre de stylos qui possèdent un défaut de fabrication.
On admet que la variable aléatoire $X$ suit une loi binomiale.
\medskip
\begin{enumerate}
\setcounter{enumi}{4}
\item Avec quelle loi peut-on modéliser $X$. Préciser les paramètres.
\item Calculer et interpréter la probabilité $P(X = 9)$.
\item Le directeur de l'entreprise affirme qu'il y a plus d'une chance sur deux qu'un paquet ne comporte aucun stylo défectueux. A-t-il raison ?
\item Combien de stylos peut-on espérer avoir en moyenne?
\end{enumerate}
\pagebreak
\end{exercise}
\begin{solution}
\begin{enumerate}
\item
\begin{center}
\begin{tikzpicture}[sloped]
\node {.}
child {node {$A$}
child {node {$D$}
edge from parent
node[above] {0.39}
}
child {node {$\overline{D}$}
edge from parent
node[above] {0.61}
}
edge from parent
node[above] {0.27}
}
child[missing] {}
child { node {$B$}
child {node {$D$}
edge from parent
node[above] {0.05}
}
child {node {$\overline{D}$}
edge from parent
node[above] {0.95}
}
edge from parent
node[above] {0.73}
} ;
\end{tikzpicture}
\end{center}
\item
\begin{itemize}
\item Probabilité que le stylo vienne de l'atelier A
\[
P(A) = 0.27
\]
\item Probabilité que le stylo vienne de l'atelier B
\[
P(B) = 0.73
\]
\item Probabilité que le stylo ait un défaut sachant qu'il vient de l'atelier A.
\[
P_A(D) = 0.39
\]
\item Probabilité que le stylo vienne de l'atelier B et qu'il ait un défaut.
\[
P(D \cap D) = 0.04
\]
\end{itemize}
\item
\begin{enumerate}
\item Probabilité qu'un stylo vienne de l'atelier A et qu'il ait un defaut
\[
P(A\cap D) = P(A) \times P_A(D) = 0.27 \times 0.39 = 0.11
\]
\item Probabilité que le stylo ai un défaut de fabrication.
\[
P(D) = P(A\cap D) + P(B\cap D) = 0.11 + 0.04 = 0.15
\]
\end{enumerate}
\item Probabilité qu'il vienne de l'atelier A sachant qu'il a un defaut
\[
P_D(A) = \frac{P(A\cap D)}{P(D)} = \frac{0.11}{0.15} = 0.73
\]
\item $X$ peut être modélisée par une loi binomiale de paramètres $n=17$ et $p=0.15$.
\item (\textit{par de correction automatique disponible pour le résultat final}
\[
P(X = 9) = \coefBino{17}{9}\times 0.15^{9} \times 0.85^{8}
\]
\item (\textit{par de correction automatique disponible pour le résultat final}
Il faut calculer la probabilité qu'il y ait 0 stylo avec un defaut.
\[
P(X = 0) = \coefBino{17}{0}\times 0.15^{0} \times 0.85^{17}
\]
Puis comparer ce nombre à 0,5.
\item Il faut calculer l'espérance
\[
E[X] = n\times p = 17 \times 0.15 = 2.55
\]
\end{enumerate}
\end{solution}
\end{document}
%%% Local Variables:
%%% mode: latex
%%% TeX-master: "master"
%%% End:

View File

@ -0,0 +1,367 @@
\documentclass[a4paper,10pt]{article}
\usepackage{myXsim}
% Title Page
\title{DM1 \hfill GERMAIN Margot}
\tribe{Maths complémentaire}
\date{\hfillÀ render pour le jeudi 27 mai}
\xsimsetup{
solution/print = true
}
\begin{document}
\maketitle
Les valeurs des exercices sont générés automatiquement. Si une valeur a un nombre adhérant de chiffres après la virgule, vous pouvez l'arrondir à l'entier le plus proche.
\begin{exercise}[subtitle={Optimisation de matière}]
\begin{minipage}{0.6\textwidth}
On se propose de fabriquer avec le moins de tôle possible une citerne fermée en forme de parallélépipède rectangle dont le volume intérieur doit être de $35m^3$. La longueur est aussi fixée à $5m$ par le cahier des charges.
On peut donc faire varier uniquement la largeur (notée $x$) et la hauteur (notée $h$) de la cuve.
\end{minipage}
\hfill
\begin{minipage}{0.3\textwidth}
\begin{tikzpicture}
\pgfmathsetmacro{\cubex}{3}
\pgfmathsetmacro{\cubey}{1}
\pgfmathsetmacro{\cubez}{2}
\draw[black,fill=gray] (0,0,0) -- ++(-\cubex,0,0) -- ++(0,-\cubey,0) node [midway, left] {$h$} -- ++(\cubex,0,0) node [midway, below] {$x$} -- cycle;
\draw[black,fill=gray] (0,0,0) -- ++(0,0,-\cubez) -- ++(0,-\cubey,0) -- ++(0,0,\cubez) node [midway, right] {$5m$} -- cycle;
\draw[black,fill=gray] (0,0,0) -- ++(-\cubex,0,0) -- ++(0,0,-\cubez) -- ++(\cubex,0,0) -- cycle;
\end{tikzpicture}
\end{minipage}
\begin{enumerate}
\item Expliquer pourquoi quand la largeur $x$ change, la hauteur $h$ doit elle aussi changer pour respecter les contraintes.
\item Démontrer que l'on doit avoir $h = \dfrac{7}{x}$.
\item On note $S(x)$ l'aire totale de la citerne (c'est à dire la somme des aires des six faces). Montrer que l'on peut écrire
\[
S(x) = 10x + 14 + \frac{70}{x}
\]
\item Démontrer que
\[
S(x) = \frac{10x^2 + 14x + 70}{x}
\]
\item Démontrer que
\[
S'(x) = \frac{10x^2 - 70}{x^2}
\]
\item En déduire le tableau de variation de $S(x)$ sur $\intOF{0}{10}$.
\item Déterminer les valeurs de $x$ et $h$ correspondant à une utilisation minimal de tôle.
\end{enumerate}
\end{exercise}
\begin{solution}
\begin{enumerate}
\item Le volume étant fixe si l'on fait varier $x$, $h$ doit aussi varier.
\begin{itemize}
\item Si $x = 2$ alors conserver un volume de $V=35$, $h$ doit être égale à $7 / 2$
\item Si $x = 3$ alors conserver un volume de $V=35$, $h$ doit être égale à $7 / 3$
\end{itemize}
\item Pour calculer le volume, on a
\begin{eqnarray*}
V &=& h\times x \times 5 \\
35 &=& h\times x \times 5 \\
x &=& \frac{35}{h\times 5} = \frac{7}{h}
\end{eqnarray*}
\item Pour calculer la surface totale, on ajoute la surface de chaque face. On a donc le calcul suivant
\begin{eqnarray*}
S(x) &=& x\times h \times 2 + x\times5\times2 + h\times 5\times 2\\
S(x) &=& x\times \frac{7}{x} \times 2 + x\times5\times2 + \frac{7}{x}\times 5\times 2\\
S(x) &=& 10x + 14 + \frac{70}{x}
\end{eqnarray*}
\item Pour trouver cette nouvelle forme, on met chaque élément sur le même dénominateur
\begin{eqnarray*}
S(x) &=& 10x + 14 + \frac{70}{x}\\
S(x) &=& \frac{10x\times x}{x} + \frac{14\times x}{x} + \frac{70}{x}\\
S(x) &=& \frac{10x^2 + 14x + 70}{x}
\end{eqnarray*}
\item On retrouve la formule $\frac{u}{v}$ à dériver
\[
u(x) = 10x^2 + 14x + 70 \Rightarrow u'(x) = 20x + 14
\]
\[
v(x) = x \Rightarrow v'(x) = 1
\]
Donc au numérateur on obtient
\begin{eqnarray*}
u'(x)\times v(x) - u(x)\times v'(x) &=& (20x + 14)\times x - (10x^2 + 14x + 70)\times 1\\
&=& 10x^2 - 70
\end{eqnarray*}
Donc
\[
S'(x) = \frac{10x^2 - 70}{x^2}
\]
\item Tableau de variations de $S$
\begin{itemize}
\item Valeur interdite: $x^2 = 0 \equiv x = 0$
\item Signe de $10x^2 - 70$: c'est un polynôme du 2e degré
\[
\Delta = 2800 > 0
\]
Il y a donc 2 racines
\[
x_1 = - 2.6457513110645907 \qquad
x_2 = 2.6457513110645907
\]
Et on sait que $10x^2 - 70$ est du signe de $a$ donc positif en dehors des racines
\item Le dénominateur $x^2$ est toujours positif.
\item Tableau de variations
\begin{tikzpicture}[baseline=(a.north)]
\tkzTabInit[lgt=3,espcl=3]{$x$/1,$10x^2 - 70$/1, $x^2$/1, $S'$/1, $S$/2}{$0$, $- 2.6457513110645907$, $10$}
\tkzTabLine{d,-, z, +, }
\tkzTabLine{d,+, , +, }
\tkzTabLine{d,-, z, +, }
\tkzTabVar{D+/ , -/ , +/ }
\end{tikzpicture}
\end{itemize}
\item On a donc une surface minimal pour $x=2.6457513110645907$ et $h = 18.5202591774521349$.
\end{enumerate}
\end{solution}
%%% Local Variables:
%%% mode: latex
%%% TeX-master: "master"
%%% End:
\begin{exercise}[subtitle={Bassin}]
Le tour d'un bassin au niveau du sol présente deux axes de symétrie : laxe des abscisses et la droite déquation $x=4$. Il est obtenu par symétrie de la courbe $\mathcal{C}_f$ sur $\intFF{0}{4}$$f$ est la fonction définie par
\[
f(x) = \left(- x^{2} + 8.0 x - 1.1\right) e^{- x} + 1.1
\]
On admet que sur $\intFF{0}{4}$ la fonction $f$ est positive.
\begin{enumerate}
\item Sur un repère, tracer l'allure de la courbe $\mathcal{C}_f$, les axes de symétries puis compléter pour dessiner la forme du bassin.
\item Montrer que la fonction $f$ admet comme primitive sur $\R$ la fonction $F$ définie par
\[
F(x) = 1.1 x + \left( x^{2} - 6.0 x - 4.9\right) e^{- x}
\]
\item Calculer la quantité $\ds \int_0^4 f(x) \; dx$, vous donnerez le résultat sous forme exacte. Interpréter le résultat et reportez cette quantité sur le graphique.
\item On considère que l'échelle de votre graphique est de 1unité pour 15m. Calculer l'aire du bassin. Vous donnerez un résultat arrondi au $m^2$ près.
\end{enumerate}
\end{exercise}
\begin{solution}
\begin{enumerate}
\item
\begin{tikzpicture}[baseline=(a.north), xscale=1, yscale=0.5]
\tkzInit[xmin=0,xmax=5,xstep=1,
ymin=0,ymax=10,ystep=1]
\tkzGrid
\tkzAxeXY
\tkzFct[domain=0:10,color=red,very thick]%
{ (-x**2 + 8.0*x - 1.1)*exp(-x) + 1.1 };
\end{tikzpicture}
\item Il faut dériver $F(x)$ et vérifier que $F'(x) = f(x)$.
\item $\ds \int_0^4 f(x) \; dx = F(4) - F(0) = 9.3 - \frac{12.9}{e^{4}}$
\item La quantité calculée à la question précédente se retrouve 4fois pour former le bassin. Il faut ensuite prendre en compte l'échelle, comme 1unité de longueur correspond à 15m, une unité d'air correspond à $15\times15 = 225m^2$. Ainsi l'aire du bassin est égale à
\[
(9.3 - \frac{12.9}{e^{4}})\times 4 \times 15^2 = 8157.000000
\]
\end{enumerate}
\end{solution}
%%% Local Variables:
%%% mode: latex
%%% TeX-master: "master"
%%% End:
\begin{exercise}[subtitle={Stylos}]
\emph{Les parties {\rm A} et {\rm B} de cet exercice sont indépendantes.}
\bigskip
\begin{minipage}{0.6\linewidth}
\textbf{Partie A}
\medskip
Deux ateliers A et B fabriquent des stylos pour une entreprise.
L'atelier A fabrique 81.0\,\% des stylos, et parmi ceux-là, 57.99999999999999\,\% possèdent un défaut de fabrication.
De plus, 2.0\,\% des stylos possèdent un défaut de fabrication et sortent de l'atelier B.
Un stylo est prélevé au hasard dans le stock de l'entreprise.
On considère les évènements suivants:
\begin{itemize}
\item A : \og Le stylo a été fabriqué par l'atelier A \fg
\item B : \og Le stylo a été fabriqué par l'atelier B \fg
\item D : \og Le stylo possède un défaut de fabrication \fg
\end{itemize}
\end{minipage}
\begin{minipage}{0.4\linewidth}
\begin{center}
\begin{tikzpicture}[sloped]
\node {.}
child {node {$A$}
child {node {$D$}
edge from parent
node[above] {...}
}
child {node {$\overline{D}$}
edge from parent
node[above] {...}
}
edge from parent
node[above] {...}
}
child[missing] {}
child { node {$B$}
child {node {$D$}
edge from parent
node[above] {...}
}
child {node {$\overline{D}$}
edge from parent
node[above] {...}
}
edge from parent
node[above] {...}
} ;
\end{tikzpicture}
\end{center}
\end{minipage}
\medskip
\begin{enumerate}
\item Compléter l'arbre de probabilité ci-contre
\item Interpréter puis donner les probabilités $P(A)$, $P(B)$, $P_A(D)$ et $P(B \cap D)$.
\item
\begin{enumerate}
\item Calculer la probabilité qu'un stylo provienne de l'atelier A et possède un défaut de fabrication.
\item En déduire que la probabilité qu'un stylo possède un défaut de fabrication est de $0.49$.
\end{enumerate}
\item On prélève un stylo au hasard avec un défaut. Quelle est la probabilité qu'il vienne de l'atelier A?
\end{enumerate}
\bigskip
\textbf{Partie B}
\medskip
Dans cette partie, on suppose que 49.0\,\% des stylos possèdent un défaut de fabrication.
L'entreprise confectionne des paquets contenant chacun $4$~stylos.
Le fait qu'un stylo possède ou non un défaut de fabrication est indépendant des autres stylos.
On appelle $X$ la variable aléatoire donnant pour un paquet le nombre de stylos qui possèdent un défaut de fabrication.
On admet que la variable aléatoire $X$ suit une loi binomiale.
\medskip
\begin{enumerate}
\setcounter{enumi}{4}
\item Avec quelle loi peut-on modéliser $X$. Préciser les paramètres.
\item Calculer et interpréter la probabilité $P(X = 14)$.
\item Le directeur de l'entreprise affirme qu'il y a plus d'une chance sur deux qu'un paquet ne comporte aucun stylo défectueux. A-t-il raison ?
\item Combien de stylos peut-on espérer avoir en moyenne?
\end{enumerate}
\pagebreak
\end{exercise}
\begin{solution}
\begin{enumerate}
\item
\begin{center}
\begin{tikzpicture}[sloped]
\node {.}
child {node {$A$}
child {node {$D$}
edge from parent
node[above] {0.58}
}
child {node {$\overline{D}$}
edge from parent
node[above] {0.42}
}
edge from parent
node[above] {0.81}
}
child[missing] {}
child { node {$B$}
child {node {$D$}
edge from parent
node[above] {0.13}
}
child {node {$\overline{D}$}
edge from parent
node[above] {0.87}
}
edge from parent
node[above] {0.19}
} ;
\end{tikzpicture}
\end{center}
\item
\begin{itemize}
\item Probabilité que le stylo vienne de l'atelier A
\[
P(A) = 0.81
\]
\item Probabilité que le stylo vienne de l'atelier B
\[
P(B) = 0.19
\]
\item Probabilité que le stylo ait un défaut sachant qu'il vient de l'atelier A.
\[
P_A(D) = 0.58
\]
\item Probabilité que le stylo vienne de l'atelier B et qu'il ait un défaut.
\[
P(D \cap D) = 0.02
\]
\end{itemize}
\item
\begin{enumerate}
\item Probabilité qu'un stylo vienne de l'atelier A et qu'il ait un defaut
\[
P(A\cap D) = P(A) \times P_A(D) = 0.81 \times 0.58 = 0.47
\]
\item Probabilité que le stylo ai un défaut de fabrication.
\[
P(D) = P(A\cap D) + P(B\cap D) = 0.47 + 0.02 = 0.49
\]
\end{enumerate}
\item Probabilité qu'il vienne de l'atelier A sachant qu'il a un defaut
\[
P_D(A) = \frac{P(A\cap D)}{P(D)} = \frac{0.47}{0.49} = 0.96
\]
\item $X$ peut être modélisée par une loi binomiale de paramètres $n=20$ et $p=0.49$.
\item (\textit{par de correction automatique disponible pour le résultat final}
\[
P(X = 14) = \coefBino{20}{14}\times 0.49^{14} \times 0.51^{6}
\]
\item (\textit{par de correction automatique disponible pour le résultat final}
Il faut calculer la probabilité qu'il y ait 0 stylo avec un defaut.
\[
P(X = 0) = \coefBino{20}{0}\times 0.49^{0} \times 0.51^{20}
\]
Puis comparer ce nombre à 0,5.
\item Il faut calculer l'espérance
\[
E[X] = n\times p = 20 \times 0.49 = 9.8
\]
\end{enumerate}
\end{solution}
\end{document}
%%% Local Variables:
%%% mode: latex
%%% TeX-master: "master"
%%% End:

View File

@ -0,0 +1,367 @@
\documentclass[a4paper,10pt]{article}
\usepackage{myXsim}
% Title Page
\title{DM1 \hfill HOKELEKLI Damla}
\tribe{Maths complémentaire}
\date{\hfillÀ render pour le jeudi 27 mai}
\xsimsetup{
solution/print = true
}
\begin{document}
\maketitle
Les valeurs des exercices sont générés automatiquement. Si une valeur a un nombre adhérant de chiffres après la virgule, vous pouvez l'arrondir à l'entier le plus proche.
\begin{exercise}[subtitle={Optimisation de matière}]
\begin{minipage}{0.6\textwidth}
On se propose de fabriquer avec le moins de tôle possible une citerne fermée en forme de parallélépipède rectangle dont le volume intérieur doit être de $10m^3$. La longueur est aussi fixée à $2m$ par le cahier des charges.
On peut donc faire varier uniquement la largeur (notée $x$) et la hauteur (notée $h$) de la cuve.
\end{minipage}
\hfill
\begin{minipage}{0.3\textwidth}
\begin{tikzpicture}
\pgfmathsetmacro{\cubex}{3}
\pgfmathsetmacro{\cubey}{1}
\pgfmathsetmacro{\cubez}{2}
\draw[black,fill=gray] (0,0,0) -- ++(-\cubex,0,0) -- ++(0,-\cubey,0) node [midway, left] {$h$} -- ++(\cubex,0,0) node [midway, below] {$x$} -- cycle;
\draw[black,fill=gray] (0,0,0) -- ++(0,0,-\cubez) -- ++(0,-\cubey,0) -- ++(0,0,\cubez) node [midway, right] {$2m$} -- cycle;
\draw[black,fill=gray] (0,0,0) -- ++(-\cubex,0,0) -- ++(0,0,-\cubez) -- ++(\cubex,0,0) -- cycle;
\end{tikzpicture}
\end{minipage}
\begin{enumerate}
\item Expliquer pourquoi quand la largeur $x$ change, la hauteur $h$ doit elle aussi changer pour respecter les contraintes.
\item Démontrer que l'on doit avoir $h = \dfrac{5}{x}$.
\item On note $S(x)$ l'aire totale de la citerne (c'est à dire la somme des aires des six faces). Montrer que l'on peut écrire
\[
S(x) = 4x + 10 + \frac{20}{x}
\]
\item Démontrer que
\[
S(x) = \frac{4x^2 + 10x + 20}{x}
\]
\item Démontrer que
\[
S'(x) = \frac{4x^2 - 20}{x^2}
\]
\item En déduire le tableau de variation de $S(x)$ sur $\intOF{0}{10}$.
\item Déterminer les valeurs de $x$ et $h$ correspondant à une utilisation minimal de tôle.
\end{enumerate}
\end{exercise}
\begin{solution}
\begin{enumerate}
\item Le volume étant fixe si l'on fait varier $x$, $h$ doit aussi varier.
\begin{itemize}
\item Si $x = 2$ alors conserver un volume de $V=10$, $h$ doit être égale à $5 / 2$
\item Si $x = 3$ alors conserver un volume de $V=10$, $h$ doit être égale à $5 / 3$
\end{itemize}
\item Pour calculer le volume, on a
\begin{eqnarray*}
V &=& h\times x \times 2 \\
10 &=& h\times x \times 2 \\
x &=& \frac{10}{h\times 2} = \frac{5}{h}
\end{eqnarray*}
\item Pour calculer la surface totale, on ajoute la surface de chaque face. On a donc le calcul suivant
\begin{eqnarray*}
S(x) &=& x\times h \times 2 + x\times2\times2 + h\times 2\times 2\\
S(x) &=& x\times \frac{5}{x} \times 2 + x\times2\times2 + \frac{5}{x}\times 2\times 2\\
S(x) &=& 4x + 10 + \frac{20}{x}
\end{eqnarray*}
\item Pour trouver cette nouvelle forme, on met chaque élément sur le même dénominateur
\begin{eqnarray*}
S(x) &=& 4x + 10 + \frac{20}{x}\\
S(x) &=& \frac{4x\times x}{x} + \frac{10\times x}{x} + \frac{20}{x}\\
S(x) &=& \frac{4x^2 + 10x + 20}{x}
\end{eqnarray*}
\item On retrouve la formule $\frac{u}{v}$ à dériver
\[
u(x) = 4x^2 + 10x + 20 \Rightarrow u'(x) = 8x + 10
\]
\[
v(x) = x \Rightarrow v'(x) = 1
\]
Donc au numérateur on obtient
\begin{eqnarray*}
u'(x)\times v(x) - u(x)\times v'(x) &=& (8x + 10)\times x - (4x^2 + 10x + 20)\times 1\\
&=& 4x^2 - 20
\end{eqnarray*}
Donc
\[
S'(x) = \frac{4x^2 - 20}{x^2}
\]
\item Tableau de variations de $S$
\begin{itemize}
\item Valeur interdite: $x^2 = 0 \equiv x = 0$
\item Signe de $4x^2 - 20$: c'est un polynôme du 2e degré
\[
\Delta = 320 > 0
\]
Il y a donc 2 racines
\[
x_1 = - 2.23606797749979 \qquad
x_2 = 2.23606797749979
\]
Et on sait que $4x^2 - 20$ est du signe de $a$ donc positif en dehors des racines
\item Le dénominateur $x^2$ est toujours positif.
\item Tableau de variations
\begin{tikzpicture}[baseline=(a.north)]
\tkzTabInit[lgt=3,espcl=3]{$x$/1,$4x^2 - 20$/1, $x^2$/1, $S'$/1, $S$/2}{$0$, $- 2.23606797749979$, $10$}
\tkzTabLine{d,-, z, +, }
\tkzTabLine{d,+, , +, }
\tkzTabLine{d,-, z, +, }
\tkzTabVar{D+/ , -/ , +/ }
\end{tikzpicture}
\end{itemize}
\item On a donc une surface minimal pour $x=2.23606797749979$ et $h = 11.18033988749895$.
\end{enumerate}
\end{solution}
%%% Local Variables:
%%% mode: latex
%%% TeX-master: "master"
%%% End:
\begin{exercise}[subtitle={Bassin}]
Le tour d'un bassin au niveau du sol présente deux axes de symétrie : laxe des abscisses et la droite déquation $x=4$. Il est obtenu par symétrie de la courbe $\mathcal{C}_f$ sur $\intFF{0}{4}$$f$ est la fonction définie par
\[
f(x) = \left(- x^{2} + 5.6 x - 1.6\right) e^{- x} + 1.6
\]
On admet que sur $\intFF{0}{4}$ la fonction $f$ est positive.
\begin{enumerate}
\item Sur un repère, tracer l'allure de la courbe $\mathcal{C}_f$, les axes de symétries puis compléter pour dessiner la forme du bassin.
\item Montrer que la fonction $f$ admet comme primitive sur $\R$ la fonction $F$ définie par
\[
F(x) = 1.6 x + \left( x^{2} - 3.6 x - 2.0\right) e^{- x}
\]
\item Calculer la quantité $\ds \int_0^4 f(x) \; dx$, vous donnerez le résultat sous forme exacte. Interpréter le résultat et reportez cette quantité sur le graphique.
\item On considère que l'échelle de votre graphique est de 1unité pour 15m. Calculer l'aire du bassin. Vous donnerez un résultat arrondi au $m^2$ près.
\end{enumerate}
\end{exercise}
\begin{solution}
\begin{enumerate}
\item
\begin{tikzpicture}[baseline=(a.north), xscale=1, yscale=0.5]
\tkzInit[xmin=0,xmax=5,xstep=1,
ymin=0,ymax=10,ystep=1]
\tkzGrid
\tkzAxeXY
\tkzFct[domain=0:10,color=red,very thick]%
{ (-x**2 + 5.6*x - 1.6)*exp(-x) + 1.6 };
\end{tikzpicture}
\item Il faut dériver $F(x)$ et vérifier que $F'(x) = f(x)$.
\item $\ds \int_0^4 f(x) \; dx = F(4) - F(0) = 8.4 - \frac{0.399999999999999}{e^{4}}$
\item La quantité calculée à la question précédente se retrouve 4fois pour former le bassin. Il faut ensuite prendre en compte l'échelle, comme 1unité de longueur correspond à 15m, une unité d'air correspond à $15\times15 = 225m^2$. Ainsi l'aire du bassin est égale à
\[
(8.4 - \frac{0.399999999999999}{e^{4}})\times 4 \times 15^2 = 7553.000000
\]
\end{enumerate}
\end{solution}
%%% Local Variables:
%%% mode: latex
%%% TeX-master: "master"
%%% End:
\begin{exercise}[subtitle={Stylos}]
\emph{Les parties {\rm A} et {\rm B} de cet exercice sont indépendantes.}
\bigskip
\begin{minipage}{0.6\linewidth}
\textbf{Partie A}
\medskip
Deux ateliers A et B fabriquent des stylos pour une entreprise.
L'atelier A fabrique 47.0\,\% des stylos, et parmi ceux-là, 25.0\,\% possèdent un défaut de fabrication.
De plus, 23.0\,\% des stylos possèdent un défaut de fabrication et sortent de l'atelier B.
Un stylo est prélevé au hasard dans le stock de l'entreprise.
On considère les évènements suivants:
\begin{itemize}
\item A : \og Le stylo a été fabriqué par l'atelier A \fg
\item B : \og Le stylo a été fabriqué par l'atelier B \fg
\item D : \og Le stylo possède un défaut de fabrication \fg
\end{itemize}
\end{minipage}
\begin{minipage}{0.4\linewidth}
\begin{center}
\begin{tikzpicture}[sloped]
\node {.}
child {node {$A$}
child {node {$D$}
edge from parent
node[above] {...}
}
child {node {$\overline{D}$}
edge from parent
node[above] {...}
}
edge from parent
node[above] {...}
}
child[missing] {}
child { node {$B$}
child {node {$D$}
edge from parent
node[above] {...}
}
child {node {$\overline{D}$}
edge from parent
node[above] {...}
}
edge from parent
node[above] {...}
} ;
\end{tikzpicture}
\end{center}
\end{minipage}
\medskip
\begin{enumerate}
\item Compléter l'arbre de probabilité ci-contre
\item Interpréter puis donner les probabilités $P(A)$, $P(B)$, $P_A(D)$ et $P(B \cap D)$.
\item
\begin{enumerate}
\item Calculer la probabilité qu'un stylo provienne de l'atelier A et possède un défaut de fabrication.
\item En déduire que la probabilité qu'un stylo possède un défaut de fabrication est de $0.35$.
\end{enumerate}
\item On prélève un stylo au hasard avec un défaut. Quelle est la probabilité qu'il vienne de l'atelier A?
\end{enumerate}
\bigskip
\textbf{Partie B}
\medskip
Dans cette partie, on suppose que 35.0\,\% des stylos possèdent un défaut de fabrication.
L'entreprise confectionne des paquets contenant chacun $4$~stylos.
Le fait qu'un stylo possède ou non un défaut de fabrication est indépendant des autres stylos.
On appelle $X$ la variable aléatoire donnant pour un paquet le nombre de stylos qui possèdent un défaut de fabrication.
On admet que la variable aléatoire $X$ suit une loi binomiale.
\medskip
\begin{enumerate}
\setcounter{enumi}{4}
\item Avec quelle loi peut-on modéliser $X$. Préciser les paramètres.
\item Calculer et interpréter la probabilité $P(X = 17)$.
\item Le directeur de l'entreprise affirme qu'il y a plus d'une chance sur deux qu'un paquet ne comporte aucun stylo défectueux. A-t-il raison ?
\item Combien de stylos peut-on espérer avoir en moyenne?
\end{enumerate}
\pagebreak
\end{exercise}
\begin{solution}
\begin{enumerate}
\item
\begin{center}
\begin{tikzpicture}[sloped]
\node {.}
child {node {$A$}
child {node {$D$}
edge from parent
node[above] {0.25}
}
child {node {$\overline{D}$}
edge from parent
node[above] {0.75}
}
edge from parent
node[above] {0.47}
}
child[missing] {}
child { node {$B$}
child {node {$D$}
edge from parent
node[above] {0.44}
}
child {node {$\overline{D}$}
edge from parent
node[above] {0.56}
}
edge from parent
node[above] {0.53}
} ;
\end{tikzpicture}
\end{center}
\item
\begin{itemize}
\item Probabilité que le stylo vienne de l'atelier A
\[
P(A) = 0.47
\]
\item Probabilité que le stylo vienne de l'atelier B
\[
P(B) = 0.53
\]
\item Probabilité que le stylo ait un défaut sachant qu'il vient de l'atelier A.
\[
P_A(D) = 0.25
\]
\item Probabilité que le stylo vienne de l'atelier B et qu'il ait un défaut.
\[
P(D \cap D) = 0.23
\]
\end{itemize}
\item
\begin{enumerate}
\item Probabilité qu'un stylo vienne de l'atelier A et qu'il ait un defaut
\[
P(A\cap D) = P(A) \times P_A(D) = 0.47 \times 0.25 = 0.12
\]
\item Probabilité que le stylo ai un défaut de fabrication.
\[
P(D) = P(A\cap D) + P(B\cap D) = 0.12 + 0.23 = 0.35
\]
\end{enumerate}
\item Probabilité qu'il vienne de l'atelier A sachant qu'il a un defaut
\[
P_D(A) = \frac{P(A\cap D)}{P(D)} = \frac{0.12}{0.35} = 0.34
\]
\item $X$ peut être modélisée par une loi binomiale de paramètres $n=19$ et $p=0.35$.
\item (\textit{par de correction automatique disponible pour le résultat final}
\[
P(X = 17) = \coefBino{19}{17}\times 0.35^{17} \times 0.65^{2}
\]
\item (\textit{par de correction automatique disponible pour le résultat final}
Il faut calculer la probabilité qu'il y ait 0 stylo avec un defaut.
\[
P(X = 0) = \coefBino{19}{0}\times 0.35^{0} \times 0.65^{19}
\]
Puis comparer ce nombre à 0,5.
\item Il faut calculer l'espérance
\[
E[X] = n\times p = 19 \times 0.35 = 6.65
\]
\end{enumerate}
\end{solution}
\end{document}
%%% Local Variables:
%%% mode: latex
%%% TeX-master: "master"
%%% End:

View File

@ -0,0 +1,367 @@
\documentclass[a4paper,10pt]{article}
\usepackage{myXsim}
% Title Page
\title{DM1 \hfill KICHENASSAMY Kévin}
\tribe{Maths complémentaire}
\date{\hfillÀ render pour le jeudi 27 mai}
\xsimsetup{
solution/print = true
}
\begin{document}
\maketitle
Les valeurs des exercices sont générés automatiquement. Si une valeur a un nombre adhérant de chiffres après la virgule, vous pouvez l'arrondir à l'entier le plus proche.
\begin{exercise}[subtitle={Optimisation de matière}]
\begin{minipage}{0.6\textwidth}
On se propose de fabriquer avec le moins de tôle possible une citerne fermée en forme de parallélépipède rectangle dont le volume intérieur doit être de $16m^3$. La longueur est aussi fixée à $4m$ par le cahier des charges.
On peut donc faire varier uniquement la largeur (notée $x$) et la hauteur (notée $h$) de la cuve.
\end{minipage}
\hfill
\begin{minipage}{0.3\textwidth}
\begin{tikzpicture}
\pgfmathsetmacro{\cubex}{3}
\pgfmathsetmacro{\cubey}{1}
\pgfmathsetmacro{\cubez}{2}
\draw[black,fill=gray] (0,0,0) -- ++(-\cubex,0,0) -- ++(0,-\cubey,0) node [midway, left] {$h$} -- ++(\cubex,0,0) node [midway, below] {$x$} -- cycle;
\draw[black,fill=gray] (0,0,0) -- ++(0,0,-\cubez) -- ++(0,-\cubey,0) -- ++(0,0,\cubez) node [midway, right] {$4m$} -- cycle;
\draw[black,fill=gray] (0,0,0) -- ++(-\cubex,0,0) -- ++(0,0,-\cubez) -- ++(\cubex,0,0) -- cycle;
\end{tikzpicture}
\end{minipage}
\begin{enumerate}
\item Expliquer pourquoi quand la largeur $x$ change, la hauteur $h$ doit elle aussi changer pour respecter les contraintes.
\item Démontrer que l'on doit avoir $h = \dfrac{4}{x}$.
\item On note $S(x)$ l'aire totale de la citerne (c'est à dire la somme des aires des six faces). Montrer que l'on peut écrire
\[
S(x) = 8x + 8 + \frac{32}{x}
\]
\item Démontrer que
\[
S(x) = \frac{8x^2 + 8x + 32}{x}
\]
\item Démontrer que
\[
S'(x) = \frac{8x^2 - 32}{x^2}
\]
\item En déduire le tableau de variation de $S(x)$ sur $\intOF{0}{10}$.
\item Déterminer les valeurs de $x$ et $h$ correspondant à une utilisation minimal de tôle.
\end{enumerate}
\end{exercise}
\begin{solution}
\begin{enumerate}
\item Le volume étant fixe si l'on fait varier $x$, $h$ doit aussi varier.
\begin{itemize}
\item Si $x = 2$ alors conserver un volume de $V=16$, $h$ doit être égale à $4 / 2$
\item Si $x = 3$ alors conserver un volume de $V=16$, $h$ doit être égale à $4 / 3$
\end{itemize}
\item Pour calculer le volume, on a
\begin{eqnarray*}
V &=& h\times x \times 4 \\
16 &=& h\times x \times 4 \\
x &=& \frac{16}{h\times 4} = \frac{4}{h}
\end{eqnarray*}
\item Pour calculer la surface totale, on ajoute la surface de chaque face. On a donc le calcul suivant
\begin{eqnarray*}
S(x) &=& x\times h \times 2 + x\times4\times2 + h\times 4\times 2\\
S(x) &=& x\times \frac{4}{x} \times 2 + x\times4\times2 + \frac{4}{x}\times 4\times 2\\
S(x) &=& 8x + 8 + \frac{32}{x}
\end{eqnarray*}
\item Pour trouver cette nouvelle forme, on met chaque élément sur le même dénominateur
\begin{eqnarray*}
S(x) &=& 8x + 8 + \frac{32}{x}\\
S(x) &=& \frac{8x\times x}{x} + \frac{8\times x}{x} + \frac{32}{x}\\
S(x) &=& \frac{8x^2 + 8x + 32}{x}
\end{eqnarray*}
\item On retrouve la formule $\frac{u}{v}$ à dériver
\[
u(x) = 8x^2 + 8x + 32 \Rightarrow u'(x) = 16x + 8
\]
\[
v(x) = x \Rightarrow v'(x) = 1
\]
Donc au numérateur on obtient
\begin{eqnarray*}
u'(x)\times v(x) - u(x)\times v'(x) &=& (16x + 8)\times x - (8x^2 + 8x + 32)\times 1\\
&=& 8x^2 - 32
\end{eqnarray*}
Donc
\[
S'(x) = \frac{8x^2 - 32}{x^2}
\]
\item Tableau de variations de $S$
\begin{itemize}
\item Valeur interdite: $x^2 = 0 \equiv x = 0$
\item Signe de $8x^2 - 32$: c'est un polynôme du 2e degré
\[
\Delta = 1024 > 0
\]
Il y a donc 2 racines
\[
x_1 = - 2 \qquad
x_2 = 2
\]
Et on sait que $8x^2 - 32$ est du signe de $a$ donc positif en dehors des racines
\item Le dénominateur $x^2$ est toujours positif.
\item Tableau de variations
\begin{tikzpicture}[baseline=(a.north)]
\tkzTabInit[lgt=3,espcl=3]{$x$/1,$8x^2 - 32$/1, $x^2$/1, $S'$/1, $S$/2}{$0$, $- 2$, $10$}
\tkzTabLine{d,-, z, +, }
\tkzTabLine{d,+, , +, }
\tkzTabLine{d,-, z, +, }
\tkzTabVar{D+/ , -/ , +/ }
\end{tikzpicture}
\end{itemize}
\item On a donc une surface minimal pour $x=2$ et $h = 8$.
\end{enumerate}
\end{solution}
%%% Local Variables:
%%% mode: latex
%%% TeX-master: "master"
%%% End:
\begin{exercise}[subtitle={Bassin}]
Le tour d'un bassin au niveau du sol présente deux axes de symétrie : laxe des abscisses et la droite déquation $x=4$. Il est obtenu par symétrie de la courbe $\mathcal{C}_f$ sur $\intFF{0}{4}$$f$ est la fonction définie par
\[
f(x) = \left(- x^{2} + 0.4 x - 6.0\right) e^{- x} + 6.0
\]
On admet que sur $\intFF{0}{4}$ la fonction $f$ est positive.
\begin{enumerate}
\item Sur un repère, tracer l'allure de la courbe $\mathcal{C}_f$, les axes de symétries puis compléter pour dessiner la forme du bassin.
\item Montrer que la fonction $f$ admet comme primitive sur $\R$ la fonction $F$ définie par
\[
F(x) = 6.0 x + \left( x^{2} + 1.6 x + 7.6\right) e^{- x}
\]
\item Calculer la quantité $\ds \int_0^4 f(x) \; dx$, vous donnerez le résultat sous forme exacte. Interpréter le résultat et reportez cette quantité sur le graphique.
\item On considère que l'échelle de votre graphique est de 1unité pour 15m. Calculer l'aire du bassin. Vous donnerez un résultat arrondi au $m^2$ près.
\end{enumerate}
\end{exercise}
\begin{solution}
\begin{enumerate}
\item
\begin{tikzpicture}[baseline=(a.north), xscale=1, yscale=0.5]
\tkzInit[xmin=0,xmax=5,xstep=1,
ymin=0,ymax=10,ystep=1]
\tkzGrid
\tkzAxeXY
\tkzFct[domain=0:10,color=red,very thick]%
{ (-x**2 + 0.4*x - 6.0)*exp(-x) + 6.0 };
\end{tikzpicture}
\item Il faut dériver $F(x)$ et vérifier que $F'(x) = f(x)$.
\item $\ds \int_0^4 f(x) \; dx = F(4) - F(0) = \frac{30.0}{e^{4}} + 16.4$
\item La quantité calculée à la question précédente se retrouve 4fois pour former le bassin. Il faut ensuite prendre en compte l'échelle, comme 1unité de longueur correspond à 15m, une unité d'air correspond à $15\times15 = 225m^2$. Ainsi l'aire du bassin est égale à
\[
(\frac{30.0}{e^{4}} + 16.4)\times 4 \times 15^2 = 15255.00000
\]
\end{enumerate}
\end{solution}
%%% Local Variables:
%%% mode: latex
%%% TeX-master: "master"
%%% End:
\begin{exercise}[subtitle={Stylos}]
\emph{Les parties {\rm A} et {\rm B} de cet exercice sont indépendantes.}
\bigskip
\begin{minipage}{0.6\linewidth}
\textbf{Partie A}
\medskip
Deux ateliers A et B fabriquent des stylos pour une entreprise.
L'atelier A fabrique 84.0\,\% des stylos, et parmi ceux-là, 85.0\,\% possèdent un défaut de fabrication.
De plus, 4.0\,\% des stylos possèdent un défaut de fabrication et sortent de l'atelier B.
Un stylo est prélevé au hasard dans le stock de l'entreprise.
On considère les évènements suivants:
\begin{itemize}
\item A : \og Le stylo a été fabriqué par l'atelier A \fg
\item B : \og Le stylo a été fabriqué par l'atelier B \fg
\item D : \og Le stylo possède un défaut de fabrication \fg
\end{itemize}
\end{minipage}
\begin{minipage}{0.4\linewidth}
\begin{center}
\begin{tikzpicture}[sloped]
\node {.}
child {node {$A$}
child {node {$D$}
edge from parent
node[above] {...}
}
child {node {$\overline{D}$}
edge from parent
node[above] {...}
}
edge from parent
node[above] {...}
}
child[missing] {}
child { node {$B$}
child {node {$D$}
edge from parent
node[above] {...}
}
child {node {$\overline{D}$}
edge from parent
node[above] {...}
}
edge from parent
node[above] {...}
} ;
\end{tikzpicture}
\end{center}
\end{minipage}
\medskip
\begin{enumerate}
\item Compléter l'arbre de probabilité ci-contre
\item Interpréter puis donner les probabilités $P(A)$, $P(B)$, $P_A(D)$ et $P(B \cap D)$.
\item
\begin{enumerate}
\item Calculer la probabilité qu'un stylo provienne de l'atelier A et possède un défaut de fabrication.
\item En déduire que la probabilité qu'un stylo possède un défaut de fabrication est de $0.75$.
\end{enumerate}
\item On prélève un stylo au hasard avec un défaut. Quelle est la probabilité qu'il vienne de l'atelier A?
\end{enumerate}
\bigskip
\textbf{Partie B}
\medskip
Dans cette partie, on suppose que 75.0\,\% des stylos possèdent un défaut de fabrication.
L'entreprise confectionne des paquets contenant chacun $4$~stylos.
Le fait qu'un stylo possède ou non un défaut de fabrication est indépendant des autres stylos.
On appelle $X$ la variable aléatoire donnant pour un paquet le nombre de stylos qui possèdent un défaut de fabrication.
On admet que la variable aléatoire $X$ suit une loi binomiale.
\medskip
\begin{enumerate}
\setcounter{enumi}{4}
\item Avec quelle loi peut-on modéliser $X$. Préciser les paramètres.
\item Calculer et interpréter la probabilité $P(X = 14)$.
\item Le directeur de l'entreprise affirme qu'il y a plus d'une chance sur deux qu'un paquet ne comporte aucun stylo défectueux. A-t-il raison ?
\item Combien de stylos peut-on espérer avoir en moyenne?
\end{enumerate}
\pagebreak
\end{exercise}
\begin{solution}
\begin{enumerate}
\item
\begin{center}
\begin{tikzpicture}[sloped]
\node {.}
child {node {$A$}
child {node {$D$}
edge from parent
node[above] {0.85}
}
child {node {$\overline{D}$}
edge from parent
node[above] {0.15}
}
edge from parent
node[above] {0.84}
}
child[missing] {}
child { node {$B$}
child {node {$D$}
edge from parent
node[above] {0.26}
}
child {node {$\overline{D}$}
edge from parent
node[above] {0.74}
}
edge from parent
node[above] {0.16}
} ;
\end{tikzpicture}
\end{center}
\item
\begin{itemize}
\item Probabilité que le stylo vienne de l'atelier A
\[
P(A) = 0.84
\]
\item Probabilité que le stylo vienne de l'atelier B
\[
P(B) = 0.16
\]
\item Probabilité que le stylo ait un défaut sachant qu'il vient de l'atelier A.
\[
P_A(D) = 0.85
\]
\item Probabilité que le stylo vienne de l'atelier B et qu'il ait un défaut.
\[
P(D \cap D) = 0.04
\]
\end{itemize}
\item
\begin{enumerate}
\item Probabilité qu'un stylo vienne de l'atelier A et qu'il ait un defaut
\[
P(A\cap D) = P(A) \times P_A(D) = 0.84 \times 0.85 = 0.71
\]
\item Probabilité que le stylo ai un défaut de fabrication.
\[
P(D) = P(A\cap D) + P(B\cap D) = 0.71 + 0.04 = 0.75
\]
\end{enumerate}
\item Probabilité qu'il vienne de l'atelier A sachant qu'il a un defaut
\[
P_D(A) = \frac{P(A\cap D)}{P(D)} = \frac{0.71}{0.75} = 0.95
\]
\item $X$ peut être modélisée par une loi binomiale de paramètres $n=17$ et $p=0.75$.
\item (\textit{par de correction automatique disponible pour le résultat final}
\[
P(X = 14) = \coefBino{17}{14}\times 0.75^{14} \times 0.25^{3}
\]
\item (\textit{par de correction automatique disponible pour le résultat final}
Il faut calculer la probabilité qu'il y ait 0 stylo avec un defaut.
\[
P(X = 0) = \coefBino{17}{0}\times 0.75^{0} \times 0.25^{17}
\]
Puis comparer ce nombre à 0,5.
\item Il faut calculer l'espérance
\[
E[X] = n\times p = 17 \times 0.75 = 12.75
\]
\end{enumerate}
\end{solution}
\end{document}
%%% Local Variables:
%%% mode: latex
%%% TeX-master: "master"
%%% End:

View File

@ -0,0 +1,367 @@
\documentclass[a4paper,10pt]{article}
\usepackage{myXsim}
% Title Page
\title{DM1 \hfill MATHIEU Allan}
\tribe{Maths complémentaire}
\date{\hfillÀ render pour le jeudi 27 mai}
\xsimsetup{
solution/print = true
}
\begin{document}
\maketitle
Les valeurs des exercices sont générés automatiquement. Si une valeur a un nombre adhérant de chiffres après la virgule, vous pouvez l'arrondir à l'entier le plus proche.
\begin{exercise}[subtitle={Optimisation de matière}]
\begin{minipage}{0.6\textwidth}
On se propose de fabriquer avec le moins de tôle possible une citerne fermée en forme de parallélépipède rectangle dont le volume intérieur doit être de $40m^3$. La longueur est aussi fixée à $4m$ par le cahier des charges.
On peut donc faire varier uniquement la largeur (notée $x$) et la hauteur (notée $h$) de la cuve.
\end{minipage}
\hfill
\begin{minipage}{0.3\textwidth}
\begin{tikzpicture}
\pgfmathsetmacro{\cubex}{3}
\pgfmathsetmacro{\cubey}{1}
\pgfmathsetmacro{\cubez}{2}
\draw[black,fill=gray] (0,0,0) -- ++(-\cubex,0,0) -- ++(0,-\cubey,0) node [midway, left] {$h$} -- ++(\cubex,0,0) node [midway, below] {$x$} -- cycle;
\draw[black,fill=gray] (0,0,0) -- ++(0,0,-\cubez) -- ++(0,-\cubey,0) -- ++(0,0,\cubez) node [midway, right] {$4m$} -- cycle;
\draw[black,fill=gray] (0,0,0) -- ++(-\cubex,0,0) -- ++(0,0,-\cubez) -- ++(\cubex,0,0) -- cycle;
\end{tikzpicture}
\end{minipage}
\begin{enumerate}
\item Expliquer pourquoi quand la largeur $x$ change, la hauteur $h$ doit elle aussi changer pour respecter les contraintes.
\item Démontrer que l'on doit avoir $h = \dfrac{10}{x}$.
\item On note $S(x)$ l'aire totale de la citerne (c'est à dire la somme des aires des six faces). Montrer que l'on peut écrire
\[
S(x) = 8x + 20 + \frac{80}{x}
\]
\item Démontrer que
\[
S(x) = \frac{8x^2 + 20x + 80}{x}
\]
\item Démontrer que
\[
S'(x) = \frac{8x^2 - 80}{x^2}
\]
\item En déduire le tableau de variation de $S(x)$ sur $\intOF{0}{10}$.
\item Déterminer les valeurs de $x$ et $h$ correspondant à une utilisation minimal de tôle.
\end{enumerate}
\end{exercise}
\begin{solution}
\begin{enumerate}
\item Le volume étant fixe si l'on fait varier $x$, $h$ doit aussi varier.
\begin{itemize}
\item Si $x = 2$ alors conserver un volume de $V=40$, $h$ doit être égale à $10 / 2$
\item Si $x = 3$ alors conserver un volume de $V=40$, $h$ doit être égale à $10 / 3$
\end{itemize}
\item Pour calculer le volume, on a
\begin{eqnarray*}
V &=& h\times x \times 4 \\
40 &=& h\times x \times 4 \\
x &=& \frac{40}{h\times 4} = \frac{10}{h}
\end{eqnarray*}
\item Pour calculer la surface totale, on ajoute la surface de chaque face. On a donc le calcul suivant
\begin{eqnarray*}
S(x) &=& x\times h \times 2 + x\times4\times2 + h\times 4\times 2\\
S(x) &=& x\times \frac{10}{x} \times 2 + x\times4\times2 + \frac{10}{x}\times 4\times 2\\
S(x) &=& 8x + 20 + \frac{80}{x}
\end{eqnarray*}
\item Pour trouver cette nouvelle forme, on met chaque élément sur le même dénominateur
\begin{eqnarray*}
S(x) &=& 8x + 20 + \frac{80}{x}\\
S(x) &=& \frac{8x\times x}{x} + \frac{20\times x}{x} + \frac{80}{x}\\
S(x) &=& \frac{8x^2 + 20x + 80}{x}
\end{eqnarray*}
\item On retrouve la formule $\frac{u}{v}$ à dériver
\[
u(x) = 8x^2 + 20x + 80 \Rightarrow u'(x) = 16x + 20
\]
\[
v(x) = x \Rightarrow v'(x) = 1
\]
Donc au numérateur on obtient
\begin{eqnarray*}
u'(x)\times v(x) - u(x)\times v'(x) &=& (16x + 20)\times x - (8x^2 + 20x + 80)\times 1\\
&=& 8x^2 - 80
\end{eqnarray*}
Donc
\[
S'(x) = \frac{8x^2 - 80}{x^2}
\]
\item Tableau de variations de $S$
\begin{itemize}
\item Valeur interdite: $x^2 = 0 \equiv x = 0$
\item Signe de $8x^2 - 80$: c'est un polynôme du 2e degré
\[
\Delta = 2560 > 0
\]
Il y a donc 2 racines
\[
x_1 = - 3.1622776601683795 \qquad
x_2 = 3.1622776601683795
\]
Et on sait que $8x^2 - 80$ est du signe de $a$ donc positif en dehors des racines
\item Le dénominateur $x^2$ est toujours positif.
\item Tableau de variations
\begin{tikzpicture}[baseline=(a.north)]
\tkzTabInit[lgt=3,espcl=3]{$x$/1,$8x^2 - 80$/1, $x^2$/1, $S'$/1, $S$/2}{$0$, $- 3.1622776601683795$, $10$}
\tkzTabLine{d,-, z, +, }
\tkzTabLine{d,+, , +, }
\tkzTabLine{d,-, z, +, }
\tkzTabVar{D+/ , -/ , +/ }
\end{tikzpicture}
\end{itemize}
\item On a donc une surface minimal pour $x=3.1622776601683795$ et $h = 31.6227766016837950$.
\end{enumerate}
\end{solution}
%%% Local Variables:
%%% mode: latex
%%% TeX-master: "master"
%%% End:
\begin{exercise}[subtitle={Bassin}]
Le tour d'un bassin au niveau du sol présente deux axes de symétrie : laxe des abscisses et la droite déquation $x=4$. Il est obtenu par symétrie de la courbe $\mathcal{C}_f$ sur $\intFF{0}{4}$$f$ est la fonction définie par
\[
f(x) = \left(- x^{2} + 6.1 x - 9.3\right) e^{- x} + 9.3
\]
On admet que sur $\intFF{0}{4}$ la fonction $f$ est positive.
\begin{enumerate}
\item Sur un repère, tracer l'allure de la courbe $\mathcal{C}_f$, les axes de symétries puis compléter pour dessiner la forme du bassin.
\item Montrer que la fonction $f$ admet comme primitive sur $\R$ la fonction $F$ définie par
\[
F(x) = 9.3 x + \left( x^{2} - 4.1 x + 5.2\right) e^{- x}
\]
\item Calculer la quantité $\ds \int_0^4 f(x) \; dx$, vous donnerez le résultat sous forme exacte. Interpréter le résultat et reportez cette quantité sur le graphique.
\item On considère que l'échelle de votre graphique est de 1unité pour 15m. Calculer l'aire du bassin. Vous donnerez un résultat arrondi au $m^2$ près.
\end{enumerate}
\end{exercise}
\begin{solution}
\begin{enumerate}
\item
\begin{tikzpicture}[baseline=(a.north), xscale=1, yscale=0.5]
\tkzInit[xmin=0,xmax=5,xstep=1,
ymin=0,ymax=10,ystep=1]
\tkzGrid
\tkzAxeXY
\tkzFct[domain=0:10,color=red,very thick]%
{ (-x**2 + 6.1*x - 9.3)*exp(-x) + 9.3 };
\end{tikzpicture}
\item Il faut dériver $F(x)$ et vérifier que $F'(x) = f(x)$.
\item $\ds \int_0^4 f(x) \; dx = F(4) - F(0) = \frac{4.8}{e^{4}} + 32.0$
\item La quantité calculée à la question précédente se retrouve 4fois pour former le bassin. Il faut ensuite prendre en compte l'échelle, comme 1unité de longueur correspond à 15m, une unité d'air correspond à $15\times15 = 225m^2$. Ainsi l'aire du bassin est égale à
\[
(\frac{4.8}{e^{4}} + 32.0)\times 4 \times 15^2 = 28879.00000
\]
\end{enumerate}
\end{solution}
%%% Local Variables:
%%% mode: latex
%%% TeX-master: "master"
%%% End:
\begin{exercise}[subtitle={Stylos}]
\emph{Les parties {\rm A} et {\rm B} de cet exercice sont indépendantes.}
\bigskip
\begin{minipage}{0.6\linewidth}
\textbf{Partie A}
\medskip
Deux ateliers A et B fabriquent des stylos pour une entreprise.
L'atelier A fabrique 92.0\,\% des stylos, et parmi ceux-là, 47.0\,\% possèdent un défaut de fabrication.
De plus, 1.0\,\% des stylos possèdent un défaut de fabrication et sortent de l'atelier B.
Un stylo est prélevé au hasard dans le stock de l'entreprise.
On considère les évènements suivants:
\begin{itemize}
\item A : \og Le stylo a été fabriqué par l'atelier A \fg
\item B : \og Le stylo a été fabriqué par l'atelier B \fg
\item D : \og Le stylo possède un défaut de fabrication \fg
\end{itemize}
\end{minipage}
\begin{minipage}{0.4\linewidth}
\begin{center}
\begin{tikzpicture}[sloped]
\node {.}
child {node {$A$}
child {node {$D$}
edge from parent
node[above] {...}
}
child {node {$\overline{D}$}
edge from parent
node[above] {...}
}
edge from parent
node[above] {...}
}
child[missing] {}
child { node {$B$}
child {node {$D$}
edge from parent
node[above] {...}
}
child {node {$\overline{D}$}
edge from parent
node[above] {...}
}
edge from parent
node[above] {...}
} ;
\end{tikzpicture}
\end{center}
\end{minipage}
\medskip
\begin{enumerate}
\item Compléter l'arbre de probabilité ci-contre
\item Interpréter puis donner les probabilités $P(A)$, $P(B)$, $P_A(D)$ et $P(B \cap D)$.
\item
\begin{enumerate}
\item Calculer la probabilité qu'un stylo provienne de l'atelier A et possède un défaut de fabrication.
\item En déduire que la probabilité qu'un stylo possède un défaut de fabrication est de $0.44$.
\end{enumerate}
\item On prélève un stylo au hasard avec un défaut. Quelle est la probabilité qu'il vienne de l'atelier A?
\end{enumerate}
\bigskip
\textbf{Partie B}
\medskip
Dans cette partie, on suppose que 44.0\,\% des stylos possèdent un défaut de fabrication.
L'entreprise confectionne des paquets contenant chacun $4$~stylos.
Le fait qu'un stylo possède ou non un défaut de fabrication est indépendant des autres stylos.
On appelle $X$ la variable aléatoire donnant pour un paquet le nombre de stylos qui possèdent un défaut de fabrication.
On admet que la variable aléatoire $X$ suit une loi binomiale.
\medskip
\begin{enumerate}
\setcounter{enumi}{4}
\item Avec quelle loi peut-on modéliser $X$. Préciser les paramètres.
\item Calculer et interpréter la probabilité $P(X = 15)$.
\item Le directeur de l'entreprise affirme qu'il y a plus d'une chance sur deux qu'un paquet ne comporte aucun stylo défectueux. A-t-il raison ?
\item Combien de stylos peut-on espérer avoir en moyenne?
\end{enumerate}
\pagebreak
\end{exercise}
\begin{solution}
\begin{enumerate}
\item
\begin{center}
\begin{tikzpicture}[sloped]
\node {.}
child {node {$A$}
child {node {$D$}
edge from parent
node[above] {0.47}
}
child {node {$\overline{D}$}
edge from parent
node[above] {0.53}
}
edge from parent
node[above] {0.92}
}
child[missing] {}
child { node {$B$}
child {node {$D$}
edge from parent
node[above] {0.17}
}
child {node {$\overline{D}$}
edge from parent
node[above] {0.83}
}
edge from parent
node[above] {0.08}
} ;
\end{tikzpicture}
\end{center}
\item
\begin{itemize}
\item Probabilité que le stylo vienne de l'atelier A
\[
P(A) = 0.92
\]
\item Probabilité que le stylo vienne de l'atelier B
\[
P(B) = 0.08
\]
\item Probabilité que le stylo ait un défaut sachant qu'il vient de l'atelier A.
\[
P_A(D) = 0.47
\]
\item Probabilité que le stylo vienne de l'atelier B et qu'il ait un défaut.
\[
P(D \cap D) = 0.01
\]
\end{itemize}
\item
\begin{enumerate}
\item Probabilité qu'un stylo vienne de l'atelier A et qu'il ait un defaut
\[
P(A\cap D) = P(A) \times P_A(D) = 0.92 \times 0.47 = 0.43
\]
\item Probabilité que le stylo ai un défaut de fabrication.
\[
P(D) = P(A\cap D) + P(B\cap D) = 0.43 + 0.01 = 0.44
\]
\end{enumerate}
\item Probabilité qu'il vienne de l'atelier A sachant qu'il a un defaut
\[
P_D(A) = \frac{P(A\cap D)}{P(D)} = \frac{0.43}{0.44} = 0.98
\]
\item $X$ peut être modélisée par une loi binomiale de paramètres $n=17$ et $p=0.44$.
\item (\textit{par de correction automatique disponible pour le résultat final}
\[
P(X = 15) = \coefBino{17}{15}\times 0.44^{15} \times 0.56^{2}
\]
\item (\textit{par de correction automatique disponible pour le résultat final}
Il faut calculer la probabilité qu'il y ait 0 stylo avec un defaut.
\[
P(X = 0) = \coefBino{17}{0}\times 0.44^{0} \times 0.56^{17}
\]
Puis comparer ce nombre à 0,5.
\item Il faut calculer l'espérance
\[
E[X] = n\times p = 17 \times 0.44 = 7.48
\]
\end{enumerate}
\end{solution}
\end{document}
%%% Local Variables:
%%% mode: latex
%%% TeX-master: "master"
%%% End:

View File

@ -0,0 +1,367 @@
\documentclass[a4paper,10pt]{article}
\usepackage{myXsim}
% Title Page
\title{DM1 \hfill MOLINIER Annelise}
\tribe{Maths complémentaire}
\date{\hfillÀ render pour le jeudi 27 mai}
\xsimsetup{
solution/print = true
}
\begin{document}
\maketitle
Les valeurs des exercices sont générés automatiquement. Si une valeur a un nombre adhérant de chiffres après la virgule, vous pouvez l'arrondir à l'entier le plus proche.
\begin{exercise}[subtitle={Optimisation de matière}]
\begin{minipage}{0.6\textwidth}
On se propose de fabriquer avec le moins de tôle possible une citerne fermée en forme de parallélépipède rectangle dont le volume intérieur doit être de $25m^3$. La longueur est aussi fixée à $5m$ par le cahier des charges.
On peut donc faire varier uniquement la largeur (notée $x$) et la hauteur (notée $h$) de la cuve.
\end{minipage}
\hfill
\begin{minipage}{0.3\textwidth}
\begin{tikzpicture}
\pgfmathsetmacro{\cubex}{3}
\pgfmathsetmacro{\cubey}{1}
\pgfmathsetmacro{\cubez}{2}
\draw[black,fill=gray] (0,0,0) -- ++(-\cubex,0,0) -- ++(0,-\cubey,0) node [midway, left] {$h$} -- ++(\cubex,0,0) node [midway, below] {$x$} -- cycle;
\draw[black,fill=gray] (0,0,0) -- ++(0,0,-\cubez) -- ++(0,-\cubey,0) -- ++(0,0,\cubez) node [midway, right] {$5m$} -- cycle;
\draw[black,fill=gray] (0,0,0) -- ++(-\cubex,0,0) -- ++(0,0,-\cubez) -- ++(\cubex,0,0) -- cycle;
\end{tikzpicture}
\end{minipage}
\begin{enumerate}
\item Expliquer pourquoi quand la largeur $x$ change, la hauteur $h$ doit elle aussi changer pour respecter les contraintes.
\item Démontrer que l'on doit avoir $h = \dfrac{5}{x}$.
\item On note $S(x)$ l'aire totale de la citerne (c'est à dire la somme des aires des six faces). Montrer que l'on peut écrire
\[
S(x) = 10x + 10 + \frac{50}{x}
\]
\item Démontrer que
\[
S(x) = \frac{10x^2 + 10x + 50}{x}
\]
\item Démontrer que
\[
S'(x) = \frac{10x^2 - 50}{x^2}
\]
\item En déduire le tableau de variation de $S(x)$ sur $\intOF{0}{10}$.
\item Déterminer les valeurs de $x$ et $h$ correspondant à une utilisation minimal de tôle.
\end{enumerate}
\end{exercise}
\begin{solution}
\begin{enumerate}
\item Le volume étant fixe si l'on fait varier $x$, $h$ doit aussi varier.
\begin{itemize}
\item Si $x = 2$ alors conserver un volume de $V=25$, $h$ doit être égale à $5 / 2$
\item Si $x = 3$ alors conserver un volume de $V=25$, $h$ doit être égale à $5 / 3$
\end{itemize}
\item Pour calculer le volume, on a
\begin{eqnarray*}
V &=& h\times x \times 5 \\
25 &=& h\times x \times 5 \\
x &=& \frac{25}{h\times 5} = \frac{5}{h}
\end{eqnarray*}
\item Pour calculer la surface totale, on ajoute la surface de chaque face. On a donc le calcul suivant
\begin{eqnarray*}
S(x) &=& x\times h \times 2 + x\times5\times2 + h\times 5\times 2\\
S(x) &=& x\times \frac{5}{x} \times 2 + x\times5\times2 + \frac{5}{x}\times 5\times 2\\
S(x) &=& 10x + 10 + \frac{50}{x}
\end{eqnarray*}
\item Pour trouver cette nouvelle forme, on met chaque élément sur le même dénominateur
\begin{eqnarray*}
S(x) &=& 10x + 10 + \frac{50}{x}\\
S(x) &=& \frac{10x\times x}{x} + \frac{10\times x}{x} + \frac{50}{x}\\
S(x) &=& \frac{10x^2 + 10x + 50}{x}
\end{eqnarray*}
\item On retrouve la formule $\frac{u}{v}$ à dériver
\[
u(x) = 10x^2 + 10x + 50 \Rightarrow u'(x) = 20x + 10
\]
\[
v(x) = x \Rightarrow v'(x) = 1
\]
Donc au numérateur on obtient
\begin{eqnarray*}
u'(x)\times v(x) - u(x)\times v'(x) &=& (20x + 10)\times x - (10x^2 + 10x + 50)\times 1\\
&=& 10x^2 - 50
\end{eqnarray*}
Donc
\[
S'(x) = \frac{10x^2 - 50}{x^2}
\]
\item Tableau de variations de $S$
\begin{itemize}
\item Valeur interdite: $x^2 = 0 \equiv x = 0$
\item Signe de $10x^2 - 50$: c'est un polynôme du 2e degré
\[
\Delta = 2000 > 0
\]
Il y a donc 2 racines
\[
x_1 = - 2.23606797749979 \qquad
x_2 = 2.23606797749979
\]
Et on sait que $10x^2 - 50$ est du signe de $a$ donc positif en dehors des racines
\item Le dénominateur $x^2$ est toujours positif.
\item Tableau de variations
\begin{tikzpicture}[baseline=(a.north)]
\tkzTabInit[lgt=3,espcl=3]{$x$/1,$10x^2 - 50$/1, $x^2$/1, $S'$/1, $S$/2}{$0$, $- 2.23606797749979$, $10$}
\tkzTabLine{d,-, z, +, }
\tkzTabLine{d,+, , +, }
\tkzTabLine{d,-, z, +, }
\tkzTabVar{D+/ , -/ , +/ }
\end{tikzpicture}
\end{itemize}
\item On a donc une surface minimal pour $x=2.23606797749979$ et $h = 11.18033988749895$.
\end{enumerate}
\end{solution}
%%% Local Variables:
%%% mode: latex
%%% TeX-master: "master"
%%% End:
\begin{exercise}[subtitle={Bassin}]
Le tour d'un bassin au niveau du sol présente deux axes de symétrie : laxe des abscisses et la droite déquation $x=4$. Il est obtenu par symétrie de la courbe $\mathcal{C}_f$ sur $\intFF{0}{4}$$f$ est la fonction définie par
\[
f(x) = \left(- x^{2} + 6.9 x - 3.3\right) e^{- x} + 3.3
\]
On admet que sur $\intFF{0}{4}$ la fonction $f$ est positive.
\begin{enumerate}
\item Sur un repère, tracer l'allure de la courbe $\mathcal{C}_f$, les axes de symétries puis compléter pour dessiner la forme du bassin.
\item Montrer que la fonction $f$ admet comme primitive sur $\R$ la fonction $F$ définie par
\[
F(x) = 3.3 x + \left( x^{2} - 4.9 x - 1.6\right) e^{- x}
\]
\item Calculer la quantité $\ds \int_0^4 f(x) \; dx$, vous donnerez le résultat sous forme exacte. Interpréter le résultat et reportez cette quantité sur le graphique.
\item On considère que l'échelle de votre graphique est de 1unité pour 15m. Calculer l'aire du bassin. Vous donnerez un résultat arrondi au $m^2$ près.
\end{enumerate}
\end{exercise}
\begin{solution}
\begin{enumerate}
\item
\begin{tikzpicture}[baseline=(a.north), xscale=1, yscale=0.5]
\tkzInit[xmin=0,xmax=5,xstep=1,
ymin=0,ymax=10,ystep=1]
\tkzGrid
\tkzAxeXY
\tkzFct[domain=0:10,color=red,very thick]%
{ (-x**2 + 6.9*x - 3.3)*exp(-x) + 3.3 };
\end{tikzpicture}
\item Il faut dériver $F(x)$ et vérifier que $F'(x) = f(x)$.
\item $\ds \int_0^4 f(x) \; dx = F(4) - F(0) = 14.8 - \frac{5.2}{e^{4}}$
\item La quantité calculée à la question précédente se retrouve 4fois pour former le bassin. Il faut ensuite prendre en compte l'échelle, comme 1unité de longueur correspond à 15m, une unité d'air correspond à $15\times15 = 225m^2$. Ainsi l'aire du bassin est égale à
\[
(14.8 - \frac{5.2}{e^{4}})\times 4 \times 15^2 = 13234.00000
\]
\end{enumerate}
\end{solution}
%%% Local Variables:
%%% mode: latex
%%% TeX-master: "master"
%%% End:
\begin{exercise}[subtitle={Stylos}]
\emph{Les parties {\rm A} et {\rm B} de cet exercice sont indépendantes.}
\bigskip
\begin{minipage}{0.6\linewidth}
\textbf{Partie A}
\medskip
Deux ateliers A et B fabriquent des stylos pour une entreprise.
L'atelier A fabrique 56.00000000000001\,\% des stylos, et parmi ceux-là, 22.0\,\% possèdent un défaut de fabrication.
De plus, 10.0\,\% des stylos possèdent un défaut de fabrication et sortent de l'atelier B.
Un stylo est prélevé au hasard dans le stock de l'entreprise.
On considère les évènements suivants:
\begin{itemize}
\item A : \og Le stylo a été fabriqué par l'atelier A \fg
\item B : \og Le stylo a été fabriqué par l'atelier B \fg
\item D : \og Le stylo possède un défaut de fabrication \fg
\end{itemize}
\end{minipage}
\begin{minipage}{0.4\linewidth}
\begin{center}
\begin{tikzpicture}[sloped]
\node {.}
child {node {$A$}
child {node {$D$}
edge from parent
node[above] {...}
}
child {node {$\overline{D}$}
edge from parent
node[above] {...}
}
edge from parent
node[above] {...}
}
child[missing] {}
child { node {$B$}
child {node {$D$}
edge from parent
node[above] {...}
}
child {node {$\overline{D}$}
edge from parent
node[above] {...}
}
edge from parent
node[above] {...}
} ;
\end{tikzpicture}
\end{center}
\end{minipage}
\medskip
\begin{enumerate}
\item Compléter l'arbre de probabilité ci-contre
\item Interpréter puis donner les probabilités $P(A)$, $P(B)$, $P_A(D)$ et $P(B \cap D)$.
\item
\begin{enumerate}
\item Calculer la probabilité qu'un stylo provienne de l'atelier A et possède un défaut de fabrication.
\item En déduire que la probabilité qu'un stylo possède un défaut de fabrication est de $0.22$.
\end{enumerate}
\item On prélève un stylo au hasard avec un défaut. Quelle est la probabilité qu'il vienne de l'atelier A?
\end{enumerate}
\bigskip
\textbf{Partie B}
\medskip
Dans cette partie, on suppose que 22.0\,\% des stylos possèdent un défaut de fabrication.
L'entreprise confectionne des paquets contenant chacun $4$~stylos.
Le fait qu'un stylo possède ou non un défaut de fabrication est indépendant des autres stylos.
On appelle $X$ la variable aléatoire donnant pour un paquet le nombre de stylos qui possèdent un défaut de fabrication.
On admet que la variable aléatoire $X$ suit une loi binomiale.
\medskip
\begin{enumerate}
\setcounter{enumi}{4}
\item Avec quelle loi peut-on modéliser $X$. Préciser les paramètres.
\item Calculer et interpréter la probabilité $P(X = 19)$.
\item Le directeur de l'entreprise affirme qu'il y a plus d'une chance sur deux qu'un paquet ne comporte aucun stylo défectueux. A-t-il raison ?
\item Combien de stylos peut-on espérer avoir en moyenne?
\end{enumerate}
\pagebreak
\end{exercise}
\begin{solution}
\begin{enumerate}
\item
\begin{center}
\begin{tikzpicture}[sloped]
\node {.}
child {node {$A$}
child {node {$D$}
edge from parent
node[above] {0.22}
}
child {node {$\overline{D}$}
edge from parent
node[above] {0.78}
}
edge from parent
node[above] {0.56}
}
child[missing] {}
child { node {$B$}
child {node {$D$}
edge from parent
node[above] {0.23}
}
child {node {$\overline{D}$}
edge from parent
node[above] {0.77}
}
edge from parent
node[above] {0.44}
} ;
\end{tikzpicture}
\end{center}
\item
\begin{itemize}
\item Probabilité que le stylo vienne de l'atelier A
\[
P(A) = 0.56
\]
\item Probabilité que le stylo vienne de l'atelier B
\[
P(B) = 0.44
\]
\item Probabilité que le stylo ait un défaut sachant qu'il vient de l'atelier A.
\[
P_A(D) = 0.22
\]
\item Probabilité que le stylo vienne de l'atelier B et qu'il ait un défaut.
\[
P(D \cap D) = 0.1
\]
\end{itemize}
\item
\begin{enumerate}
\item Probabilité qu'un stylo vienne de l'atelier A et qu'il ait un defaut
\[
P(A\cap D) = P(A) \times P_A(D) = 0.56 \times 0.22 = 0.12
\]
\item Probabilité que le stylo ai un défaut de fabrication.
\[
P(D) = P(A\cap D) + P(B\cap D) = 0.12 + 0.1 = 0.22
\]
\end{enumerate}
\item Probabilité qu'il vienne de l'atelier A sachant qu'il a un defaut
\[
P_D(A) = \frac{P(A\cap D)}{P(D)} = \frac{0.12}{0.22} = 0.55
\]
\item $X$ peut être modélisée par une loi binomiale de paramètres $n=19$ et $p=0.22$.
\item (\textit{par de correction automatique disponible pour le résultat final}
\[
P(X = 19) = \coefBino{19}{19}\times 0.22^{19} \times 0.78^{0}
\]
\item (\textit{par de correction automatique disponible pour le résultat final}
Il faut calculer la probabilité qu'il y ait 0 stylo avec un defaut.
\[
P(X = 0) = \coefBino{19}{0}\times 0.22^{0} \times 0.78^{19}
\]
Puis comparer ce nombre à 0,5.
\item Il faut calculer l'espérance
\[
E[X] = n\times p = 19 \times 0.22 = 4.18
\]
\end{enumerate}
\end{solution}
\end{document}
%%% Local Variables:
%%% mode: latex
%%% TeX-master: "master"
%%% End:

View File

@ -0,0 +1,367 @@
\documentclass[a4paper,10pt]{article}
\usepackage{myXsim}
% Title Page
\title{DM1 \hfill MOUHOUBI Maïssa}
\tribe{Maths complémentaire}
\date{\hfillÀ render pour le jeudi 27 mai}
\xsimsetup{
solution/print = true
}
\begin{document}
\maketitle
Les valeurs des exercices sont générés automatiquement. Si une valeur a un nombre adhérant de chiffres après la virgule, vous pouvez l'arrondir à l'entier le plus proche.
\begin{exercise}[subtitle={Optimisation de matière}]
\begin{minipage}{0.6\textwidth}
On se propose de fabriquer avec le moins de tôle possible une citerne fermée en forme de parallélépipède rectangle dont le volume intérieur doit être de $35m^3$. La longueur est aussi fixée à $5m$ par le cahier des charges.
On peut donc faire varier uniquement la largeur (notée $x$) et la hauteur (notée $h$) de la cuve.
\end{minipage}
\hfill
\begin{minipage}{0.3\textwidth}
\begin{tikzpicture}
\pgfmathsetmacro{\cubex}{3}
\pgfmathsetmacro{\cubey}{1}
\pgfmathsetmacro{\cubez}{2}
\draw[black,fill=gray] (0,0,0) -- ++(-\cubex,0,0) -- ++(0,-\cubey,0) node [midway, left] {$h$} -- ++(\cubex,0,0) node [midway, below] {$x$} -- cycle;
\draw[black,fill=gray] (0,0,0) -- ++(0,0,-\cubez) -- ++(0,-\cubey,0) -- ++(0,0,\cubez) node [midway, right] {$5m$} -- cycle;
\draw[black,fill=gray] (0,0,0) -- ++(-\cubex,0,0) -- ++(0,0,-\cubez) -- ++(\cubex,0,0) -- cycle;
\end{tikzpicture}
\end{minipage}
\begin{enumerate}
\item Expliquer pourquoi quand la largeur $x$ change, la hauteur $h$ doit elle aussi changer pour respecter les contraintes.
\item Démontrer que l'on doit avoir $h = \dfrac{7}{x}$.
\item On note $S(x)$ l'aire totale de la citerne (c'est à dire la somme des aires des six faces). Montrer que l'on peut écrire
\[
S(x) = 10x + 14 + \frac{70}{x}
\]
\item Démontrer que
\[
S(x) = \frac{10x^2 + 14x + 70}{x}
\]
\item Démontrer que
\[
S'(x) = \frac{10x^2 - 70}{x^2}
\]
\item En déduire le tableau de variation de $S(x)$ sur $\intOF{0}{10}$.
\item Déterminer les valeurs de $x$ et $h$ correspondant à une utilisation minimal de tôle.
\end{enumerate}
\end{exercise}
\begin{solution}
\begin{enumerate}
\item Le volume étant fixe si l'on fait varier $x$, $h$ doit aussi varier.
\begin{itemize}
\item Si $x = 2$ alors conserver un volume de $V=35$, $h$ doit être égale à $7 / 2$
\item Si $x = 3$ alors conserver un volume de $V=35$, $h$ doit être égale à $7 / 3$
\end{itemize}
\item Pour calculer le volume, on a
\begin{eqnarray*}
V &=& h\times x \times 5 \\
35 &=& h\times x \times 5 \\
x &=& \frac{35}{h\times 5} = \frac{7}{h}
\end{eqnarray*}
\item Pour calculer la surface totale, on ajoute la surface de chaque face. On a donc le calcul suivant
\begin{eqnarray*}
S(x) &=& x\times h \times 2 + x\times5\times2 + h\times 5\times 2\\
S(x) &=& x\times \frac{7}{x} \times 2 + x\times5\times2 + \frac{7}{x}\times 5\times 2\\
S(x) &=& 10x + 14 + \frac{70}{x}
\end{eqnarray*}
\item Pour trouver cette nouvelle forme, on met chaque élément sur le même dénominateur
\begin{eqnarray*}
S(x) &=& 10x + 14 + \frac{70}{x}\\
S(x) &=& \frac{10x\times x}{x} + \frac{14\times x}{x} + \frac{70}{x}\\
S(x) &=& \frac{10x^2 + 14x + 70}{x}
\end{eqnarray*}
\item On retrouve la formule $\frac{u}{v}$ à dériver
\[
u(x) = 10x^2 + 14x + 70 \Rightarrow u'(x) = 20x + 14
\]
\[
v(x) = x \Rightarrow v'(x) = 1
\]
Donc au numérateur on obtient
\begin{eqnarray*}
u'(x)\times v(x) - u(x)\times v'(x) &=& (20x + 14)\times x - (10x^2 + 14x + 70)\times 1\\
&=& 10x^2 - 70
\end{eqnarray*}
Donc
\[
S'(x) = \frac{10x^2 - 70}{x^2}
\]
\item Tableau de variations de $S$
\begin{itemize}
\item Valeur interdite: $x^2 = 0 \equiv x = 0$
\item Signe de $10x^2 - 70$: c'est un polynôme du 2e degré
\[
\Delta = 2800 > 0
\]
Il y a donc 2 racines
\[
x_1 = - 2.6457513110645907 \qquad
x_2 = 2.6457513110645907
\]
Et on sait que $10x^2 - 70$ est du signe de $a$ donc positif en dehors des racines
\item Le dénominateur $x^2$ est toujours positif.
\item Tableau de variations
\begin{tikzpicture}[baseline=(a.north)]
\tkzTabInit[lgt=3,espcl=3]{$x$/1,$10x^2 - 70$/1, $x^2$/1, $S'$/1, $S$/2}{$0$, $- 2.6457513110645907$, $10$}
\tkzTabLine{d,-, z, +, }
\tkzTabLine{d,+, , +, }
\tkzTabLine{d,-, z, +, }
\tkzTabVar{D+/ , -/ , +/ }
\end{tikzpicture}
\end{itemize}
\item On a donc une surface minimal pour $x=2.6457513110645907$ et $h = 18.5202591774521349$.
\end{enumerate}
\end{solution}
%%% Local Variables:
%%% mode: latex
%%% TeX-master: "master"
%%% End:
\begin{exercise}[subtitle={Bassin}]
Le tour d'un bassin au niveau du sol présente deux axes de symétrie : laxe des abscisses et la droite déquation $x=4$. Il est obtenu par symétrie de la courbe $\mathcal{C}_f$ sur $\intFF{0}{4}$$f$ est la fonction définie par
\[
f(x) = \left(- x^{2} + 1.1 x - 6.1\right) e^{- x} + 6.1
\]
On admet que sur $\intFF{0}{4}$ la fonction $f$ est positive.
\begin{enumerate}
\item Sur un repère, tracer l'allure de la courbe $\mathcal{C}_f$, les axes de symétries puis compléter pour dessiner la forme du bassin.
\item Montrer que la fonction $f$ admet comme primitive sur $\R$ la fonction $F$ définie par
\[
F(x) = 6.1 x + \left( x^{2} + 0.9 x + 7.0\right) e^{- x}
\]
\item Calculer la quantité $\ds \int_0^4 f(x) \; dx$, vous donnerez le résultat sous forme exacte. Interpréter le résultat et reportez cette quantité sur le graphique.
\item On considère que l'échelle de votre graphique est de 1unité pour 15m. Calculer l'aire du bassin. Vous donnerez un résultat arrondi au $m^2$ près.
\end{enumerate}
\end{exercise}
\begin{solution}
\begin{enumerate}
\item
\begin{tikzpicture}[baseline=(a.north), xscale=1, yscale=0.5]
\tkzInit[xmin=0,xmax=5,xstep=1,
ymin=0,ymax=10,ystep=1]
\tkzGrid
\tkzAxeXY
\tkzFct[domain=0:10,color=red,very thick]%
{ (-x**2 + 1.1*x - 6.1)*exp(-x) + 6.1 };
\end{tikzpicture}
\item Il faut dériver $F(x)$ et vérifier que $F'(x) = f(x)$.
\item $\ds \int_0^4 f(x) \; dx = F(4) - F(0) = \frac{26.6}{e^{4}} + 17.4$
\item La quantité calculée à la question précédente se retrouve 4fois pour former le bassin. Il faut ensuite prendre en compte l'échelle, comme 1unité de longueur correspond à 15m, une unité d'air correspond à $15\times15 = 225m^2$. Ainsi l'aire du bassin est égale à
\[
(\frac{26.6}{e^{4}} + 17.4)\times 4 \times 15^2 = 16098.00000
\]
\end{enumerate}
\end{solution}
%%% Local Variables:
%%% mode: latex
%%% TeX-master: "master"
%%% End:
\begin{exercise}[subtitle={Stylos}]
\emph{Les parties {\rm A} et {\rm B} de cet exercice sont indépendantes.}
\bigskip
\begin{minipage}{0.6\linewidth}
\textbf{Partie A}
\medskip
Deux ateliers A et B fabriquent des stylos pour une entreprise.
L'atelier A fabrique 23.0\,\% des stylos, et parmi ceux-là, 66.0\,\% possèdent un défaut de fabrication.
De plus, 68.0\,\% des stylos possèdent un défaut de fabrication et sortent de l'atelier B.
Un stylo est prélevé au hasard dans le stock de l'entreprise.
On considère les évènements suivants:
\begin{itemize}
\item A : \og Le stylo a été fabriqué par l'atelier A \fg
\item B : \og Le stylo a été fabriqué par l'atelier B \fg
\item D : \og Le stylo possède un défaut de fabrication \fg
\end{itemize}
\end{minipage}
\begin{minipage}{0.4\linewidth}
\begin{center}
\begin{tikzpicture}[sloped]
\node {.}
child {node {$A$}
child {node {$D$}
edge from parent
node[above] {...}
}
child {node {$\overline{D}$}
edge from parent
node[above] {...}
}
edge from parent
node[above] {...}
}
child[missing] {}
child { node {$B$}
child {node {$D$}
edge from parent
node[above] {...}
}
child {node {$\overline{D}$}
edge from parent
node[above] {...}
}
edge from parent
node[above] {...}
} ;
\end{tikzpicture}
\end{center}
\end{minipage}
\medskip
\begin{enumerate}
\item Compléter l'arbre de probabilité ci-contre
\item Interpréter puis donner les probabilités $P(A)$, $P(B)$, $P_A(D)$ et $P(B \cap D)$.
\item
\begin{enumerate}
\item Calculer la probabilité qu'un stylo provienne de l'atelier A et possède un défaut de fabrication.
\item En déduire que la probabilité qu'un stylo possède un défaut de fabrication est de $0.83$.
\end{enumerate}
\item On prélève un stylo au hasard avec un défaut. Quelle est la probabilité qu'il vienne de l'atelier A?
\end{enumerate}
\bigskip
\textbf{Partie B}
\medskip
Dans cette partie, on suppose que 83.0\,\% des stylos possèdent un défaut de fabrication.
L'entreprise confectionne des paquets contenant chacun $4$~stylos.
Le fait qu'un stylo possède ou non un défaut de fabrication est indépendant des autres stylos.
On appelle $X$ la variable aléatoire donnant pour un paquet le nombre de stylos qui possèdent un défaut de fabrication.
On admet que la variable aléatoire $X$ suit une loi binomiale.
\medskip
\begin{enumerate}
\setcounter{enumi}{4}
\item Avec quelle loi peut-on modéliser $X$. Préciser les paramètres.
\item Calculer et interpréter la probabilité $P(X = 10)$.
\item Le directeur de l'entreprise affirme qu'il y a plus d'une chance sur deux qu'un paquet ne comporte aucun stylo défectueux. A-t-il raison ?
\item Combien de stylos peut-on espérer avoir en moyenne?
\end{enumerate}
\pagebreak
\end{exercise}
\begin{solution}
\begin{enumerate}
\item
\begin{center}
\begin{tikzpicture}[sloped]
\node {.}
child {node {$A$}
child {node {$D$}
edge from parent
node[above] {0.66}
}
child {node {$\overline{D}$}
edge from parent
node[above] {0.34}
}
edge from parent
node[above] {0.23}
}
child[missing] {}
child { node {$B$}
child {node {$D$}
edge from parent
node[above] {0.88}
}
child {node {$\overline{D}$}
edge from parent
node[above] {0.12}
}
edge from parent
node[above] {0.77}
} ;
\end{tikzpicture}
\end{center}
\item
\begin{itemize}
\item Probabilité que le stylo vienne de l'atelier A
\[
P(A) = 0.23
\]
\item Probabilité que le stylo vienne de l'atelier B
\[
P(B) = 0.77
\]
\item Probabilité que le stylo ait un défaut sachant qu'il vient de l'atelier A.
\[
P_A(D) = 0.66
\]
\item Probabilité que le stylo vienne de l'atelier B et qu'il ait un défaut.
\[
P(D \cap D) = 0.68
\]
\end{itemize}
\item
\begin{enumerate}
\item Probabilité qu'un stylo vienne de l'atelier A et qu'il ait un defaut
\[
P(A\cap D) = P(A) \times P_A(D) = 0.23 \times 0.66 = 0.15
\]
\item Probabilité que le stylo ai un défaut de fabrication.
\[
P(D) = P(A\cap D) + P(B\cap D) = 0.15 + 0.68 = 0.83
\]
\end{enumerate}
\item Probabilité qu'il vienne de l'atelier A sachant qu'il a un defaut
\[
P_D(A) = \frac{P(A\cap D)}{P(D)} = \frac{0.15}{0.83} = 0.18
\]
\item $X$ peut être modélisée par une loi binomiale de paramètres $n=17$ et $p=0.83$.
\item (\textit{par de correction automatique disponible pour le résultat final}
\[
P(X = 10) = \coefBino{17}{10}\times 0.83^{10} \times 0.17^{7}
\]
\item (\textit{par de correction automatique disponible pour le résultat final}
Il faut calculer la probabilité qu'il y ait 0 stylo avec un defaut.
\[
P(X = 0) = \coefBino{17}{0}\times 0.83^{0} \times 0.17^{17}
\]
Puis comparer ce nombre à 0,5.
\item Il faut calculer l'espérance
\[
E[X] = n\times p = 17 \times 0.83 = 14.11
\]
\end{enumerate}
\end{solution}
\end{document}
%%% Local Variables:
%%% mode: latex
%%% TeX-master: "master"
%%% End:

View File

@ -0,0 +1,367 @@
\documentclass[a4paper,10pt]{article}
\usepackage{myXsim}
% Title Page
\title{DM1 \hfill PERDRIX Camille}
\tribe{Maths complémentaire}
\date{\hfillÀ render pour le jeudi 27 mai}
\xsimsetup{
solution/print = true
}
\begin{document}
\maketitle
Les valeurs des exercices sont générés automatiquement. Si une valeur a un nombre adhérant de chiffres après la virgule, vous pouvez l'arrondir à l'entier le plus proche.
\begin{exercise}[subtitle={Optimisation de matière}]
\begin{minipage}{0.6\textwidth}
On se propose de fabriquer avec le moins de tôle possible une citerne fermée en forme de parallélépipède rectangle dont le volume intérieur doit être de $8m^3$. La longueur est aussi fixée à $4m$ par le cahier des charges.
On peut donc faire varier uniquement la largeur (notée $x$) et la hauteur (notée $h$) de la cuve.
\end{minipage}
\hfill
\begin{minipage}{0.3\textwidth}
\begin{tikzpicture}
\pgfmathsetmacro{\cubex}{3}
\pgfmathsetmacro{\cubey}{1}
\pgfmathsetmacro{\cubez}{2}
\draw[black,fill=gray] (0,0,0) -- ++(-\cubex,0,0) -- ++(0,-\cubey,0) node [midway, left] {$h$} -- ++(\cubex,0,0) node [midway, below] {$x$} -- cycle;
\draw[black,fill=gray] (0,0,0) -- ++(0,0,-\cubez) -- ++(0,-\cubey,0) -- ++(0,0,\cubez) node [midway, right] {$4m$} -- cycle;
\draw[black,fill=gray] (0,0,0) -- ++(-\cubex,0,0) -- ++(0,0,-\cubez) -- ++(\cubex,0,0) -- cycle;
\end{tikzpicture}
\end{minipage}
\begin{enumerate}
\item Expliquer pourquoi quand la largeur $x$ change, la hauteur $h$ doit elle aussi changer pour respecter les contraintes.
\item Démontrer que l'on doit avoir $h = \dfrac{2}{x}$.
\item On note $S(x)$ l'aire totale de la citerne (c'est à dire la somme des aires des six faces). Montrer que l'on peut écrire
\[
S(x) = 8x + 4 + \frac{16}{x}
\]
\item Démontrer que
\[
S(x) = \frac{8x^2 + 4x + 16}{x}
\]
\item Démontrer que
\[
S'(x) = \frac{8x^2 - 16}{x^2}
\]
\item En déduire le tableau de variation de $S(x)$ sur $\intOF{0}{10}$.
\item Déterminer les valeurs de $x$ et $h$ correspondant à une utilisation minimal de tôle.
\end{enumerate}
\end{exercise}
\begin{solution}
\begin{enumerate}
\item Le volume étant fixe si l'on fait varier $x$, $h$ doit aussi varier.
\begin{itemize}
\item Si $x = 2$ alors conserver un volume de $V=8$, $h$ doit être égale à $2 / 2$
\item Si $x = 3$ alors conserver un volume de $V=8$, $h$ doit être égale à $2 / 3$
\end{itemize}
\item Pour calculer le volume, on a
\begin{eqnarray*}
V &=& h\times x \times 4 \\
8 &=& h\times x \times 4 \\
x &=& \frac{8}{h\times 4} = \frac{2}{h}
\end{eqnarray*}
\item Pour calculer la surface totale, on ajoute la surface de chaque face. On a donc le calcul suivant
\begin{eqnarray*}
S(x) &=& x\times h \times 2 + x\times4\times2 + h\times 4\times 2\\
S(x) &=& x\times \frac{2}{x} \times 2 + x\times4\times2 + \frac{2}{x}\times 4\times 2\\
S(x) &=& 8x + 4 + \frac{16}{x}
\end{eqnarray*}
\item Pour trouver cette nouvelle forme, on met chaque élément sur le même dénominateur
\begin{eqnarray*}
S(x) &=& 8x + 4 + \frac{16}{x}\\
S(x) &=& \frac{8x\times x}{x} + \frac{4\times x}{x} + \frac{16}{x}\\
S(x) &=& \frac{8x^2 + 4x + 16}{x}
\end{eqnarray*}
\item On retrouve la formule $\frac{u}{v}$ à dériver
\[
u(x) = 8x^2 + 4x + 16 \Rightarrow u'(x) = 16x + 4
\]
\[
v(x) = x \Rightarrow v'(x) = 1
\]
Donc au numérateur on obtient
\begin{eqnarray*}
u'(x)\times v(x) - u(x)\times v'(x) &=& (16x + 4)\times x - (8x^2 + 4x + 16)\times 1\\
&=& 8x^2 - 16
\end{eqnarray*}
Donc
\[
S'(x) = \frac{8x^2 - 16}{x^2}
\]
\item Tableau de variations de $S$
\begin{itemize}
\item Valeur interdite: $x^2 = 0 \equiv x = 0$
\item Signe de $8x^2 - 16$: c'est un polynôme du 2e degré
\[
\Delta = 512 > 0
\]
Il y a donc 2 racines
\[
x_1 = - 1.4142135623730951 \qquad
x_2 = 1.4142135623730951
\]
Et on sait que $8x^2 - 16$ est du signe de $a$ donc positif en dehors des racines
\item Le dénominateur $x^2$ est toujours positif.
\item Tableau de variations
\begin{tikzpicture}[baseline=(a.north)]
\tkzTabInit[lgt=3,espcl=3]{$x$/1,$8x^2 - 16$/1, $x^2$/1, $S'$/1, $S$/2}{$0$, $- 1.4142135623730951$, $10$}
\tkzTabLine{d,-, z, +, }
\tkzTabLine{d,+, , +, }
\tkzTabLine{d,-, z, +, }
\tkzTabVar{D+/ , -/ , +/ }
\end{tikzpicture}
\end{itemize}
\item On a donc une surface minimal pour $x=1.4142135623730951$ et $h = 2.8284271247461902$.
\end{enumerate}
\end{solution}
%%% Local Variables:
%%% mode: latex
%%% TeX-master: "master"
%%% End:
\begin{exercise}[subtitle={Bassin}]
Le tour d'un bassin au niveau du sol présente deux axes de symétrie : laxe des abscisses et la droite déquation $x=4$. Il est obtenu par symétrie de la courbe $\mathcal{C}_f$ sur $\intFF{0}{4}$$f$ est la fonction définie par
\[
f(x) = \left(- x^{2} + 8.3 x - 5.0\right) e^{- x} + 5.0
\]
On admet que sur $\intFF{0}{4}$ la fonction $f$ est positive.
\begin{enumerate}
\item Sur un repère, tracer l'allure de la courbe $\mathcal{C}_f$, les axes de symétries puis compléter pour dessiner la forme du bassin.
\item Montrer que la fonction $f$ admet comme primitive sur $\R$ la fonction $F$ définie par
\[
F(x) = 5.0 x + \left( x^{2} - 6.3 x - 1.3\right) e^{- x}
\]
\item Calculer la quantité $\ds \int_0^4 f(x) \; dx$, vous donnerez le résultat sous forme exacte. Interpréter le résultat et reportez cette quantité sur le graphique.
\item On considère que l'échelle de votre graphique est de 1unité pour 15m. Calculer l'aire du bassin. Vous donnerez un résultat arrondi au $m^2$ près.
\end{enumerate}
\end{exercise}
\begin{solution}
\begin{enumerate}
\item
\begin{tikzpicture}[baseline=(a.north), xscale=1, yscale=0.5]
\tkzInit[xmin=0,xmax=5,xstep=1,
ymin=0,ymax=10,ystep=1]
\tkzGrid
\tkzAxeXY
\tkzFct[domain=0:10,color=red,very thick]%
{ (-x**2 + 8.3*x - 5.0)*exp(-x) + 5.0 };
\end{tikzpicture}
\item Il faut dériver $F(x)$ et vérifier que $F'(x) = f(x)$.
\item $\ds \int_0^4 f(x) \; dx = F(4) - F(0) = 21.3 - \frac{10.5}{e^{4}}$
\item La quantité calculée à la question précédente se retrouve 4fois pour former le bassin. Il faut ensuite prendre en compte l'échelle, comme 1unité de longueur correspond à 15m, une unité d'air correspond à $15\times15 = 225m^2$. Ainsi l'aire du bassin est égale à
\[
(21.3 - \frac{10.5}{e^{4}})\times 4 \times 15^2 = 18997.00000
\]
\end{enumerate}
\end{solution}
%%% Local Variables:
%%% mode: latex
%%% TeX-master: "master"
%%% End:
\begin{exercise}[subtitle={Stylos}]
\emph{Les parties {\rm A} et {\rm B} de cet exercice sont indépendantes.}
\bigskip
\begin{minipage}{0.6\linewidth}
\textbf{Partie A}
\medskip
Deux ateliers A et B fabriquent des stylos pour une entreprise.
L'atelier A fabrique 66.0\,\% des stylos, et parmi ceux-là, 35.0\,\% possèdent un défaut de fabrication.
De plus, 28.000000000000004\,\% des stylos possèdent un défaut de fabrication et sortent de l'atelier B.
Un stylo est prélevé au hasard dans le stock de l'entreprise.
On considère les évènements suivants:
\begin{itemize}
\item A : \og Le stylo a été fabriqué par l'atelier A \fg
\item B : \og Le stylo a été fabriqué par l'atelier B \fg
\item D : \og Le stylo possède un défaut de fabrication \fg
\end{itemize}
\end{minipage}
\begin{minipage}{0.4\linewidth}
\begin{center}
\begin{tikzpicture}[sloped]
\node {.}
child {node {$A$}
child {node {$D$}
edge from parent
node[above] {...}
}
child {node {$\overline{D}$}
edge from parent
node[above] {...}
}
edge from parent
node[above] {...}
}
child[missing] {}
child { node {$B$}
child {node {$D$}
edge from parent
node[above] {...}
}
child {node {$\overline{D}$}
edge from parent
node[above] {...}
}
edge from parent
node[above] {...}
} ;
\end{tikzpicture}
\end{center}
\end{minipage}
\medskip
\begin{enumerate}
\item Compléter l'arbre de probabilité ci-contre
\item Interpréter puis donner les probabilités $P(A)$, $P(B)$, $P_A(D)$ et $P(B \cap D)$.
\item
\begin{enumerate}
\item Calculer la probabilité qu'un stylo provienne de l'atelier A et possède un défaut de fabrication.
\item En déduire que la probabilité qu'un stylo possède un défaut de fabrication est de $0.51$.
\end{enumerate}
\item On prélève un stylo au hasard avec un défaut. Quelle est la probabilité qu'il vienne de l'atelier A?
\end{enumerate}
\bigskip
\textbf{Partie B}
\medskip
Dans cette partie, on suppose que 51.0\,\% des stylos possèdent un défaut de fabrication.
L'entreprise confectionne des paquets contenant chacun $4$~stylos.
Le fait qu'un stylo possède ou non un défaut de fabrication est indépendant des autres stylos.
On appelle $X$ la variable aléatoire donnant pour un paquet le nombre de stylos qui possèdent un défaut de fabrication.
On admet que la variable aléatoire $X$ suit une loi binomiale.
\medskip
\begin{enumerate}
\setcounter{enumi}{4}
\item Avec quelle loi peut-on modéliser $X$. Préciser les paramètres.
\item Calculer et interpréter la probabilité $P(X = 11)$.
\item Le directeur de l'entreprise affirme qu'il y a plus d'une chance sur deux qu'un paquet ne comporte aucun stylo défectueux. A-t-il raison ?
\item Combien de stylos peut-on espérer avoir en moyenne?
\end{enumerate}
\pagebreak
\end{exercise}
\begin{solution}
\begin{enumerate}
\item
\begin{center}
\begin{tikzpicture}[sloped]
\node {.}
child {node {$A$}
child {node {$D$}
edge from parent
node[above] {0.35}
}
child {node {$\overline{D}$}
edge from parent
node[above] {0.65}
}
edge from parent
node[above] {0.66}
}
child[missing] {}
child { node {$B$}
child {node {$D$}
edge from parent
node[above] {0.81}
}
child {node {$\overline{D}$}
edge from parent
node[above] {0.19}
}
edge from parent
node[above] {0.34}
} ;
\end{tikzpicture}
\end{center}
\item
\begin{itemize}
\item Probabilité que le stylo vienne de l'atelier A
\[
P(A) = 0.66
\]
\item Probabilité que le stylo vienne de l'atelier B
\[
P(B) = 0.34
\]
\item Probabilité que le stylo ait un défaut sachant qu'il vient de l'atelier A.
\[
P_A(D) = 0.35
\]
\item Probabilité que le stylo vienne de l'atelier B et qu'il ait un défaut.
\[
P(D \cap D) = 0.28
\]
\end{itemize}
\item
\begin{enumerate}
\item Probabilité qu'un stylo vienne de l'atelier A et qu'il ait un defaut
\[
P(A\cap D) = P(A) \times P_A(D) = 0.66 \times 0.35 = 0.23
\]
\item Probabilité que le stylo ai un défaut de fabrication.
\[
P(D) = P(A\cap D) + P(B\cap D) = 0.23 + 0.28 = 0.51
\]
\end{enumerate}
\item Probabilité qu'il vienne de l'atelier A sachant qu'il a un defaut
\[
P_D(A) = \frac{P(A\cap D)}{P(D)} = \frac{0.23}{0.51} = 0.45
\]
\item $X$ peut être modélisée par une loi binomiale de paramètres $n=19$ et $p=0.51$.
\item (\textit{par de correction automatique disponible pour le résultat final}
\[
P(X = 11) = \coefBino{19}{11}\times 0.51^{11} \times 0.49^{8}
\]
\item (\textit{par de correction automatique disponible pour le résultat final}
Il faut calculer la probabilité qu'il y ait 0 stylo avec un defaut.
\[
P(X = 0) = \coefBino{19}{0}\times 0.51^{0} \times 0.49^{19}
\]
Puis comparer ce nombre à 0,5.
\item Il faut calculer l'espérance
\[
E[X] = n\times p = 19 \times 0.51 = 9.69
\]
\end{enumerate}
\end{solution}
\end{document}
%%% Local Variables:
%%% mode: latex
%%% TeX-master: "master"
%%% End:

View File

@ -0,0 +1,367 @@
\documentclass[a4paper,10pt]{article}
\usepackage{myXsim}
% Title Page
\title{DM1 \hfill POISON Lorette}
\tribe{Maths complémentaire}
\date{\hfillÀ render pour le jeudi 27 mai}
\xsimsetup{
solution/print = true
}
\begin{document}
\maketitle
Les valeurs des exercices sont générés automatiquement. Si une valeur a un nombre adhérant de chiffres après la virgule, vous pouvez l'arrondir à l'entier le plus proche.
\begin{exercise}[subtitle={Optimisation de matière}]
\begin{minipage}{0.6\textwidth}
On se propose de fabriquer avec le moins de tôle possible une citerne fermée en forme de parallélépipède rectangle dont le volume intérieur doit être de $35m^3$. La longueur est aussi fixée à $5m$ par le cahier des charges.
On peut donc faire varier uniquement la largeur (notée $x$) et la hauteur (notée $h$) de la cuve.
\end{minipage}
\hfill
\begin{minipage}{0.3\textwidth}
\begin{tikzpicture}
\pgfmathsetmacro{\cubex}{3}
\pgfmathsetmacro{\cubey}{1}
\pgfmathsetmacro{\cubez}{2}
\draw[black,fill=gray] (0,0,0) -- ++(-\cubex,0,0) -- ++(0,-\cubey,0) node [midway, left] {$h$} -- ++(\cubex,0,0) node [midway, below] {$x$} -- cycle;
\draw[black,fill=gray] (0,0,0) -- ++(0,0,-\cubez) -- ++(0,-\cubey,0) -- ++(0,0,\cubez) node [midway, right] {$5m$} -- cycle;
\draw[black,fill=gray] (0,0,0) -- ++(-\cubex,0,0) -- ++(0,0,-\cubez) -- ++(\cubex,0,0) -- cycle;
\end{tikzpicture}
\end{minipage}
\begin{enumerate}
\item Expliquer pourquoi quand la largeur $x$ change, la hauteur $h$ doit elle aussi changer pour respecter les contraintes.
\item Démontrer que l'on doit avoir $h = \dfrac{7}{x}$.
\item On note $S(x)$ l'aire totale de la citerne (c'est à dire la somme des aires des six faces). Montrer que l'on peut écrire
\[
S(x) = 10x + 14 + \frac{70}{x}
\]
\item Démontrer que
\[
S(x) = \frac{10x^2 + 14x + 70}{x}
\]
\item Démontrer que
\[
S'(x) = \frac{10x^2 - 70}{x^2}
\]
\item En déduire le tableau de variation de $S(x)$ sur $\intOF{0}{10}$.
\item Déterminer les valeurs de $x$ et $h$ correspondant à une utilisation minimal de tôle.
\end{enumerate}
\end{exercise}
\begin{solution}
\begin{enumerate}
\item Le volume étant fixe si l'on fait varier $x$, $h$ doit aussi varier.
\begin{itemize}
\item Si $x = 2$ alors conserver un volume de $V=35$, $h$ doit être égale à $7 / 2$
\item Si $x = 3$ alors conserver un volume de $V=35$, $h$ doit être égale à $7 / 3$
\end{itemize}
\item Pour calculer le volume, on a
\begin{eqnarray*}
V &=& h\times x \times 5 \\
35 &=& h\times x \times 5 \\
x &=& \frac{35}{h\times 5} = \frac{7}{h}
\end{eqnarray*}
\item Pour calculer la surface totale, on ajoute la surface de chaque face. On a donc le calcul suivant
\begin{eqnarray*}
S(x) &=& x\times h \times 2 + x\times5\times2 + h\times 5\times 2\\
S(x) &=& x\times \frac{7}{x} \times 2 + x\times5\times2 + \frac{7}{x}\times 5\times 2\\
S(x) &=& 10x + 14 + \frac{70}{x}
\end{eqnarray*}
\item Pour trouver cette nouvelle forme, on met chaque élément sur le même dénominateur
\begin{eqnarray*}
S(x) &=& 10x + 14 + \frac{70}{x}\\
S(x) &=& \frac{10x\times x}{x} + \frac{14\times x}{x} + \frac{70}{x}\\
S(x) &=& \frac{10x^2 + 14x + 70}{x}
\end{eqnarray*}
\item On retrouve la formule $\frac{u}{v}$ à dériver
\[
u(x) = 10x^2 + 14x + 70 \Rightarrow u'(x) = 20x + 14
\]
\[
v(x) = x \Rightarrow v'(x) = 1
\]
Donc au numérateur on obtient
\begin{eqnarray*}
u'(x)\times v(x) - u(x)\times v'(x) &=& (20x + 14)\times x - (10x^2 + 14x + 70)\times 1\\
&=& 10x^2 - 70
\end{eqnarray*}
Donc
\[
S'(x) = \frac{10x^2 - 70}{x^2}
\]
\item Tableau de variations de $S$
\begin{itemize}
\item Valeur interdite: $x^2 = 0 \equiv x = 0$
\item Signe de $10x^2 - 70$: c'est un polynôme du 2e degré
\[
\Delta = 2800 > 0
\]
Il y a donc 2 racines
\[
x_1 = - 2.6457513110645907 \qquad
x_2 = 2.6457513110645907
\]
Et on sait que $10x^2 - 70$ est du signe de $a$ donc positif en dehors des racines
\item Le dénominateur $x^2$ est toujours positif.
\item Tableau de variations
\begin{tikzpicture}[baseline=(a.north)]
\tkzTabInit[lgt=3,espcl=3]{$x$/1,$10x^2 - 70$/1, $x^2$/1, $S'$/1, $S$/2}{$0$, $- 2.6457513110645907$, $10$}
\tkzTabLine{d,-, z, +, }
\tkzTabLine{d,+, , +, }
\tkzTabLine{d,-, z, +, }
\tkzTabVar{D+/ , -/ , +/ }
\end{tikzpicture}
\end{itemize}
\item On a donc une surface minimal pour $x=2.6457513110645907$ et $h = 18.5202591774521349$.
\end{enumerate}
\end{solution}
%%% Local Variables:
%%% mode: latex
%%% TeX-master: "master"
%%% End:
\begin{exercise}[subtitle={Bassin}]
Le tour d'un bassin au niveau du sol présente deux axes de symétrie : laxe des abscisses et la droite déquation $x=4$. Il est obtenu par symétrie de la courbe $\mathcal{C}_f$ sur $\intFF{0}{4}$$f$ est la fonction définie par
\[
f(x) = \left(- x^{2} + 6.1 x - 3.4\right) e^{- x} + 3.4
\]
On admet que sur $\intFF{0}{4}$ la fonction $f$ est positive.
\begin{enumerate}
\item Sur un repère, tracer l'allure de la courbe $\mathcal{C}_f$, les axes de symétries puis compléter pour dessiner la forme du bassin.
\item Montrer que la fonction $f$ admet comme primitive sur $\R$ la fonction $F$ définie par
\[
F(x) = 3.4 x + \left( x^{2} - 4.1 x - 0.7\right) e^{- x}
\]
\item Calculer la quantité $\ds \int_0^4 f(x) \; dx$, vous donnerez le résultat sous forme exacte. Interpréter le résultat et reportez cette quantité sur le graphique.
\item On considère que l'échelle de votre graphique est de 1unité pour 15m. Calculer l'aire du bassin. Vous donnerez un résultat arrondi au $m^2$ près.
\end{enumerate}
\end{exercise}
\begin{solution}
\begin{enumerate}
\item
\begin{tikzpicture}[baseline=(a.north), xscale=1, yscale=0.5]
\tkzInit[xmin=0,xmax=5,xstep=1,
ymin=0,ymax=10,ystep=1]
\tkzGrid
\tkzAxeXY
\tkzFct[domain=0:10,color=red,very thick]%
{ (-x**2 + 6.1*x - 3.4)*exp(-x) + 3.4 };
\end{tikzpicture}
\item Il faut dériver $F(x)$ et vérifier que $F'(x) = f(x)$.
\item $\ds \int_0^4 f(x) \; dx = F(4) - F(0) = 14.3 - \frac{1.1}{e^{4}}$
\item La quantité calculée à la question précédente se retrouve 4fois pour former le bassin. Il faut ensuite prendre en compte l'échelle, comme 1unité de longueur correspond à 15m, une unité d'air correspond à $15\times15 = 225m^2$. Ainsi l'aire du bassin est égale à
\[
(14.3 - \frac{1.1}{e^{4}})\times 4 \times 15^2 = 12852.00000
\]
\end{enumerate}
\end{solution}
%%% Local Variables:
%%% mode: latex
%%% TeX-master: "master"
%%% End:
\begin{exercise}[subtitle={Stylos}]
\emph{Les parties {\rm A} et {\rm B} de cet exercice sont indépendantes.}
\bigskip
\begin{minipage}{0.6\linewidth}
\textbf{Partie A}
\medskip
Deux ateliers A et B fabriquent des stylos pour une entreprise.
L'atelier A fabrique 47.0\,\% des stylos, et parmi ceux-là, 95.0\,\% possèdent un défaut de fabrication.
De plus, 50.0\,\% des stylos possèdent un défaut de fabrication et sortent de l'atelier B.
Un stylo est prélevé au hasard dans le stock de l'entreprise.
On considère les évènements suivants:
\begin{itemize}
\item A : \og Le stylo a été fabriqué par l'atelier A \fg
\item B : \og Le stylo a été fabriqué par l'atelier B \fg
\item D : \og Le stylo possède un défaut de fabrication \fg
\end{itemize}
\end{minipage}
\begin{minipage}{0.4\linewidth}
\begin{center}
\begin{tikzpicture}[sloped]
\node {.}
child {node {$A$}
child {node {$D$}
edge from parent
node[above] {...}
}
child {node {$\overline{D}$}
edge from parent
node[above] {...}
}
edge from parent
node[above] {...}
}
child[missing] {}
child { node {$B$}
child {node {$D$}
edge from parent
node[above] {...}
}
child {node {$\overline{D}$}
edge from parent
node[above] {...}
}
edge from parent
node[above] {...}
} ;
\end{tikzpicture}
\end{center}
\end{minipage}
\medskip
\begin{enumerate}
\item Compléter l'arbre de probabilité ci-contre
\item Interpréter puis donner les probabilités $P(A)$, $P(B)$, $P_A(D)$ et $P(B \cap D)$.
\item
\begin{enumerate}
\item Calculer la probabilité qu'un stylo provienne de l'atelier A et possède un défaut de fabrication.
\item En déduire que la probabilité qu'un stylo possède un défaut de fabrication est de $0.95$.
\end{enumerate}
\item On prélève un stylo au hasard avec un défaut. Quelle est la probabilité qu'il vienne de l'atelier A?
\end{enumerate}
\bigskip
\textbf{Partie B}
\medskip
Dans cette partie, on suppose que 95.0\,\% des stylos possèdent un défaut de fabrication.
L'entreprise confectionne des paquets contenant chacun $4$~stylos.
Le fait qu'un stylo possède ou non un défaut de fabrication est indépendant des autres stylos.
On appelle $X$ la variable aléatoire donnant pour un paquet le nombre de stylos qui possèdent un défaut de fabrication.
On admet que la variable aléatoire $X$ suit une loi binomiale.
\medskip
\begin{enumerate}
\setcounter{enumi}{4}
\item Avec quelle loi peut-on modéliser $X$. Préciser les paramètres.
\item Calculer et interpréter la probabilité $P(X = 12)$.
\item Le directeur de l'entreprise affirme qu'il y a plus d'une chance sur deux qu'un paquet ne comporte aucun stylo défectueux. A-t-il raison ?
\item Combien de stylos peut-on espérer avoir en moyenne?
\end{enumerate}
\pagebreak
\end{exercise}
\begin{solution}
\begin{enumerate}
\item
\begin{center}
\begin{tikzpicture}[sloped]
\node {.}
child {node {$A$}
child {node {$D$}
edge from parent
node[above] {0.95}
}
child {node {$\overline{D}$}
edge from parent
node[above] {0.05}
}
edge from parent
node[above] {0.47}
}
child[missing] {}
child { node {$B$}
child {node {$D$}
edge from parent
node[above] {0.95}
}
child {node {$\overline{D}$}
edge from parent
node[above] {0.05}
}
edge from parent
node[above] {0.53}
} ;
\end{tikzpicture}
\end{center}
\item
\begin{itemize}
\item Probabilité que le stylo vienne de l'atelier A
\[
P(A) = 0.47
\]
\item Probabilité que le stylo vienne de l'atelier B
\[
P(B) = 0.53
\]
\item Probabilité que le stylo ait un défaut sachant qu'il vient de l'atelier A.
\[
P_A(D) = 0.95
\]
\item Probabilité que le stylo vienne de l'atelier B et qu'il ait un défaut.
\[
P(D \cap D) = 0.5
\]
\end{itemize}
\item
\begin{enumerate}
\item Probabilité qu'un stylo vienne de l'atelier A et qu'il ait un defaut
\[
P(A\cap D) = P(A) \times P_A(D) = 0.47 \times 0.95 = 0.45
\]
\item Probabilité que le stylo ai un défaut de fabrication.
\[
P(D) = P(A\cap D) + P(B\cap D) = 0.45 + 0.5 = 0.95
\]
\end{enumerate}
\item Probabilité qu'il vienne de l'atelier A sachant qu'il a un defaut
\[
P_D(A) = \frac{P(A\cap D)}{P(D)} = \frac{0.45}{0.95} = 0.47
\]
\item $X$ peut être modélisée par une loi binomiale de paramètres $n=15$ et $p=0.95$.
\item (\textit{par de correction automatique disponible pour le résultat final}
\[
P(X = 12) = \coefBino{15}{12}\times 0.95^{12} \times 0.05^{3}
\]
\item (\textit{par de correction automatique disponible pour le résultat final}
Il faut calculer la probabilité qu'il y ait 0 stylo avec un defaut.
\[
P(X = 0) = \coefBino{15}{0}\times 0.95^{0} \times 0.05^{15}
\]
Puis comparer ce nombre à 0,5.
\item Il faut calculer l'espérance
\[
E[X] = n\times p = 15 \times 0.95 = 14.25
\]
\end{enumerate}
\end{solution}
\end{document}
%%% Local Variables:
%%% mode: latex
%%% TeX-master: "master"
%%% End:

View File

@ -0,0 +1,367 @@
\documentclass[a4paper,10pt]{article}
\usepackage{myXsim}
% Title Page
\title{DM1 \hfill RODRIGUEZ Teddy}
\tribe{Maths complémentaire}
\date{\hfillÀ render pour le jeudi 27 mai}
\xsimsetup{
solution/print = true
}
\begin{document}
\maketitle
Les valeurs des exercices sont générés automatiquement. Si une valeur a un nombre adhérant de chiffres après la virgule, vous pouvez l'arrondir à l'entier le plus proche.
\begin{exercise}[subtitle={Optimisation de matière}]
\begin{minipage}{0.6\textwidth}
On se propose de fabriquer avec le moins de tôle possible une citerne fermée en forme de parallélépipède rectangle dont le volume intérieur doit être de $50m^3$. La longueur est aussi fixée à $5m$ par le cahier des charges.
On peut donc faire varier uniquement la largeur (notée $x$) et la hauteur (notée $h$) de la cuve.
\end{minipage}
\hfill
\begin{minipage}{0.3\textwidth}
\begin{tikzpicture}
\pgfmathsetmacro{\cubex}{3}
\pgfmathsetmacro{\cubey}{1}
\pgfmathsetmacro{\cubez}{2}
\draw[black,fill=gray] (0,0,0) -- ++(-\cubex,0,0) -- ++(0,-\cubey,0) node [midway, left] {$h$} -- ++(\cubex,0,0) node [midway, below] {$x$} -- cycle;
\draw[black,fill=gray] (0,0,0) -- ++(0,0,-\cubez) -- ++(0,-\cubey,0) -- ++(0,0,\cubez) node [midway, right] {$5m$} -- cycle;
\draw[black,fill=gray] (0,0,0) -- ++(-\cubex,0,0) -- ++(0,0,-\cubez) -- ++(\cubex,0,0) -- cycle;
\end{tikzpicture}
\end{minipage}
\begin{enumerate}
\item Expliquer pourquoi quand la largeur $x$ change, la hauteur $h$ doit elle aussi changer pour respecter les contraintes.
\item Démontrer que l'on doit avoir $h = \dfrac{10}{x}$.
\item On note $S(x)$ l'aire totale de la citerne (c'est à dire la somme des aires des six faces). Montrer que l'on peut écrire
\[
S(x) = 10x + 20 + \frac{100}{x}
\]
\item Démontrer que
\[
S(x) = \frac{10x^2 + 20x + 100}{x}
\]
\item Démontrer que
\[
S'(x) = \frac{10x^2 - 100}{x^2}
\]
\item En déduire le tableau de variation de $S(x)$ sur $\intOF{0}{10}$.
\item Déterminer les valeurs de $x$ et $h$ correspondant à une utilisation minimal de tôle.
\end{enumerate}
\end{exercise}
\begin{solution}
\begin{enumerate}
\item Le volume étant fixe si l'on fait varier $x$, $h$ doit aussi varier.
\begin{itemize}
\item Si $x = 2$ alors conserver un volume de $V=50$, $h$ doit être égale à $10 / 2$
\item Si $x = 3$ alors conserver un volume de $V=50$, $h$ doit être égale à $10 / 3$
\end{itemize}
\item Pour calculer le volume, on a
\begin{eqnarray*}
V &=& h\times x \times 5 \\
50 &=& h\times x \times 5 \\
x &=& \frac{50}{h\times 5} = \frac{10}{h}
\end{eqnarray*}
\item Pour calculer la surface totale, on ajoute la surface de chaque face. On a donc le calcul suivant
\begin{eqnarray*}
S(x) &=& x\times h \times 2 + x\times5\times2 + h\times 5\times 2\\
S(x) &=& x\times \frac{10}{x} \times 2 + x\times5\times2 + \frac{10}{x}\times 5\times 2\\
S(x) &=& 10x + 20 + \frac{100}{x}
\end{eqnarray*}
\item Pour trouver cette nouvelle forme, on met chaque élément sur le même dénominateur
\begin{eqnarray*}
S(x) &=& 10x + 20 + \frac{100}{x}\\
S(x) &=& \frac{10x\times x}{x} + \frac{20\times x}{x} + \frac{100}{x}\\
S(x) &=& \frac{10x^2 + 20x + 100}{x}
\end{eqnarray*}
\item On retrouve la formule $\frac{u}{v}$ à dériver
\[
u(x) = 10x^2 + 20x + 100 \Rightarrow u'(x) = 20x + 20
\]
\[
v(x) = x \Rightarrow v'(x) = 1
\]
Donc au numérateur on obtient
\begin{eqnarray*}
u'(x)\times v(x) - u(x)\times v'(x) &=& (20x + 20)\times x - (10x^2 + 20x + 100)\times 1\\
&=& 10x^2 - 100
\end{eqnarray*}
Donc
\[
S'(x) = \frac{10x^2 - 100}{x^2}
\]
\item Tableau de variations de $S$
\begin{itemize}
\item Valeur interdite: $x^2 = 0 \equiv x = 0$
\item Signe de $10x^2 - 100$: c'est un polynôme du 2e degré
\[
\Delta = 4000 > 0
\]
Il y a donc 2 racines
\[
x_1 = - 3.162277660168379 \qquad
x_2 = 3.162277660168379
\]
Et on sait que $10x^2 - 100$ est du signe de $a$ donc positif en dehors des racines
\item Le dénominateur $x^2$ est toujours positif.
\item Tableau de variations
\begin{tikzpicture}[baseline=(a.north)]
\tkzTabInit[lgt=3,espcl=3]{$x$/1,$10x^2 - 100$/1, $x^2$/1, $S'$/1, $S$/2}{$0$, $- 3.162277660168379$, $10$}
\tkzTabLine{d,-, z, +, }
\tkzTabLine{d,+, , +, }
\tkzTabLine{d,-, z, +, }
\tkzTabVar{D+/ , -/ , +/ }
\end{tikzpicture}
\end{itemize}
\item On a donc une surface minimal pour $x=3.162277660168379$ et $h = 31.622776601683790$.
\end{enumerate}
\end{solution}
%%% Local Variables:
%%% mode: latex
%%% TeX-master: "master"
%%% End:
\begin{exercise}[subtitle={Bassin}]
Le tour d'un bassin au niveau du sol présente deux axes de symétrie : laxe des abscisses et la droite déquation $x=4$. Il est obtenu par symétrie de la courbe $\mathcal{C}_f$ sur $\intFF{0}{4}$$f$ est la fonction définie par
\[
f(x) = \left(- x^{2} + 1.0 x - 3.6\right) e^{- x} + 3.6
\]
On admet que sur $\intFF{0}{4}$ la fonction $f$ est positive.
\begin{enumerate}
\item Sur un repère, tracer l'allure de la courbe $\mathcal{C}_f$, les axes de symétries puis compléter pour dessiner la forme du bassin.
\item Montrer que la fonction $f$ admet comme primitive sur $\R$ la fonction $F$ définie par
\[
F(x) = 3.6 x + \left( x^{2} + x + 4.6\right) e^{- x}
\]
\item Calculer la quantité $\ds \int_0^4 f(x) \; dx$, vous donnerez le résultat sous forme exacte. Interpréter le résultat et reportez cette quantité sur le graphique.
\item On considère que l'échelle de votre graphique est de 1unité pour 15m. Calculer l'aire du bassin. Vous donnerez un résultat arrondi au $m^2$ près.
\end{enumerate}
\end{exercise}
\begin{solution}
\begin{enumerate}
\item
\begin{tikzpicture}[baseline=(a.north), xscale=1, yscale=0.5]
\tkzInit[xmin=0,xmax=5,xstep=1,
ymin=0,ymax=10,ystep=1]
\tkzGrid
\tkzAxeXY
\tkzFct[domain=0:10,color=red,very thick]%
{ (-x**2 + 1.0*x - 3.6)*exp(-x) + 3.6 };
\end{tikzpicture}
\item Il faut dériver $F(x)$ et vérifier que $F'(x) = f(x)$.
\item $\ds \int_0^4 f(x) \; dx = F(4) - F(0) = \frac{24.6}{e^{4}} + 9.8$
\item La quantité calculée à la question précédente se retrouve 4fois pour former le bassin. Il faut ensuite prendre en compte l'échelle, comme 1unité de longueur correspond à 15m, une unité d'air correspond à $15\times15 = 225m^2$. Ainsi l'aire du bassin est égale à
\[
(\frac{24.6}{e^{4}} + 9.8)\times 4 \times 15^2 = 9226.000000
\]
\end{enumerate}
\end{solution}
%%% Local Variables:
%%% mode: latex
%%% TeX-master: "master"
%%% End:
\begin{exercise}[subtitle={Stylos}]
\emph{Les parties {\rm A} et {\rm B} de cet exercice sont indépendantes.}
\bigskip
\begin{minipage}{0.6\linewidth}
\textbf{Partie A}
\medskip
Deux ateliers A et B fabriquent des stylos pour une entreprise.
L'atelier A fabrique 64.0\,\% des stylos, et parmi ceux-là, 4.0\,\% possèdent un défaut de fabrication.
De plus, 6.0\,\% des stylos possèdent un défaut de fabrication et sortent de l'atelier B.
Un stylo est prélevé au hasard dans le stock de l'entreprise.
On considère les évènements suivants:
\begin{itemize}
\item A : \og Le stylo a été fabriqué par l'atelier A \fg
\item B : \og Le stylo a été fabriqué par l'atelier B \fg
\item D : \og Le stylo possède un défaut de fabrication \fg
\end{itemize}
\end{minipage}
\begin{minipage}{0.4\linewidth}
\begin{center}
\begin{tikzpicture}[sloped]
\node {.}
child {node {$A$}
child {node {$D$}
edge from parent
node[above] {...}
}
child {node {$\overline{D}$}
edge from parent
node[above] {...}
}
edge from parent
node[above] {...}
}
child[missing] {}
child { node {$B$}
child {node {$D$}
edge from parent
node[above] {...}
}
child {node {$\overline{D}$}
edge from parent
node[above] {...}
}
edge from parent
node[above] {...}
} ;
\end{tikzpicture}
\end{center}
\end{minipage}
\medskip
\begin{enumerate}
\item Compléter l'arbre de probabilité ci-contre
\item Interpréter puis donner les probabilités $P(A)$, $P(B)$, $P_A(D)$ et $P(B \cap D)$.
\item
\begin{enumerate}
\item Calculer la probabilité qu'un stylo provienne de l'atelier A et possède un défaut de fabrication.
\item En déduire que la probabilité qu'un stylo possède un défaut de fabrication est de $0.09$.
\end{enumerate}
\item On prélève un stylo au hasard avec un défaut. Quelle est la probabilité qu'il vienne de l'atelier A?
\end{enumerate}
\bigskip
\textbf{Partie B}
\medskip
Dans cette partie, on suppose que 9.0\,\% des stylos possèdent un défaut de fabrication.
L'entreprise confectionne des paquets contenant chacun $4$~stylos.
Le fait qu'un stylo possède ou non un défaut de fabrication est indépendant des autres stylos.
On appelle $X$ la variable aléatoire donnant pour un paquet le nombre de stylos qui possèdent un défaut de fabrication.
On admet que la variable aléatoire $X$ suit une loi binomiale.
\medskip
\begin{enumerate}
\setcounter{enumi}{4}
\item Avec quelle loi peut-on modéliser $X$. Préciser les paramètres.
\item Calculer et interpréter la probabilité $P(X = 9)$.
\item Le directeur de l'entreprise affirme qu'il y a plus d'une chance sur deux qu'un paquet ne comporte aucun stylo défectueux. A-t-il raison ?
\item Combien de stylos peut-on espérer avoir en moyenne?
\end{enumerate}
\pagebreak
\end{exercise}
\begin{solution}
\begin{enumerate}
\item
\begin{center}
\begin{tikzpicture}[sloped]
\node {.}
child {node {$A$}
child {node {$D$}
edge from parent
node[above] {0.04}
}
child {node {$\overline{D}$}
edge from parent
node[above] {0.96}
}
edge from parent
node[above] {0.64}
}
child[missing] {}
child { node {$B$}
child {node {$D$}
edge from parent
node[above] {0.18}
}
child {node {$\overline{D}$}
edge from parent
node[above] {0.82}
}
edge from parent
node[above] {0.36}
} ;
\end{tikzpicture}
\end{center}
\item
\begin{itemize}
\item Probabilité que le stylo vienne de l'atelier A
\[
P(A) = 0.64
\]
\item Probabilité que le stylo vienne de l'atelier B
\[
P(B) = 0.36
\]
\item Probabilité que le stylo ait un défaut sachant qu'il vient de l'atelier A.
\[
P_A(D) = 0.04
\]
\item Probabilité que le stylo vienne de l'atelier B et qu'il ait un défaut.
\[
P(D \cap D) = 0.06
\]
\end{itemize}
\item
\begin{enumerate}
\item Probabilité qu'un stylo vienne de l'atelier A et qu'il ait un defaut
\[
P(A\cap D) = P(A) \times P_A(D) = 0.64 \times 0.04 = 0.03
\]
\item Probabilité que le stylo ai un défaut de fabrication.
\[
P(D) = P(A\cap D) + P(B\cap D) = 0.03 + 0.06 = 0.09
\]
\end{enumerate}
\item Probabilité qu'il vienne de l'atelier A sachant qu'il a un defaut
\[
P_D(A) = \frac{P(A\cap D)}{P(D)} = \frac{0.03}{0.09} = 0.33
\]
\item $X$ peut être modélisée par une loi binomiale de paramètres $n=16$ et $p=0.09$.
\item (\textit{par de correction automatique disponible pour le résultat final}
\[
P(X = 9) = \coefBino{16}{9}\times 0.09^{9} \times 0.91^{7}
\]
\item (\textit{par de correction automatique disponible pour le résultat final}
Il faut calculer la probabilité qu'il y ait 0 stylo avec un defaut.
\[
P(X = 0) = \coefBino{16}{0}\times 0.09^{0} \times 0.91^{16}
\]
Puis comparer ce nombre à 0,5.
\item Il faut calculer l'espérance
\[
E[X] = n\times p = 16 \times 0.09 = 1.44
\]
\end{enumerate}
\end{solution}
\end{document}
%%% Local Variables:
%%% mode: latex
%%% TeX-master: "master"
%%% End:

View File

@ -0,0 +1,367 @@
\documentclass[a4paper,10pt]{article}
\usepackage{myXsim}
% Title Page
\title{DM1 \hfill SAINT CYR Louis}
\tribe{Maths complémentaire}
\date{\hfillÀ render pour le jeudi 27 mai}
\xsimsetup{
solution/print = true
}
\begin{document}
\maketitle
Les valeurs des exercices sont générés automatiquement. Si une valeur a un nombre adhérant de chiffres après la virgule, vous pouvez l'arrondir à l'entier le plus proche.
\begin{exercise}[subtitle={Optimisation de matière}]
\begin{minipage}{0.6\textwidth}
On se propose de fabriquer avec le moins de tôle possible une citerne fermée en forme de parallélépipède rectangle dont le volume intérieur doit être de $45m^3$. La longueur est aussi fixée à $5m$ par le cahier des charges.
On peut donc faire varier uniquement la largeur (notée $x$) et la hauteur (notée $h$) de la cuve.
\end{minipage}
\hfill
\begin{minipage}{0.3\textwidth}
\begin{tikzpicture}
\pgfmathsetmacro{\cubex}{3}
\pgfmathsetmacro{\cubey}{1}
\pgfmathsetmacro{\cubez}{2}
\draw[black,fill=gray] (0,0,0) -- ++(-\cubex,0,0) -- ++(0,-\cubey,0) node [midway, left] {$h$} -- ++(\cubex,0,0) node [midway, below] {$x$} -- cycle;
\draw[black,fill=gray] (0,0,0) -- ++(0,0,-\cubez) -- ++(0,-\cubey,0) -- ++(0,0,\cubez) node [midway, right] {$5m$} -- cycle;
\draw[black,fill=gray] (0,0,0) -- ++(-\cubex,0,0) -- ++(0,0,-\cubez) -- ++(\cubex,0,0) -- cycle;
\end{tikzpicture}
\end{minipage}
\begin{enumerate}
\item Expliquer pourquoi quand la largeur $x$ change, la hauteur $h$ doit elle aussi changer pour respecter les contraintes.
\item Démontrer que l'on doit avoir $h = \dfrac{9}{x}$.
\item On note $S(x)$ l'aire totale de la citerne (c'est à dire la somme des aires des six faces). Montrer que l'on peut écrire
\[
S(x) = 10x + 18 + \frac{90}{x}
\]
\item Démontrer que
\[
S(x) = \frac{10x^2 + 18x + 90}{x}
\]
\item Démontrer que
\[
S'(x) = \frac{10x^2 - 90}{x^2}
\]
\item En déduire le tableau de variation de $S(x)$ sur $\intOF{0}{10}$.
\item Déterminer les valeurs de $x$ et $h$ correspondant à une utilisation minimal de tôle.
\end{enumerate}
\end{exercise}
\begin{solution}
\begin{enumerate}
\item Le volume étant fixe si l'on fait varier $x$, $h$ doit aussi varier.
\begin{itemize}
\item Si $x = 2$ alors conserver un volume de $V=45$, $h$ doit être égale à $9 / 2$
\item Si $x = 3$ alors conserver un volume de $V=45$, $h$ doit être égale à $9 / 3$
\end{itemize}
\item Pour calculer le volume, on a
\begin{eqnarray*}
V &=& h\times x \times 5 \\
45 &=& h\times x \times 5 \\
x &=& \frac{45}{h\times 5} = \frac{9}{h}
\end{eqnarray*}
\item Pour calculer la surface totale, on ajoute la surface de chaque face. On a donc le calcul suivant
\begin{eqnarray*}
S(x) &=& x\times h \times 2 + x\times5\times2 + h\times 5\times 2\\
S(x) &=& x\times \frac{9}{x} \times 2 + x\times5\times2 + \frac{9}{x}\times 5\times 2\\
S(x) &=& 10x + 18 + \frac{90}{x}
\end{eqnarray*}
\item Pour trouver cette nouvelle forme, on met chaque élément sur le même dénominateur
\begin{eqnarray*}
S(x) &=& 10x + 18 + \frac{90}{x}\\
S(x) &=& \frac{10x\times x}{x} + \frac{18\times x}{x} + \frac{90}{x}\\
S(x) &=& \frac{10x^2 + 18x + 90}{x}
\end{eqnarray*}
\item On retrouve la formule $\frac{u}{v}$ à dériver
\[
u(x) = 10x^2 + 18x + 90 \Rightarrow u'(x) = 20x + 18
\]
\[
v(x) = x \Rightarrow v'(x) = 1
\]
Donc au numérateur on obtient
\begin{eqnarray*}
u'(x)\times v(x) - u(x)\times v'(x) &=& (20x + 18)\times x - (10x^2 + 18x + 90)\times 1\\
&=& 10x^2 - 90
\end{eqnarray*}
Donc
\[
S'(x) = \frac{10x^2 - 90}{x^2}
\]
\item Tableau de variations de $S$
\begin{itemize}
\item Valeur interdite: $x^2 = 0 \equiv x = 0$
\item Signe de $10x^2 - 90$: c'est un polynôme du 2e degré
\[
\Delta = 3600 > 0
\]
Il y a donc 2 racines
\[
x_1 = - 3 \qquad
x_2 = 3
\]
Et on sait que $10x^2 - 90$ est du signe de $a$ donc positif en dehors des racines
\item Le dénominateur $x^2$ est toujours positif.
\item Tableau de variations
\begin{tikzpicture}[baseline=(a.north)]
\tkzTabInit[lgt=3,espcl=3]{$x$/1,$10x^2 - 90$/1, $x^2$/1, $S'$/1, $S$/2}{$0$, $- 3$, $10$}
\tkzTabLine{d,-, z, +, }
\tkzTabLine{d,+, , +, }
\tkzTabLine{d,-, z, +, }
\tkzTabVar{D+/ , -/ , +/ }
\end{tikzpicture}
\end{itemize}
\item On a donc une surface minimal pour $x=3$ et $h = 27$.
\end{enumerate}
\end{solution}
%%% Local Variables:
%%% mode: latex
%%% TeX-master: "master"
%%% End:
\begin{exercise}[subtitle={Bassin}]
Le tour d'un bassin au niveau du sol présente deux axes de symétrie : laxe des abscisses et la droite déquation $x=4$. Il est obtenu par symétrie de la courbe $\mathcal{C}_f$ sur $\intFF{0}{4}$$f$ est la fonction définie par
\[
f(x) = \left(- x^{2} + 8.0 x - 0.9\right) e^{- x} + 0.9
\]
On admet que sur $\intFF{0}{4}$ la fonction $f$ est positive.
\begin{enumerate}
\item Sur un repère, tracer l'allure de la courbe $\mathcal{C}_f$, les axes de symétries puis compléter pour dessiner la forme du bassin.
\item Montrer que la fonction $f$ admet comme primitive sur $\R$ la fonction $F$ définie par
\[
F(x) = 0.9 x + \left( x^{2} - 6.0 x - 5.1\right) e^{- x}
\]
\item Calculer la quantité $\ds \int_0^4 f(x) \; dx$, vous donnerez le résultat sous forme exacte. Interpréter le résultat et reportez cette quantité sur le graphique.
\item On considère que l'échelle de votre graphique est de 1unité pour 15m. Calculer l'aire du bassin. Vous donnerez un résultat arrondi au $m^2$ près.
\end{enumerate}
\end{exercise}
\begin{solution}
\begin{enumerate}
\item
\begin{tikzpicture}[baseline=(a.north), xscale=1, yscale=0.5]
\tkzInit[xmin=0,xmax=5,xstep=1,
ymin=0,ymax=10,ystep=1]
\tkzGrid
\tkzAxeXY
\tkzFct[domain=0:10,color=red,very thick]%
{ (-x**2 + 8.0*x - 0.9)*exp(-x) + 0.9 };
\end{tikzpicture}
\item Il faut dériver $F(x)$ et vérifier que $F'(x) = f(x)$.
\item $\ds \int_0^4 f(x) \; dx = F(4) - F(0) = 8.7 - \frac{13.1}{e^{4}}$
\item La quantité calculée à la question précédente se retrouve 4fois pour former le bassin. Il faut ensuite prendre en compte l'échelle, comme 1unité de longueur correspond à 15m, une unité d'air correspond à $15\times15 = 225m^2$. Ainsi l'aire du bassin est égale à
\[
(8.7 - \frac{13.1}{e^{4}})\times 4 \times 15^2 = 7614.000000
\]
\end{enumerate}
\end{solution}
%%% Local Variables:
%%% mode: latex
%%% TeX-master: "master"
%%% End:
\begin{exercise}[subtitle={Stylos}]
\emph{Les parties {\rm A} et {\rm B} de cet exercice sont indépendantes.}
\bigskip
\begin{minipage}{0.6\linewidth}
\textbf{Partie A}
\medskip
Deux ateliers A et B fabriquent des stylos pour une entreprise.
L'atelier A fabrique 88.0\,\% des stylos, et parmi ceux-là, 75.0\,\% possèdent un défaut de fabrication.
De plus, 4.0\,\% des stylos possèdent un défaut de fabrication et sortent de l'atelier B.
Un stylo est prélevé au hasard dans le stock de l'entreprise.
On considère les évènements suivants:
\begin{itemize}
\item A : \og Le stylo a été fabriqué par l'atelier A \fg
\item B : \og Le stylo a été fabriqué par l'atelier B \fg
\item D : \og Le stylo possède un défaut de fabrication \fg
\end{itemize}
\end{minipage}
\begin{minipage}{0.4\linewidth}
\begin{center}
\begin{tikzpicture}[sloped]
\node {.}
child {node {$A$}
child {node {$D$}
edge from parent
node[above] {...}
}
child {node {$\overline{D}$}
edge from parent
node[above] {...}
}
edge from parent
node[above] {...}
}
child[missing] {}
child { node {$B$}
child {node {$D$}
edge from parent
node[above] {...}
}
child {node {$\overline{D}$}
edge from parent
node[above] {...}
}
edge from parent
node[above] {...}
} ;
\end{tikzpicture}
\end{center}
\end{minipage}
\medskip
\begin{enumerate}
\item Compléter l'arbre de probabilité ci-contre
\item Interpréter puis donner les probabilités $P(A)$, $P(B)$, $P_A(D)$ et $P(B \cap D)$.
\item
\begin{enumerate}
\item Calculer la probabilité qu'un stylo provienne de l'atelier A et possède un défaut de fabrication.
\item En déduire que la probabilité qu'un stylo possède un défaut de fabrication est de $0.7$.
\end{enumerate}
\item On prélève un stylo au hasard avec un défaut. Quelle est la probabilité qu'il vienne de l'atelier A?
\end{enumerate}
\bigskip
\textbf{Partie B}
\medskip
Dans cette partie, on suppose que 70.0\,\% des stylos possèdent un défaut de fabrication.
L'entreprise confectionne des paquets contenant chacun $4$~stylos.
Le fait qu'un stylo possède ou non un défaut de fabrication est indépendant des autres stylos.
On appelle $X$ la variable aléatoire donnant pour un paquet le nombre de stylos qui possèdent un défaut de fabrication.
On admet que la variable aléatoire $X$ suit une loi binomiale.
\medskip
\begin{enumerate}
\setcounter{enumi}{4}
\item Avec quelle loi peut-on modéliser $X$. Préciser les paramètres.
\item Calculer et interpréter la probabilité $P(X = 14)$.
\item Le directeur de l'entreprise affirme qu'il y a plus d'une chance sur deux qu'un paquet ne comporte aucun stylo défectueux. A-t-il raison ?
\item Combien de stylos peut-on espérer avoir en moyenne?
\end{enumerate}
\pagebreak
\end{exercise}
\begin{solution}
\begin{enumerate}
\item
\begin{center}
\begin{tikzpicture}[sloped]
\node {.}
child {node {$A$}
child {node {$D$}
edge from parent
node[above] {0.75}
}
child {node {$\overline{D}$}
edge from parent
node[above] {0.25}
}
edge from parent
node[above] {0.88}
}
child[missing] {}
child { node {$B$}
child {node {$D$}
edge from parent
node[above] {0.34}
}
child {node {$\overline{D}$}
edge from parent
node[above] {0.66}
}
edge from parent
node[above] {0.12}
} ;
\end{tikzpicture}
\end{center}
\item
\begin{itemize}
\item Probabilité que le stylo vienne de l'atelier A
\[
P(A) = 0.88
\]
\item Probabilité que le stylo vienne de l'atelier B
\[
P(B) = 0.12
\]
\item Probabilité que le stylo ait un défaut sachant qu'il vient de l'atelier A.
\[
P_A(D) = 0.75
\]
\item Probabilité que le stylo vienne de l'atelier B et qu'il ait un défaut.
\[
P(D \cap D) = 0.04
\]
\end{itemize}
\item
\begin{enumerate}
\item Probabilité qu'un stylo vienne de l'atelier A et qu'il ait un defaut
\[
P(A\cap D) = P(A) \times P_A(D) = 0.88 \times 0.75 = 0.66
\]
\item Probabilité que le stylo ai un défaut de fabrication.
\[
P(D) = P(A\cap D) + P(B\cap D) = 0.66 + 0.04 = 0.7
\]
\end{enumerate}
\item Probabilité qu'il vienne de l'atelier A sachant qu'il a un defaut
\[
P_D(A) = \frac{P(A\cap D)}{P(D)} = \frac{0.66}{0.7} = 0.94
\]
\item $X$ peut être modélisée par une loi binomiale de paramètres $n=15$ et $p=0.7$.
\item (\textit{par de correction automatique disponible pour le résultat final}
\[
P(X = 14) = \coefBino{15}{14}\times 0.7^{14} \times 0.3^{1}
\]
\item (\textit{par de correction automatique disponible pour le résultat final}
Il faut calculer la probabilité qu'il y ait 0 stylo avec un defaut.
\[
P(X = 0) = \coefBino{15}{0}\times 0.7^{0} \times 0.3^{15}
\]
Puis comparer ce nombre à 0,5.
\item Il faut calculer l'espérance
\[
E[X] = n\times p = 15 \times 0.7 = 10.5
\]
\end{enumerate}
\end{solution}
\end{document}
%%% Local Variables:
%%% mode: latex
%%% TeX-master: "master"
%%% End:

View File

@ -0,0 +1,367 @@
\documentclass[a4paper,10pt]{article}
\usepackage{myXsim}
% Title Page
\title{DM1 \hfill SAVIN Lou-Ann}
\tribe{Maths complémentaire}
\date{\hfillÀ render pour le jeudi 27 mai}
\xsimsetup{
solution/print = true
}
\begin{document}
\maketitle
Les valeurs des exercices sont générés automatiquement. Si une valeur a un nombre adhérant de chiffres après la virgule, vous pouvez l'arrondir à l'entier le plus proche.
\begin{exercise}[subtitle={Optimisation de matière}]
\begin{minipage}{0.6\textwidth}
On se propose de fabriquer avec le moins de tôle possible une citerne fermée en forme de parallélépipède rectangle dont le volume intérieur doit être de $18m^3$. La longueur est aussi fixée à $3m$ par le cahier des charges.
On peut donc faire varier uniquement la largeur (notée $x$) et la hauteur (notée $h$) de la cuve.
\end{minipage}
\hfill
\begin{minipage}{0.3\textwidth}
\begin{tikzpicture}
\pgfmathsetmacro{\cubex}{3}
\pgfmathsetmacro{\cubey}{1}
\pgfmathsetmacro{\cubez}{2}
\draw[black,fill=gray] (0,0,0) -- ++(-\cubex,0,0) -- ++(0,-\cubey,0) node [midway, left] {$h$} -- ++(\cubex,0,0) node [midway, below] {$x$} -- cycle;
\draw[black,fill=gray] (0,0,0) -- ++(0,0,-\cubez) -- ++(0,-\cubey,0) -- ++(0,0,\cubez) node [midway, right] {$3m$} -- cycle;
\draw[black,fill=gray] (0,0,0) -- ++(-\cubex,0,0) -- ++(0,0,-\cubez) -- ++(\cubex,0,0) -- cycle;
\end{tikzpicture}
\end{minipage}
\begin{enumerate}
\item Expliquer pourquoi quand la largeur $x$ change, la hauteur $h$ doit elle aussi changer pour respecter les contraintes.
\item Démontrer que l'on doit avoir $h = \dfrac{6}{x}$.
\item On note $S(x)$ l'aire totale de la citerne (c'est à dire la somme des aires des six faces). Montrer que l'on peut écrire
\[
S(x) = 6x + 12 + \frac{36}{x}
\]
\item Démontrer que
\[
S(x) = \frac{6x^2 + 12x + 36}{x}
\]
\item Démontrer que
\[
S'(x) = \frac{6x^2 - 36}{x^2}
\]
\item En déduire le tableau de variation de $S(x)$ sur $\intOF{0}{10}$.
\item Déterminer les valeurs de $x$ et $h$ correspondant à une utilisation minimal de tôle.
\end{enumerate}
\end{exercise}
\begin{solution}
\begin{enumerate}
\item Le volume étant fixe si l'on fait varier $x$, $h$ doit aussi varier.
\begin{itemize}
\item Si $x = 2$ alors conserver un volume de $V=18$, $h$ doit être égale à $6 / 2$
\item Si $x = 3$ alors conserver un volume de $V=18$, $h$ doit être égale à $6 / 3$
\end{itemize}
\item Pour calculer le volume, on a
\begin{eqnarray*}
V &=& h\times x \times 3 \\
18 &=& h\times x \times 3 \\
x &=& \frac{18}{h\times 3} = \frac{6}{h}
\end{eqnarray*}
\item Pour calculer la surface totale, on ajoute la surface de chaque face. On a donc le calcul suivant
\begin{eqnarray*}
S(x) &=& x\times h \times 2 + x\times3\times2 + h\times 3\times 2\\
S(x) &=& x\times \frac{6}{x} \times 2 + x\times3\times2 + \frac{6}{x}\times 3\times 2\\
S(x) &=& 6x + 12 + \frac{36}{x}
\end{eqnarray*}
\item Pour trouver cette nouvelle forme, on met chaque élément sur le même dénominateur
\begin{eqnarray*}
S(x) &=& 6x + 12 + \frac{36}{x}\\
S(x) &=& \frac{6x\times x}{x} + \frac{12\times x}{x} + \frac{36}{x}\\
S(x) &=& \frac{6x^2 + 12x + 36}{x}
\end{eqnarray*}
\item On retrouve la formule $\frac{u}{v}$ à dériver
\[
u(x) = 6x^2 + 12x + 36 \Rightarrow u'(x) = 12x + 12
\]
\[
v(x) = x \Rightarrow v'(x) = 1
\]
Donc au numérateur on obtient
\begin{eqnarray*}
u'(x)\times v(x) - u(x)\times v'(x) &=& (12x + 12)\times x - (6x^2 + 12x + 36)\times 1\\
&=& 6x^2 - 36
\end{eqnarray*}
Donc
\[
S'(x) = \frac{6x^2 - 36}{x^2}
\]
\item Tableau de variations de $S$
\begin{itemize}
\item Valeur interdite: $x^2 = 0 \equiv x = 0$
\item Signe de $6x^2 - 36$: c'est un polynôme du 2e degré
\[
\Delta = 864 > 0
\]
Il y a donc 2 racines
\[
x_1 = - 2.4494897427831783 \qquad
x_2 = 2.4494897427831783
\]
Et on sait que $6x^2 - 36$ est du signe de $a$ donc positif en dehors des racines
\item Le dénominateur $x^2$ est toujours positif.
\item Tableau de variations
\begin{tikzpicture}[baseline=(a.north)]
\tkzTabInit[lgt=3,espcl=3]{$x$/1,$6x^2 - 36$/1, $x^2$/1, $S'$/1, $S$/2}{$0$, $- 2.4494897427831783$, $10$}
\tkzTabLine{d,-, z, +, }
\tkzTabLine{d,+, , +, }
\tkzTabLine{d,-, z, +, }
\tkzTabVar{D+/ , -/ , +/ }
\end{tikzpicture}
\end{itemize}
\item On a donc une surface minimal pour $x=2.4494897427831783$ et $h = 14.6969384566990698$.
\end{enumerate}
\end{solution}
%%% Local Variables:
%%% mode: latex
%%% TeX-master: "master"
%%% End:
\begin{exercise}[subtitle={Bassin}]
Le tour d'un bassin au niveau du sol présente deux axes de symétrie : laxe des abscisses et la droite déquation $x=4$. Il est obtenu par symétrie de la courbe $\mathcal{C}_f$ sur $\intFF{0}{4}$$f$ est la fonction définie par
\[
f(x) = \left(- x^{2} + 2.7 x - 6.1\right) e^{- x} + 6.1
\]
On admet que sur $\intFF{0}{4}$ la fonction $f$ est positive.
\begin{enumerate}
\item Sur un repère, tracer l'allure de la courbe $\mathcal{C}_f$, les axes de symétries puis compléter pour dessiner la forme du bassin.
\item Montrer que la fonction $f$ admet comme primitive sur $\R$ la fonction $F$ définie par
\[
F(x) = 6.1 x + \left( x^{2} - 0.7 x + 5.4\right) e^{- x}
\]
\item Calculer la quantité $\ds \int_0^4 f(x) \; dx$, vous donnerez le résultat sous forme exacte. Interpréter le résultat et reportez cette quantité sur le graphique.
\item On considère que l'échelle de votre graphique est de 1unité pour 15m. Calculer l'aire du bassin. Vous donnerez un résultat arrondi au $m^2$ près.
\end{enumerate}
\end{exercise}
\begin{solution}
\begin{enumerate}
\item
\begin{tikzpicture}[baseline=(a.north), xscale=1, yscale=0.5]
\tkzInit[xmin=0,xmax=5,xstep=1,
ymin=0,ymax=10,ystep=1]
\tkzGrid
\tkzAxeXY
\tkzFct[domain=0:10,color=red,very thick]%
{ (-x**2 + 2.7*x - 6.1)*exp(-x) + 6.1 };
\end{tikzpicture}
\item Il faut dériver $F(x)$ et vérifier que $F'(x) = f(x)$.
\item $\ds \int_0^4 f(x) \; dx = F(4) - F(0) = \frac{18.6}{e^{4}} + 19.0$
\item La quantité calculée à la question précédente se retrouve 4fois pour former le bassin. Il faut ensuite prendre en compte l'échelle, comme 1unité de longueur correspond à 15m, une unité d'air correspond à $15\times15 = 225m^2$. Ainsi l'aire du bassin est égale à
\[
(\frac{18.6}{e^{4}} + 19.0)\times 4 \times 15^2 = 17407.00000
\]
\end{enumerate}
\end{solution}
%%% Local Variables:
%%% mode: latex
%%% TeX-master: "master"
%%% End:
\begin{exercise}[subtitle={Stylos}]
\emph{Les parties {\rm A} et {\rm B} de cet exercice sont indépendantes.}
\bigskip
\begin{minipage}{0.6\linewidth}
\textbf{Partie A}
\medskip
Deux ateliers A et B fabriquent des stylos pour une entreprise.
L'atelier A fabrique 47.0\,\% des stylos, et parmi ceux-là, 45.0\,\% possèdent un défaut de fabrication.
De plus, 3.0\,\% des stylos possèdent un défaut de fabrication et sortent de l'atelier B.
Un stylo est prélevé au hasard dans le stock de l'entreprise.
On considère les évènements suivants:
\begin{itemize}
\item A : \og Le stylo a été fabriqué par l'atelier A \fg
\item B : \og Le stylo a été fabriqué par l'atelier B \fg
\item D : \og Le stylo possède un défaut de fabrication \fg
\end{itemize}
\end{minipage}
\begin{minipage}{0.4\linewidth}
\begin{center}
\begin{tikzpicture}[sloped]
\node {.}
child {node {$A$}
child {node {$D$}
edge from parent
node[above] {...}
}
child {node {$\overline{D}$}
edge from parent
node[above] {...}
}
edge from parent
node[above] {...}
}
child[missing] {}
child { node {$B$}
child {node {$D$}
edge from parent
node[above] {...}
}
child {node {$\overline{D}$}
edge from parent
node[above] {...}
}
edge from parent
node[above] {...}
} ;
\end{tikzpicture}
\end{center}
\end{minipage}
\medskip
\begin{enumerate}
\item Compléter l'arbre de probabilité ci-contre
\item Interpréter puis donner les probabilités $P(A)$, $P(B)$, $P_A(D)$ et $P(B \cap D)$.
\item
\begin{enumerate}
\item Calculer la probabilité qu'un stylo provienne de l'atelier A et possède un défaut de fabrication.
\item En déduire que la probabilité qu'un stylo possède un défaut de fabrication est de $0.24$.
\end{enumerate}
\item On prélève un stylo au hasard avec un défaut. Quelle est la probabilité qu'il vienne de l'atelier A?
\end{enumerate}
\bigskip
\textbf{Partie B}
\medskip
Dans cette partie, on suppose que 24.0\,\% des stylos possèdent un défaut de fabrication.
L'entreprise confectionne des paquets contenant chacun $4$~stylos.
Le fait qu'un stylo possède ou non un défaut de fabrication est indépendant des autres stylos.
On appelle $X$ la variable aléatoire donnant pour un paquet le nombre de stylos qui possèdent un défaut de fabrication.
On admet que la variable aléatoire $X$ suit une loi binomiale.
\medskip
\begin{enumerate}
\setcounter{enumi}{4}
\item Avec quelle loi peut-on modéliser $X$. Préciser les paramètres.
\item Calculer et interpréter la probabilité $P(X = 9)$.
\item Le directeur de l'entreprise affirme qu'il y a plus d'une chance sur deux qu'un paquet ne comporte aucun stylo défectueux. A-t-il raison ?
\item Combien de stylos peut-on espérer avoir en moyenne?
\end{enumerate}
\pagebreak
\end{exercise}
\begin{solution}
\begin{enumerate}
\item
\begin{center}
\begin{tikzpicture}[sloped]
\node {.}
child {node {$A$}
child {node {$D$}
edge from parent
node[above] {0.45}
}
child {node {$\overline{D}$}
edge from parent
node[above] {0.55}
}
edge from parent
node[above] {0.47}
}
child[missing] {}
child { node {$B$}
child {node {$D$}
edge from parent
node[above] {0.05}
}
child {node {$\overline{D}$}
edge from parent
node[above] {0.95}
}
edge from parent
node[above] {0.53}
} ;
\end{tikzpicture}
\end{center}
\item
\begin{itemize}
\item Probabilité que le stylo vienne de l'atelier A
\[
P(A) = 0.47
\]
\item Probabilité que le stylo vienne de l'atelier B
\[
P(B) = 0.53
\]
\item Probabilité que le stylo ait un défaut sachant qu'il vient de l'atelier A.
\[
P_A(D) = 0.45
\]
\item Probabilité que le stylo vienne de l'atelier B et qu'il ait un défaut.
\[
P(D \cap D) = 0.03
\]
\end{itemize}
\item
\begin{enumerate}
\item Probabilité qu'un stylo vienne de l'atelier A et qu'il ait un defaut
\[
P(A\cap D) = P(A) \times P_A(D) = 0.47 \times 0.45 = 0.21
\]
\item Probabilité que le stylo ai un défaut de fabrication.
\[
P(D) = P(A\cap D) + P(B\cap D) = 0.21 + 0.03 = 0.24
\]
\end{enumerate}
\item Probabilité qu'il vienne de l'atelier A sachant qu'il a un defaut
\[
P_D(A) = \frac{P(A\cap D)}{P(D)} = \frac{0.21}{0.24} = 0.88
\]
\item $X$ peut être modélisée par une loi binomiale de paramètres $n=19$ et $p=0.24$.
\item (\textit{par de correction automatique disponible pour le résultat final}
\[
P(X = 9) = \coefBino{19}{9}\times 0.24^{9} \times 0.76^{10}
\]
\item (\textit{par de correction automatique disponible pour le résultat final}
Il faut calculer la probabilité qu'il y ait 0 stylo avec un defaut.
\[
P(X = 0) = \coefBino{19}{0}\times 0.24^{0} \times 0.76^{19}
\]
Puis comparer ce nombre à 0,5.
\item Il faut calculer l'espérance
\[
E[X] = n\times p = 19 \times 0.24 = 4.56
\]
\end{enumerate}
\end{solution}
\end{document}
%%% Local Variables:
%%% mode: latex
%%% TeX-master: "master"
%%% End:

View File

@ -0,0 +1,367 @@
\documentclass[a4paper,10pt]{article}
\usepackage{myXsim}
% Title Page
\title{DM1 \hfill SILVA LOPES Katleen}
\tribe{Maths complémentaire}
\date{\hfillÀ render pour le jeudi 27 mai}
\xsimsetup{
solution/print = true
}
\begin{document}
\maketitle
Les valeurs des exercices sont générés automatiquement. Si une valeur a un nombre adhérant de chiffres après la virgule, vous pouvez l'arrondir à l'entier le plus proche.
\begin{exercise}[subtitle={Optimisation de matière}]
\begin{minipage}{0.6\textwidth}
On se propose de fabriquer avec le moins de tôle possible une citerne fermée en forme de parallélépipède rectangle dont le volume intérieur doit être de $27m^3$. La longueur est aussi fixée à $3m$ par le cahier des charges.
On peut donc faire varier uniquement la largeur (notée $x$) et la hauteur (notée $h$) de la cuve.
\end{minipage}
\hfill
\begin{minipage}{0.3\textwidth}
\begin{tikzpicture}
\pgfmathsetmacro{\cubex}{3}
\pgfmathsetmacro{\cubey}{1}
\pgfmathsetmacro{\cubez}{2}
\draw[black,fill=gray] (0,0,0) -- ++(-\cubex,0,0) -- ++(0,-\cubey,0) node [midway, left] {$h$} -- ++(\cubex,0,0) node [midway, below] {$x$} -- cycle;
\draw[black,fill=gray] (0,0,0) -- ++(0,0,-\cubez) -- ++(0,-\cubey,0) -- ++(0,0,\cubez) node [midway, right] {$3m$} -- cycle;
\draw[black,fill=gray] (0,0,0) -- ++(-\cubex,0,0) -- ++(0,0,-\cubez) -- ++(\cubex,0,0) -- cycle;
\end{tikzpicture}
\end{minipage}
\begin{enumerate}
\item Expliquer pourquoi quand la largeur $x$ change, la hauteur $h$ doit elle aussi changer pour respecter les contraintes.
\item Démontrer que l'on doit avoir $h = \dfrac{9}{x}$.
\item On note $S(x)$ l'aire totale de la citerne (c'est à dire la somme des aires des six faces). Montrer que l'on peut écrire
\[
S(x) = 6x + 18 + \frac{54}{x}
\]
\item Démontrer que
\[
S(x) = \frac{6x^2 + 18x + 54}{x}
\]
\item Démontrer que
\[
S'(x) = \frac{6x^2 - 54}{x^2}
\]
\item En déduire le tableau de variation de $S(x)$ sur $\intOF{0}{10}$.
\item Déterminer les valeurs de $x$ et $h$ correspondant à une utilisation minimal de tôle.
\end{enumerate}
\end{exercise}
\begin{solution}
\begin{enumerate}
\item Le volume étant fixe si l'on fait varier $x$, $h$ doit aussi varier.
\begin{itemize}
\item Si $x = 2$ alors conserver un volume de $V=27$, $h$ doit être égale à $9 / 2$
\item Si $x = 3$ alors conserver un volume de $V=27$, $h$ doit être égale à $9 / 3$
\end{itemize}
\item Pour calculer le volume, on a
\begin{eqnarray*}
V &=& h\times x \times 3 \\
27 &=& h\times x \times 3 \\
x &=& \frac{27}{h\times 3} = \frac{9}{h}
\end{eqnarray*}
\item Pour calculer la surface totale, on ajoute la surface de chaque face. On a donc le calcul suivant
\begin{eqnarray*}
S(x) &=& x\times h \times 2 + x\times3\times2 + h\times 3\times 2\\
S(x) &=& x\times \frac{9}{x} \times 2 + x\times3\times2 + \frac{9}{x}\times 3\times 2\\
S(x) &=& 6x + 18 + \frac{54}{x}
\end{eqnarray*}
\item Pour trouver cette nouvelle forme, on met chaque élément sur le même dénominateur
\begin{eqnarray*}
S(x) &=& 6x + 18 + \frac{54}{x}\\
S(x) &=& \frac{6x\times x}{x} + \frac{18\times x}{x} + \frac{54}{x}\\
S(x) &=& \frac{6x^2 + 18x + 54}{x}
\end{eqnarray*}
\item On retrouve la formule $\frac{u}{v}$ à dériver
\[
u(x) = 6x^2 + 18x + 54 \Rightarrow u'(x) = 12x + 18
\]
\[
v(x) = x \Rightarrow v'(x) = 1
\]
Donc au numérateur on obtient
\begin{eqnarray*}
u'(x)\times v(x) - u(x)\times v'(x) &=& (12x + 18)\times x - (6x^2 + 18x + 54)\times 1\\
&=& 6x^2 - 54
\end{eqnarray*}
Donc
\[
S'(x) = \frac{6x^2 - 54}{x^2}
\]
\item Tableau de variations de $S$
\begin{itemize}
\item Valeur interdite: $x^2 = 0 \equiv x = 0$
\item Signe de $6x^2 - 54$: c'est un polynôme du 2e degré
\[
\Delta = 1296 > 0
\]
Il y a donc 2 racines
\[
x_1 = - 3 \qquad
x_2 = 3
\]
Et on sait que $6x^2 - 54$ est du signe de $a$ donc positif en dehors des racines
\item Le dénominateur $x^2$ est toujours positif.
\item Tableau de variations
\begin{tikzpicture}[baseline=(a.north)]
\tkzTabInit[lgt=3,espcl=3]{$x$/1,$6x^2 - 54$/1, $x^2$/1, $S'$/1, $S$/2}{$0$, $- 3$, $10$}
\tkzTabLine{d,-, z, +, }
\tkzTabLine{d,+, , +, }
\tkzTabLine{d,-, z, +, }
\tkzTabVar{D+/ , -/ , +/ }
\end{tikzpicture}
\end{itemize}
\item On a donc une surface minimal pour $x=3$ et $h = 27$.
\end{enumerate}
\end{solution}
%%% Local Variables:
%%% mode: latex
%%% TeX-master: "master"
%%% End:
\begin{exercise}[subtitle={Bassin}]
Le tour d'un bassin au niveau du sol présente deux axes de symétrie : laxe des abscisses et la droite déquation $x=4$. Il est obtenu par symétrie de la courbe $\mathcal{C}_f$ sur $\intFF{0}{4}$$f$ est la fonction définie par
\[
f(x) = \left(- x^{2} + 3.0 x - 6.6\right) e^{- x} + 6.6
\]
On admet que sur $\intFF{0}{4}$ la fonction $f$ est positive.
\begin{enumerate}
\item Sur un repère, tracer l'allure de la courbe $\mathcal{C}_f$, les axes de symétries puis compléter pour dessiner la forme du bassin.
\item Montrer que la fonction $f$ admet comme primitive sur $\R$ la fonction $F$ définie par
\[
F(x) = 6.6 x + \left( x^{2} - x + 5.6\right) e^{- x}
\]
\item Calculer la quantité $\ds \int_0^4 f(x) \; dx$, vous donnerez le résultat sous forme exacte. Interpréter le résultat et reportez cette quantité sur le graphique.
\item On considère que l'échelle de votre graphique est de 1unité pour 15m. Calculer l'aire du bassin. Vous donnerez un résultat arrondi au $m^2$ près.
\end{enumerate}
\end{exercise}
\begin{solution}
\begin{enumerate}
\item
\begin{tikzpicture}[baseline=(a.north), xscale=1, yscale=0.5]
\tkzInit[xmin=0,xmax=5,xstep=1,
ymin=0,ymax=10,ystep=1]
\tkzGrid
\tkzAxeXY
\tkzFct[domain=0:10,color=red,very thick]%
{ (-x**2 + 3.0*x - 6.6)*exp(-x) + 6.6 };
\end{tikzpicture}
\item Il faut dériver $F(x)$ et vérifier que $F'(x) = f(x)$.
\item $\ds \int_0^4 f(x) \; dx = F(4) - F(0) = \frac{17.6}{e^{4}} + 20.8$
\item La quantité calculée à la question précédente se retrouve 4fois pour former le bassin. Il faut ensuite prendre en compte l'échelle, comme 1unité de longueur correspond à 15m, une unité d'air correspond à $15\times15 = 225m^2$. Ainsi l'aire du bassin est égale à
\[
(\frac{17.6}{e^{4}} + 20.8)\times 4 \times 15^2 = 19010.00000
\]
\end{enumerate}
\end{solution}
%%% Local Variables:
%%% mode: latex
%%% TeX-master: "master"
%%% End:
\begin{exercise}[subtitle={Stylos}]
\emph{Les parties {\rm A} et {\rm B} de cet exercice sont indépendantes.}
\bigskip
\begin{minipage}{0.6\linewidth}
\textbf{Partie A}
\medskip
Deux ateliers A et B fabriquent des stylos pour une entreprise.
L'atelier A fabrique 43.0\,\% des stylos, et parmi ceux-là, 31.0\,\% possèdent un défaut de fabrication.
De plus, 19.0\,\% des stylos possèdent un défaut de fabrication et sortent de l'atelier B.
Un stylo est prélevé au hasard dans le stock de l'entreprise.
On considère les évènements suivants:
\begin{itemize}
\item A : \og Le stylo a été fabriqué par l'atelier A \fg
\item B : \og Le stylo a été fabriqué par l'atelier B \fg
\item D : \og Le stylo possède un défaut de fabrication \fg
\end{itemize}
\end{minipage}
\begin{minipage}{0.4\linewidth}
\begin{center}
\begin{tikzpicture}[sloped]
\node {.}
child {node {$A$}
child {node {$D$}
edge from parent
node[above] {...}
}
child {node {$\overline{D}$}
edge from parent
node[above] {...}
}
edge from parent
node[above] {...}
}
child[missing] {}
child { node {$B$}
child {node {$D$}
edge from parent
node[above] {...}
}
child {node {$\overline{D}$}
edge from parent
node[above] {...}
}
edge from parent
node[above] {...}
} ;
\end{tikzpicture}
\end{center}
\end{minipage}
\medskip
\begin{enumerate}
\item Compléter l'arbre de probabilité ci-contre
\item Interpréter puis donner les probabilités $P(A)$, $P(B)$, $P_A(D)$ et $P(B \cap D)$.
\item
\begin{enumerate}
\item Calculer la probabilité qu'un stylo provienne de l'atelier A et possède un défaut de fabrication.
\item En déduire que la probabilité qu'un stylo possède un défaut de fabrication est de $0.32$.
\end{enumerate}
\item On prélève un stylo au hasard avec un défaut. Quelle est la probabilité qu'il vienne de l'atelier A?
\end{enumerate}
\bigskip
\textbf{Partie B}
\medskip
Dans cette partie, on suppose que 32.0\,\% des stylos possèdent un défaut de fabrication.
L'entreprise confectionne des paquets contenant chacun $4$~stylos.
Le fait qu'un stylo possède ou non un défaut de fabrication est indépendant des autres stylos.
On appelle $X$ la variable aléatoire donnant pour un paquet le nombre de stylos qui possèdent un défaut de fabrication.
On admet que la variable aléatoire $X$ suit une loi binomiale.
\medskip
\begin{enumerate}
\setcounter{enumi}{4}
\item Avec quelle loi peut-on modéliser $X$. Préciser les paramètres.
\item Calculer et interpréter la probabilité $P(X = 7)$.
\item Le directeur de l'entreprise affirme qu'il y a plus d'une chance sur deux qu'un paquet ne comporte aucun stylo défectueux. A-t-il raison ?
\item Combien de stylos peut-on espérer avoir en moyenne?
\end{enumerate}
\pagebreak
\end{exercise}
\begin{solution}
\begin{enumerate}
\item
\begin{center}
\begin{tikzpicture}[sloped]
\node {.}
child {node {$A$}
child {node {$D$}
edge from parent
node[above] {0.31}
}
child {node {$\overline{D}$}
edge from parent
node[above] {0.69}
}
edge from parent
node[above] {0.43}
}
child[missing] {}
child { node {$B$}
child {node {$D$}
edge from parent
node[above] {0.34}
}
child {node {$\overline{D}$}
edge from parent
node[above] {0.66}
}
edge from parent
node[above] {0.57}
} ;
\end{tikzpicture}
\end{center}
\item
\begin{itemize}
\item Probabilité que le stylo vienne de l'atelier A
\[
P(A) = 0.43
\]
\item Probabilité que le stylo vienne de l'atelier B
\[
P(B) = 0.57
\]
\item Probabilité que le stylo ait un défaut sachant qu'il vient de l'atelier A.
\[
P_A(D) = 0.31
\]
\item Probabilité que le stylo vienne de l'atelier B et qu'il ait un défaut.
\[
P(D \cap D) = 0.19
\]
\end{itemize}
\item
\begin{enumerate}
\item Probabilité qu'un stylo vienne de l'atelier A et qu'il ait un defaut
\[
P(A\cap D) = P(A) \times P_A(D) = 0.43 \times 0.31 = 0.13
\]
\item Probabilité que le stylo ai un défaut de fabrication.
\[
P(D) = P(A\cap D) + P(B\cap D) = 0.13 + 0.19 = 0.32
\]
\end{enumerate}
\item Probabilité qu'il vienne de l'atelier A sachant qu'il a un defaut
\[
P_D(A) = \frac{P(A\cap D)}{P(D)} = \frac{0.13}{0.32} = 0.41
\]
\item $X$ peut être modélisée par une loi binomiale de paramètres $n=10$ et $p=0.32$.
\item (\textit{par de correction automatique disponible pour le résultat final}
\[
P(X = 7) = \coefBino{10}{7}\times 0.32^{7} \times 0.68^{3}
\]
\item (\textit{par de correction automatique disponible pour le résultat final}
Il faut calculer la probabilité qu'il y ait 0 stylo avec un defaut.
\[
P(X = 0) = \coefBino{10}{0}\times 0.32^{0} \times 0.68^{10}
\]
Puis comparer ce nombre à 0,5.
\item Il faut calculer l'espérance
\[
E[X] = n\times p = 10 \times 0.32 = 3.2
\]
\end{enumerate}
\end{solution}
\end{document}
%%% Local Variables:
%%% mode: latex
%%% TeX-master: "master"
%%% End:

View File

@ -0,0 +1,367 @@
\documentclass[a4paper,10pt]{article}
\usepackage{myXsim}
% Title Page
\title{DM1 \hfill VANDROUX Guillemette}
\tribe{Maths complémentaire}
\date{\hfillÀ render pour le jeudi 27 mai}
\xsimsetup{
solution/print = true
}
\begin{document}
\maketitle
Les valeurs des exercices sont générés automatiquement. Si une valeur a un nombre adhérant de chiffres après la virgule, vous pouvez l'arrondir à l'entier le plus proche.
\begin{exercise}[subtitle={Optimisation de matière}]
\begin{minipage}{0.6\textwidth}
On se propose de fabriquer avec le moins de tôle possible une citerne fermée en forme de parallélépipède rectangle dont le volume intérieur doit être de $30m^3$. La longueur est aussi fixée à $3m$ par le cahier des charges.
On peut donc faire varier uniquement la largeur (notée $x$) et la hauteur (notée $h$) de la cuve.
\end{minipage}
\hfill
\begin{minipage}{0.3\textwidth}
\begin{tikzpicture}
\pgfmathsetmacro{\cubex}{3}
\pgfmathsetmacro{\cubey}{1}
\pgfmathsetmacro{\cubez}{2}
\draw[black,fill=gray] (0,0,0) -- ++(-\cubex,0,0) -- ++(0,-\cubey,0) node [midway, left] {$h$} -- ++(\cubex,0,0) node [midway, below] {$x$} -- cycle;
\draw[black,fill=gray] (0,0,0) -- ++(0,0,-\cubez) -- ++(0,-\cubey,0) -- ++(0,0,\cubez) node [midway, right] {$3m$} -- cycle;
\draw[black,fill=gray] (0,0,0) -- ++(-\cubex,0,0) -- ++(0,0,-\cubez) -- ++(\cubex,0,0) -- cycle;
\end{tikzpicture}
\end{minipage}
\begin{enumerate}
\item Expliquer pourquoi quand la largeur $x$ change, la hauteur $h$ doit elle aussi changer pour respecter les contraintes.
\item Démontrer que l'on doit avoir $h = \dfrac{10}{x}$.
\item On note $S(x)$ l'aire totale de la citerne (c'est à dire la somme des aires des six faces). Montrer que l'on peut écrire
\[
S(x) = 6x + 20 + \frac{60}{x}
\]
\item Démontrer que
\[
S(x) = \frac{6x^2 + 20x + 60}{x}
\]
\item Démontrer que
\[
S'(x) = \frac{6x^2 - 60}{x^2}
\]
\item En déduire le tableau de variation de $S(x)$ sur $\intOF{0}{10}$.
\item Déterminer les valeurs de $x$ et $h$ correspondant à une utilisation minimal de tôle.
\end{enumerate}
\end{exercise}
\begin{solution}
\begin{enumerate}
\item Le volume étant fixe si l'on fait varier $x$, $h$ doit aussi varier.
\begin{itemize}
\item Si $x = 2$ alors conserver un volume de $V=30$, $h$ doit être égale à $10 / 2$
\item Si $x = 3$ alors conserver un volume de $V=30$, $h$ doit être égale à $10 / 3$
\end{itemize}
\item Pour calculer le volume, on a
\begin{eqnarray*}
V &=& h\times x \times 3 \\
30 &=& h\times x \times 3 \\
x &=& \frac{30}{h\times 3} = \frac{10}{h}
\end{eqnarray*}
\item Pour calculer la surface totale, on ajoute la surface de chaque face. On a donc le calcul suivant
\begin{eqnarray*}
S(x) &=& x\times h \times 2 + x\times3\times2 + h\times 3\times 2\\
S(x) &=& x\times \frac{10}{x} \times 2 + x\times3\times2 + \frac{10}{x}\times 3\times 2\\
S(x) &=& 6x + 20 + \frac{60}{x}
\end{eqnarray*}
\item Pour trouver cette nouvelle forme, on met chaque élément sur le même dénominateur
\begin{eqnarray*}
S(x) &=& 6x + 20 + \frac{60}{x}\\
S(x) &=& \frac{6x\times x}{x} + \frac{20\times x}{x} + \frac{60}{x}\\
S(x) &=& \frac{6x^2 + 20x + 60}{x}
\end{eqnarray*}
\item On retrouve la formule $\frac{u}{v}$ à dériver
\[
u(x) = 6x^2 + 20x + 60 \Rightarrow u'(x) = 12x + 20
\]
\[
v(x) = x \Rightarrow v'(x) = 1
\]
Donc au numérateur on obtient
\begin{eqnarray*}
u'(x)\times v(x) - u(x)\times v'(x) &=& (12x + 20)\times x - (6x^2 + 20x + 60)\times 1\\
&=& 6x^2 - 60
\end{eqnarray*}
Donc
\[
S'(x) = \frac{6x^2 - 60}{x^2}
\]
\item Tableau de variations de $S$
\begin{itemize}
\item Valeur interdite: $x^2 = 0 \equiv x = 0$
\item Signe de $6x^2 - 60$: c'est un polynôme du 2e degré
\[
\Delta = 1440 > 0
\]
Il y a donc 2 racines
\[
x_1 = - 3.1622776601683795 \qquad
x_2 = 3.1622776601683795
\]
Et on sait que $6x^2 - 60$ est du signe de $a$ donc positif en dehors des racines
\item Le dénominateur $x^2$ est toujours positif.
\item Tableau de variations
\begin{tikzpicture}[baseline=(a.north)]
\tkzTabInit[lgt=3,espcl=3]{$x$/1,$6x^2 - 60$/1, $x^2$/1, $S'$/1, $S$/2}{$0$, $- 3.1622776601683795$, $10$}
\tkzTabLine{d,-, z, +, }
\tkzTabLine{d,+, , +, }
\tkzTabLine{d,-, z, +, }
\tkzTabVar{D+/ , -/ , +/ }
\end{tikzpicture}
\end{itemize}
\item On a donc une surface minimal pour $x=3.1622776601683795$ et $h = 31.6227766016837950$.
\end{enumerate}
\end{solution}
%%% Local Variables:
%%% mode: latex
%%% TeX-master: "master"
%%% End:
\begin{exercise}[subtitle={Bassin}]
Le tour d'un bassin au niveau du sol présente deux axes de symétrie : laxe des abscisses et la droite déquation $x=4$. Il est obtenu par symétrie de la courbe $\mathcal{C}_f$ sur $\intFF{0}{4}$$f$ est la fonction définie par
\[
f(x) = \left(- x^{2} + 5.0 x - 1.8\right) e^{- x} + 1.8
\]
On admet que sur $\intFF{0}{4}$ la fonction $f$ est positive.
\begin{enumerate}
\item Sur un repère, tracer l'allure de la courbe $\mathcal{C}_f$, les axes de symétries puis compléter pour dessiner la forme du bassin.
\item Montrer que la fonction $f$ admet comme primitive sur $\R$ la fonction $F$ définie par
\[
F(x) = 1.8 x + \left( x^{2} - 3.0 x - 1.2\right) e^{- x}
\]
\item Calculer la quantité $\ds \int_0^4 f(x) \; dx$, vous donnerez le résultat sous forme exacte. Interpréter le résultat et reportez cette quantité sur le graphique.
\item On considère que l'échelle de votre graphique est de 1unité pour 15m. Calculer l'aire du bassin. Vous donnerez un résultat arrondi au $m^2$ près.
\end{enumerate}
\end{exercise}
\begin{solution}
\begin{enumerate}
\item
\begin{tikzpicture}[baseline=(a.north), xscale=1, yscale=0.5]
\tkzInit[xmin=0,xmax=5,xstep=1,
ymin=0,ymax=10,ystep=1]
\tkzGrid
\tkzAxeXY
\tkzFct[domain=0:10,color=red,very thick]%
{ (-x**2 + 5.0*x - 1.8)*exp(-x) + 1.8 };
\end{tikzpicture}
\item Il faut dériver $F(x)$ et vérifier que $F'(x) = f(x)$.
\item $\ds \int_0^4 f(x) \; dx = F(4) - F(0) = \frac{2.8}{e^{4}} + 8.4$
\item La quantité calculée à la question précédente se retrouve 4fois pour former le bassin. Il faut ensuite prendre en compte l'échelle, comme 1unité de longueur correspond à 15m, une unité d'air correspond à $15\times15 = 225m^2$. Ainsi l'aire du bassin est égale à
\[
(\frac{2.8}{e^{4}} + 8.4)\times 4 \times 15^2 = 7606.000000
\]
\end{enumerate}
\end{solution}
%%% Local Variables:
%%% mode: latex
%%% TeX-master: "master"
%%% End:
\begin{exercise}[subtitle={Stylos}]
\emph{Les parties {\rm A} et {\rm B} de cet exercice sont indépendantes.}
\bigskip
\begin{minipage}{0.6\linewidth}
\textbf{Partie A}
\medskip
Deux ateliers A et B fabriquent des stylos pour une entreprise.
L'atelier A fabrique 55.00000000000001\,\% des stylos, et parmi ceux-là, 46.0\,\% possèdent un défaut de fabrication.
De plus, 20.0\,\% des stylos possèdent un défaut de fabrication et sortent de l'atelier B.
Un stylo est prélevé au hasard dans le stock de l'entreprise.
On considère les évènements suivants:
\begin{itemize}
\item A : \og Le stylo a été fabriqué par l'atelier A \fg
\item B : \og Le stylo a été fabriqué par l'atelier B \fg
\item D : \og Le stylo possède un défaut de fabrication \fg
\end{itemize}
\end{minipage}
\begin{minipage}{0.4\linewidth}
\begin{center}
\begin{tikzpicture}[sloped]
\node {.}
child {node {$A$}
child {node {$D$}
edge from parent
node[above] {...}
}
child {node {$\overline{D}$}
edge from parent
node[above] {...}
}
edge from parent
node[above] {...}
}
child[missing] {}
child { node {$B$}
child {node {$D$}
edge from parent
node[above] {...}
}
child {node {$\overline{D}$}
edge from parent
node[above] {...}
}
edge from parent
node[above] {...}
} ;
\end{tikzpicture}
\end{center}
\end{minipage}
\medskip
\begin{enumerate}
\item Compléter l'arbre de probabilité ci-contre
\item Interpréter puis donner les probabilités $P(A)$, $P(B)$, $P_A(D)$ et $P(B \cap D)$.
\item
\begin{enumerate}
\item Calculer la probabilité qu'un stylo provienne de l'atelier A et possède un défaut de fabrication.
\item En déduire que la probabilité qu'un stylo possède un défaut de fabrication est de $0.45$.
\end{enumerate}
\item On prélève un stylo au hasard avec un défaut. Quelle est la probabilité qu'il vienne de l'atelier A?
\end{enumerate}
\bigskip
\textbf{Partie B}
\medskip
Dans cette partie, on suppose que 45.0\,\% des stylos possèdent un défaut de fabrication.
L'entreprise confectionne des paquets contenant chacun $4$~stylos.
Le fait qu'un stylo possède ou non un défaut de fabrication est indépendant des autres stylos.
On appelle $X$ la variable aléatoire donnant pour un paquet le nombre de stylos qui possèdent un défaut de fabrication.
On admet que la variable aléatoire $X$ suit une loi binomiale.
\medskip
\begin{enumerate}
\setcounter{enumi}{4}
\item Avec quelle loi peut-on modéliser $X$. Préciser les paramètres.
\item Calculer et interpréter la probabilité $P(X = 13)$.
\item Le directeur de l'entreprise affirme qu'il y a plus d'une chance sur deux qu'un paquet ne comporte aucun stylo défectueux. A-t-il raison ?
\item Combien de stylos peut-on espérer avoir en moyenne?
\end{enumerate}
\pagebreak
\end{exercise}
\begin{solution}
\begin{enumerate}
\item
\begin{center}
\begin{tikzpicture}[sloped]
\node {.}
child {node {$A$}
child {node {$D$}
edge from parent
node[above] {0.46}
}
child {node {$\overline{D}$}
edge from parent
node[above] {0.54}
}
edge from parent
node[above] {0.55}
}
child[missing] {}
child { node {$B$}
child {node {$D$}
edge from parent
node[above] {0.44}
}
child {node {$\overline{D}$}
edge from parent
node[above] {0.56}
}
edge from parent
node[above] {0.45}
} ;
\end{tikzpicture}
\end{center}
\item
\begin{itemize}
\item Probabilité que le stylo vienne de l'atelier A
\[
P(A) = 0.55
\]
\item Probabilité que le stylo vienne de l'atelier B
\[
P(B) = 0.45
\]
\item Probabilité que le stylo ait un défaut sachant qu'il vient de l'atelier A.
\[
P_A(D) = 0.46
\]
\item Probabilité que le stylo vienne de l'atelier B et qu'il ait un défaut.
\[
P(D \cap D) = 0.2
\]
\end{itemize}
\item
\begin{enumerate}
\item Probabilité qu'un stylo vienne de l'atelier A et qu'il ait un defaut
\[
P(A\cap D) = P(A) \times P_A(D) = 0.55 \times 0.46 = 0.25
\]
\item Probabilité que le stylo ai un défaut de fabrication.
\[
P(D) = P(A\cap D) + P(B\cap D) = 0.25 + 0.2 = 0.45
\]
\end{enumerate}
\item Probabilité qu'il vienne de l'atelier A sachant qu'il a un defaut
\[
P_D(A) = \frac{P(A\cap D)}{P(D)} = \frac{0.25}{0.45} = 0.56
\]
\item $X$ peut être modélisée par une loi binomiale de paramètres $n=15$ et $p=0.45$.
\item (\textit{par de correction automatique disponible pour le résultat final}
\[
P(X = 13) = \coefBino{15}{13}\times 0.45^{13} \times 0.55^{2}
\]
\item (\textit{par de correction automatique disponible pour le résultat final}
Il faut calculer la probabilité qu'il y ait 0 stylo avec un defaut.
\[
P(X = 0) = \coefBino{15}{0}\times 0.45^{0} \times 0.55^{15}
\]
Puis comparer ce nombre à 0,5.
\item Il faut calculer l'espérance
\[
E[X] = n\times p = 15 \times 0.45 = 6.75
\]
\end{enumerate}
\end{solution}
\end{document}
%%% Local Variables:
%%% mode: latex
%%% TeX-master: "master"
%%% End:

Binary file not shown.

View File

@ -13,8 +13,11 @@
\begin{document}
\maketitle
Les valeurs des exercices sont générés automatiquement. Si une valeur a un nombre adhérant de chiffres après la virgule, vous pouvez l'arrondir à l'entier le plus proche.
\Block{include "./tpl_optimisation.tex"}
\Block{include "./tpl_bassin.tex"}
\Block{include "./tpl_stylos.tex"}
\end{document}

View File

@ -1,50 +1,49 @@
%- set latex = sympy.latex
%- set sqrt = sympy.sqrt
%- set exp = sympy.functions.exp
%- set integrate = sympy.integrate
\begin{exercise}[subtitle={Bassin}]
%- set Vinit = randint(1, 10)*100000
%- set tx = round((random()+1)/2, 1)
Le clinker est un constituant du ciment qui résulte de la cuisson d'un mélange composé de calcaire et d'argile. La fabrication du clinker nécessite des fours à très haute température qui libèrent dans l'air une grande quantité de dioxyde de carbone (CO$_2$).
%- set a = round(random()*10, 1)
%- set b = round(random()*10, 1)
%- set x = sympy.symbols("x")
%- set f = -(x**2 - a*x + b)*exp(-x) + b
%- set F = integrate(f, x)
Le tour d'un bassin au niveau du sol présente deux axes de symétrie : laxe des abscisses et la droite déquation $x=4$. Il est obtenu par symétrie de la courbe $\mathcal{C}_f$ sur $\intFF{0}{4}$$f$ est la fonction définie par
Dans une cimenterie, la fabrication du clinker s'effectue de 7 h 30 à 20 h, dans une pièce de volume \np{\Var{Vinit}}~dm$^3$.
À 20 h, après une journée de travail, le taux volumique de CO$_2$ dans la pièce est de \Var{tx}\,\%.
\[
f(x) = \Var{latex(f)}
\]
On admet que sur $\intFF{0}{4}$ la fonction $f$ est positive.
\begin{enumerate}
%- set v20 = int(Vinit*tx/100)
\item Justifier que le volume de CO$_2$ présent dans cette pièce à 20 h est de \np{\Var{v20}}~dm$^3$ .
%- set q = round(random()/10, 2)
%- set c = randint(20, 60)*10
%- set v0 = int(v20 - c)
%- set t = sympy.symbols("t")
%- set V = v0*exp(- q*t) + c
%- set Vp = V.diff()
\item On modélise le volume de CO$_2$ présent dans la pièce par une fonction du temps $t$ écoulé après 20h (exprimé en minutes) qui pour formule $V(t) = V_0e^{-\Var{q}t} + \Var{c}$
\begin{enumerate}
\item Démontrer que $V_0$ est égale à \np{\Var{v0}}.
%- set decal = randint(1, 4)
\item Quel sera, au dm$^3$ près, le volume de CO$_2$ dans cette pièce à \Var{20+decal} h ?
\item Démontrer que $V'(t) = \Var{latex(Vp)}$.
\item Étudier le signe de $V'(t)$ puis en déduire le sens de variation de $V(t)$.
\item Que peut-on dire du volume de CO$_2$ quand $t$ devient grand?
\end{enumerate}
\item Sur un repère, tracer l'allure de la courbe $\mathcal{C}_f$, les axes de symétries puis compléter pour dessiner la forme du bassin.
\item Montrer que la fonction $f$ admet comme primitive sur $\R$ la fonction $F$ définie par
\[
F(x) = \Var{latex(F) | replace("1.0", "")}
\]
\item Calculer la quantité $\ds \int_0^4 f(x) \; dx$, vous donnerez le résultat sous forme exacte. Interpréter le résultat et reportez cette quantité sur le graphique.
\item On considère que l'échelle de votre graphique est de 1unité pour 15m. Calculer l'aire du bassin. Vous donnerez un résultat arrondi au $m^2$ près.
\end{enumerate}
\end{exercise}
\begin{solution}
\begin{enumerate}
\item Volume à 20h: $\Var{Vinit}\times \Var{tx/100} = \Var{v20}$
\item
\begin{enumerate}
\item $t=0$ correspond à 20h.
Donc $V(0) = \Var{v20} = V_0e^{-\Var{q}\times 0} + \Var{c} = V_0 + \Var{c}$
Donc $V_0 = \Var{v20} - \Var{c} = \Var{v0}$
\item Il faut calculer $V(t)$ pour $t = \Var{decal}$ donc
\[
V(\Var{decal}) = \Var{round(V.subs(t, str(decal)), 2)}
\]
\item Pas de correction pour cette question.
\item Pas de correction pour cette question.
\item Pas de correction pour cette question.
\end{enumerate}
\begin{tikzpicture}[baseline=(a.north), xscale=1, yscale=0.5]
\tkzInit[xmin=0,xmax=5,xstep=1,
ymin=0,ymax=10,ystep=1]
\tkzGrid
\tkzAxeXY
\tkzFct[domain=0:10,color=red,very thick]%
{ \Var{f} };
\end{tikzpicture}
\item Il faut dériver $F(x)$ et vérifier que $F'(x) = f(x)$.
%- set surf = integrate(f, (x, 0, 4))
\item $\ds \int_0^4 f(x) \; dx = F(4) - F(0) = \Var{latex(surf)}$
\item La quantité calculée à la question précédente se retrouve 4fois pour former le bassin. Il faut ensuite prendre en compte l'échelle, comme 1unité de longueur correspond à 15m, une unité d'air correspond à $15\times15 = 225m^2$. Ainsi l'aire du bassin est égale à
\[
(\Var{latex(surf)})\times 4 \times 15^2 = \Var{round(sympy.N(surf*4*15**2, 10), 0)}
\]
\end{enumerate}
\end{solution}

View File

@ -0,0 +1,195 @@
\begin{exercise}[subtitle={Stylos}]
%- set pA = round(random(), 2)
%- set pB = round(1 - pA, 2)
%- set pD_A = round(random(), 2)
%- set pD_B = round(random(), 2)
%- set pDB = round(pB*pD_B, 2)
%- set pDA = round(pA*pD_A, 2)
%- set pD = round(pDA + pDB, 2)
\emph{Les parties {\rm A} et {\rm B} de cet exercice sont indépendantes.}
\bigskip
\begin{minipage}{0.6\linewidth}
\textbf{Partie A}
\medskip
Deux ateliers A et B fabriquent des stylos pour une entreprise.
L'atelier A fabrique \Var{pA*100 | round(2)}\,\% des stylos, et parmi ceux-là, \Var{pD_A*100 | round(2)}\,\% possèdent un défaut de fabrication.
De plus, \Var{pDB*100 | round(2)}\,\% des stylos possèdent un défaut de fabrication et sortent de l'atelier B.
Un stylo est prélevé au hasard dans le stock de l'entreprise.
On considère les évènements suivants:
\begin{itemize}
\item A : \og Le stylo a été fabriqué par l'atelier A \fg
\item B : \og Le stylo a été fabriqué par l'atelier B \fg
\item D : \og Le stylo possède un défaut de fabrication \fg
\end{itemize}
\end{minipage}
\begin{minipage}{0.4\linewidth}
\begin{center}
\begin{tikzpicture}[sloped]
\node {.}
child {node {$A$}
child {node {$D$}
edge from parent
node[above] {...}
}
child {node {$\overline{D}$}
edge from parent
node[above] {...}
}
edge from parent
node[above] {...}
}
child[missing] {}
child { node {$B$}
child {node {$D$}
edge from parent
node[above] {...}
}
child {node {$\overline{D}$}
edge from parent
node[above] {...}
}
edge from parent
node[above] {...}
} ;
\end{tikzpicture}
\end{center}
\end{minipage}
\medskip
\begin{enumerate}
\item Compléter l'arbre de probabilité ci-contre
\item Interpréter puis donner les probabilités $P(A)$, $P(B)$, $P_A(D)$ et $P(B \cap D)$.
\item
\begin{enumerate}
\item Calculer la probabilité qu'un stylo provienne de l'atelier A et possède un défaut de fabrication.
\item En déduire que la probabilité qu'un stylo possède un défaut de fabrication est de $\Var{pD}$.
\end{enumerate}
\item On prélève un stylo au hasard avec un défaut. Quelle est la probabilité qu'il vienne de l'atelier A?
\end{enumerate}
\bigskip
\textbf{Partie B}
%- set nbr = randint(10, 20)
%- set k = randint(int(nbr/2), nbr)
\medskip
Dans cette partie, on suppose que \Var{pD*100 | round(2)}\,\% des stylos possèdent un défaut de fabrication.
L'entreprise confectionne des paquets contenant chacun $4$~stylos.
Le fait qu'un stylo possède ou non un défaut de fabrication est indépendant des autres stylos.
On appelle $X$ la variable aléatoire donnant pour un paquet le nombre de stylos qui possèdent un défaut de fabrication.
On admet que la variable aléatoire $X$ suit une loi binomiale.
\medskip
\begin{enumerate}
\setcounter{enumi}{4}
\item Avec quelle loi peut-on modéliser $X$. Préciser les paramètres.
\item Calculer et interpréter la probabilité $P(X = \Var{k})$.
\item Le directeur de l'entreprise affirme qu'il y a plus d'une chance sur deux qu'un paquet ne comporte aucun stylo défectueux. A-t-il raison ?
\item Combien de stylos peut-on espérer avoir en moyenne?
\end{enumerate}
\pagebreak
\end{exercise}
\begin{solution}
\begin{enumerate}
\item
\begin{center}
\begin{tikzpicture}[sloped]
\node {.}
child {node {$A$}
child {node {$D$}
edge from parent
node[above] {\Var{pD_A}}
}
child {node {$\overline{D}$}
edge from parent
node[above] {\Var{round(1-pD_A, 2)}}
}
edge from parent
node[above] {\Var{pA}}
}
child[missing] {}
child { node {$B$}
child {node {$D$}
edge from parent
node[above] {\Var{pD_B}}
}
child {node {$\overline{D}$}
edge from parent
node[above] {\Var{round(1-pD_B, 2)}}
}
edge from parent
node[above] {\Var{pB}}
} ;
\end{tikzpicture}
\end{center}
\item
\begin{itemize}
\item Probabilité que le stylo vienne de l'atelier A
\[
P(A) = \Var{pA}
\]
\item Probabilité que le stylo vienne de l'atelier B
\[
P(B) = \Var{pB}
\]
\item Probabilité que le stylo ait un défaut sachant qu'il vient de l'atelier A.
\[
P_A(D) = \Var{pD_A}
\]
\item Probabilité que le stylo vienne de l'atelier B et qu'il ait un défaut.
\[
P(D \cap D) = \Var{pDB}
\]
\end{itemize}
\item
\begin{enumerate}
\item Probabilité qu'un stylo vienne de l'atelier A et qu'il ait un defaut
\[
P(A\cap D) = P(A) \times P_A(D) = \Var{pA} \times \Var{pD_A} = \Var{pDA}
\]
\item Probabilité que le stylo ai un défaut de fabrication.
\[
P(D) = P(A\cap D) + P(B\cap D) = \Var{pDA} + \Var{pDB} = \Var{pD}
\]
\end{enumerate}
\item Probabilité qu'il vienne de l'atelier A sachant qu'il a un defaut
\[
P_D(A) = \frac{P(A\cap D)}{P(D)} = \frac{\Var{pDA}}{\Var{pD}} = \Var{round(pDA/pD, 2)}
\]
\item $X$ peut être modélisée par une loi binomiale de paramètres $n=\Var{nbr}$ et $p=\Var{pD}$.
\item (\textit{par de correction automatique disponible pour le résultat final}
\[
P(X = \Var{k}) = \coefBino{\Var{nbr}}{\Var{k}}\times \Var{pD}^{\Var{k}} \times \Var{round(1 - pD, 2)}^{\Var{nbr-k}}
\]
\item (\textit{par de correction automatique disponible pour le résultat final}
Il faut calculer la probabilité qu'il y ait 0 stylo avec un defaut.
\[
P(X = 0) = \coefBino{\Var{nbr}}{0}\times \Var{pD}^{0} \times \Var{round(1 - pD, 2)}^{\Var{nbr}}
\]
Puis comparer ce nombre à 0,5.
\item Il faut calculer l'espérance
\[
E[X] = n\times p = \Var{nbr} \times \Var{pD} = \Var{round(nbr*pD, 2)}
\]
\end{enumerate}
\end{solution}